Download as pdf or txt
Download as pdf or txt
You are on page 1of 152

Krishna's 1000+

MCQs
B.Sc. Objective

Differential Equations and


Integral Transforms
(For B.Sc. II year Students of all Colleges affiliated to Universities in U.P.)

As Per U.P. Unified Syllabus

By

Manoj Garg
M.Sc., M.Phil, Ph.D.
C.S.I.R. NET (JRF)

Associate Professor & Head, Dep’t of Mathematics


Nehru P.G. College
Chhibramau, Kannauj (U.P.)

KRISHNA Prakashan Media (P) Ltd.


KRISHNA HOUSE, 11, Shivaji Road, Meerut-250 001 (U.P.), India
Syllabus
Differential Equations and Integral Transforms
B.Sc. II Year; II Paper

As per U.P. UNIFIED Syllabus (w.e.f. 2012-13)


M.M. : 33/65

Differential Equations
Unit 1
Formation of a differential equation (D.E.), Degree, order and solution of a D.E., Equations of first order and first
degree : Separation of variables method, Solution of homogeneous equations, linear equations and exact
equations, Linear differential equations with constant coefficients, Homogeneous linear differential equations.

Unit 2
Differential equations of the first order but not of the first degree, Clairaut’s equations and singular solutions,
Orthogonal trajectories, Simultaneous linear differential equations with constant coefficients, Linear differential
equations of the second order (including the method of variation of parameters).

Unit 3
Series solutions of second order differential equations, Legendre and Bessel functions (Pn and Jn only) and their
properties.

Order, degree and formation of partial differential equations, Partial differential equations of the first order,
Lagrange’s equations, Charpit’s general method, Linear partial differential equations with constant coefficients.

Unit 4 (i)
Partial differential equations of the second order, Monge’s method.

Integral Transforms
Unit 4 (ii)
The concept of transform, Integral transforms and kernel, Linearity property of transforms, Laplace transform,
Inverse Laplace transform, Convolution theorem, Applications of Laplace transform to solve ordinary differential
equations.

Unit 5
Fourier transforms (finite and infinite), Fourier integral, Applications of Fourier transform to boundary value
problems, Fourier series.

(iii)
Brief Contents
Syllabus.... ..............................................................................................III
Brief Contents..........................................................................................IV

Differential Equations and Integral Transforms

Unit-I
Chapter 1: Differential Equations of First Order and First Degree.................................................(01-11)
Chapter 2: Linear Differential Equations with Constant Coefficients.............................................(12-20)
Chapter 3: Homogeneous Linear Differential Equations..............................................................(21-26)

Unit-II
Chapter 4: Differential Equation of First Order but not of the First Degree...................................(27-34)
Chapter 5: Orthogonal Trajectories..............................................................................................(35-39)
Chapter 6: Ordinary Simultaneous Differential Equations............................................................(40-45)
Chapter 7: Linear Equations of Second Order With Variable Coefficients....................................(46-57)
Chapter 8: Partial Differential Equations of the First Order...........................................................(58-69)
Chapter 9: Linear Partial Differential Equation of Second and Higher Order with
Constant Coefficients.................................................................................................(70-80)

Unit-III
Chapter 10: Partial Differential Equations of Second Order with Variable
Coefficients : Monge’s Method...................................................................................(81-88)
Chapter 11: Legendre's Functions..................................................................................................(89-95)
Chapter 12: Bessel’s Functions....................................................................................................(96-100)
Chapter 13: Series Solutions of Differential Equations...............................................................(101-105)
Chapter 14: The Laplace Transform..........................................................................................(106-112)

Unit-IV
Chapter 15: The Inverse Laplace Transform..............................................................................(113-119)
Chapter 16: Applications of Laplace Transform.........................................................................(120-125)

Unit-V
Chapter 17: Fourier Transforms.................................................................................................(126-133)
Chapter 18: Finite Fourier Transform.........................................................................................(134-139)
Chapter 19: Applications of Fourier Transforms in Initial and Boundary Value Problems...........(140-143)
Chapter 20: Fourier Series.........................................................................................................(144-152)

(iv)
B.Sc. Objective Mathematics 2.2

Book-2
Differential Equations and Integral Transforms
Unit-I
Chapter -1 : Differential Equations of First Order and First Degree
Chapter -2 : Linear Differential Equations with Constant Coefficients
Chapter -3 : Homogeneous Linear Differential Equations

Unit-II
Chapter -4 : Differential Equation of First Order but not of the First Degree
Chapter -5 : Orthogonal Trajectories
Chapter -6 : Ordinary Simultaneous Differential Equations
Chapter -7 : Linear Equations of Second Order With Variable Coefficients
Chapter -8 : Partial Differential Equations of the First Order
Chapter -9 : Linear Partial Differential Equation of Second and Higher Order with
Constant Coefficients

Unit-III
Chapter -10 : Partial Differential Equations of Second Order with Variable
Coefficients : Monge’s Method
Chapter -11 : Legendre's Functions
Chapter -12 : Bessel’s Functions
Chapter -13 : Series Solutions of Differential Equations
Chapter -14 : The Laplace Transform

Contd...
...Contd.

Unit-IV
Chapter -15 : The Inverse Laplace Transform
Chapter -16 : Applications of Laplace Transform

Unit-V
Chapter -17 : Fourier Transforms
Chapter -18 : Finite Fourier Transform
Chapter -19 : Applications of Fourier Transforms in Initial and Boundary Value
Problems
Chapter -20 : Fourier Series
Unit-3I
C HAPTER
1 Differential Equations of First Order and First Degree
DEFINITIONS Mdx + Ndy = 0
1. An equation which involves differential where M and N are some functions of x and y
coefficients or derivatives is called a differential or are constants.
equation. 2. If the differential equation can be written in the
2. A differential equation is said to be partial if form f (x ) dx = φ(y)dy
there are two or more independent variables. where f (x ) is a function of x only and φ( y) is a
3. The order of a differential equation is the order function of y only then we get the solution by
of the highest derivative occuring in the integrating both sides and we say that the
equation. variables are separable.
4. The degree of a differential equation is the 3. A differential equation of the form
highest power of the highest order derivative dy f (x , y)
present in it when the derivative occurs in the = where f (x , y) and φ (x , y) are
dx φ (x , y)
form of polynomials i.e. made free from
homogeneous functions of x and y of the same
radicals and fractions so far as derivatives are
degree, is called a homogeneous equations in
concerned. dy dv
such cases we put y = vx and =v+ x
5. A differential equation is said to be linear if the dx dx
dependent variable and all its derivatives occur and then solve it by variables separable
in the first degree and they do not occur method.
together with multiple operation, otherwise
4. The differential equation of the form
called non-linear.
dy ax + by + c
6. The function y = f (x ) is said to be the solution = can be reduced to the
dx Ax + By + c
of a differential equation if, when putting in the
equation it reduces the equation to an identity. homogeneous form by substitution
dy dY
7. If in a differential equation, the independent x = X + h, y = Y + k and so =
dx dX
arbitrary constants are equal to the order of the
differential equation then the solution is called The given differential equation reduced to
the general solution. dY aX + bY + ah + bk + c
= , choose h and
8. A solution obtained by giving particular values dX AX + BY + Ah + Bk + C
to the arbitrary constants in the general solution
k such that
is called a particular solution or integral.
ah + bk + c = 0 and Ah + Bk + C = 0
9. The solution of an n th order differential
equation may contain n arbitrary constants. dY aX + bY
Then given equation becomes =
dX AX + BY
DIFFERENTIAL EQUATIONS OF FIRST ORDER and solve by substituting Y = VX .
AND FIRST DEGREE
This is called equations reducible to
1. The differential equation of first order and of
homogeneous form.
first degree is represented by
4

dy ∫ Mdx + ∫ Ndy = C
5. An equation of the form + Py = Q where P
dx y as constant Only those
terms in
and Q are functions of x only is called a linear N which do
differential equation of the first order. In such not contain x
Pdx
case, multiply both sides by e∫ i.e.
EQUATION REDUCIBLE TO THE EXACT
 dy 
e ∫ Pdx .  + Py = Q. e ∫ Pdx EQUATIONS (INTEGRATING FACTORS)
 dx 
1. A differential equation which is not exact can
d  Pdx  Pdx
i.e.  y . e∫  = Q ⋅ e∫ sometimes be made exact by multiplying by
dx  
some suitable function of x and y then such
Integrating both sides, we get function is called an integrating factor. This
Pdx Pdx
y e∫ = Q. e∫
∫ + C which is a solution. integrating factor can by formed by inspection.

dy 2. If the differential equation Mdx + Ndy = 0 is of


6. The differential equation + Py = Qy n the form
dx
where P and Q are functions of x alone is called [ f (x , y)] ydx + [φ (x , y)] x dy = 0 and
Bernoulli’s equation. 1
Mx − Ny ≠ 0, then is an integrating
Mx − Ny
On dividing both sides by y n, we get
factor.
dy
y −n + Py − n + 1 = Q
dx 3. If the differential equation Mdx + Ndy = 0 is
dy dV homogeneous equation and Mx + Ny ≠ 0
Put y − n + 1 = V so (1 − n) y − n = it 1
dx dx then is an integrating factor.
dV Mx + Ny
reduces to + (1 − n) P. V = (1 − n)Q which is
dx 1  ∂M ∂N 
linear. 4. If  −  = f (x ) is a function of x alone
N  ∂y ∂x 
It is called equations reducible to the linear then the integrating factor for Mdx + Ndy = 0
from. f( x)dx
is e∫ .
7. An exact differential equation is formed by
1  ∂N ∂M 
directly differentiating its primitive i.e. solution 5. If  −  = f (y) is a function of y alone,
M  ∂x ∂y 
without any other process.
then the integrating factor for Mdx + Ndy = 0
The necessary and sufficient condition for the f( y) dy
is e ∫ .
arbitrary differential equation Mdx + Ndy = 0
to be exact is that 6. If the differential equation is
∂M ∂N x ay b (mydx + nxdy) + x c y d ( pydx + qxdy) = 0
=
∂y ∂x Where a, b, m, n c, d, p, q are constants, then
If the differential equation Mdx + Ndy = 0 is its integrating factor is x hy k , where h and k can
exact then its solution is be obtained by multiplying the given equation
by x hy k and equation must become exact.
5

EXERCISE
MULTIPLE CHOICE QUESTIONS 11. The solution of
dy
+ y = e − x is
x −x dx
1. y = Ae + Be , is the general solution of the
(K.U. 2015) (a) ye x = x + c (b) xe y = x + c
differential equation
(c) xe y = y + c (d) ye x = y + c
d 2y d 2y
(a) =−y (b) + y+ 1= 0
dx 2 dx 2 dy
12. The integrating factor of (x + 2y 3) = y is
2
d y 2
d y dx
(c) − y=1 (d) =y 1 1
dx 2 dx 2 (a) (b) y (c) x (d)
x y
2
d y
2. The differential equation + sin y = sin x is 13. y = Ax − A2, is the general solution of the
dx 2
(K.U. 2014) differential equation
(a) Linear of order 2 (b) Non linear of order 2 (a) (y ′)2 − xy ′ + y = 0 (b) y ′′ + y ′ = 0
(c) Linear of degree 2 (d) Non linear of degree 2 2
(c) (y ′) + y = 0 (d) y ′′ − y ′ = A
3. The order and degree of the differential equation
3
14. The differential equation whose solution is
 d 2y  4
dy y = a cos (x + 3), is
x  +   + y 2 = 0 are
 dx 2   dx  (a) y ′ + tan(x + 3) = 0 (b) y ′′ + tan (x + 3) = 0
(a) 2 and 3 (b) 3 and 2 (c) 2 and 1 (d) 3 and 4 (c) y ′ − tan (x + 3) = 0 (d) y ′′ − tan (x + 3) = 0

4. The differential equation corresponding to the 15. The solution of sec2 x tan y dx + sec2 y tan x dy
equation y = A cos x + B sin x is = 0 is
(a) y ′′ + y ′ + y = 0 (b) y" + 2y ′ + y = 0 (a) cot x tan y = c (b) tan x cot y = c
(c) y ′′ + y = 0 (d) y ′ + y = 0 (c) cot x cot y = c (d) tan x tan y = c
5. The solution of (1 + x) ydx + (1 − y) x dy = 0 is 16. The solution of cos (x + y)dx = dy is
x−y x+y (a) tan (x + y) = c (b) cos (x + y) = c
(a) x + y = ce (b) x − y = ce
x + y x + y
(c) xy = ce x−y
(d) xy = ce y − x (c) sin   = x + c (d) tan   =x+ c
 2   2 
dy dy y 2
6. If for differential equation = ex +y
, y (1) = 1 then 17. The solution of x + = y is
dx dx x
y (−1) is equal to
(a) cx = e x / y (b) cy = e x / y
(a)1 (b) − 1 (c) 0 (d) 2
(c) cx = e x − y (d) cy = e y − x
dy y2
7. The solution of = is 18. The differential equation yx ′ = y − 1, y(0) = 1 has
dx xy − x 2
(a) Infinite number of solutions
(a) x = ke y / x (b) y = ke x / y (b) Unique solution
(c) y = ke y / x (d) y = ke x (c) No solution (d) Many solutions
dy
8. If the function 1 and − e −3x are the fundamental 19. The integrating factor of cos 2 x + y = tan x is
dx
solutions of the equation y ′′ + py ′ − 3y = 0 then, p is
(a) tan x (b) cot x (c) e cot x
(d) e tan x
(a) 0 (b) 1 (c) –1 (d) 3
20. The solution of (y cos x + 1)dx + sin x dy = 0 is
9. The degree of the differential equation
(a) y + x sin x = c (b) y + x cos x = c
y ′′ + 1 + y ′ = 0 is
(c) x + y sin x = c (d) x + y cos x = c
(a) 1 (b) 2 (c) 3 (d) 0
dy
dy 2 21. The integrating factor of x + y = y 2 log x is
10. The integrating factor of + 2xy = e − x is dx
dx
1 1 1 1
2 3
(a) (b) (c) (d)
(a) e x (b) e x (c) e x (d) xe x
x2 x y y2
6

22. The differential equation Mdx + Ndy = 0 is exact 32. In homogeneous equation the degree of each term
when is
(a) Mx = Ny (b) My = Nx (a) Different (b) One
∂M ∂N ∂M ∂N (c) Two (d) Same
(c) = (d) =
∂x ∂y ∂y ∂x 33. If the equation (x 2 + ay + x)dx + (y 2 + x − y) dy
23. The solution of (1 + e x / y )dx + e x / y (1 − x / y) dy = 0 is exact then a is
= 0 is (a) 0 (b) 1 (c) 2 (d) Not exist
(a) x + y e x / y = c (b) y + e x / y = c 34. If the subnormal is constant then the curve is
(c) x + e x / y = c (d) None of these (a) Circle (b) Ellipse
(c) Hyperbola (d) Parabola
24. If x hy k be the integrating factor of
2 2 3
(y + 2x y)dx + (2x − xy)dy = 0 then h and k are dx 1 tan −1 y
35. The differential equation + x = is
2
dy y 1 + y2
(a) −5/ 2, 1 / 2 (b) 5/ 2, 1 / 2
(c) 5/ 2, − 1 / 2 (d) − 5/ 2, − 1 / 2 linear only when dependent variable is/are
(a) x (b) y
25. The integrating factor of
(c) Either x or y (d) Neither x nor y
(x 2y 2 + xy + 1)ydx + (x 2 y 2 − xy + 1)x dy = 0 is
1 1 36. The differential equation (y 2e x + 2xy)dx − x 2dy
(a) (b)
2xy 2x 2y = 0 is
1 1 (a) Exact (b) Not exact
(c) (d)
2 2 2 (c) Linear (d) Homogeneous
2xy 2x y

26. If the polar subtangent is constant then the curve is 37. The number of arbitrary constants in the solution of
(a) r(θ − c) = a (b) r θ = a differential equation of order 2 is
(c) r (θ + c) + a = 0 (d) r + θ = a (a) 1 (b) 2 (c) 3 (d) 4

27. If the equation y sin x dx − sin x dy = 0 is exact 38. The order of the differential equation
then x is equal to 2/ 3
 2
 3   d 2y
(a) 0 (b) π/ 2 (c) π/ 3 (d) π/ 4 4 +  d y   = + 1 is
dy y
  dx 3   dx 2
28. The solution of + = x 2 if y(1) = 1 is  
dx x (a) 1 (b) 2 (c) 3 (d) 4
x2 x4 3
(a) xy = + c (b) xy = + 39. The degree of the differential equation
4 4 4
3/ 2
x2 3 x2 3 2 2
 dy  + 2 =  d y 
(c) xy = + (d) xy = + is
4 4 4 4  dx   dx 2 
29. The differential equation (a) 1 (b) 3 (c) 2 (d) 3/2
xdy − ydx
xdx + ydy + = 0 is 40. The solution of (xy + x)dy − (xy + y)dx = 0 is
x 2 + y2
(a) x = cye y − x (b) y = cxe y − x
(a) Homogeneous (b) Of order 2
(c) xy = ce y − x (d) xy = ce x −y
(c) Linear (d) Exact
dy
41. The solution of = 1 + tan (y − x) is
30. The solution of (x 2 + y 2) dx − 2xy dy = 0 is dx
x
(a) x − y = cx (b) x 2 + y 2 = cx (a) sin (y + x) = ce (b) cos (y − x) = ce x
2 2
(c) x − y = cx (d) x + y = cx (c) tan (y − x) = ce x (d) sin (y − x) = ce x
dy dy y y
31. The solution of x = y (log y − log x + 1) is 42. The solution of = tan + is
dx dx x x
(a) y = xe cx (b) x = ye cx (a) tan y / x = cx (b) sin y / x = cx
(c) y = x 2 + c (d) x = y + c (c) cos y / x = cx (d) cot y / x = cx
7

43. The solution of 53. If the differential equation Mdx + Ndy = 0 is


x cos x dy + (xy sin x + y cos x − 1) dx = 0 homogeneous and Mx + Ny ≠ 0 then its integrating
(a) xy = c sin x ⋅ cos x factor is
(b) x − y = sin x + c cos x (a) Mx + Ny (b) Mx − Ny
1 1
(c) x + y = sin x + c cos x (c) (d)
Mx + Ny Mx − Ny
(d) xy = sin x + c cos x
dy 1  ∂M ∂N 
44. The solution of + tan x tan y = cos x sec y is 54. If f (x) =  −  is a function of x alone in
dx N  ∂y ∂x 
(a) sin x sec y = x + c (b) sin y sec x = x + c the differential equation Mdx + Ndy = 0 then
(c) sin x sec x = y + c (d) sin x cos y = x + c Integrating factor is
ydx xdy f ( y) dx f ( x )dx
45. If x hy k be the integrating factor of (a) e ∫ (b) e ∫ (c) e ∫ (d) e ∫
(3y − 2xy 3)dx + (4x − 3x 2y 2) dy = 0 then h and k is 55. The solution of cos (x + y)dx = dy is
x + y x + y
equal to (a) tan   = y + c (b) tan   =x+ c
 2   2 
(a) 2, 3 (b) 3, 2 (c) 2, 4 (d) 3, 4
x + y
dy (c) tan   =x+ y+ c
46. The integrating factor of + Py = Q is  2 
dx
Qdy Pdy Qdx Pdx (d) None of these
(a) e ∫ (b) e ∫ (c) e ∫ (d) e ∫
56. The differential equation corresponding to
47. The integrating factor of (y + 2x 3) dx − x dy = 0 is 2x −3x x
y = Ae + Be + Ce is
(a) 1/ x 2 (b) x (c) e x (d) e − x
(a) y ′′′ − 5y ′ + 3y = 0 (b) y ′′′ − 7 y ′ + 6y = 0
48. The solution of y ′ + y tan x = cos x , y(0) = 0 is (c) y ′′′ − 7 y ′ + y = 0 (d) y ′′′ + 7 y ′ − 3y = 0
(a) x sin x (b) x cos x (c) sin x (d) cos x 57. The differential equation corresponding to
49. In solving
dy x + y + 2
= the substitution are y = e x ( A cos x + B sin x) is
dx x − y + 3
(a) y ′′ − 2y ′ + 2y = 0 (b) y ′′ − 2y ′ + y = 0
(a) x = X + h, y = Y − k (c) y ′′ − y ′ + 2y = 0 (d) y ′′ − 2y ′ + 3y = 0
(b) x = X 2 + h, y = Y 2 + k
d 2y 2 dy
58. The solution of + = 0 is
(c) x = X + h, y = Y + k 2 x dx
dx
(d) x = X + 1, y = Y + 1
(a) y = Ax + B (b) y = A/ x + B
3/ 2
dy (c) x = A/ y + B (d) x = Ay + B
50. The differential equation   + y=x 2/ 3
is
 dx  dy −2xy
59. The solution of = at y(1) = 2 is
(a) Linear (b) Non linear of order 1 dx x 2 + 1
(c) Non linear of order 3/2
(a) y(x 2 + 1) = 1 (b) x(y 2 + 1) = 4
(d) Linear of order 1
dy (c) y(x 2 + 1) = 4 (d) x(y 2 + 1) = 1
51. The differential equation + Py = Qy n can be
dx dy y 2
reduced to linear form by substituting 60. The solution of x + = y at y(1) = 1 is
dx x
1 1
(a) =V (b) =V (a) ex = e x / y (b) x = e x / y
y n−1 yn
1 1 (c) ey = e x / y (d) y = e x / y
(c) =V (d) = V
yn + 1 y dy y y
61. The solution of = + tan at y(1) = π / 2 is
52. The differential equation yx ′ = y − 1 with x(1) = 0 dx x x
y y
has (a) cos   = x (b) tan   = x
x x
(a) Unique solution
y x
(b) Infinite number of solutions (c) sin   = x (d) cos   = x
x  y
(c) No solution (d) Only two solutions
8

62. The differential equation 65. The differential equation of all circles passing
2 2 through the origin and having their cenres on the
(3a x + by cos x) dx + (2 sin x − 4ay ′)dy = 0 is exact
when y-axis is
(a) a = 2, b = 3 (b) a = 3, b = 2 (a) (x 2 − y 2) y ′ + xy = 0
(c) a = 3, b = 3 (d) a = 2, b = 2 (b) (y 2 − x 2) y ′ + y = 0
63. The solution of the differential equation (c) (y 2 − x 2) y ′ + 2xy = 0
dy π
sin x = y log y with y = e is (d) y ′′ + xy ′ + 2xy = 0
dx 2
dy dy 
(a) e cot x/2
(b) e sin x / 2 (c) e cos x / 2 (d) e tan x / 2 66. The solution of y − x = a  y 2 +  is
dx  dx 
dy
64. The solution of = 2 y with y(0) = 0 is (a) y = c (1 − ay) (x + a) (b) y = ay (x + a)
dx
(c) y = c (1 + ay) x (d) y = c (1 + ay) (x − a)
(a) No solution (b) Unique solution
(c) Infinite solution (d) Finite solution

ANSWERS
MULTIPLE CHOICE QUESTIONS
1. (d) 2. (b) 3. (a) 4. (c) 5. (d) 6. (b) 7. (c) 8. (a) 9. (b) 10. (b)
11. (a) 12. (d) 13. (a) 14. (a) 15. (d) 16. (d) 17. (a) 18. (b) 19. (d) 20. (c)
21. (b) 22. (d) 23. (a) 24. (d) 25. (d) 26. (c) 27. (d) 28. (b) 29. (d) 30. (c)
31. (a) 32. (d) 33. (b) 34. (d) 35. (a) 36. (b) 37. (b) 38. (c) 39. (b) 40. (a)
41. (d) 42. (b) 43. (d) 44. (b) 45. (a) 46. (d) 47. (a) 48. (b) 49. (c) 50. (b)
51. (a) 52. (a) 53. (c) 54. (d) 55. (b) 56. (b) 57. (a) 58. (b) 59. (c) 60. (a)
61. (c) 62. (b) 63. (d) 64. (b) 65. (c) 66. (a)

HINTS AND SOLUTIONS


1. y = Ae x + Be − x differentiating two times dy y2 dy dv
7. = put y = vx i.e. =v+ x
2 dx xy − x 2 dx dx
dy d y
= Ae x − Be − x , = Ae x + Be − x
dx dx 2
dv v 2x 2 v2
d 2y v+ x =
2
=
so, =y dx (v − 1)x v−1
dx 2
5. (1 + x) ydx + (1 − y) xdy = 0 dv v2 v
so x = −v =
dx v − 1 v −1
 1 + x  dx +  1 − y  dy = 0
  v − 1 dx 1 dx
 x   y  i.e.   dv = ⇒  1 −  dv − =0
 v  x  v x
1 1 
⇒  + 1 dx +  − 1 dy = 0 Integrating v − log v − log x = log c
x  y 
xv
integrating it we get log x + x + log y − y = log c ⇒ log c   = v
 c 
xy
i.e. log   = y − x i.e. xy = ce y − x i.e. log cy = y / x ⇒ cy = e y / x ⇒ y = k e y / x
 c 
2xdx 2
10. The integrating factor is e ∫ = ex
9

dy 1 dx dy 1 dy 1 log x
11. (D + 1) y = e − x i.e. + y = e − x I.F. is e ∫ = ex 21. x + y = y 2 log x or + =
dx dx y 2 dx xy x
So solution is y e x = ∫ e x , e − x dx + C = x + C 1 1 dy dv
put = v i.e. − =
y y 2 dx dx
13. y = Ax − A2, y′ = A
dv v log x dv 1 log x
we get − + = ⇒ − v=
So (y ′)2 − xy ′ + y = A2 − Ax + Ax − A2 = 0 dx x x dx x x
1
sec2 x sec2 y − ∫ dx 1
15. dx + dy = 0 integrating, we get So. I.F. is e x = e − log x = elog 1/ x =
tan x tan y x
log tan x + log tan y = log c 24. If x h y k be the I.F. then the equation
log (tan x tan y) = log c ⇒ tan x tan y = c (x yh k + 2 + 2x h+ 2y k +1) dx
dy dv
16. Put x + y = v, 1 + = put it in + (2x h+ 3y k − x h + 1 y k + 1)dy = 0 is exact
dx dx
cos (x + y)dy = dx ∂M ∂N
So =
∂y ∂x
dv cos v dv
We get cos v  − 1 = 1 i.e. = dx
 dx  1 + cos v i.e. (k + 2) y k + 1x h + 2x h + 2 (k + 1) y k

 1  = 2(h + 3)x h+ 2y k …… (h + 1) x hy k + 1
i. e. 1 −  dv = dx
 1 + cos v  Comparing the coefficient we get

or  1 − 1 sec2 v  dv = dx k + 2 = − h − 1 and 2k + 2 = 2h + 6
 2 2
after solving we get h = − 5/ 2, y = − 1 / 2
v
integrating we get v − tan =x+ C 25. The given equation is (x 2y 2 + xy + 1)y dx
2
x+ y + (x 2y 2 − xy + 1)x dy = 0
⇒ x + y − tan   =x+c
 2 
It is of the form [ f (x, y)]ydx + [φ (x, y)] xdy = 0
x + y
i.e. y = c + tan   1
 2  so its integrating factor is
Mx − Ny
dx  1
18. y = y − 1 ⇒  1 −  dy = dx 1
dy  y i.e.
xy(x 2y 2 + xy + 1) − xy (x 2y 2 − xy + 1)
Integrating y − log y = x + log c ⇒ y − x = log cy 1
=
i.e. cy = e y − x now put y(0) = 1 2x 2y 2
we get c = e dθ
26. The polar sub tangant is r 2 =a
So required solution is ey = e y−x
i.e. unique dt
a
solution. i.e. dθ = dr integrating it
r2
20. (y cos x + 1) dx + sin x dy = 0 a
θ+ c = − i.e. r (θ + c) + a = 0
∂M ∂ N r
Here M = y cos x + 1, N = sin x so =
∂y ∂x ∂ M ∂N
27. y sin x dx + sin x dy = 0 is exact then =
∂y ∂x
i.e. given differential equation is exact so integrating
it where M = y sin x, N = sin x
so sin x = cos x ⇒ x = π/ 4
∫ (y cos x + 1) dx taking y constant
dy x 2 + y 2
+ sin x dy = c taking free from. 30. Given that = , put y = vx
∫ dx 2xy

y sin x + x = c dy dv
i.e. =v+ x then come get
dx dx
10
y
dv x 2 (1 + v 2) 1 + v 2 i.e. sin = cx
v+ x = = x
dx 2x 2v 2v
dy
44. Given + tan x tan y = cos x sec y
2v dx dx
i.e. dv = integrating it
1 − v2 x dy
i.e. cos y + tan x . sin y = cos x , put sin y = v
dx
log (1 − V 2) = − log x + log c
dy dv dv
i.e. cos y = we get + tan x . v = cos x
⇒ log {x (1 − v 2)} = log c dx dx dx
tan x dx
I.F. is e ∫ = elog sec x = sec x
 y2 
or x(1 − v 2) = c or x 1 − =c So solution is v . sec x = ∫ sec x ⋅ cos x dx + c
 x2
i.e. sin y ⋅ sec x = x + c
i.e. x 2 − y 2 = cx
47. Given that (2x 3 + y)dx − x dy = 0
2 2
33. If (x + ay + x)dx + (y + x − y)dy = 0 is exact ∂M ∂N
here M = 2x 3 + y, N = − x i.e. = 1, = −1
∂M ∂N ∂y ∂x
then = i.e. a = 1
∂y ∂x
1  ∂M ∂N  1 2
So,  −  = − (1 + 1) = − = f (x)
40. Given that (1 + y)xdy − (1 + x)ydx = 0 N  ∂y ∂x  x x
1 + y 2 1
i.e.   1 + x  dx = 0 −∫ dx log 1
 dy −  so I.F. is e x = e −2log x = e x2 =
 y  x 
x2
1  1  dy
 + 1 dy −  + 1 dx = 0 integrating it, we get 48. + tan x. y = cos x, y(0) = 0
y  x dx
y I.F. is e ∫ tan x dx = elog sec x = sec x
log y + y − log x − x = log c ⇒ log =x−y
xc
so solution is y sec x = ∫ sec x ⋅ sin dx + c = x + c
 xc  y y−x
i.e. log   = − x + y ⇒ x= e
 y c i.e. y sec x = x + c
y−x put y (0) = 0 ⇒ 0 = 0 + c ⇒ c = 0
⇒ x = c′ y e
so required solution is y sec x = x ⇒ y = x cos x
dy dv
41. Put y− x =v i.e. = +1 in dx
dx dx 52. Given that x = y − 1 and x(1) = 0
dy dy
= 1 + tan (y − x)
dx x 2 y2
i.e. x dx = (y + 1) dy integrating it= −y+ c
dv 2 2
we get + 1 = 1 + tan(v) ⇒ cot v dv = dx 1 1
dx put x(1) = 0 we get 0 = − 1 + c ⇒ c =
2 2
sin (y − x) 
i.e. log sin v = x + log c ⇒ log   =x 2 2  1
So required solution is x − y = 2  − y
 c  2 
sin (y − x)
⇒ = e x ⇒ sin (y − x) = ce x i.e. a unique solution.
c
56. Given y = Ae 2x + Be −3x + Ce x
y
42. Put = v i.e. y = vx
x differentiate it y ′ = 2 A e 2x − 3Be −3x + Ce x
dy dv dy y y y ′′ = 4 Ae 2x + 9 Be −3x + Ce x
or =v+ x in = tan +
dx dx dx x x
dv dv y ′′′ = 8 Ae 2x − 27 Be −3x + Ce x
v+ x = tan v + v ⇒ x = tan v
dx dx after solving we get y ′′′ − 7 y′ + 6y = 0
dx d 2y 2 dy dy d 2v dv
i.e. dv cot v − =0 58. + = 0, put = v, =
x 2
dx x dx dx dx 2 dx
Integrating it, we get dv 2
we get + v=0
log sin v − log x = log c ⇒ sin v = cx dx x
11
dv 2 x 2 + (y − α)2 = r 2 …(i)
⇒ + dx = 0 integrating it
v x
since it pass through (0, 0) so we have
dy c
log v + 2 log x = log c ⇒ vx 2 = c ⇒
= α2 = r2
dx x 2
c A So (1) reduces to x 2 − (y − α)2 = α 2
again integrating, we get y = − + B = + B
x x ⇒ x 2 + y 2 = 2αy …(ii)
2
59. We have 2xydx + (x + 1)dy = 0 Differentiating it w.r.t. x
∂M ∂N
2 dy dy
Here, M = 2xy, N = x + 1 so = = 2x 2x + 2y = 2α
∂y ∂x dx dx
dy dy
i.e. equation is exact, so integrating it 2xy + 2y 2 = (x 2 + y 2)
2 dx dx
∫ 2xydx + ∫ (x + 1)dy = c
i.e., (y 2 − x 2) y ′ + 2xy = 0
y is constant Free from x
dy dy  dy dx
x 2y + y = c ∵ y(1) = 2 we get c = 4 66. Given y − x = a  y 2 + ⇒ =
dx  dx  y − ay 2 x+ a
∴ y(x 2 + 1) = 4
1 a  1
y  +  dy = dx ,
61. Put = v we get required solution.  y 1 − ay  x+ a
x
65. Let Co-ordinates of the centre of the circles be (0, α) Integrating both sides, we get
and their radii r then equation of circle is log y − log (1 − ay) = log (x + a) + log c
y
 y 
log   = log {c(x + a)}
1 − ay 
⇒ y = c (1 − ay) (x + a)
mmm
(0, α)
9z
x
12
Unit-I
C HAPTER
2
Linear Differential Equations
with Constant Coefficients
DEFINITIONS Case III. If two roots m1 = α + iB and
1. A linear differential equation is an equation in m2 = α − iB are complex the C.F. is
which the dependent variable and its y = e αx (C 1 cos βx + C 2 sin βx )
derivatives appear only in the first degree.
or C 1e αx cos (βx + C 2 )
d ny d n− 1y d n− 2 y
+ a1 + a2
dx n dx n− 1 dx n− 2 If the imaginary roots also repeated i.e. α + iβ
dy and α − iβ occur twice then C.F. is
+ … + a n− 1 + a ny = Q
dx y = e αx {(C 1 + C 2 x ) cos βx
is a linear differential equation of order n with
+ (C 3 + C 4 x ) sin βx }
constant coefficients. Here a 1, a 2 , … a n are all
constants and Q is a function of x. Case IV. If a pair of the roots are irrational i.e.
2. If y = f (x ) and y = g (x ) are the general solution α ± β then C.F. is
(Complementary function) and particular e αx(C 1 cos h β x + C 2 sin h β x )
solution (P.I.) respectively then y = f (x ) + g (x )
or C 1e αx sin h ( β x + C 2 )
i.e. y = C. F. + P. I.
or C 1e αx cos h ( β x + C 2 )
DETERMINATION OF COMPLEMENTARY
FUNCTION (C.F.) THE PARTICULAR INTEGRAL (P.I.)
Consider a linear differential equation of order Let the linear differential equation of n th order
n with constant coefficient f (D) y = 0 is
i.e. [D n + a 1D n − 1 + a 2 D n− 2 (D n + a 1D n− 1 + a 2 D n− 2 + … + a n)y =r (x ), r (x ) ≠ 0
+ … + a n] y = 0 …(1)
i.e. f (D)y = Q (x ) then its particular integral
then its auxiliary equation (A.E.) is 1
(P.I.) is Q (x ).
m n + a 1 m n − 1 + … + a n = 0 if it will given n F (D)
roots say m1, m2 … mn then Case I. If differential equation is f (D) y = x m
Case I. If all the roots m1, m2 … mn are distinct 1
then P.I. is x m = [ f (D)]−1 x m expending
then general solution (C.F.) is f (D)
y = C 1e m1 x + C 2 e m2 x + … + C ne mn x [ f (D)]−1 in ascending powers of D by using
Case II. If two roots m1 = m2 are equal then binomial theorem.
C.F. is Case II. For f (D) = e ax φ( x )
m1 x m3 x mn x
(C 1 + C 2 x ) e + C3e + … + C ne
1 1
P.I. is e ax φ(x ) = e ax φ(x )
Similarly, if three roots m1 = m2 = m3 are equal f (D) f (D + a)
then C.F. is
Case III. For f (D) = sin ax or cos ax
(C 1 + C 2 x + C 3 x ) e m1 x
+ C 4 e m4 x + … + C ne mn x
13

1 1 1 1 ax
then P.I. is sin ax = sin ax , e ax = e if f (a) ≠ 0
f (D )2 2
f (− a ) f (D) f (a)
1
provided f (− a ) ≠ 0 2
=x e ax if f (a) = 0
f ′ (a)
1 1 1
and cos ax = cos ax , provided = x2 e ax if f ′(a) = 0
f (D 2 ) f (− a 2 ) f ′′(a)
f (− a 2 ) ≠ 0 = e ax
1
. 1 if f (a) = 0
f (D + a)
If f (− a 2 ) = 0 then
1
1 1 Case V. φ(x ) = e ax ∫e
− ax
φ(x ) dx
sin ax = x ⋅ sin ax , similar for D−a
f (D 2 ) f ′ (− a 2 )
cos ax Case VI. For f (D) = xV, V is any function of x

If f ′(− a 2 ) = 0 then 1
(xV) = x
1
V−
f ′(D)
⋅V
1 1 f (D) f (D) { f (D)} 2
sin ax = x 2 sin ax , similar for
2
f (D ) f ′′ (− a 2 ) Case VII.
1
φ (x ) = e − ax ax
D+a ∫e φ (x ) dx
cos ax and so on.
Case IV. If f (D) = e ax then

EXERCISE
MULTIPLE CHOICE QUESTIONS (a)
1
x e− x (b)
1 2 −x
x e
2 6 6
d y 1 3 −x 1
1. The solution of + 4y = 0 is (c) x e (d) x 4 e − x
dx 2 6 6
(a) C1 cos x + C2 sin x 5. The solution of (D 2 + 1) y = 0 when y(0) = 2 and
(b) C1 cos 2x + C2 sin 2 x y( π / 2) = − 2 is
(c) C1e 2x + C2e −2x (a) cos x + sin x (b) 2 cos x + 2 sin x
x −x (c) 2 cos x + sin x (d) 2 cos x − 2 sin x
(d) C1e + C2e
6. The particular integral of (D 2 + 9)y = cos 4x is
2. The particular integral of y ′′ − 5y ′ + 6 y = sin 3x is 1
(a) cos 4x (b) cos 4x
1 7
(a) (cos 3x − sin 3x)
12 1 1
(c) cos 4x (d) − cos 4x
1 5 7
(b) (5 cos 3x − sin 3x)
12 7. The particular integral of (D 2 − 4D + 4)y = x 2 is
1 3
(c) (5 cos 3x − sin 3x) (a) x 2 + 2x (b) x 2 + 2x +
78 2
1 1 2 3 1 2 3
(d) (5 cos 3x + sin 3x) (c)  x + 2x +  (d)  x + 2x + 
78 4 2 3 2

3. The complementary function of (D 4 − D 2) y = 2 is 8. The particular integral of (D 2 − 6D + 9)y = 6e 3x is

(a) C1 + C2 x + C3x 2 (b) C1 + C2x + C3e x (a) 3e 3x (b) 6 e 3x (c) 3x e 3x (d) 3x 2 e 3x

(c) (C1 + C2x)x 2 + C3 (d) C1 + C2x + C3x 3 9. The particular integral of (D 3 + 1) y = e 2x is

4. The particular integral of y ′′′ + 3y ′′ + 3y ′ + y = e − x ex e 2x e− x e −2x


(a) (b) (c) (d)
3 9 9 3
is
14

10. The particular integral of (D 2 − 5D + 6)y = cos 3x 19. The particular integral of (D 2 + 9) y = cos 2x
is + sin 2x is
1 1 1
(a) (cos 3x − sin 3x) (a) (cos 2x + sin 2x) (b) cos (2x + sin 2x)
78 5 3
1 1
(b) (3 cos 3x − sin 3x) (c) (cos 2x + sin 2x) (d) None of these
78 2
1
(c) (−5 sin 3x − cos 3x) 1
78 20. The value of sin 4x is
1 D2
(d) (5 cos 3x + sin 3x) 1 1
78 (a) − sin 4x (b) − sin 4x
1 4 16
11. e ax . V is 1 1
f (D) (c) − cos 4x (d) − cos 4x
1 1 16 4
(a) e ax V (b) e ax V
f (D) f (D + a) 1
1 1 21. The value of sin 3 x is
(c) e ax V (d) e ax V V D2 + 3
f (D − a) f (D + a)
x x
1 (a) − cos 3x (b) cos 3x
12. sin 2x is equal to 2 3 2 3
D2 + 4
x x
x cos 2x x cos 2x (c) sin 3x (d) − sin 3x
(a) (b) − 2 3 2 3
2 2
x cos 2x x cos 2x dy
(c) (d) − 22. + Py = Q is a linear differential equation of
4 4 dx
13. The solution of (D 2 + 1)y = 0 with y(0) = 0, y ′(0) = 0 degree first order if
is (a) P, Q are functions of x only
(a) y = cos x (b) y = sin x (b) P, Q are functions of y only
(c) y = 0 (d) Infinite solution
(c) P and Q are functions of x and y
14. The solution of ordinary differential equation of n
order contains (d) None of these
(a) n-arbitrary constants dy
23. The differential equation + p (x) y = q (x) is
(b) More than n-arbitrary constants dx
(c) Less then n-arbitrary constants
homogeneous when
(d) No arbitrary constants
(a) p(x) = 0 (b) q(x) ≠ 0
15. The equation ydx + xdy = 0 is (c) p(x) ≠ 0 (d) q(x) = 0
(a) Exact (b) Not-exact
24. If x hy k is an integrating function of
(c) Partial differential equation 7 2 8
(x y + 3y)dx + (3x y − x)dy = 0 then
(d) Differential equation of order 2
(a) h = − 7, k = 1 (b) h = 7, k = 1
16. The differential equation derived from y = Ae x (c) h = − 7, h = − 1 (d) h = 7, k = − 1
+ Be − x have the order
25. A differential equation of first order and first degree
(a) 1 (b) 2 (c) 3 (d) Infinite is homogeneous if
y
d 2y
dy (a) y ′ = φ   (b) y ′ = φ( x )
17. The solution of −2 + y = 0 is x
2 dx
dx (c) y ′ = constant (d) None of these
(a) C1e x + C2e − x (b) (C1 + C2x) e x dy 1
26. The differential equation − ⋅ y = 2x with
(c) (C1 + C2)e x (d) C1 + C2xe x dx x
y(0) = 0 has
18. The particular integral of y ′′ − 2y ′ + y = x 2e x is
(a) Unique solution (b) Finite solution
x 4 ex x 2e 2x x 4 ex x 2e 2x (c) No solution (d) Infinite solutions
(a) (b) (c) (d)
4 12 12 4
15

27. The solution of (D 4 + D 2)y = 0 with 36. The particular integral of (D 4 + 4)y = x 4 is
y (0) = y ′(0) = y ′′(0) = 0, y ′′′(0) = 1 is x4 − x2 x4 + 4
(a) (b)
4 4
(a) y = sin x (b) y = x + sin x
x4 − 6 x4 − x2 − 6
(c) y = x − cos x (d) y = x − sin x (c) (d)
4 4
28. The particular integral of (D 2 + 2D + 2)y = sin hx is
37. Particular integral of (D 2 + 1)y = x 2 is
x −x x −x
e − 5e e + e (a) x 2 + 2 (b) x 2 − 2
(a) (b)
10 10
(c) x 2 + 1 (d) x 2 − 1
5e x − e − x
(c) (d) None of these
10 38. The solution of (D 3 + 2D 2 + D)y = 0 is
1 (a) y = C1 + (C2 + C3x)e x
29. sin ax is equal to
D 2 + a2
(b) y = C1 + (C2 + C3x)e − x
x x
(a) sin ax (b) − sin ax
2a 2a (c) y = (C1 + C2x + C3x 2)e x
x x (d) y = C1x + C2x 2 + C3e x
(c) cos ax (d) − cos ax
2a 2a
1
30. The differential equation of the solution 39. sec 2x is equal to
D − 2i
y = A cos (ωx − θ) is
(a) ∫ e −2 ix sec 2x dx (b) e 2 ix ∫ sec 2x dx
d 2y d 2y
(a) =y (b) =0
dx 2 dx 2 (c) e −2 ix ∫ e 2 ix sec 2x dx (d) e 2 ix ∫ e −2 ix sec 2xdx
d 2y
(c) = − ω 2y (d) None of these 40. Which one of the following is not an integrating
dx 2
function for xy ′ = y
31. The order and degree of e y′′′ − y ′′ + xy = 0 are 1 1 1 1
(a) (b) (c) (d)
(a) 3, 1 (b) 3, infinite x+ y xy x2 y2
(c) 3, not defined (d) 3, finite
41. The solution of (D 3 + 6D 2 + 12D + 8)y = 0 with
1
32. sin 2x is equal to y(0) = y ′(0) = 0, y ′′(0) = 2 is
D 2 − 2D + 2
(a) x 2e 2x + x + 1 (b) x 2e −2x − 2x
1 1
(a) sin 2x (b) cos 2 x
2 2 2 2 (c) x 2e −2x (d) x 2e 2x
1 1 dy
(c) sin 2x (d) cos 2x 42. Integrating factor of cos 2 x + y = tan x is
2 2 dx
1 (a) e sin x (b) e sec x (c) e cosec x (d) e tan x
33. If φ( x ) be any function then φ(x) is equal to
D−a
43. If a equation contains x-arbitrary constants then the
(a) e − ax ax
∫ e φ(x) dx (b) e ax ∫ φ(x) dx
order of differential equation derived is
− ax
(c) e ∫ φ(x) dx (d) e ax ∫ e − ax φ(x)dx (a) n − 1 (b) n (c) n + 1 (d) 1

34. The homogeneous differential equation 1 e− x


44. ⋅ is equal to
2
Mdx + Ndy = 0 can be reduced to a differential D + 2D + 1 x2
equation in which the variables are separated by
(a) e x log x + x 2 + 1 (b) − e − x log x
the substitution
(a) y = vx (b) x + y = v (c) − e − x + log x + 1 (d) e − x − log x − 1
(c) xy = v (d) x − y = v 45. The solution of the differential equation
35. The solution obtained from complete primitive by dy
(x − y − 1) = 1 is
giving particular values to arbitrary constants is dx
(a) Complete solution (b) Singular solution (a) x − y − z = Ce y (b) y − z − x = Ce z
(c) Particular integral (d) Non-singular solution (c) z − x − y = Ce x (d) None of these
16

46. The solution of y ′′ + 2y ′ + y = 0, y(0) = 1, y′(0) = − 1 55. The particular integral of (D 2 + 1)y = x 2 is
is
(a) 1 + x 2 (b) x 2 = 2 (c) − x 2 + 2 (d) x − 2
(a) xe x (b) − x e − x (c) e − x (d) − e − x
56. The P.I. of (D − a)2 y = e ax f ′′(x) is
47. The differential equation whose auxiliary equation
(a) e − ax f (x) (b) f ′(x)
has the roots 0, 1, 1 is
(c) e ax f (x) (d) e ax f ′(x)
(a) (D 3 − 2D 2 + D) y = 0
57. The particular integral of (D 2 − 2 D + 4)y
(b) (D 3 + 2D 2 + D) y = 0
= e x cos x is
(c) (D 3 − D 2 + 2D) y = 0 1 1 x
(a) e x cos x (b) e sin x
(d) (D 3 + D 2 + 2D)y = 0 2 2
(c) e x cos x (d) e x sin x
48. The particular integral of (D 2 − 1) y = cos x is
1 1 58. e − x (C1 cos 3x + C2 sin 3x) + C3e 2x is the
(a) cos x (b) − cos x
2 2 general solution of
1
(c) − sin x (d) None of these (a) (D 3 − 4) y = 0 (b) (D 3 − 8) y = 0
2
(c) (D 3 + 8)y = 0 (d) (D 3 + 4) y = 0
d 2y 1 dy 12 log x
49. The particular integral of + = is
dx 2 x dx x2 59. (C1 + C2x) + (C3 + C4 x)e − x is the solution of
(a) 2 (log x) (b) 2 (log x)2 (a) (D 4 + D 3 + D 2)y = 0
(c) 2 (log x)3 (d) (log x)3 (b) (D 4 + D 2) y = 0
50. The solution of the differential equation (c) (D 4 + 2D 3 + D 2)y = 0
(D 2 + 1) y = 0 with y(0) = 1, y ( π / 2) = 2 is (d) (D 4 − D 2) y = 0
(a) y = 2 cos x + sin x (b) y = cos x + 2 sin x
60. The P.I. of (D 3 + 1) y = (e x + 1)2 is
(c) y = cos x + sin x (d) y = 2 cos x + 2 sin x
e 2x ex
1 (a) e 2x + e x + 1 (b) + +1
51. x 2 sin ax is equal to 9 2
f (D)
e 2x
(a) e i ax
1
x2 (c) e 2x − 2e x + 1 (d) − ex + 1
f (D + i a) 2

(b) Real part of e i ax


1
x2 61. The P.I. of (D 2 + D − 6)y = x is
f (D + ia) 6x + 1 6x + 1
(a) (b) −
1 12 18
(c) e − iax x2
f (D − ia) 6x + 1 6x + 1
(c) (d) −
1 18 36
(d) Imaginary part of e iax x2
f (D + ia) 62. The P.I. of (D 2 − 4D + 3)y = e 2x sin 3x is
52. The solution of y ′′ + a 2y = 0 with y(0) = 0, y ′(0) = a is e 2x sin 3x e 2x sin 3x
(a) − (b) −
(a) sin ax (b) cos ax (c) sec ax (d) cosec ax 10 6
53. The solution of y ′′ + 2 y ′ + 2y = 0, with e 2x sin 3x
(c) (d) None of these
y(0) = 0, y′ (0) = 1 is 8

(a) e x sin x (b) e − x sin x 63. The differential equation derived from
x −x x −2x 3x
(c) e cos x (d) e cos x y = Ae + Be + Ce have the order
1 (a) 1 (b) 2 (c) 3 (d) 4
54. x is equal to
D2 − 2 D + 2 64. The differential equation derived from
x +1 x x −1 x x
y = Ae + Be −x
have the degree
(a) (b) (c) (d) −
2 2 2 2
(a) 1 (b) 2 (c) 3 (d) 0
17
6x − 5 2x − 5
65. The particular integral of (D − 2)2 y = e 2x f ′′′(x) is (c) (d)
36 18
(a) e 2x f (x) (b) − e 2x f ′(x)
69. The particular integral of (D 3 + 8)y = x 4 + 2x + 1 is
2x
(c) e f ′′(x) (d) e 2x f ′(x) x3 + x − 1 x4 − x + 1
(a) (b)
1 8 8
66. x 3 cos x is equal to
f (D) x5 + x4 − 1 x 4 − 2x + 3
(c) (d)
1 8 8
(a) e ix x3
f (D + i) d 2y
1 70. The solution of = a + bx + cx 2 with y ′ = 0
(b) Real part of e ix x3 dx 2
f (D + i)
when x = 0 and y = d when x = 0 is
1
(c) Real part of e ix x3 bx 2 cx 3 d
f (D − i) (a) y = ax + + +
2 3 4
1
(d) e ix x 3 cos x ax 2 bx 3 cx 4
f (D + i) (b) y = + + +d
2 6 12
1
67. xe x sin x is equal to ax 2 bx 3 cx 4
D2 − 2D + 1 (c) y = + + + d
2 3 4
(a) e x (x cos x + sin x) bx cx 3 dx 4
−x (d) y = a + + +
(b) − e (x cos x + sin x) 2 4 8
x
(c) e (x sin x + 2 cos x) d 2y
71. The solution of − y = 1, y(0) = 0, y (−α) → finite
(d) − e x (x sin x + 2 cos x) dx 2
1 is
68. x is equal to
D 2 − 5D + 6 (a) e x (b) e x + 1 (c) e x − 1 (d) − e x − 1
6x + 5 2x + 5
(a) (b)
36 18

ANSWERS
MULTIPLE CHOICE QUESTIONS
1. (b) 2. (c) 3. (b) 4. (c) 5. (d) 6. (d) 7. (c) 8. (d) 9. (b) 10. (c)
11. (b) 12. (d) 13. (c) 14. (a) 15. (a) 16. (b) 17. (b) 18. (c) 19. (a) 20. (b)
21. (a) 22. (a) 23. (d) 24. (a) 25. (a) 26. (d) 27. (d) 28. (a) 29. (d) 30. (c)
31. (c) 32. (b) 33. (d) 34. (a) 35. (c) 36. (c) 37. (b) 38. (a) 39. (d) 40. (a)
41. (c) 42. (d) 43. (b) 44. (b) 45. (a) 46. (c) 47. (a) 48. (b) 49. (c) 50. (b)
51. (d) 52. (a) 53. (b) 54. (a) 55. (b) 56. (c) 57. (a) 58. (b) 59. (c) 60. (b)
61. (d) 62. (a) 63. (c) 64. (a) 65. (d) 66. (b) 67. (d) 68. (a) 69. (b) 70. (b)
71. (c)

HINTS AND SOLUTIONS


1. Given (D 2 + 4)y = 0 A.E. is m 2 + 4 = 0 i.e. m = ± 2i
1
So solution is y = C1 cos 2x + C2 sin 2x Its particular integral is sin 3x
2
D2 − 5 D + 6
2. Given differential equation is (D − 5D + 6)y = sin 3x.
1 1
= sin 3x = sin 3x
−9 − 5 D + 6 −5 D − 3
18

=−
(5D − 3)
sin 3x i.e. (m − 1)2 = 0 ⇒ m = 1, 1
(5D + 3) (5D − 3)
So solution is
(5D − 3)
=− sin 3x
25D 2 − 9 y = (C1 + C2x)e x
1 1
=− (15 cos 3x − 3 sin 3x) 18. The particular integral is ex x 2
−225 − 9 D 2 − 2D + 1
1
= (15 cos 3x − 3 sin 3x) 1 1 ex x 4
234 = ex x 2 = ex x2=
2 2 12
1 (D + 1) − 2(D + 1) + 1 D
= (5 cos 3x − sin 3x)
78 24. If x hy k be the I.F. of (x 7 y 2 + 3y)dx
1
4. e− x
D 3 + 3D 2 + 3 D + 1 + (3x 8y − x)dy = 0

= e− x
1
.1 then (x h+7 y k + 2 + 3x hy k +1)dx
3 2
(D − 1) + 3 (D − 1) + 3(D − 1) + 1 + (3x h+ 8y k +1 − x h+1y k ) dy = 0
−x
e ∂M ∂N
= is exact i.e. =
D 3 − 1 − 3D (D − 1) + 3D 2 + 3 − 6D + 3D − 2 ∂y ∂x
1 x 3e − x 1 3 − x
= − e− x ⋅ .1 = = x e i.e., (k + 3)x h + 7 y k + 1 + 3(k + 1)x hy k
D3 3× 2 6
= 3 (h + 8)x h + 7 y k + 1 − (h + 1) x h y k
1 1 1
6. cos 4x = cos 4x = − cos 4x
2
D + 9 − 16 + 9 7 Equating the coefficient, we get

−1 k + 3 = 3h + 24 & 3k + 3 = − h − 1
1 1 2  D2  2
7. x =  1 − D +  ⋅x After solving, we get h = − 7, k = 1
2
D − 4D + 4 4  4 
27. Given
1 D2 (−2) ⋅ D 2  2 (D 4 + D 2) y = 0, y(0) = y ′(0) = y ′′(0), y ′′′(0) = 1
= 1 + D − + (−1) x
4  4 2 
A.E. is m 4 + m 2 = 0 i.e. m = 0, 0, + i, − i
Neglecting higher terms
so its solution is y = C1 + C2x + C3 cos x + C4 sin x
1 3 1 3
= 1 + D + D 2  x 2 =  x 2 + 2x + 
4  4  4  2  so y ′ = C2 − C3 sin x + C4 cos x

1 1 1 y ′′ = − C3 cos x − C4 sin x,
9. e 2x = e 2x = e 2x
D3 + 1 8+ 1 9 y ′′′ = C3 sin x − C4 cos x
y(0) = 0 ⇒ C1 + C3 = 0
13. Given that (D 2 + 1) y = 0, y(0) = 0 = y ′(0)
y ′(0) = 0 ⇒ C2 + C4 = 0
A.E. is m 2 + 1 = 0 i.e., m = ± i so its solution is y ′′(0) = 0 ⇒ − C3 = 0 ⇒ C3 = 0 so C1 = 0
y = C1 cos x + C2 sin x, y ′ = − C1 sin x + C2 cos x y ′′′(0) = 1 ⇒ 1 = − C4 ⇒ C4 = − 1∴C2 = 1
If y(0) = 0 ⇒ 0 = C1 So the required solution is
If y ′(0) = 0 ⇒ 0 = C2 so y = 0 y = x − sin x

16. Given y = Ae x + Be − x , y ′ = Ae − x − Be − x 1 1  ex − e− x 
28. sin hx =  
2 2 2
D + 2D + 2 D + 2D + 2  
Adding we get y ′ + y = 2 Ae x
1 1 1 1
Again differentiate y ′′ + y ′ = 2 Ae x = ex − e− x
2 (D 2 + 2D + 2) 2 (D 2 + 2D + 2)
Subtracting we get y ′′ − y = 0 so its order is 2. 1 1 x 1 1 − x e x − 5e − x
= . e − . e =
2 2 2 5 2 1 10
17. Given (D − 2D + 1)y = 0 A.E. is m − 2m + 1 = 0
19

dy So its solution is xe − y = − ∫ e − y (y + 1)dy + C


30. y = A cos (ωx − θ), = − Aw sin (ωx − θ)
dx
d 2y xe − y = + (y + 1) e − y − ∫ e − y dy + C
= − Aω2 cos (ωx − θ)
dx 2 = (y + 1) e − y + e − y + C
2
d y
so differential equation is = − ω 2y xe − y = (y + 2) e − y + C ⇒ (x − y − z) e − y = C
dx 2
i.e., x − y − z = Ce y
1 1
32. sin 2x= sin 2x
D 2 − 2D + 2 −2 − 2 D + 2 d 2y 1 dy 12 log x
49. Given that + =
1 dx 2 x dx x2
=− sin 2x
2D d 2y dy
D 1 i.e. x2 + x = 12 log x
=− sin 2x = − 2 cos 2 x dx 2 dx
2D 2 2(−2)
1 put x = e 2 we get [D (D − 1) + D] y = 12z, where
= cos 2x
2 2 d
Z = log x & D ≡
−1 dz
1 1  D4 
36. x4 =  1 +  x4 Thus, we have D 2y = 12z so P.I. is
1
12z
D + 4 4 4  4 
D2
1  D4 
12 . z 3
=  1 − + … x 4 = = 2z 3 = 2(log x)3
4  4  6
1  4 24  1 4 1
= x −  = (x − 6) 55. Given P.I. is x 2 = (1 + D 2)−1 ⋅ x 2
4 4 4 D2 + 1
41. Given that (D 3 + 6D 2 + 12D + 8) y = 0,
= (1 − D 2 + …) x 2 = x 2 − 2
y(0) = y ′ (0) = 0, y ′′(0) = 2
1 1
A.E. is m 3 + 6m 2 + 12m + 8 = 0 56. e ax f ′′(x) = e ax f ′′(x)
(D − a)2 (D + a − a)2
m 2(m + 2) + 4m (m + 2) + 4 (m + 2) = 0
1
= e ax f ′′(x) = e ax f (x)
⇒ (m 2 + 4m + 4) (m + 2) = 0 D2
−4 ± 16 − 16 1
m = − 2, = − 2, − 2, − 2 57. e x cos x
2
2 (D − 2D + 4)
So solution is y = (C1 + C2x + C3x 2) e −2x 1
= ex cos x
y ′ = − 2C1 e −2x + C2 (e −2x − 2x e −2x (D + 1)2 − 2 (D + 1) + 4

+ C3 (2x e −2x − 2x 2 e −2x ) 1 1 e x cos x


= ex cos x = e x cos x =
2 −1 + 3 2
y ′′ = 4C1 e −2x + C2 (−4e −2x + 4xe −2x ) D + 3

+ C3 (−8xe −2x + 4x 2e −2x + 2e −2x ) 58. Consider (D 3 − 8) y = 0, A.E. is m 3 − 8 = 0

y (0) = 0 ⇒ 0 = C1 ⇒ C1 = 0 2± 4 − 16
(m − 2) (m 2 + 4 − 2m) i.e. m = 2,
y′ (0) = 0 ⇒ 0 = − 2C1 + C2 ⇒ C2 = 0 2

y ′′ (0) = 2 ⇒ 2 = 4C1 − 4C2 + 2C3 ⇒ C3 = 1 2 ± 2 3i


i.e. m = 2, = 2, 1 ± 3i
2 −2x 2
So required solution is x e .
dy dx So solution is e − x (C1 cos 3x
45. Given that (x − y − 1) = 1⇒ = x − y −1
dx dy + C2 sin 3x) + C3e 2x
dx −1 dy 1 1
i.e. − x = − (y + 1) I.F. is e ∫ = e− y 60. (e x + 1)2 = (e 2x + e x + 1)
dy (D 3 + 1) (D 3 + 1)
20
−1
1 1 1  5D − D 2 
= e 2x + ex + −1 68.
1
x=
1
 1 −  x
3 3 3
D +1 D +1 D +1 D2 − 5 D + 6 6  6 
e 2x ex e 2x ex 1  5 D − D2  1 5 6x + 5
= + + (1 + D 3)−1 − 1 = + +1 =  1 − + … =  x +  =
9 2 9 2 6 6  6 6  36
 
1
62. e 2x sin 3x 69. Particular integral is
1
x 4 + 2x + 1
2
D − 4D + 3 D3 + 8
1 −1
= e 2x sin 3x 1 D3  4
(D + 2)2 − 4 (D + 2) + 3 = 1 +  (x + 2x + 1)
8 8 
1 1
= e 2x sin 3x = e 2x sin 3x 1 D3 
D2 − 1 −9 − 1 =  1 − + ... (x 4 + 2x + 1)
8 8 
e 2x sin 3x 1 4 1
=− = (x + 2x + 1) − D 3 (x 4 + 2x + 1)
10 8 8
y = Ae x + B e −2x differentiating it 1 4 1
63. = (x + 2x + 1 − 3x) = (x 4 − x + 1)
8 8
dy
= Ae x − 2Be −2x 2
d y
dx 70. Given = a + bx + cx 2, y ′(0) = 0, y(0) = d
dy dx 2
Subtracting − y = − 3 Be −2x …(1)
dx dy bx 2 cx 3
Integrating = ax + + + e
dx 2 3
d 2y dy
Again differentiating − = 6Be −2x …(2) Again integrating
2 dx
dx
ax 2 bx 3 cx 4
y= + + + ex + f …(1)
Multiply equation (1) by 2 and add with equation 2 6 12
(2), we get
y (0) = d ⇒ f = d
d 2y dy y′(0) = 0 ⇒ e = 0
+ − 2y = 0 so its degree is 1
dx 2 dx put these values in equation (1), we get
1 2x 2x 1
65. e f ′′′(x) = e f ′′′(x) ax 2 bx 3 cx 4
(D − 2)2 (D + 2 − 2)2 y= + + + d
2 6 12
e 2x 71. Given the (D 2 − 1) y = 1, y(0) = 0, y(−α) → finite
= f ′′′(x) = e 2x ⋅ f ′(x)
2
D
A.E. is m 4 − 1 = 0 ⇒ m = ± 1
ix
66. ∵ cos x = Real part of e so
C.F. is C1e x + C2e − x
1 1
x 3 cos x = Real part of e ix x3 1
f (D) f (D + i) P.I. 1 = − (1 − D 2)−1 .1
D2 − 1
1 1
67. x e x sin x = (e x ⋅ x sin x) = − (1 + D 2 + …). 1 = − 1
D2 − 2 D + 1 (D − 1)2
So, solution is y = C1e x + C2 e − x − 1 …(1)
x 1 x 1
=e x sin x = e x sin x
(D + 1 − 1)2 D2 y(0) = 0 ⇒ 0 = C1 + C2 − 1 ⇒ C1 + C2 = 1
 1 y(−α) = finite ⇒ C2 = 0 so C1 = 1
1  1 
= ex x sin x − 2D  sin x  
2 2   2  put these values in equation (1) we get
 D D  D 
2 y = ex − 1
x  
= e − x sin x − sin x 
 D3  mmm
= − e x (x sin x + 2 cos x)
Unit21-I
C HAPTER
3 Homogeneous Linear Differential Equations
DEFINITIONS Case I. If m1, m2 , … mn are distinct roots of the
1. A differential equation of the form auxiliary equation of f (D)y = 0 then C.F. of
n
d y d n− 1
y dy f (D) y = Q is
xn + a1 + … + a n− 1x + a ny = P
dx n
dx n− 1 dx y = C 1e m1 z + C 2 e m2 z + …… C ne mn ⋅ z

where a 1, a 2 … a n are constants and P is either or y = C 1 x m1 + C 2 x m2 + …… C n x mn


a constant or a function of x is called a
homogeneous linear differential equation. Case II. If r roots are equal say m and others
2. To solve homogeneous linear differential (n − r ) are different then C.F. is
mr + 1 z
equation, we put x = e z or log x = z such that (C 1 + C 2 z + …… C r z r − 1) e mz + C r + 1e
1 dz
= + … + C ne mn z
x dx
Then
dy dy dz 1 dy
= ⋅ = i.e. [C 1 + C 2 log x + … + C r (log x )r − 1] x m
dx dz dx x dz mr + 1
+ Cr + 1 x + … C nx mn
dy dy d
So x = = Dy where D ≡
dx dz dz Case III. If the roots are imaginary i.e. α ± iβ
d 2y d  dy  d  1 dy 
Again =   =   then
dx 2 dx  dx  dx  x dz 
C.F. is e αx (C 1 cos βz + C 2 sin βz )
1 d  dy  dy  1 
=  + − 
x dx  dz  dz  x 2  = x α [C 1 cos (β log x ) + C 2 sin (β log x )] or we

1 d 2 y dz 1 dy 1 d 2y 1 dy can write C 1e αz cos (βz + C 2 )


= ⋅ − = −
x dz dx x dz x 2 dz 2 x 2 dz
2 2
i.e. C 1x α cos (β log x + C 2 )
d 2y d 2y dy
or x2 = − = D(D − 1)y Case IV. If α ± iβ occur r times then C.F. is
2 2 dz
dx dz
e αz [(C 1 + C 2 z + … C r z r − 1)
3
d y cos βz + (d 1 + d 2 z + … d rz r − 1) sin βz ]
Similarly, x 3 = D(D − 1) (D − 2)y and so
dx 3
i.e. x α [{ C 1 + C 2 log x + …
on. Thus, the new transformed equation is
d + C r (log x )r− 1} cos (β log x ) +
f (D)y = Q, where D ≡ and Q is a function of
d2 { d 1 + d 2 log x + … + dr (log x )r − 1}
z. The general solution of this equation is the sin (β log x)]
sum of particular solution of this equation and 4. Now we discuss the cases to find the particular
general solution of f (D) y = 0. integral (P.I.) of f (D)y = Q
3. Now we discuss all the cases to find the 1 1 1 az
Case I. Q= e az = e , provided
complementary function (C.F.) of, f (D)y = 0. f (D) f (D) f (a)
f (a) ≠ 0
22

1 1 where V is a function of z.
Case II. cos az = cos az
2 2 1 1  d 1 
f (D ) f (− a )
Case V. (zV) = z V+ ⋅ V
1 1 f (D) f (D)  dD f (D)
and sin az = sin az , provided
f (D 2 ) f (− a 2 ) 5. If the differential equation is
f (− a 2 ) ≠ 0 d ny d n− 1y
(a + bx )n + a 1 (a + bx )n − 1. + ....
1 dx n dx n− 1
Case III. z m = [ f (D)]−1 z m and then
f (D) a ny = X then it can be reduced to linear
expend
differential equation with constant coefficient
[ f (D)]−1 in ascending powers of D.
by substitute a + bx = e z .
1 1
Case IV. e az V = e az V
f (D) f (D + a)

EXERCISE
MULTIPLE CHOICE QUESTIONS d 2y d 2y
(a) − y = ez (b) + y = ez
2 2 2
1. The complementary function of (x D + 3xD + 1) y dz dz 2
1 d 2y dy d 2y dy
= is (c) + + y = ez (d) + = ez
(1 − x)2 dz 2 dz dz 2 dz
(a) C1 + C2 log x (b) C1x + C2x ⋅ log x 7. The particular integral of (x 2D 2 + xD + 1)y
1 C = sin (log x 2) is
(c) (C1 + C2 log x) (d) 1 + C2x log x
x x 1 1
(a) cos (log x 2) (b) − cos (log x 2)
2. The complementary function of 3 3
1 1
(x 2D 2 − 4xD + 6) y = x 2 is (c) sin (log x 2) (d) − sin (log x 2)
3 3
(a) C1x + C2x 2 (b) C1 + C2x
8. The P.I. of (x 2D 2 + xD + 1)y = x is
3
(c) C1x + C2x (d) C1x + C2 x x x
(a) x (b) (c) (d) x 2 +
3. For solving the homogeneous linear equations we 3 2 3
d d 2y 9. The complementary function of (x 2D 2 − 3xD + 4)y
substitute x = e z and D ≡ than x 2 is
= 2x 2 is
dz dx 2
(a) D 2y (b) (D − 1)y (a) (C1 + C2) x 2 (b) (C1 + C2 log x) x 2
2
(c) D(D + 1)y (d) D(D − 1)y (c) C1x + C2x log x (d) C1 + C2x log x

4. The solution of (x 2D 2 + xD + 1)y = 0 is 10. The particular integral of (x 2D 2 + xD − 1)y = x 2e x


(a) C1 cos (log x) + C2 sin (log x) is
(a) xe x (b) (1 − x) e x
(b) C1 cos x + C2 sin x
1 1
(c) (C1 + C2x) cos (log x) (c)  1 −  e x (d)  x −  e x
 x  x
(d) (C2 + C2x) ⋅ sin (log x)
11. If we substitute x = e z in (x 2D 2 + 3xD + 1) y = 0
5. The solution of (x 2D 2 − xD) y = 0 is
then the transformed differential equation is
(a) C1x + C2x 2 (b) C1 + C2x (a) (D ′2 + 1)y = 0 (b) (D ′2 + 2)y = 0
(c) C1 + C2x 2 (d) C1x + C2x 3 (c) (D ′ + 1)2 y = 0 (d) (D ′ + 2)2 y = 0
d 2y dy 12. The P.I. of (x + a)2 D 2y − 4(x + a)Dy + 6y = x is
6. On putting x = e z , x 2 + x + y = x is
2 dx
dx x a x a
(a) x + a (b) + a (c) x + (d) +
transformed to 2 3 2 3
23

13. If we substitute 5 + 2x = e z in the differential 1


21. e x φ(x) is equal to
d 2y dy f (D)
equation (5 + 2x)2 − 6(5 + 2x) + 8y = 0
dx 2 dx 1 1
d (a) e x φ(x) (b) e x φ(x)
and D ≡ then it transform to f (D + 1) f (D)
dz
1 1
(a) (2D 2 − 3D + 1)y = 0 (b) (D 2 − 4D + 2)y = 0 (c) e − x φ(x) (d) e x φ(x)
f (D) f (D − 1)
(c) (2D 2 + 3D + 1)y = 0 (d) (2D 2 + 4D + 2)y = 0
22. The nth order linear differential homogeneous
14. To transform (x 2D 2 + xD + 1)y = 0 into the linear equation have
differential equation with constant coefficients, we (a) Finite singular solution
put
(b) n-singular solution
(a) x = log z (b) x = e z
(c) No singular solution
(c) x = z 2 (d) x = e 2z
(d) Unique singular solution
15. The P.I. of (x 3D 3 + 3x 2D 2 + xD + 1)y = x is
42
23. The particular integral of (x 2D 2 − 4xD + 6)y =
(a) x 2 + 2x (b) x 2 − x + 1 x4
x
(c) (d) x 2 + x is
2 1 1 1 1
(a) (b) (c) (d)
16. The P.I. of (x 2D 2 − xD − 3)y = x 2 log x is x x3 x4 x5
x2
(a) x 2 (3 log x + 2) (b) − (3 log x + 2) 24. If we substitute x = e z in (x 2D 2 + 4xD + 2)y = 0
3
x2 then the transformed differential equation is
(c) − (3 log x + 2) (d) − x (3 log x + 2)
9 (a) (D ′2 + 3D ′ + 2)y = 0 (b) (D ′2 − 3D ′ + 2)y = 0
z d
17. If we substitute x =e and D≡ in
dz (c) (D ′2 − 3D ′ + 4)y = 0 (d) (D ′2 + 3D ′ + 4)y = 0
d 3y d 2y
dy
x3 + 3x 2+ x + y=0 then 25. The particular integral of (x 2D 2 − 4xD + 6)y = x 4 is
3
dx dx 2 dx
transformed equation is (a) x 4 + x 2 − 1 (b) x 2 − x
(a) (D 3 + 3D 2 + D + 1)y = 0 x+1 x4
(c) (d)
(b) (D 3 + D + 1)y = 0 12 2
26. The P.I. of (x 3D 3 + 2x 2D 2 + 2)y = 10 x is
(c) (D 3 + 4)y = 0
(d) (D 3 + 1)y = 0 (a) 10x (b) 20x (c) 5x (d) 15x
2 2
27. The P.I. of (x D − xD + 2)y = x log x is
18. The particular integral of (x 2D 2 + 1)y = 3x 2 is
2 log x − 3  2 log x + 3 
(a) x (b) x 2 (c) 3x 2 (d) 2x 2 (a)  x (b)  x
 4   4 
19. The solution of (x 2 D 2 − 4xD + 6)y = 0 is
2 log x − 3
(c) (d) x log x
(a) C1x + C2x 2 (b) C1x 2 + C2x 3 4
(c) C1 + C2x (d) C1x + C2x 4
d d 3y
28. If D stands for and x = e z then x 3 is
2 2
20. The particular integral of (x D − 3xD + 5)y dz dx 3

= x 2 sin (log x) is (a) D(D − 1) (D − 2)y


2 (b) D(D − 1)(D − 1)(D − 3)y
x
(a) − log x cos (log x)
2 (c) D 3(D − 1)y
(b) − sin x (log x)
(d) D(D − 1)y
1
(c) − log x
x 29. The particular integral of (x 3D 3 + 2x 2D 2
1 + 3xD − 3)y = x 2 is
(d) − sin x (log x)
x
24

x2 + 1 x2 (a) sin log (1 + x)


(a) (b)
5 7 (b) 2 log (1 + x) sin log (1 + x)
(c) log (1 + x) sin log (1 + x)
x 3 + 7x + 3 x2 − 3
(c) (d) (d) log (1 + x) cos log (1 + x)
7 7
1 d 32. By substitution y = z 2, the homogeneous equation
30. Q is equal to, where D stands for
D−a dz of
2
(a) e − az ∫ e az Qdz (b) ∫ e − az Qdz d 2y dy dy
2x 2 y + 4y 2 − x 2   − 2xy  = 0 is
2  dx   dx 
dx
(c) ∫ e az Q (d) e az ∫ e − az Qdz (a) (x 2D 2 − xD + 1) z = 0

31. The particular integral of (b) (x 2D 2 + 1)z = 0

d 2y dy (c) (x 2D 2 − 2xD − 1) z = 0
(1 + x)2 + (1 + x) + y = 2 cos log (1 + x) is
dx 2 dx (d) (x 2D 2 − 2) z = 0

ANSWERS
MULTIPLE CHOICE QUESTIONS
1. (c) 2. (c) 3. (d) 4. (a) 5. (c) 6. (b) 7. (d) 8. (c) 9. (b) 10. (c)
11. (c) 12. (d) 13. (b) 14. (b) 15. (c) 16. (c) 17. (d) 18. (c) 19. (b) 20. (a)
21. (a) 22. (c) 23. (c) 24. (a) 25. (d) 26. (c) 27. (d) 28. (a) 29. (b) 30. (d)
31. (c) 32. (a)

HINTS AND SOLUTIONS


1. Given differential equation is (x 2D 2 + 3xD d 2y dy
6. Given x2 + x + y = x, put x = ez
1 d dx 2 dx
+ 1)y = put x = e z and D ≡ we get
2 dz
(1 − x) [D(D − 1) + D + 1] y = e z
1
[D(D − 1) + 3D + 1]y =
(1 − e z )2 ⇒ (D 2 + 1)y = e z

i.e. (D 2 + 2D + 1)y =
1 7. Given (x 2D 2 + xD + 1)y = sin (log x 2) put
z 2
(1 − e ) d
x = ez , D ′ = we get
A.E. is m 2 + 2m + 1 = 0 i. e. m = − 1, − 1 dz
1 [D ′(D ′ − 1) + D ′ + 1]y = sin (2z)
So, C.F. is (C1 + C2z)e − z = (C1 + C2 log x) ⋅
x i.e. (D ′2 + 1)y = sin 2z
4. Given that (x 2D 2 + xD + 1)y = 0, put so its P.I. is
z d 1 1 1
x = e , D′ ≡ sin 2z = + sin 2z = − sin 2z
dz (D ′2 + 1) −4 + 1 3
We get [D ′(D ′ − 1) + D ′ + 1]y = 0 i.e. (D ′2 + 1)y = 0 1 1 2
= − sin (2 log x) = − sin (log x )
3 3
A.E. is m 2 + 1 = 0 ⇒ m = i, − i 10. Given differential equation is
d
So solution is C1 cos z + C2 sin z = C1 cos (log x) (x 2D 2 + xD − 1)y = x 2e x put x = e z , D ′ ≡ , we
dz
+ C2 sin (log x) get
(D ′(D ′ − 1) + D ′ − 1)y = e 2z ee z
25

⇒ (D ′2 − 1)y = e 2z ee z = x 2e x 23. Given homogeneous equation is (x 2D 2 − 4xD


1 1  1
P.I. is x 2e x =  x 2e x  + 6)y =
4z
put x = e z and D ′ ≡
d
we get
D ′2 − 1 D′ + 1  D′ − 1 
x4 dz
1 2 x − 1−1 1 x
= (x x e x dx) = xe [D ′(D ′ − 1) − 4 D ′ + 6] y = 4ze −4 z
D′ + 1 ∫ D′ + 1
⇒ (D ′2 − 5D ′ + 6)y = 4ze −4 z
ex
= x −1 ∫ xe x x 1−1dx = x −1 (x − 1)e x = e x − 1
x P.I. is 4ze −4 z
2
1 D ′ − 5D ′ + 6
=  1 −  e x
 x 1
⇒ = 4ze −4 z .1
12. Given that (x + a)2 D 2y − 4 (x + a) Dy + 6y = x put (D ′ − 4)2 − 5(D ′ − 4) + 6
1
x + a = e z we get = 4z e − 4 z .1
D ′2 − 13D ′ + 4z
[D ′(D ′ − 1) − 4 (D ′) + 6] y = e z − a −1
4z −4 z  13D ′ D ′2 
= e 1 − +  .1
i.e. (D ′2 − 5 D ′ + 6)y = e z − a 4z 
 4z 4z 
1 1 4z −4 z  1 + 13 D ′ + … .1 = e −4 z = 1
So, P.I. is ez − a .1 = e  
(D ′2 − 5D ′+ 6) D ′2 − 5 D ′+ 6 4z  4z  x4

1 z a  5D ′ D ′2 
−1
25. Given (x 2D 2 − 4xD + 6)y = x 4 , put
= e − 1 − +  .1 d
2 6  6 6  x = ez , D ′ ≡

dz
ez a x + a a x a
= − = − = + We have [D ′(D ′ − 1) − 4 D ′ + 6] y = e 4 z
2 6 2 6 2 3

d 2y dy i.e. (D ′2 − 5D ′ + 6)y = e 4 z
13. Given that (2x + 5)2 − 6(2x + 5) + 8y = 0
2
dx dx 1 1 4z x 4
P.I. e4 z = e =
2 2 2
put 2x + 5 = e z we get [4D ′(D ′ − 1) − 12D ′ + 8]y= 0 (D ′ − 5D ′ + 6)

⇒ [(D ′2 − D ′) − 3 D ′ + 2]y = 0 27. Given differential equation is


i.e. (D ′2 − 4D ′ + 2)y = 0 (x 2D 2 − xD + 2)y = x log x

Differential equation is (x 2D 2 − xD − 3)y = x 2 log x d


16. put x = e z , D ′ =
dz
d
put x = e z , D ′ ≡ , we get
dz we get (D ′(D ′ − 1) − D ′ + 2)y = ze z
2z
[D ′(D ′− 1) − D ′ − 3]y = ze i. e., (D ′2 − 2D ′ + 2)y = ze z

⇒ (D ′2 − 2D ′ − 3)y = ze 2z P.I. is
1
ze z
2
D ′ − 2D ′ + 2
1
P.I. is ze 2z
2 1
(D ′ − 2D ′ − 3) = ez z
2
(D ′ + 1) − 2 (D ′ + 1) + 2
1
= e 2z z
(D ′ + 2)2 − 2 (D ′ + 2) − 3 1
= ez z = e z (1 + D ′2)−1 ⋅ 2 = ze z
2
−1
D′ + 1
e 2z e 2z  2D ′ D ′2  = x log x
= z2 = − 1 − −  z
D ′2 + 2D ′ − 3 3  3 3 

29. Given (x 3D 3 + 2 x 2D 2 + 3xD − 3)y = x 2
2z 2z
e  2D ′ e  2 d
=− 1 + + … z = − z +  put x = e z & D ′ ≡ , we get
3  3  3  3 dz

x2  2 x2 [D ′(D ′ − 1) (D ′ − 2) + 2D ′(D ′ − 1) + 3D ′ − 3] y = e 2z
=− log x +  = − (3 log x + 2)
3  3  9
or (D ′ − 1) (D ′2 + 3)y = e 2z
26

1 e 2z e 2z 32. Given differential equation is


So P.I. e 2z = =
2 (2 − 1) (4 + 3) 7 2
(D ′ − 1) (D ′ + 3) d 2y dy dy
2x 2y + 4y 2 − x 2   − 2xy =0
x2 dx 2  dx  dx
=
7 dy dz
Put y = z 2, = 2z ,
31. Given differential equation is dx dx
2
d 2y dy d 2y d 2z dz
(1 + x)2 + (1 + x) + y = 2 cos log (1 + x) = 2z + 2   , we get
dx 2 dx dx 2 dx 2  dx 
2 2
Put 1 + x = e z , we get  d 2z dz  dz
2x 2z 2  2z + 2    + 4z 4 − 4x 2z 2  
2  dx    dx 
[D (D − 1) + D + 1] y = 2 cos z  dx 
⇒ (D 2 + 1)y = 2 cos z − 2xz 2 ⋅ 2z
dz
=0
d dx
D≡
dz d 2z dz
or x2 −x + z=0
The particular integral is
1
2 cos z dx 2 dx
D2 + 1 or (x 2D 2 − xD + 1)z = 0
2z
= sin z = 2 sin z = log (1 + x) sin log (1 + x) mmm
2
Unit27
-II

C HAPTER
4
Differential Equation of First
Order but not of the First Degree
DEFINITIONS 4. For the equations solvable for x, let the
1. A differential equation of the form differential equation is
pn + A1 pn − 1 + A2 pn − 2 + … An− 1 p + An = 0 x = f (y, p) …(1)
is called a differential equation of the first and Differentiating (1) w.r.t. y we get
dy
n th degree, where p stands for and 1  dp
dx = φ  y, p,  …(2)
A1, A2 , A3 … An all are functions of x and y. p  dy 
2. For the equations solvable for p, let the Let the solution of (2) is
differential equation in above can be resolved F (y, p, c) = 0 …(3)
into n linear factors such that Eliminating p between (1) and (3) we get the
{p − f1(x, y)} {p − f2 (x, y)} … {p − fn (x, y)} = 0
required solution of (1) such that
Now we can equate each factor to zero and (x , y, c) = 0
than solve the resulting differential equations of
If it is not possible to eliminate p, we solve (1)
the first order and first degree i.e. the solution is
and (3) for x and y such that
φ 1(x , y, c1) = 0, φ 2 (x , y, c2 ) = 0
x = f 1( p, c), y = f 2 ( p, c) …(4)
… φ n(x , y, cn) = 0
Which is the required solution of (1).
Consider a unique constant C then n solutions
are φ 1(x , y, c) = 0, φ 2 (x , y, c) = 0 … φ n CLAIRAUT’S EQUATION
(x , y, c) = 0
1. The differential eqution of the form
Thus, the combined solution is
y = px + f ( p) is called Clairaut’s equation
φ 1 (x , y, c) φ 2 (x , y, c) … φ n (x , y, c) = 0 where f ( p) is a function of p only.
3. For the equations solvable for y let the Differentiating it w.r.t. x
differential equation is dp dp
p= p+ x + f ′( p)
y = f (x , p) …(1) dx dx
Differentiating (1) w.r.t. x we get dp
or { x + f ′( p)} = 0
 dp  dx
p = φ  x , p,  …(2) dp
 dx  ∴ = 0 or x + f ′( p) = 0
dy dx
where p stands for First eduation gives p = constant = c. Thus, we
dx
obtained y = cx + f (c).
Let the solution of (2) is
2. If we eliminate p between y = px + f ( p) and
F (x , p, c) = 0 … (3)
x + f ′( p) = 0 we get the singular solution of
Eliminating p between (1) and (3) we get the given differential equation.
required solution of (1) say
3. Some differentiable equations by suitable
φ( x , y, c) = 0
change of variables (may be given in questions)
If we cannot eliminate p between (1) and (3) may be reduced to Clairaut’s form.
then we may solve (1) and (3) such that
x = f1(p, c) and y = f2 (p, c) …(4) 4. Let f (x , y, p) = 0 be the differential equation of
first order and its general solution is
which are the solution of (1).
28

φ (x , y, c) = 0 it represents a curve for each Similarly, the condition for the equation (1) inp
value of c i.e, it represents a family of curves. to have two equal roots is obtained on
Hence every differential equation of the first eliminating p between (1) and
degree represents a family of curves. ∂
f (x , y, p) = 0 …(5)
∂p
SINGULAR SOLUTIONS and the p-eliminant between (1) and (5) is
1. The singular solution of a differential equation called the p-discriminant relation.
is given by the envelope of family of curves
3. If p occurs only in the first degree in the
represented by that differential equation.
differential equation, there will be no singular
Thus, if the envelope of the family of curves solution.
φ(x , y, c) = 0 …(1)
4. The locus common to both the c-discriminant
represented by the general solution of the and the p-discriminant relations gives us the
differential equation singular solutions of (1).
f (x , y, p) = 0 …(2)
5. Let the general solution of the Clairaut’s
exists, the equation of the envelope is the equation y = px + f ( p) …(1)
singular solution of the differental equation (2). is y = c (x) + f (c) …(2)
2. Let the given differential equation be Differentiating (2) partically w.r.t. c
f (x , y, p) = 0 …(1) 0 = x + f ′(c) …(3)
and its general solution is φ( x , y, c) = 0 …(2) The singular solution, which is envelope of (2)
The envelope of the family of curves (2) is is obtained by eliminating c between (2) and
contained in the locus obtained by eliminating (3).
C, between (2) and Again differentiating (1) partially w.r.t. p, we

φ ( x , y, c) = 0 …(3) get
∂C
0 = x + f ′( p) …(4)
so the c-eliminant between (2) and (3) be
The singular solution of the Clairaut’s equation
ψ ( x , y) = 0 …(4)
(1) is obtained by eliminating p between (1)
This is also called c-discriminant relation.
and (4).

EXERCISE
MULTIPLE CHOICE QUESTIONS 3. The solution of the differential equation
2 2
1. The solution of p − 5p + 6 = 0 is y = px + p + 4 is
(a) y − 2x = c (a) (y − cx)2 = 4 cx (b) (y − cx)2 + 4c 2 = 0
(b) y − 3x = c
(c) (y − cx)2 − 4c 2 = 0 (d) (y − cx)2 − c 2 = 4
(c) (y − 2x − c) (y − 3x − c) = 0
4. The singular solution of the differential equation
(d) y 2 − 5x + 6 = c
(xp − y)2 = p 2 − 1 is
a
2. Solution of the equation y = px + is
p (a) x 2 + y 2 = 1 (b) x 2 − y 2 = 1
(a) cy = c 2x + a (b) y = x + a (c) y 2 − x 2 = 1 (d) x 2 − 2y 2= 1
a
(c) cy = cx + (d) y = cx + a
c
29

5. The solution of p 2 − ax 3 = 0 is 14. The differential equation of first degree always


(a) 25(y + c)2 − 4ax 5 = 0 represents a
(a) Unique curve (b) Unique surface
(b) 5(y + C)2 − 4ax 5 = 0
(c) Family of curve (d) Family of surfaces
(c) 25(y − c)2 − ax 5= 0
15. The equation y = px + f (p) is called
(d) 5(y − c)2 − ax 5 = 0 (a) Linear equation (b) Exact equation
6. The solution of the differential equation (c) Clairaut’s equation (d) Bernoulli’s equation
xp 2 − (y − x) p − y = 0 is dy  dy 
3
16. The solution of y = x +   is
(a) (y + x − c) (xy − c) = 0 dx  dx 
(b) (y + 2x + c) (xy − c) = 0 (a) y = cx (b) y = cx + c
(c) (2y + x − c) (xy − c) = 0 1
(c) y = cX + (d) y = cx + c 3
(d) (y − x − c) (xy − c) = 0 c3
7. The solution of y = sin p − p cos p is 17. The singular solution of a differential equation
(a) x = cos p − c, y = sin p contains
(b) x = c − cos p, y = sin p − cos p (i) Can be obtained from general solution
(c) x = c − cos p, y = − cos p
(ii) Arbitrary constant
(d) x = c − cos p, y = sin p + cos p
(iii)Cannot be obtained from general solution
8. The solution of the differential equation
(iv) Do not contain arbitrary constant
p = log (px − y) is
(a) (iii) and (iv) are true (b) (i) and (ii) are true
(a) c = log (x − y) (b) y = cx + e − p
(c) (i) and (iv) are true (d) (ii) and (iii) are true
(c) cx = log (x − y) (d) y = cx − e p
18. The solution of y = 2xp + f (xp 2) is
9. The general solution of
(a) y = 4cx (b) (y − c 2)2 = 4cx
y 2 − 2pxy + p 2 (x 2 − 1) = m 2 is
(a) (y − cx)2 = c 2 + m 2 (c) (y − 4cx)2 = c 2 (d) (y + c)2 = − 4cx

(b) y 2 − 2cxy + c 2(x 2 − 1) m 2 = 0 19. The solution of sin px cos y = p + cos px sin y is

(c) (y − cx) = c 2 2 (a) y = cx + sin c (b) y − cx = sin cx cos y


(c) y = cx − sin −1 c (d) y = cx + cos −1 c
(d) y 2 − 2xy + c 2m 2 = 0
a 20. The singular solution of
10. The singular solution of y = px + is
p y 2 − 2pxy + p 2 (x 2 − 1) = m 2 is
(a) x 2 = 4ay (b) x 2 + y 2 = a 2 (a) y 2 + m 2 x 2 = m 2 (b) m 2y 2 + x 2 = m 2
2 2 2 2
(c) y = 4ax (d) x − y = a (c) y 2 + x 2 = m 2 (d) y 2 − x 2 = m 2
11. The general solution of 8p 3 − 27 y = 0 is 21. The singular solution of 27 y − 8p 3 = 0 is
2 2 2
(a) y = (x + c) (b) y = (x + c) (a) 27 y − 8c 3 = 0 (b) 27 y − 8x 3 = 0
2 3 2 3
(c) y = 8 (x + c) (d) y = (x + c) (c) x = 0 (d) y = 0
12. The general solution of (y − px)2 + a 2p = 0 is 22. The general solution of y 2 + p 2 = 1 is
2 2 2 2
(a) (y − x) = a c (b) (y − cx) = a (a) y 2 + c 2 = 1 (b) y = cos (x + c)
2 2 2 2
(c) (y − cx) + a c = 0 (d) (y − x) + a c = 0 (c) y = sin (x + c) (d) None of these

13. The singular solution of p 2 + y 2 = 1 is 23. The singular solution of p 2 + px − y = 0 is


(a) y = 1 only (b) y = − 1 only (a) y 2 = 4x (b) x 2 + 4y = 0
(c) x = ± 1 (d) y = ± 1 (c) x 2 + y 2 = 0 (d) y 2 + 4x = 0
30

24. The solution of y = xp + x 1 + p 2 is 36. Which of the following equation is Clairaut’s


equation
(a) x 2 + y 2 − xc = 0 (b) x 3 + y 2 − xc = 0
(a) y = px + f (p) (b) y = p + f (p)
(c) x 3 + y 2 − 2cx = 0 (d) x 3 − y 2 − 2cx = 0
(c) y = x + f (p) (d) y = px + f (c)
25. The solution of p 2(x 2 − a 2) − 2pxy + y 2 − b2 = 0 is 37. The solution of p = xy is
(a) (y − cx)2 = b2 + a 2c 2 (a) y = ce x (b) y = ce x / 2
(b) (y + cx)2 = b2 + a 2c 2
2
(c) y = ce x / 2 (d) y = ce x
2 2 2 2
(c) (y − cx) = b − a c
38. The order and degree of the differential equation
(d) (y + cx)2 = b2 − a 2c 2
p 3 + 3p 2 + p − 1 = 0 is
26. The solution of y = px + ap (1 − p) is (a) 3, 1 (b) 1, 3 (c) 3, 3 (d) 1, 1
(a) y − cx = ac 2 (b) y − cx = a − c 2
39. By differentiating w.r.t. x the equation y = 2px − p 2,
(b) y − cx = ac (1 + c) (d) y − cx = ac (1 − c)
the general solution x = f (p) is
27. The singular solution of y = px + a 2p 2 + b2 is 2
(a) x = 2p + cp 2 (b) x = p + cp 2
2 2 2 2 2 2 2 2 3
(a) x + y = a + b (b) a x + b y = 1
2
2 2 2 2 2 2 (c) x = 2p + cp −2 (d) x = p + cp −2
(c) b x + a y = a b (d) a 2x 2 + b2y 2 = a 2b2 3
28. The singular solution of (y − px)2 + pa 2 = 0 is 40. The solution of y 2 + x 2p 2 − 2xyp = 4p 2 is
2
a (a) y 2 + x 2c 2 = 4c 2 (b) y 2 + x 2c 2 − 2xy = 0
(a) xy = a (b) x − y =
4
a2 (c) (y− cx)2 = 4/ c 2 (d) (y − cx)2 = 2/ c
(c) xy = a 2 (d) xy =
4 41. The solution of (x − a)p 2 + (x − y)p − y = 0 is
29. The envelope of family of curves represented by the (a) (y − cx)2 = 4c (b)y 2 + (x − y)2 + x − c = 0
differential equation is called
2
(a) Complete solution (b) Particular integral (c) (x − a)c + (x − y) c − y = 0
(c) Singular solution (d) General solution (d) (x − a)2 + (x − y) c = c
30. Singular solution of p 2 + y 2 = 1 is 42. The number of constant is a singular solution of nth
2 2
(a) x + y = 1 (b) xy = 1 degree equation is
2 (a) 0 (b) n (c) n + 1 (d) n − 1
(c) y = 4x (d) Not exist
31, The solution of the equation p 2 + pxy = y 2 log y is 43. By differentiating w.r.t. y, the equation
2 2 3
(a) log y = x + cX (b) log x = y + cy xp = a + bp, The general solution x = f (p) is
(c) c 2 + cxy = y 2 log y (d) log y = x 2 + cx + c b a b
(a) x = ap + (b) x = +
p p p2
32. The singular solution of y = px is a b a b
(a) y 2 = 4ax (b) x 2 + y 2 = a 2 (c) x = + (d) x = +
p3 p4 p3 p2
(c) y = mx + c (d) Not exist
44. By differentiating w.r.t. y, the equation
33. The solution of (y − px) = p 2(1 − p 2)2 is
2
p 3 − 4xy p + 8y 2= 0 the general solution y = f (p) is
(a) y − cx = x 2 (b) y = cx + c − c 3
2 2 (a) p = cy (b) p 2 = cy
(c) y − 2x + 2cx = 0 (d) y = cx + c 2
(c) p 2 + 2p = cy (d) None of these
34. The solution of (y − px), (p − 1) = p is
(a) (y − x) (c − 1) = c (b) (y + x) (c − 1) = c 45. Using the transformation x 2 = u, y 2 = v, the
(c) (y − cx) (c − 1) = c (d) (y − cx) c = c 2 2
solution of x (y − px) = yp is 2

2 2
35. The singular solution of xp − (x − a) = 0 is (a) y = cx + c 2 (b) y 2 = cx + c 2
2 2
(a) y = 0 (b) xc − (x − a) = 0 (c) y 2 = cx + c (d) y 2 = cx 2 + c 2
2
(c) x − (x − a) = 0 (d) x = 0
31

46. The singular solution of the differential equation 53. The p-discriminant of f (x, y, p) = 0 is
2 2
y (1 + p ) = a is 2 ∂
(a) f (x, y, p) = 0 (b) f (x, y, p) = 0
∂p
(a) y 2(1 + x 2) = a 2 (b) y 2 = a 2x
(c) (a) or (b) only (d) Both (a) and (b)
(c) y = ± ax (d) y = ± a
dy
47. The singular solution of p 2(1 − x 2) = 1 − y 2 is, 54. The solution of y = 2x + 3e dy / dx is
dx
(a) x = ± 1 only (b) y = ± 1 only (a) y = 2cx + 3e c (b) y = 2x + 3e x
(c) x = 1, y = 1 (d) x = ± 1, y = ± 1
(c) y = 2cx + 3e x (d) y = 2x + 3e c
48. The solution of y = 2px + y 2p 3 by substitution
55. One singular solution of p 3 − 4pxy + 8y 2 = 0 is
y 2 = v is
4 3 4 3
(a) y 2 = x (b) y = x
c3 c3
(a) y 2 = cx + (b) y 2 = cx 2 + 27 27
8 8 27 3 27 3
c c (c) y = x (d) y 2 = x
(c) y 2 = x + (d) y 2 = cx + 4 4
2 2
56. By differentiating w.r.t. x the equation
49. By differentiating w.r.t. x, the equation
y = 2px + f (xp 2), the general solution in the form
y = a 1 + p 2 the general solution x = f (p) is
x = f (p) is
(a) y = a 1 + x 2 + p (a) px = c (b) x p = c
(c) px 3/ 2 = c (d) p x = c
(b)x = a log {p + 1 + p2} + c
57. The singular solution of pxy + x 2 = a is
(c) y = a log {p + 1 + p2} + c
(a) x 2 + xy = a (b) x + y 2 = a
2
(d) y = a log { 1 + p } + c (c) y 2 + x = a (d) No singular solution

50. The general solution of y 1 + p 2 = r is 58. The singular solution of y = px + f (p), where p is
linear in p is
(a) x 2 + y 2 = r 2 (b) y 2 + (x + c)2 = r 2
(a) y = cx + f (c) (b) y = cx
(c) x 2 + (y + c)2 = r 2 (d) y 1 + x 2 = r 2 (c) x = cy + f (c) (d) No singular solution

51. The singular solution of y = px + a 1 + p 2 is 59. The degree of the differential equation
2 2 2 3 2
(a) y = 4ax (b) x − y = 1 p + 3p + 2p + x = 0 is
2 2 2 2
(c) x + y = a (d) x = 4ay (a) 1 (b) 2 (c) 3 (d) 0

52. By differentiating w.r.t. x the equation 60. The order of the differential equation
2 3 2 2
y = 2px + p y, the general solution y = f (p) is p − 4xyp + 8y = 0 is
(a) p 2y + p + 2 = 0 (b) p 2y = c (a) 3 (b) 2
(c) py + p = a (d) py = c (c) 1 (d) None of these

ANSWERS
MULTIPLE CHOICE QUESTIONS
1. (c) 2. (a) 3. (d) 4. (b) 5. (a) 6. (d) 7. (b) 8. (d) 9. (a) 10. (c)
11. (d) 12. (c) 13. (d) 14. (c) 15. (c) 16. (d) 17. (a) 18. (b) 19. (c) 20. (a)
21. (d) 22. (b) 23. (b) 24. (c) 25. (a) 26. (d) 27. (c) 28. (d) 29. (c) 30. (d)
31. (a) 32. (d) 33. (b) 34. (c) 35. (d) 36. (a) 37. (c) 38. (b) 39. (d) 40. (c)
41. (c) 42. (a) 43. (d) 44. (b) 45. (d) 46. (d) 47. (d) 48. (a) 49. (b) 50. (b)
51. (c) 52. (d) 53. (d) 54. (a) 55. (b) 56. (d) 57. (d) 58. (d) 59. (c) 60. (c)
32

HINTS AND SOLUTIONS


1. p 2 − 5p + 6 = 0 ⇒ (p − 2) (p − 3) = 0 (1) and (2) are the required solution.
dy 9. Given that y 2 − 2pxy + p 2(x 2 − 1) = m 2
If p − 2 = 0 ⇒ − 2 = 0 ⇒ y − 2x − c = 0
dx
dy i.e. (y − px)2 = p 2 + m 2 ⇒ y = px ± p 2 + m2
If p − 3 = 0 ⇒ − 3 = 0 ⇒ y − 3x − c = 0
dx which is Clairaut’s form, so solution is
Thus, the solution is (y − 2x − c) (y − 3x − c) = 0 y = cx ± c 2 + m 2 ⇒ (y − cx)2 = c 2 + m 2
a
2. Given equation is y = px + which is Clairaut’s a
p 10. Given differential equation is y = px +
p
a
equation so solution is y = cx + ⇒ cy = c 2x + a i.e. xp 2 − yp + a = 0
c
its discriminant is y 2 − 4ax = 0
4. Given equation is (xp − y)2 = p 2 − 1
So singular solution is y 2 = 4ax
2
So, y = px ± p − 1 so its solution is dy
13. Given that p 2 + y 2 = 1 ⇒ =± 1 − y2
2
(xc− y) = c − 12 dx
dy
i.e. c (x − 1) − 2xyc + y 2 + 1 = 0.
2 2 or dx = ± integrating it,
1 − y2
The envelope of the family of above curve is the
we get, x + c = ± cos −1 y
required singular solution.
⇒ y = cos (x + c) which is general solution.
The c-discriminant relation is
Again p-discriminant is y 2 − 1 = 0 ⇒ y = ± 1 which
4x 2y 2 − 4 (x 2 − 1) (y 2 + 1) = 0
satisfy the given differential equation, so it is singular
i.e. x 2 − y 2 = 1 which is singular solution. solution.
dy 18. Given that y = 2px + f (xp 2) ...(1)
5. Given p 2 = ax 3 ⇒ p = = a1/ 2x 3/ 2 Integrating it
dx dp
Differentiating w.r.t. xp = 2p + 2x
2a1/ 2 x 5/ 2 dx
y= + C
dp 
5 + f ′ (xp 2) p 2 + 2xp 
 dx 
6. Given that xp 2 + (y − x)p − y = 0
or  p + 2x dp  [1 + pf ′ (xp 2)] = 0
 
i.e. xp (p − 1) + y (p − 1) = 0 or (p − 1) (xp + y) = 0  dx 
dy dp dp − dx
If p − 1 = 0 ⇒ = 1⇒y = x + c …(1) If p + 2x = 0⇒ = integrating it
dx dx p 2x
dy y dy dx
If xp + y = 0 ⇒ p = =− ⇒ + =0 −1
dx x y x log p = log x + log c ⇒ px 1/ 2 = c
2
Integrating log (xy) = log c ⇒ xy = c …(2) c
⇒ p= put it in (1) we get y = 2c x + f (c 2)
So the solution is (y − x − c) (xy − c) = 0 x

7. Given differential equation is 19. Given sin px cos y − cos px sin y = p i.e.
sin (− y + px) = p
y = sin p − p cos p …(1)
⇒ y = px − sin −1 p which is Clairaut’s equation so
differentiating w.r.t., x, we get
its solution is
dp
p = (cos p − cos p + p sin p) y = cx − sin −1 c
dx
dy 3 1/ 3
dp 21. Given 27 y − 8p 3 = 0 ⇒ p = = y
or p = p sin p ⇒ sin p dp = dx dx 2
dx 2
Integrating x = c − cos p …(2) ⇒ dy y −1/ 3  = dx Integrating it y 2/ 3 = x + c
3 
33

⇒ y 2 = (x + c)3 which is general solution. 39. Given y = 2px − p 2 differentiate w.r.t. x we get
Differentiating it w.r.t. c x + c = 0, put x = − c dp dp dp dp
p = 2p + 2x − 2p ⇒ p + 2x − 2p =0
dx dx dx dx
in (1) we get y = 0 so p = 0 i.e. singular solution
dx 2(p − x) dx  2 
is y = 0. or = ⇒ +  x = 2
dp p dp  p 
23. Given that p 2 + px − y = 0 its p-discriminant is 2
∫ dp
x 2 + 4y = 0 which is singular solution. its I.F. is e p
= e 2log p = p 2 so its solution is
2
27. Given differential equation is y = px + a 2p 2 + b2 xp 2 = ∫ p 2.2dp + c = p 3 + c
3
or (y − px)2 = a 2p 2 + b2 i. e.
2
x = p+
c
2 2 2 2 2 3 p2
⇒ (x − a )p − 2xyp + y − b = 0
41. Given (x − a)p 2 + (x − y) p − y = 0
So p-discriminant is
4x 2y 2 − 4(x 2 − a 2) (y 2 − b2) = 0 ⇒ xp 2 − ap 2 + xp − yp − y = 0

⇒ b2x 2 + a 2y 2 = a 2b2 which is singular solution. or p(− y + xp) − (y − xp) = ap 2


⇒ (y − px)(−1 − p) = ap 2
31. Given differential equation is y 2 log y = pxy + p 2
y log y p ap 2
or x= − differentiating w.r.t. y or y = px − which is Clairant's form so its
p y 1+ p
ac 2
1 log y y y log y dp p dp 1 solution is y = cx −
= + − + − − 1+ c
2 dy
p p yp p y 2 dy y
44. Given p 3 − 4xyp + 8y 2 = 0
p  y2  1 dp  y2  p 2 2y
or 1 + log y = 1 + log y x= + differentiating w.r.t. y, we get
2 2  y dy  2 
y  p   p  4x p

p 1 dp dp dy 1 p2 2p dp 2 2y dp
or = i. e. = integrating it =− + + −
2
y2 y dx p y p 4y 4y dy p p 2 dy

log p = log y + log c or p = cy, dp  p 2y  p2 1 p  p 2y 


or  −  = − =  − 
put it in original equation we get required solution dy  2y p 2  4y 2 p 2y  2y p 2 
log y = cx + c 2 dp p 2dp dy
So, = ⇒ = integrating, we get
p dy 2y p y
34. y = px = which is Clairaut’s form so its
p −1 log p 2 = log cy ⇒ p 2 = cy
c
solution is y = cx +
c −1 45. Given equation is x 2(y − px) = yp 2 and

35. Given that xp 2 − (x − a)2 = 0 x 2 = u, y 2 = v


dy dy x dv
⇒ p= = ± (x 1/ 2 − ax −1/ 2) i.e. 2xdx = du and 2y dy = dv so =p=
dx dx y du
i. e. dy = ± (x 1/ 2 − ax −1/ 2)dx integrating it put it in original equation, we get
2
2
y + c = ±  x 3/ 2 − 2ax 1/ 2   x dv  x 2  dv 
3  x2 y − x ⋅  = y ⋅. 2  
 y du  y du
⇒ 9(y + c)2 = 4x(x − 3a)2 …(1) 2 2
dv  dv  dv  dv 
or y2 − x 2 =   ⇒v − u = 
Differentiating at w.r.t.c. we get y + c = 0 …(2) du  du  du  du 
2 2
Also p − discriminant is x(x − a) = 0 so singular dv  dv  dv
i.e. v = u +   ⇒v = uP + P 2, when =P
solution is x = 0 du  du  du
34

This is Clairaut’s form so its solution is v = uc + c 2 55. Given that p 3 − 4pxy + 8y 2 = 0 …(1)
⇒ y 2 = cx 2 + c 2 Differentiate w.r.t. x 3p 2 − 4xy = 0

48. Given y = 2px + y 2p 3 and y 2 = v xy


i.e. p=2 …(2)
3
dy dv dv P
i.e. 2y = ⇒ 2py = P, where P = or p = Eliminate p between (1) and (2) we get 27 y − 4x 3.
dx dx dx 2y
4 3
 P3  Also y = x satisfy the given differential equation.
P
put it in above equation, y = 2x + y2  27
2y  3 
 8y 
57. pxy + x 2 = a is linear in p so no singular solution
3 3
P P exist.
⇒ y 2 = xP + or v = xP + which is in
8 8
mmm
2 c3
Clairaut’s form so its solution is y = cx + .
8
35

C HAPTER
5 Orthogonal Trajectories
DEFINITIONS (ii) In this differential equation, replace
dy dx
1. A trajectory of a given system of curves is by − and we obtained the differential
dx dy
defined to be a curve which cuts all the
equation of the orthogonal trajectories is
members of the family according to a given
obtained.
law. If the curve cuts at constant angle α, it is
(iii) Find the general solution of this differential
called an α-trajectory.
equation. This is the required equation of
If the angle is at right angle, the trajectories are orthogonal trajectories.
called orthogonal trajectories. 3. In polar co-ordinates, let the equation of given
If the angle is other than 90° the trajectories are family of curver be f (r , θ, c) = 0 ...(1)
called oblique trajectories. Differentiating it w.r.t. θ and eliminating c, the
differential equation of the given family (1) be
2. In cartesian co-ordinates, the equation of the dr
φ  r,θ,  = 0 ...(2)
orthogonal trajectories can be obtained as  dθ 
follows : The differential equation of the required
(i) Differentiate the equation of the given family of trajectories is

curves and eliminate the arbitrary parameter. φ  r,θ,− r 2  = 0
 dr 
Thus, the differential equation of given family
Solving this we shall get the required equation
of curves is obtained.
of the orthogonal trajectories.

EXERCISE
MULTIPLE CHOICE QUESTIONS (a) x 2 + y 2 = cy + 1 (b) x 2 + y 2 = cy
1. The orthogonal trajectories of the family of curves (c) x 2 − y 2 = cy (d) x 2 + y 2 = cy 2
2
y = ax is
4. The orthogonal trajectories of the cardioids
(a) x 2 + y 2 = c 2 (b) x 2 + 3y 2 = c 2 r = a (1 + cos θ); where a is the parameter is
(c) x 2 + 2y 2 = c 2 (d) x 2 − 2y 2 = c 2
(a) r = b cosθ (b) r = b(1 − cos θ)
2. The orthogonal trajectories of the family of curves
(c) r = b cos 2 θ + 1 (d) r = b(1 + cos θ)
ay 2 = x 3 is
3 2 5. The differential equation of family of orthogonal
(a) x 2 − y = c2 (b) x 2 + 3y 2 = c 2
2 trajectories of the family of circles x 2 + y 2 = 2ax is
3
(c) x 2 + y 2 = c 2 (d) x 2 − 3y 2 = c 2 x 2 − y2 x 2 + y2
2 (a) y′ = (b) y′ =
xy 2xy
3. The orthogonal trajectories of the family of circles
x 2 − y2 x 2 + y2
2 2 (c) y′ = (d) y′ =
(x − 1) + y + 2ax = 0, where a is parameter 2xy xy
36

6. The orthogonal trajectories of the family of curves 16. The orthogonal trajectories of r n = a n cos nθ, a is a
y = ax 4 is parameter is
2
(a) x + ny = c 2 2 2
(b) x + y = n2 2 (a) r n = c n cos nθ (b) r n sin nθ = c n
n n
(c) r cos nθ = c (d) r n = c n sin nθ
(c) nx 2 + y 2 = c 2 (d) x 2y 2 = n2c 2
17. The differential equation of the family of circles
7. The orthogonal trajectories of the rθ = a
2 x 2 + y 2 + 2 fy + 1 = 0, where f is parameter is
(a) r = ae θ (b) r = ae θ
2 (a) x 2 + y 2 + 2xy′ = 0
(c) r 2 = ae θ (d) r 2 = ae θ
(b) x 2 − y 2 − 2xyx ′+1 = 0
8. The orthogonal trajectories of the family of circles
(c) x 2 − y 2 + yx ′+1 = 0
x 2 + y 2 = a 2, where a is parameter is
(d) x 2 + y 2 + yx ′−1 = 0
(a) xy = c (b) x 2y = c
18. The differential equation of the orthogonal
(c) x 2 = cy (d) x = cy
trajectories of the curve r = e aθ is
9. The differential equation of orthogonal trajectories dθ log r dθ rθ
(a) =− (b) =−
of the family of curve y 2 = 4a (x + a) is dr rθ dr log r
(a) yy′2 +2xy′ = y (b) xy′2 + xy′ = y dθ log r dθ rθ
(c) = (d) =
2 2 dr rθ dr log r
(c) yy′ + y′ = x (d) yy′ + xy′ = 2y
19. The differential equation of the orthogonal
10. The orthogonal trajectories of the family of
trajectories of the family of cardioids r = a (1 + cos θ)
rectangular hyperboloid xy = c 2 is
is
(a) x + y = k (b) x − y = k dr 1 + cos θ dr r sin θ
(a) = (b) =
(c) x 2 + y 2 = k (d) x 2 − y 2 = k dθ sin θ dθ 1 + cos θ
dr r (1 + cos θ) dr r sin θ
11. The orthogonal trajectories of r = e aθ is (c) = (d) =−
dθ sin θ dθ 1 + cos θ
(a) θ + log r = k (b) θ2 + (log r)2 = k
20. The equation of the family of curves that is
(c) θ2 + log r = k (d) θ + (log r)2 = k
orthogonal to ax 2 + y 2 = 1 is
n n 2
+ y2
12. The orthogonal trajectories of r sin nθ = a is (a) y = ce x + y (b) y 2 = ce x
(a) r n cosec nθ = bn (b) r n sec nθ = bn (c) y 2 = ce x
2
+y
(d) y 2 = ce x + y
2

n n n n
(c) r cos nθ = b (d) r tan nθ = b
21. The differential equation of the orthogonal
13. The differential equation of family of curves of
trajectories of the family of curves
y 2 − x 2 + 4xy − 2cx = 0 is 2/ 3 2/ 3 2/ 3
x +y =a , where a is parameter is
(a) 2x (2x + y) y′ = x 2 + y 2
(a) xdx = ydy (b) x 2/ 3dx = y 2/ 3dy
(b) (x + 2y) y′ = x 2 + y 2
(c) x 1/ 3dx = y1/ 3dy (d) None of these
(c) 2x (x + y) y′ = x 2 + y 2
(d) (2x + y) y′ = x 2 + y 2 22. The orthogonal trajectories of the curve
x 2 + y 2 = a 2, where a is the parameter is
14. The orthogonal trajectories of the family of coaxial
x
circles x 2 + y 2 + 2gx + c = 0, g is parameter is (a) xy = c (b) = c
y
(a) x 2 + y 2 + a 2 (b) x 2 + y 2 + x + c = 0 y2
2 2 2 2 (c) x 2y = c (d) =c
(c) x + y + 2gy + c = 0 (d) x + y + 2y + c = 0 x
15. The orthogonal trajectories of the family of circles 23. To find the orthogonal trajectories in polar curve,
x 2 + y 2 = 2ax , a is parameter is dθ
r is replaced by
dr
(a) x 2 = 4ay (b) x 2 + y 2 = bx dr 1 dr dr 1 dr
(a) r (b) (c) −r (d) −
(c) y 2 = 4ax (d) x 2 + y 2 = by dθ r dθ dθ r dθ
37

24. The differential equation of the orthogonal (a) Concentric circles (b) Parabolas
dy (c) Ellipses (d) Itself
trajectories we replace by
dx
dx dx dx dx 28. The orthogonal trajectories of the family of
(a) (b) − (c) −x (d) −y
dy dy dy dy parabolas y 2 = a (x + a) where a is a parameter a

25. The differential equation of orthogonal trajectories family of


2 (a) Liner (b) Ellipses
of the curve xy = a is given by
(c) Parabolas (d) Hyperbolas.
(a) dx = ydy (b) xdx = dy
29. The equation of the system of orthogonal
(c) ydx = xdy (d) xdx = ydy 2a
trajectories of the parabolas r = , a is
26. A curve which cuts every member of a given family 1 + cosθ
of curves according to some given law is called a parameter is
(a) Orthogonal trajectories (a) r = 2b(1 + cos θ) (b) r = 2b sinθ
2b
(b) Trajectories (c) r = 2b cosθ (d)
1 − cosθ
(c) Self orthogonal families 30. The orthogonal trajectories of the family of curves
(d) Orthogonal curves r 2 = a 2 cos 2θ is
b2
27. The orthogonal trajectories of a system of (a) r 2 = b2 sin 2θ (b) r 2 =
sin 2 θ
concurrent straight lines is a system of 2 2
(c) r = b tan θ 2
(d) r = b2 cot 2θ
2

ANSWERS
MULTIPLE CHOICE QUESTIONS
1. (c) 2. (c) 3. (a) 4. (b) 5. (c) 6. (a) 7. (d) 8. (d) 9. (a) 10. (d)
11. (b) 12. (c) 13. (a) 14. (c) 15. (d) 16. (d) 17. (b) 18. (a) 19. (c) 20. (b)
21. (c) 22. (b) 23. (d) 24. (b) 25. (d) 26. (b) 27. (a) 28. (c) 29. (d) 30. (a)

HINTS AND SOLUTIONS


dy dy
1. Given y = ax 2, differentiating it, = 2ax x 2 − 2x + 1 + y 2 +  2 − 2x − 2y  x = 0
dx  dx 
eliminate a between them we get dy
y 2 − x 2 − 2xy +1= 0
1 dy 2 dx
= this is the differential equation of the
y dx x
So the differential equation of orthogonal
given family of curves so differential equation of the
trajectories is
1  dx  2
orthogonal trajectories is −  = i.e. dx
y  dy  x y 2 − x 2 + 2xy +1= 0
dy
xdx + 2ydy = 0
x2 1 or 2xydx + (y 2 − x 2 + 1) dy = 0 = Mdx + Ndy
integrating it, + y 2 = c 2 ⇒ x 2 + 2y 2 = c
2 2 ∂M ∂N
= 2x, = −2x
This is the required orthogonal trajectories. ∂y ∂x
3. Given curve x 2 − 2x + 1 + y 2 + 2ax = 0
is 1  ∂N ∂M  1 2
dy So,  −  = (− 4x) = −
differentiate it 2x − 2 + 2y + 2a = 0 Thus, the 
M ∂x ∂y  2xy y
dx 1
So integrating factor is e − f 2/ ydy =
differential equation of family of curves is
y2
38

2x  1− x2  10. Given curve is xy = c 2, differentiate it w.r.t. x,


Thus dx +  1 + dy = 0 is exact so
y  y2  xdy
+ y = 0, this is the differential equation of the
integrating it we get dx
x2 1
+ y − = c ⇒ x 2 + y 2 − 1 = cy given family of curves. So differential equation of
y y  dx 
the orthogonal trajectories is x  −  + y = 0
dr  dy 
4. Givn curve is r = a (1 + cos θ), = − a sin θ
dθ ⇒ xdx = ydy integrating it x 2 − y 2 = c
eliminate a between two equation
(1 + cos θ) dr 12. Given family of curve is r n sin nθ = a n
r=−
sin θ dθ Differentiate it w.r.t. θ we get
dr
This is the differential equation of family of curves. nr n cos n θ + nr n−1 sin n θ =0

So differential equation of family of orthogonal
dr
trajectories is i. e., = − r cot n θ which is the differential equation
− (1 + cos θ)  2 dθ  dθ
r=  −r  of the given family of curves.
sin θ  dr 
1 + cos θ  rdθ So the differential equation of the orthogonal
so 1 =  
 sin θ  dr dθ
trajectories is − r 2 = − r cot n θ
dr 1 + cos θ dr
i.e. − dθ = 0 1
r sin θ i. e. dr = tan n θ d θ
r
dr 2 cos 2 θ / 2
Integrating it ∫ −∫ dθ = log b 1
r 2 sin θ / 2 cos θ / 2 Integrating it, log r = − log(cos nθ) + log c
n
log r − 2 log sin θ / 2 = log b
n log r = − log cos n θ + n log c
θ b
r = b sin 2 = (1 − cos θ) = c (1 − cos θ)
2 2 ⇒ r n = c n / cos n θ ⇒ r n cos n θ = c n

6. Given y = ax n, differentiate it w.r.t. x we get 15. Given family of curve is x 2 + y 2 = 2ax


dy
= anx n−1, eliminate a between them we get the dy
dx differentiate it w.r.t. x, 2x + 2y = 2a
dx
differential equation of family of curves i. e.
dx dy
x = ny, so differential equation of orthogonal i. e., x + y − a = 0, eliminate a between two
dy dx
dy
dy equations x 2 + 2xy
− y 2 = 0 which is the
trajectories is x  −  = ny ⇒ − xdx + nydy = 0 dx
 dx 
differential equation of family of curves. So
Integrating we get x 2 + ny 2 = c 2
differential equation of orthogonal trajectories is
9. Given curve is y 2 = 4a (x + a)  dx  dx x 2 − y 2
x 2 + 2xy  −  − y 2 = 0 ⇒ =
dy  dy  dy 2xy
Differentiate it 2y = 4a, eliminate a between
dx dx ydv
them we get the differential equation of given family put x = vy, =v+ we get,
dy dy
of curves i. e.
y 2 = 2y
dy 
x +
y dy 
 ydv v 2 − 1 2vdv dy
= −v ⇒ = − , integrating
dx  2 dx  dy 2v 1 + v2 y
So the differential equation of family of orthogonal x2 b
it log(1 + v 2) = − log y + log b ⇒ 1 + =
trajectories is y 2 y
dx  y dx 
y = −2 x −  ∴ 2 2
x + y = by
dy  2 dy 
2 y  −2(2xy′− y) 17. Given family of curves is x 2 + y 2 + 2 fy + 1 = 0
i. e. y=− x −  =
y′  2y′  2y′2 Differentiate w.r.t. x, 2x + 2y
dy
+ 2f
dy
=0
⇒ yy′2 + 2xy′ = y dx dx
39

dx dx 22. Given that x 2 + y 2 = a 2, differentiate it w.r.t. x we


i. e., x + y + f = 0 ⇒ f = −y − x
dy dy dy x
get = − , it is the differential equation of the
eliminate f between two equations we get dx y
dx given family of curves. So differential equation of
x 2 − y 2 − 2xy +1= 0
dy the orthogonal trajectories is
dx x dx dy
18. Given r = e aθ differentiate w.r.t. θ we get − =− ⇒ =
dy y x y
dr
= ae aθ = ar
dθ Integrating it, log x = log y + log c ⇒ x = cy

Eliminate a between two equations we get 25. Given that xy = a 2


1 dr log r differentiate w.r.t. x, we get
=
r dθ θ xdy
+ y = 0 so differential equation of the family of
Which is the differential equation of given family of dx
curves. So differential equation of orthogonal orthogonal trajectories is
trajectories is  dx 
x  −  + y = 0 ⇒ xdx = ydy
rdθ log r dθ log r  dy 
− = ⇒ =−
dr θ dr rθ 2a dr 2a sin θ
29. r= , differentiating w.r.t. θ, =
20. 2 2
Given that ax + y = 1, differentiate it w.r.t. x we 1 + cosθ dθ (1 + cos θ)2
dy ax eliminate a between two equations, we get
get = − , eliminate a between them we get
dx y dr r sin θ
= , so the differential equation of
dy y 2 − 1 dθ 1 + cos θ
= . So differential equation of orthogonal
dx xy
r 2d θ r sin θ
2 2 orthogonal trajectories is − =
dx y − 1 1− y dr 1 + cos θ
trajectories is − = i. e., xdx = dy,
dy xy y dr 1 + cos θ θ
i. e. =− dθ = − cot dθ
integrating we get r sin θ 2
1 2 1 1
x = log y − y 2 − log c integrating we get, log r = −2 log sin
θ
+ log c
2 2 2 2
⇒ x 2 = log y 2 − y 2 − log c or log
r
= log
1
⇒r =
c
=
2c
c 2 2 1 − cos θ
2
+ y2 sin θ / 2 sin θ / 2
log(y 2 / c) = x 2 + y 2 or y 2 = ce x
mmm
Unit-II
C HAPTER
6 Ordinary Simultaneous Differential Equations
INTRODUCTION i.e. F (D)x = F (t)
1. In this chapter, we shall discuss ordinary which is linear differential equation with
differential equations containing one constant coefficients in x and t. After solving it
independent variable and two or more then we find x, then put this x in (1) or (2) we get
two dependent variables. value of y.
Let x and y be two dependent variables and t 4. Method of Differentiation : Let the two
be the independent variable. Then the given equation (1) and (2) connect four
simultaneous linear differential equations are dx dy
quantities x , y, and . Differentiating (1)
f 1(D)x + f 2 (D)y = F1(t) ...(1) dt dt
and φ 1 (D) x + φ 2 (D) y = F2 (t) ...(2) and (2) w.r.t. t we have four equations
where F1(t) and F2 (t) are functions of t. dx dy d 2 x d 2y
containing x , y, , , and .
2. There are two methods to solve equations (1) dt dt dt 2 dt 2
and (2) first method is method of elimination dy d 2y
Eliminate y, and from these four
and second method is method of differentiation. dt dt 2
3. Method of Elimination : Operating both equations we get a differential equations of
sides of (1) by φ (D) and of (2) by f 2 (D) and then order two in x and t. Solving it we get x in terms
of t. Then by other equations we get the
subtracting, we get
variable y.
[φ 2 (D) f 1 (D) − f 2(D) φ 1 (D)] x
= φ 2 (D) F1 (t) − f 2 (D) F2 (t)

EXERCISE
MULTIPLE CHOICE QUESTIONS (a) A cos t + B sin t + t (b) A cos t − B sin t + t 2 + 1
dx dy 2
1. In equations = −ωy, = ωx, the value of x is (c) A cos t + B sin t + t − 1
dt dt (d) None of these
(a) e ωt ( A cos ωt + B sin ωt) dx dy
4. If = y + 1 and = x + 1 then x is
(b) A cos ωt + B sin ωt dt dt
(c) e − ωt ( A cos ωt + B sin ωt) (a) Ae t + Be − t (b) Ae 2t + Be − t
(d) e ωt ( Aωt + B) (c) Ae t + Be −2t (d) Ae t + Be − t − 1

2. Solution for x of the following equations 5. If Dx = 2y, Dy = 2z, Dz = 2x then the differential
(D − 7)x + y = 0 and −2x + (D − 5) y = 0 is equation for x is
(a) e 6t ( A cos t + B sin t) (b) e 4 t ( A cos t − B sin t) (a) (D 3 − 8) x = 0 (b) (D 3 − 2D) x = 0

(c) e −6t ( A cos t − B sin t) (d) e −4 t ( A cos t + B sin t) (c) (D 3 − 4D + 2) x = 0 (d) (D 3 + 8) x = 0

3. If
dx
− y = t and
dy
= t 2 − 5 then x is 6. If D 2y = x and D 2x = y then the solution for y is
dt dt (a) Ae t + Be − t + C cos t − D sin t
41

(b) Ae t + Be − t − C cos 2t − D sin 2t 16. If (D − 3) x + 8y = 0 and x + (D + 3) y = 0 then x is


2t
(c) Ae + Be −2t
− C cos t − D sin t (a) Ae 2t + Be −2t (b) x = Ae t + Be − t
t −2t
(d) Ae + Be 2t −2t
− C cos 2t − D sin 2t (c) Ae + Be (d) Ae 2t + Be − t
dx dy dz 17. If (D 2 − 3) x − 4y = 0 and x + (D 2 + 1) y = 0 then the
7. The solution of = = is
yz zx xy
differential equation for y is
(a) Unique (b) Finite (c) Infinite (d) Not exist
(a) (D 2 + 1) y = 0 (b) (D 2 + 1)2 y = 0
dx dy
8. If + 4y = 0 and −4x + = 0 then x is (c) (D 2 − 1)2 y = 0 (d) (D 2 − 1)y = 0
dt dt
(a) A sin 4t + B cos 4t (b) A sin 4t − B cos 4t 18. The locus of the point satisfying the equation
Dx = −ωy and Dy = + ωx is
(c) − A sin 4t + B cos 4t (d) − A sin 4t − B cos 4t
(a) Line (b) Parabola
9. If D 2x + y = sin t and D 2y + x = cos t then the
(c) Ellipse (d) Circle
differential equation containing x is
dx dy dz
(a) (D 4 − 1)x = − sin t − cos t 19. The solution of = = is
x y z
4
(b) (D − 1) x = sin t + cos t
(a) x = c1 2, y = c 2z (b) x = c1y, y = c 2z 2
(c) (D 4 − 1) x = sin t − cos t
(c) x = c1z 2, y = c 2z (d) x = c1y 2, y = c 2z
(d) (D 4 − 1) x = − sin t + cos t
xdx dy dz
dx dy 20. The solution of = = is
10. If + 5y = 0 and − 5x = 0 then y is 2
y z xz y 2
dt dt
(a) A cosh 5t + B sinh 5t 5t
(b) Ae + Be −5t (a) x − y = c1, x 2 − z 2 = c 2
2 2

(c) ( A + Bt) e 5t (d) A cos 5t + B sin 5t (b) x 3 − y 3 = c1, x 2 − z 2 = c 2


11. If Dx + Dy + 2x + y = 0 and Dy + 5x + 3y = 0 then x (c) x 3 + y 3 = c1, x 2 + z 2 = c 2
is
(a) A cosh t + B sinh t (b) Ae t + Be − t (d) x 2 + y 2 = c1, x 2 + z 2 = c 2
(c) A cos t + B sin t (d) ( A + Bt) e t 21. The solution of
dx
=
dy
=
dz
x (y 2 − z 2) y (z 2 − x 2) z (x 2 − y 2)
12. If Dx = ny − mz, Dy = lz − nx, Dz = mx − ly then the
differential equation for x is is
2 2 2
(a) (D − l − m − n ) x = lc 2 (a) x 2 + y 2 + z 2 = c1, x + y + z = c 2

(b) (D 2 + l 2 + m 2 + n2) x = lc (b) x 2y 2z 2 = c1, x + y + z = c 2

(c) (D 2 + l 2 + m 2 + n2) x = (l + m) c (c) x 2 + y 2 + z 2 = c1, xyz = c 2

(d) (D 2 − l 2 − m 2 − n2) x = (l + m + n) c (d) x + y − z = c1, xyz = c 2


dy xdz 22. If Dx + y = sin t, Dy + x = cos t with x (0) = 0,
13. If x + z = 0 and + y = 0 then the differential
dx dx x ′(0) = 1 then x is
equation for y is 1
(a) 2e t + 3e − t (b) e t + e − t
(a) (D 2 − 1) y = 0 (b) (D 2 + 1)y = 0 2
t 1 −t
(c) (D 2 − 2D + 1) y = 0 (d) None of these (c) 2e + e (d) e t − e − t
3
14. Is t dx = (t − 2x) dt and t dy = (tx + ty + 2x − t) then dx dy dz
23. The solution of = = is
2 2
the solution for x is y z −x z x 2y
A t A t A t A
(a) t + (b) + (c) + (d) + (a) f (x 3 + y 3, y 2 + z 2) = 0
t 2 t2 4 t 3 t2
dx dy (b) f (x 3 + y 3 + z 3, y 2 + z 2) = 0
15. In equation = −ωy, = ωx the value of y is
dt dt (c) f (x 3 + y 3 + z 3, x 2 + y 2 + z 2) = 0
ωt
(a) A sin ωt − B cos ωt (b) e ( A sin ωt + B cos ωt)
(d) f (x 3 + y 3 + z 3, xyz) = 0
(c) e ( A cos ωt − B sin ωt) (d) e ωt ( A cos ωt + B sin ωt)
ωt
42

dx dy dz dx dy dz
24. The solution of = = is 30. The solution of = = is
x 2 − y2 − z 2 2xy 2x 2 mz − ny nx − lz ly − mx
y (a) lx + my + nz = c1, xyz = c 2
(a) f  , x + y + z  = 0
2  (b) lx + my + nz = c1, x + y + z = c 2
x
(b) f  , x 2 + y 2 + z 2  = 0 (c) lx + my + nz = c1, e xy = c 2z
2 
(d) lx + my + nz = c1, x 2 + y 2 + z 2 = c 2
 x 
(c) f  , xyz  = 0
2  dx dy dz
31. The solution of = = is
 y x 2 + y2 + z 2  z (x + y) z (x − y) x 2 + y 2
(d) f  ,  =0
z z  (a) f (xy − z 2, x 2 + y 2 + z 2) = 0
dx dy dz (b) f (xy − z 2, x 2 − y 2 − z 2) = 0
25. The solution of = =
x 2 − y2 y 2 − zx z 2 − xy (c) f (xyz, x 2 − y 2 − z 2) = 0
y−z (d) f (xy − z 2, xyz) = 0
(a) φ  x 2 + y 2 + z 2,  =0
 z − x
dx dy dz
x −y y−z 32. The solution of = = is
(b) φ  ,  =0 1+ y 1+ x z
 y− z z − x
x + y+ z
x −y  (a) f  zx − zy,  =0
(c) φ  ,x + y + z = 0  z 
y−z 
x + y
 x − y  (b) f  zx − zy,  =0
(d) φ  , xyz  = 0  z 
y−z 
x + y+ z
26. The solution of
dx
=
dy
=
dz
is (c) f  xy − yz,  =0
 z 
x (y − z) y (z − x) z (x − y)
1 1 1 (d) f (xy − yz, x − y + z) = 0
(a) + + = c1, xyz = c 2
x y z dy dx
33. If = x and = y + e 2t then the value of x is
(b) xy + yz + zx = c1, xyz = c 2 dt dt
(c) x 2 + y 2 + z 2 = c1, xyz = c 2 2
(a) c1e t + c 2e − t + e 2t (b) c1e t + c 2e − t + e 2t
3
(d) (x + y + z) = c1, xyz = c 2
t −t 1 2t
dx dy (c) c1e + c 2e + e (d) None of these
27. If t + y = 0, t + x = 0 then x is 3
dt dt dx dy
34. If + 2x = 3y and + 2y = 3x then x is
(a) c1t + c 2 / t (b) c1t + c 2t dt dt
(c) c1t 2 + c 2t (d) (c1 + c 2t) t (a) c1e t + c 2e 3t (b) c1e t + c 2e −3t

28. The solution of


dx
=
dy
=
dz
is (c) c1e t + c 2e 5t (d) c1e t + c 2e −5t
x (y − z) y (z − x) z (x − y)
dx 2dy
35. If + − 2x + 2y = 0 and
(a) f (x − y + z, y + z) = 0 dt dt
(b) f (x + y + z, xyz) = 0 dx dy
3 + + 2x + y = 0 then x is
(c) f (x + y + z, xy) = 0 dt dt
(d) f (x 2 + y 2 + z 2, x + y + z) = 0 (a) ce −6/ 5t (b) c1e t + c 2e −2t
dx dy dz (c) c1e 2t + c 2e −3t (d) ce 6/ 5t
29. The solution of = = is
y x z
dy dx
36. If = x and = y then x is
(a) x + y = c1z, x 2 + y 2 = c 2 dt dt
(b) x + z = c1y, x 2 − y 2 = c 2 (a) c1 cos t + c 2 sin t (b) (c1 + c 2t) e t
2 2
(c) x + y = c1z, x − y = c 2
(c) c1e t + c 2e − t (d) (c1 + c 2t)e − t
2 2
(d) x + z = c1y, x + y = c 2
43

dx 2 dy 1 d 2x dx
37. If + (x − y) = 0 and + (x + 5y) = 0 then 42. Elimination of y between t 2 +t + 2y = 0,
dt t dt t dt 2 dt
differential equation for x is d 2y dy
2 2 t2 +t − 2x = 0 gives
(a) (t D − 4tD + 6) x = 0 dx 2 dt
(b) (t 2D 2 − 8D + 8) x = 0 (a) (D 4 + 4) x = 0
(c) (t 2D 2 − 4tD + 12) x = 0 (b) (D 4 + 2D 2 + 3) x = 0
(d) (t 2D 2 + 8tD + 12) x = 0 (c) (D 3 + 4) x = 0
38. If we choose 1, x, z or multipliers in (d) (D 2 + 4) x = 0
xdx dy dz
= = then 43. The differential equation in x from
z 2 − 2yz − y 2 y + z y − z
dx 2dy
3 + −4x + 3y = 0 and
(a) x + y + z = c (b) x 2 + y 2 + z 2 = c dt dt
(c) xyz = c (d) None of these dx dy
4 + + 3x + 4y = 0 is
dt dt
39. If we choose x, − y, − z as multipliers in
dx dy dz (a) (D 2 + D + 3) x = 0
= = then
z (x + y) z (x − y) x 2 + y 2 (b) (D 2 + 3D + 5) x = 0
(a) x 2 + y 2 + z 2 = c (b) x 2 − y 2 + z 2 = c (c) (D 2 − 2D + 5) x = 0
(c) x 2 + y 2 − z 2 = c (d) x 2 − y 2 − z 2 = c (d) (D 2 + 2D + 5) x = 0
40. By choosing x, y, z as multipliers in dx dy
44. The solution of + + 2x + y = 0 and
dx dy dz dt dt
= = , we have
x (y 2 − z 2) − y (z 2 + x 2) z (x 2 + y 2) dy
+ 5x + 3y = 0 for x is
2 2 2 2 2 2 dt
(a) x + y + z = c (b) x y z = c
(a) (c1 + c 2t) e t (b) c1 cos t + c 2 sin t
(c) xyz = c (d) x + y + z = c
t −t
(c) c1e + c 2e (d) (c1 + c 2t) cos t
1 1 1
41. By choosing , , as multipliers in
x y 3z 45. The differential equation in x from
dx
=
dy
=
dz
we have d 2x dy dx dy
− = 2x + 2t and +4 = 3y is
3 4 4 3 3 3
y x − 2x 2y − x y 9z (x − y ) dt 2 dt dt dt

(a) xyz = c (b) x 1/ 3yz = c (a) (D 3 + D 2 − D + 3) x = 4 − 3t

(c) xy1/ 3z = c (d) xyz1/ 3 = c (b) (D 3 − D 2 − 4D + 1) x = 4 − 3t


(c) (D 2 − 3D + 4) x = 4 − 3 t
(d) (2D 3 − D 2 − 4D + 3) x = 4 − 3t

ANSWERS

MULTIPLE CHOICE QUESTIONS


1. (b) 2. (a) 3. (d) 4. (d) 5. (a) 6. (a) 7. (d) 8. (c) 9. (a) 10. (d)
11. (c) 12. (b) 13. (d) 14. (d) 15. (a) 16. (d) 17. (c) 18. (d) 19. (a) 20. (b)
21. (c) 22. (d) 23. (a) 24. (d) 25. (b) 26. (c) 27. (a) 28. (b) 29. (c) 30. (d)
31. (b) 32. (a) 33. (a) 34. (d) 35. (a) 36. (c) 37. (d) 38. (b) 39. (d) 40. (a)
41. (d) 42. (a) 43. (d) 44. (b) 45. (d)
44

HINTS AND SOLUTIONS


dx
1. Given = −ωy, differentiate it w.r.t. t or (D 2 + 1) x = 0, A.E. is m 2 + 1 = 0, m = ± i
dt
d 2x dy So solution is x = A cos t + B sin t
= −ω = −ω(ωx) = −ω2x
2 dt dy dz
dt 13. Given x = − z and x = −y
dx dx
So (D 2 + ω2) x = 0 A.E. is m 2 + ω2 = 0
Differentiate equation first we get
or m = ± ωi so x = A cos ωt + B sin ωt d 2y dy dz y
dx dy +x + =− =
3. Given − y = t and = t2 − 5 dx 2 dx dx x
dt dt
Differentiate first equation w.r.t. t we get d 2y dy
i. e. x2 +x −y= 0
2 dx
d 2x dy d 2x dx
− =1 ⇒ − t2 + 5 = 1
dt 2 dt df 2 14. Given tdx = (t − 2x) dt
dx 2x dx 2
d 2x = 1− ⇒ + x =1
i. e. = t 2 − 4 integrating it dt t dt t
dt 2 2/ tdt
I.F. e ∫ = t 2, So solution is xt 2 = ∫ t 2dt + A
dx t 3
= − 4t + A, again integrating
dt 3 t3 t A
xt 2 ± + A ⇒ x= +
t4 3 3 t2
x= − 2t 2 + At + B
12 17. Given (D 2 − 3) x − 4y = 0 …(1)
5. Dx = 2y, Dy = 2z, Dz = 2x and 2
x + (D + 1) y = 0 ...(2)
Differentiate first equation D 2x = 2Dy = 4z
Multiplying (2) by (D 2 − 3) and then subtracting, we
3 get
Again differentiate D x = 4Dz = 8x ⇒
(D 3 − 8) x = 0 or (D 2 − 1)2 y = 0

6. Given that D 2y = x and D 2x = y 18. After solving x= A cos ωt + B sin ωt and


y = A sin ωt − B cos ωt
Differentiate first equation twice we get
x 2 + y 2 = A2 + B 2 i. e. a circle
D 4 y = D 2x = y ⇒ (D 4 − 1) y = 0
dx dy dz
20. Given that = =
⇒ (D 2 + 1)(D 2 − 1) y = 0 2
y z/x xz yz
So A. E. is (m 2 + 1)(m 2 − 1) = 0 ⇒ m = 1, − 1,i, − i By first and second term x 2dx − y 2dy = 0
t −t
So y = Ae + Be + C cos t + D sin t ⇒ x 3 − y 3 = c1
9. Given that D 2x + y = sin t ...(1) and by second and third term ydy − zdz = 0
and 2
x + D y = cos t ...(2) ⇒ y2 − z 2 = c 2
dx dy dz
Multiplying equation (1) by D 2 and then subtract 21. Given = =
2 2 2 2
equation (2) x (y − z ) y (z − x ) z (x − y 2)
2

(D 2 − 1) x = − sin t − cos t Consider


xdx + ydy + zdz
we get x 2 + y 2 + z 2 = c1
0
11. Given that
1 1 1
(D + 2) x + (D + 1) y = 0 ...(1) Choosing , , as multiplier we have each
x y z
5x + (D + 3) y = 0 ...(2) fraction
dx dy dz
Multiplying (1) by (D + 3), (2) by (D + 1) and then + +
x y z so dy + dy + dz = 0
subtracting, we get
0 x y z
[(D + 2)(D + 3) − 5(D + 1)] x = 0
Integrating log xyz = log c 2 ⇒ xyz = c 2
45

22. Given that Dx + y = sin t ...(1) dx dy x + 5y


37. Given t + 2x − 2y = 0 and =−
dt dt t
x + Dy = cos t ...(2)
Differentiating first equation w.r.t. t we get
Multiplying (1) by D and subtracting (2) we get
d 2x dx 2dx 2dy
(D 2 − 1) x = cos t − cos t = 0 t + + − =0
2 dt dt dt
dt
A.E. is m 2 − 1 = 0 ⇒ m = 1, − 1 d 2x 3 dx 2(x + 5y)
or t + + =0
t −t t −t 2 dt t
So solution is x = c1e + c 2e , x ′ = c1e − c 2e dt
2
∵ x (0) = 0, x ′(0) = 1 so we get 0 = c1 + c 2 and d x 3t dx 2 + 5 / 2 + dx / dt + 5x 
or t 2 + + 2   =0
1 = c1 − c 2 ⇒ c1 = 1, c 2 = −1 so we get dt 2 dt  t 

x = et − e− t After solving we get (t 2D 2 + 8tD + 12) x = 0


dx dy dz dx dy dz
24. = = 39. Given = =
x 2 − y2 − z 2 2xy 2xz z (x + y) z (x − y) x 2 + y 2
dy dz Choosing x, − y, − z as multipliers, we get
Taking last two terms we get =
y z
xdx − ydy − zdz
⇒ log y = log z + log c1 i. e.
x 2z + xyz − xyz + y 2z − x 2z − y 2z
y
= c1 xdx − ydy − zdz = 0
z
xdx + ydy + zdz dz integrating x 2 − y 2 − z 2 = c
consider =
x (x 2 + y 2 + z 2) 2xz dx dy dz
41. Given that = =
2 2 2
Integrating it log(x + y + z ) = log z + log c 2 y 3x − 2x 4 2y 4 − x 3y 9z (x 3 − y 3)
1 1 1
x 2 + y2 + z 2 Choosing , , as multipliers we get
i. e. = c2 x y 3z
z
 y x 2 + y2 + z 2  dy dy dz
So solution is f  , + +
 =0 x y 3z
z z 
y 3 − 2x 3 + 2y 3 − x 3 + 3x 3 − 3y 3
dx dy
27. Given that t + y = 0 and t ≠x=0 dx dy dz
dt dt i. e. + + = 0, integrating xyz1/ 3 = c
x y 3z
Differentiating first equation w.r.t. t we get
43. Given (3D − 4) x + (2D + 3) y = 0 ...(1)
td 2x dx dy d 2x dx x
+ + = 0⇒t + − =0 and (4D + 3) x + (D + 4) y = 0 ...(2)
dt 2 dt dt dt 2 dt t
Multiplying (1) by (D + 4) and (2) by (2D + 3) and
d 2x dx
i. e. t2 +t −x = 0 subtracting, we get
dt 2 dt
d [(3D − 4)(D + 4) − (4D + 3)(2D + 3)] x = 0
put it e z we get [D (D − 1) + D − 1] x = 0 D ≡
d2 i. e. (D 2 + 2D + 5) y = 0

(D 2 − 1) x = 0 A.E. m 2 − 1 = 0 ⇒ m = 1, − 1 45. Given equations are


(D 2 − 2) x − Dy = 2t ...(1)
So solution x = c1e z + c 2e − z = c1t + c 2 / t
and Dx + (4D − 3)y = 0 ...(2)
35. Given that (D − 2) x + (2D + 2) y = 0 ...(1)
Eliminating y from (1) and (2), we get
and (3D + 2) x + (D + 1) y = 0 ...(2)
[(4D − 3)(D 2 − 2) + D 2] x = (4D − 3) 2t
Multiplying (2) by 2 and subtracting from (1) we get
(5D + 6)x = 0 A.E. is 5m + 6 = 0 ⇒ m = −
6 i. e. (2D 3 − D 2 − 4D + 3) x = 4 − 3t
5
mmm
So solution is x = ce −6/ 5 t
46
Unit-II
C HAPTER
7
Linear Equations of Second Order
with Variable Coefficients
LINEAR DIFFERENTIAL EQUATION OF SECOND REMOVAL OF THE FIRST DERIVATIVE
ORDER (REDUCTION TO NORMAL FORM)
d 2y dy 1. If we are unable to obtain a part of C.F. of the
1. +P + Qy = R, where P, Q and R are
dx 2 dx solution of the equation
functions of x only, is called the linear d 2y dy
+P + Qy = R ...(1)
differential equation of second order. dx 2 dx
2. If an integral included in the complementary then we change the dependent variable from y
function of such an equation be known then to v in equation (1) by putting y = uv (where u is
the general solution can be found in terms of
a function of x) we get
the known integral.
d 2v  2 du  dv
3. Let y = x be known integral i. e. solution of u + u P +
dx 2
 u dx  dx
d 2y dy
+P + Qy = 0  d 2u du 
dx 2 dx +v 
2
+P + Qu = R ...(2)
 dx dx 
and y = uv be the solution of
d 2y dy To remove the first derivative term, we take
+P + Qy = R
dx 2 dx 2 du
P+ =0
then after solving, we get u dx
1
e − ∫ Pdx − ∫ Pdx
y = c1u ∫ dx + c2u ⇒ u=e 2 ...(3)
u2
So equation (2) becomes
 e −∫ Pdx  1
Pdx
d 2v  1 dP P 2  ∫ Pdx
+u ∫ 
 u 2
∫ u Re∫ dx  dx +  Q − −  v = R e2

 dx 2  2 dx 4
Where last term shows the particular integral. d 2v
i. e. + Xv = Y ...(4)
d 2y dy dx 2
4. The solution of +P + Qy = 0 can be 1
dx 2 dx 1 dP P 2 ∫ Pdx
where X = Q − − and Y = R e 2
find by the following results. 2 dx 4
(i) y = e x is a part of complementary function if Equation (4) is called the normal form of
1+ P + Q= 0 equation (1) and can be solve easily.
(ii) y = e − x is a part of (C.F.) if 1 − P + Q = 0 TRANSFORMATION OF THE EQUATION BY
(iii) y = e mx is part of C.F. if m 2 + mP + Q = 0 CHANGING THE INDEPENDENT VARIABLE x
(iv) y = x is a part of C.F. if P + Qx = 0 1. Let the differential equation is
2 2 d 2y dy
(v) y = x is a part of C.F. if 2 + 2Px + Qx = 0 +P + Qy = R
dx 2 dx
(vi) y = x m is a part of C.F. if m (m − 1) + Pmx Then by changing the independent variable be
+ Qx 2 = 0 changed from x to z, we get
47

d 2y dy dy
= A1 + B1v + Au1 + Bv1
+ P1 + Q1y = R1
dz 2 dz dx
Choose A and B such that
d 2z dz
+P A1u + B1v = 0 ...(4)
2 dx Q
where P1 = dx , Q1 = ,
2 2 So by (1), (2) and (4) we get
 dz   dz 
    A1u1 + B1v1 = R1 ...(5)
 dx   dx 
R Solving (4) and (5), we get
R1 =
2
 dz  vR uR
  A1 = − and B1 =
 dx  u1v − uv1 v1u − vu1
where P1, Q1 and R1 are functions of x. so A = ∫
vRdx
+ C1
2 u1v − uv1
 dz 
Choose   = Q or variable part of Q such uR
 dx  and B=∫ dx + C2
v1u − vu1
dz
that it is a perfect square then = Q then
dx Substitute A and B in (3) we get the complete
calculate P1, Q1 and R1. primitive of (1).
d 2y dy
If P1 is zero or constant, then this method is Now, consider +P + Qy = R ...(1)
2 dx
applicable. dx
Then P.I. of (1) is
METHOD OF VARIATION OF PARAMETERS − u∫
vR
dx + v∫
uR
dx
Let the linear equation of second order be w w
d 2y dy vR uR
+P + Qy = R ...(1) = −u ∫ dx + v∫ dx ...(2)
dx 2 dx uv1 − vu1 uv1 − vu1

and let y = Au + Bv Where u and v are defined by y = c1u + c 2 v

Is the complementary function of (1), where u u v


and Wronskian of u and v is W =
and v are functions of x and A and B are u1 v1
constants. ∴ C.S. = C.F. + P.I.
So ( Au 2 + Bv 2 ) + P ( Au 1 + Bv 1)
+ Q ( Au + Bv) = 0
METHOD OF OPERATIONAL FACTORS
Let the linear equation of second order is
i. e. u 2 + Pu1 + Qu = 0
d 2y dy
and v2 + Pv1 + Qv = 0 ...(2) +P + Qy = R
dx 2 dx
Again let y = Au + Bv ...(3)
i. e. f (D) y = R ⇒ f1(D) f2(D)y = R
is the complete primitive of (1) then A and B
are functions of x so by (3)
48

EXERCISE
MULTIPLE CHOICE QUESTIONS (a) y = x (b) y = e x (c) y = x 2 (d) y = e − x
1. y = ex is the part of solution of d 2y dy 2
2 9. If y = e − x is a part of C.F. of +P − y = x2
d y dy 2 dx x
+P + Qy = 0 if : dx
2 dx
dx then value of P is :
(a) 1 − P + Q = 0 (b) P + Qx = 0 (a) x − 2 (b) x + 2
(c) 1 + P + Q = 0 (d) Px + Q = 0 x−2 x+2
(c) (d)
x x
d 2y dy
2. For P +Q + Ry = X, if P − Q + R = 0, then a
dx 2 dx 10. If y = ex is a part of C.F. of
2
part of the C.F. is : d y dy
x + (1 − x) + Qy = e x then Q is :
2
(a) y = e − x (b) y = e x dx dx
(c) y = x (d) y = x 2 (a) 2x − 1 (b) 2x + 1
(c) +1 (d) −1
3. The part of C.F. of the differential equation :
11. A part of C.F. of
d 2y dy
x − (2x − 1) + (x − 1) y = 0 is d 2y dy
dx 2 dx x2 − 2x (1 + x) + 2 (1 + x) y = x 3 is :
2 dx
dx
(a) y = e − x (b) y = x
(a) y = x (b) y = x 3
(c) y = x 2 (d) y = e x
(c) y = e x (d) y = e 2x
4. y = x is a part of complementary function of
d 2y dy
d 2y dy 12. y = e − x is a part of C.F. of +P + Qy = R if :
+P + Qy = R if : 2 dx
dx
dx 2 dx
(a) 1 − P + Q = 0 (b) 1 + P + Q = 0
(a) 1 + P + Q = 0 (b) 1 − P + Q = 0
(c) P + Q = 0 (d) P − Q = 0
(c) Q + Px = 0 (d) P + Qx = 0
d 2y dy
5. A part of the complementary function of the differe- 13. The solution of x 2 +x − 9y = 0 , given that
dx 2 dx
d 2y dy
-ntial equation x − (2x − 1) + (x − 1)y = 0 is : y = x 3 is a solution is :
dx 2 dx
c c2
(a) x (b) x 2 (c) e x (d) e 2x (a) y = c1x 3 + 2 (b) y = c1x 3 +
x x2
6. The part of C.F. of the differential equation 3 c2
(c) y = c1x + (d) None of these
dy  d 2y  x3
x − y = (x − 1)  − x + 1 is :
dx  2 
 dx  d 2y dy
2 −x
14. A part of C.F. of − x2 + xy = x is :
(a) y = x (b) y = e 2 dx
dx
(c) y = e x (d) y = x (a) y = e x (b) y = e − x

7. A part of the C.F. of the solution of the differential (c) y = x (d) y = x 2


2
equation
d y
+ (1 − cot x)
dy
− y cot x = sin 2 x is : d 2y dy
2 15. In reducing the equation +P + Qy = R in
dx dx dx 2 dx
(a) y = e − x (b) y = e x d 2v
3
the normal form + Xv = Y, value of X is :
(c)y = x (d) y = x dx 2
1 1
8. A part of the C.F. of the solution of the differential −
2
∫ Pdx ∫ Pdx
2
(a) e (b) Re 2
d y 2(x + 1) dy x + 2 x−2 x
equation − + y= e is : 1 dP P 2
dx 2 x dx x x (c) Q − − (d) None of these
2 dx 4
49

16. The normal form of the differential equation (a) e x (b) e x


2
(c) e − x (d) e − x
2

d 2y dy 2
− 4x + (4x 2 − 3) y = e x is : 24. In reducing the equation
2 dx
dx
2
(a) (D + 1) v = 1 2
(b) (D + 1) v = x d 2y 2 dy  2 x
− + 1 +  y = xe
dx 2 x dx  x2
(c) (D 2 − 1) v = e x (d) (D 2 + 2) v = 1
d 2v
17. To change the equation in normal form + Xv = Y then X is :
dx 2
d 2y dy 1
+ 2x + (x 2 + 1) y = x 3 + 3x in normal form (a) x (b) 1 (c) e x (d)
dx 2 dx x
the value of u is : 25. In reducing the equation
(a)e x
(b) e x2
(c) e −x 2 / 2
(d) e −x 2 d  2 dy  2 d 2v
 cos x  + y cos x = 0 in + Xv = y ,
2 dx  dx  dx 2
d y dy
18. In reducing the equation +P + Qy = R in value of y is :
dx 2 dx
(a) secx (b) 2 (c) 2x (d) 0
the normal form the value of u is :
1 1 d 2y dy
∫ Pdx ∫ Pdx
− 26. If the equation cos x + sin x − 2y cos 3 x = 0
(a) e 2 (b) e 2 dx 2 dx
1 1
− ∫ Rdx − ∫ Qdx d 2y dy
(c) e 2 (d) e 2 is changed to + P1 + Q1y = 0 then P1 is :
dz 2 dz
d 2y dy
19. y = e mx is a part of C.F. of +P + Qy = R if : (a) 0 (b) −2 (c) 2 cos 2 x (d) x
2 dx
dx
d 2y dy
(a) 2 + 2Px + Qx 2 = 0 (b) m 2 + Pm + Q = 0 27. If the equation x − − 4x 3y = 0 is
2 dx 2 dx
(c) mP + Qx = 0 (d) P + mQ + m = 0
d 2y dy
transformed into + P1 + Q1y = 0 then Q1 is :
20. The part of C.F. of the differential equation dz 2 dz
d 2y dy
(2x − 1) −2 + (3 − 2x) y = 2e x is (a) −1 (b) 2sin z
dx 2 dx
(c) −x (d) None of these
(a) y = x (b) y = x 2 (c) y = e − x (d) y = e x 2
d y dy
28. If the equation + (tan x − 1)2 − n(n − 1)y
2 d 2y dx 2 dx
21. If y = cot x is a solution of sin x = 2y then
dx 2 d 2y dy
complete solution is : sec4 x = 0 is transformed to + P1 + Q, y = 0
dz 2 dz
(a) y = c1 cot x + c 2x cot x
then P1 is :
(b) y = c1 cot x + (1 − x cot x) c 2 (a) 1 (b) n (n − 1)
(c) y = c1 cot x + c 2 sin x (c) − n (n − 1) (d) None of these
(d) y = c1 cot x + (1 + cot x) c 2
29. The transformed equation of
dy d 2y d 2y dy 1
22. In reducing the equation +P + Qy = R in x6 + 3x 5 + a 2y = from x to z variable is :
dx 2 dx
dx 2 dx x2
d 2v (a) (D 2 − a 2) y = 0 (b) (D 2 + a 2) y = z
the normal form + Xv = Y, value of Y is :
2
dx
1 1 (c) (D 2 + a 2) y = −2z (d) (D 2 − a 2) y = −2z
∫ Pdx − ∫ Pdx
(a) R e2 (b) R e 2
30. If y = A cos 2x + B sin 2x be the complete primitive
1 1
∫ Pdx − ∫ Pdx d 2y
(c) e2 (d) e 2 of + 4y = 4 tan 2x then Wronskian is :
dx 2
23. To change the differential equation (a) 1 (b) 2 (c) 3 (d) 4
d 2y dy 2
− 4x + (4x 2 − 1) y = −3e x sin 2x in normal dy d 2y
dx 2 dx 31. The Wronskian of −2 + y = e x log x is :
2 dx
dx
form the value of u is :
(a) e x (b) e 2x (c) 1 (d) 0
50

d 2y d 2y dy
32. The Wronskian of + n2y = sec nx is : 40. If the equation + (3 sin x − cot x) + 2y sin 2 x
2
dx dx 2 dx
(a) x 2 (b) secnx (c) nx (d) n d 2y dy
= e − cos x sin 2 x be transformed into + P1
33. The Wronskian of the differential equation dz 2 dz
d 2y dy + Q1y = R1 then R1 is :
x2 +x − y = x 2e x is :
dx 2 dx (a) 1 (b) e − z (c) e + z (d) −1
2 2 1 2
(a) 1 (b) − (c) (d) d y dy x

x x x 41. If the equation − (1 + 4e x ) + 3e 2x = e 2( x + e )


2 dx
dx
34. If y = vx be the solution of
d 2y dy
d 2y dy be transformed into + P1 + Q1y = R1 then P1
2 2
x − (x + 2x) + (x + 2) y = x 3e x then the dz 2 dz
dx 2 dx
is :
transformed equation is :
(a) 3 (b) 4 (c) e 2z (d) e 2x
2 x 2 x
(a) (D − 1) v = e (b) (D + 1) v = e
42. The differential equation [3x 2D 2 + (2 + 6x − 6x 2)D
(c) (D 2 − 1) v = e − x (d) (D 2 + 1) v = e − x
−4] y = 0 is equivalent to :
3 dy 2 d 2y (a) (3x 2D + 2) (D − 2) y = 0
35. If y = vx be the solution of x +x − 9y = 0
2 dx
dx (b) (3x 2D + 2) (D + 2) y = 0
then transformed equation is :
(c) (D − 2) (3x 2D + 2) y = 0
7
(a) (D 2 + 7 D) v = 0 (b)  D 2 + D  v = 0
 x  (d) (D + 2) (3x 2D − 2) y = 0
7
(c) (D 2 − 7 D) v = 0 (d)  D 2 − D  v = 0 43. The differential equation [xD 2 − (x + 2) D + 2] y
 x 
= x 3 is equivalent to :
36. If y = ve x be the solution of
(a) (D − 1) (xD − 2) y = x 3
d 2y dy
x
2
− (2x − 1) + (x − 1) = 0 then transformed (b) (D + 1) (xD + 2) y = x 3
dx dx
(c) (xD − 2) (D + 1) y = x 3
equation is :
(a) (xD 2 − D) v = 0 (b) (xD 2 + D) v = 0 (d) (xD − 2) (D − 1) y = x 3

(c) (xD 2 + 1) v = 0 (d) (xD 2 − 1) v = 0 d 2y dy


44. The differential equation x + (x − 1) −y
2 dx
37. If y = v sec x be the solution of dx

d y 2
dy = x 2 is equivalent to :
− 2 tan x + 5y = sec xe x then transformed (a) (xD + 1) (D − 1) y = x 2
dx 2 dx
equation is : (b) (D − 1) (xD − 1) y = x 2
(a) (D 2 + 6) v = e x (b) (D 2 − 6) v = e x (c) (xD − 1) (D + 1) y = x 2
(c) (D 2 + 5) v = xe x (d) (D 2 + 6) v = sec xe x (d) (D + 1) (xD + 1) y = x 2

38. To change the equation 45. The differential equation [3x 2D 2 + (2 − 6x 2)D − 4]y
d 2y dy = 0 is equivalent to :
x2 − 2 (x 2 + x ) + (x 2 + 2x + 2) y = 0 into
dx 2 dx (a) (3x 2D + 2) (D + 2) y = 0
normal form, the value of u is : (b) (3x 2D + 2) (D − 2) y = 0
(a) x (b) e x (c) xe − x (d) xe x
(c) (D + 2) (3x 2D + 2) y = 0
39. The part of C.F. of the differential equation
(d) None of these
d 2y dy
(3 − x ) − (9 − 4x) + (6 − 3x) y = 0 is : 46. The differential equation [xD 2 + (x − 2) D − 2] y = 0
2 dx
dx
is equivalent to :
(a) y = x (b) y = x 2 (c) y = e x (d) y = e 2x
(a) (xD − 2) (D + 1) y = 0 (b) (xD + 2) (D + 1) y = 0
51

(c) (D + 1) (xD − 2) y = 0 (d) (D − 1) (xD + 2) y = 0 d 2y dy


57. If +P + Qy = R is transformed to
47. If y = A cos x + B sin x be the general solution of dx 2 dx
(D 2 + 1) y = x then A is : d 2y dy
+ P1 + Q1y = R1 then P1 is :
2 dz
(a) x sin x − cos x + c (b) x cos x − sin x + c dz
(c) x cos x + sin x + c (d) x sin x + cos x + c  d 2z dz 
d 2z  + 
 2 dx 
48. If y = A cos ax + B sin ax be the general solution of 2  dx 
(a) dx (b)
(D 2 + a 2) y = cosec ax then A is :  dz 
2
 dz 
2
   
x x  dx   dx 
(a) ax + c (b) + c (c) − + c (d) − ax + c
a a d 2z dz d 2z dz
−P +P
49. If y = Au + Bv be the complete primitive of 2 dx 2 dx
(c) dx (d) dx
D 2y + PDy + Qy = R then A and B are functions of :  dz 
2
 dz 
2
   
(a) x and y (b) x only (c) y only (d) Constant  dx   dx 
50. If y = Ax + Bxe 2x
be the complete primitive of 58. If y = e x is a part of C.F. of
[x 2D 2 − 2x (1 + x) D + 2 (1 + x)] y = x 3 then A is : d 2y dy
− cot x − Qy = e x sin x then Q is :
x x dx 2 dx
(a) x + c (b) + c (c) − + c (d) − x + c
2 2 (a) 1 + cot x (b) cot x
(c) 1 − cot x (d) e x cot x
51. The normal form of (D 2 + PD + Q) y = R is :
(a) (D + x)v = Y (b) (D 2 + xD)v = 0 59. If y = A cos ax + B sin ax be the general solution of
(D 2 + a 2) y = sec ax then B is :
(c) (D + xD)v = 0 (d) (D 2 + x)v = Y
(a) ax + c (b) x + c
 2  2  x a
52. The normal form of  D 2 − D +  1 +   y = 0 is (c) + c (d) + c
 x  x2 a x
B
(a) (D 2 + 1) v = x (b) (D 2 + 1) v = 0 60. If y = Ax +be the general solution of
x
(c) (D 2 − 1) v = x (d) (D 2 − 1) v = 0 (x D + xD − 1) y = x 2e x then A is :
2 2

e− x ex
53. The equation [x 2D 2 + D − (1 + x 2)] y = 0 is (a) (b) e x (c) e − x (d)
2 2
equivalent to :
(a) (D − 1) [x 2(D + 1) + 1] y = 0 61. If y = ve x be the solution of
2
d y dy
(b) [x 2(D − 1) − 1] (D + 1) y = 0 x + (1 − x) − y = ex then transformed
2 dx
dx
(c) [x 2(D − 1) + 1] (D + 1) y = 0
equation is :
(d) [x 2(D + 1) + 1] (D − 1) y = 0 d 2v dv
(a) x + (1 + x) =1
2 dx
54. If y = A cos x + B sin x be the general solution of dx
(D 2 + 1) y = x then B is : d 2v 1 + x  dv
(b) +   =0
2
 x  dx
(a) x sin x + cos x + c (b) x cos x + sin x + c dx
(c) x sin x − cos x + c (d) x cos x − sin x + c d 2v dv
(c) x + (1 − x) =1
55. If y = A cos 2x + B sin 2x be the general solution of dx 2 dx

(D 2 + 4) y = 4 tan 2x then B is : d 2v 1 − x  dv
(d) +   =0
(a) − sin 2x + c (b) − tan 2x + c dx 2  x  dx

(c) − cos 2x + c (d) cos 2x + c d 2y


2 dy  2 2
62. If −+ a +  y = 0 is transformed into
2 dx 2
x dx  x2
56. If  D 2 + D + n2  y = 0 is reduced to normal form
 x  d 2v
normal form + Xv = Y then X is :
(D 2 + X) v = Y then X is : dx 2
2
1 (a) 0 (b) a (c) a 2 (d)
(a) 0 (b) n (c) n2 (d) a2
x
52

d 2y dy 1 1
63. If + (1 + 2 tan 2 x) y = sec x tan x is
+ 2 tan x (a) 1 (b) (c) e (d)
dx 2 dx e e2
d 2v dy
transformed into normal form + Xv = Y then y 68. The solution of x 2 + 2xy − x + 1 = 0 when
dx 2 dx
is :
x = 1, y = 0 is :
(a) tan x (b) sec2 x
1 1 1 1 1 1
(c) sec x tan x (d) sec2 x tan x (a) − + (b) + −
z x 2x 2 z x 2x 2
64. In reducing (D 2 − 2 tan x + 5) y = 0 into normal form 1 1 1 1
(c) 1 − + (d) 1 + −
choosen such that : x x2 x x2
(a) u = tan x (b) u = sec x
69. If complete solution of (D 2 + PD + Q) y = 0 is
(c) u = cot x (d) u = cosec x
 D 2 − 1 D + 4x 2  y = x 4 y = c1e − x + c 2e −3x then P and Q are :
65. If the equation   be
 x  (a) P = 3, Q = 4 (b) P = 4, Q = 3
2 (c) P = 3, Q = 3 (d) P = 4, Q = 4
d y dy
transformed into + P1 + Q1y = R1 then P1 is :
dz 2 dz d 2x dx
70. If +3 + 2x = 5 then lim x (t) is :
(a) 0 (b) 2 (c) 4 (d) 1 dt2 dt t→ 8
2 5 3 1 2
d y dy (a) (b) (c) (d)
66. If the equation − cot x − y sin 2 x = 0 be 2 2 2 3
dx 2 dx
d 2y dy
dy d 2y 71. To change the equation x2 − 2x
transformed into + P1 + Q1y = R1 then P1 is : dx 2 dx
dz 2 dz
2 d 2v
(a) 0 (b) 1 (c) −1 (d) 2 + (2 + x ) y = 0 into + Xv = Y, X is equal to:
dx 2
dy 2 ln x
67. If x 2 + 2xy = and y (1) = 0, then y(e) is : x2
dx x (a) x (b) 1 (c) (d) 0
2

ANSWERS
MULTIPLE CHOICE QUESTIONS
1. (c) 2. (a) 3. (d) 4. (d) 5. (c) 6. (d) 7. (a) 8. (b) 9. (c) 10. (d)
11. (a) 12. (a) 13. (c) 14. (c) 15. (c) 16. (a) 17. (c) 18. (b) 19. (b) 20. (d)
21. (b) 22. (a) 23. (b) 24. (b) 25. (d) 26. (a) 27. (a) 28. (a) 29. (c) 30. (b)
31. (b) 32. (d) 33. (b) 34. (a) 35. (b) 36. (b) 37. (a) 38. (d) 39. (c) 40. (c)
41. (b) 42. (c) 43. (d) 44. (c) 45. (b) 46. (a) 47. (b) 48. (c) 49. (b) 50. (c)
51. (d) 52. (b) 53. (d) 54. (a) 55. (c) 56. (c) 57. (d) 58. (c) 59. (c) 60. (d)
61. (a) 62. (c) 63. (d) 64. (b) 65. (a) 66. (a) 67. (d) 68. (a) 69. (b) 70. (a)
71. (b)

HINTS AND SOLUTIONS


d 2y 2x + 1  dy  x − 1  d 2y x dy 1
3. Given −   +  y=0 6. Given that − + y = x −1
2
dx  x  dx  x  dx 2 x − 1 dx x − 1

d 2y dy d 2y dy
Comparing +P + Qy = R we have after comparing +P + Qy = R we have
dx 2 dx dx 2 dx
1 + P + Q = 0 so e x is a part of C.F. P + Qx = 0, so y = x is the part of C.F.
53

d 2y dy 2 d 2y d 2v dv
9. If y = e − x is a part of C.F. of +P − y = x2 = cot x − 2 cosec 2 x + 2 cosec 2 x cot x.v
2 2 2 dx
dx dx x dx dx
then 1− P + Q = 0 Put in
given equation, we get
or
2
1− P − = 0 d 2v 2 dv
− =0
x dx 2 sin x cos x dx
2 x−2
⇒ P = 1− =
x x dp dp 2 sec2 x
Put = p we have = dx
dx p tan x
d 2y dy
11. Given that x 2 − 2x (1 + x) + 2 (1 + x) y = x 3
2
dx dx Integrating we get p = c1 tan 2 x
2
d y 1 + x  dy 1+ x 
or − 2   + 2  y=x i. e.
dv
= c1(sec2 x − 1)
dx 2  x  dx  x2  dx
1+ x  1 + x  = 0, soy = x y
Here P + Qx = −2   + 2  Again integrating v = = c1 tan x − c1x + c 2
 x   x  cot x
is a part of C.F. or y = c1 − c1x cot x + c 2 cot x
2
d y dy 1 1
13. Given x 2 +x − 9y = 0 − ∫ Pdx − ∫ −4 x dx 2

dx 2 dx 23. u=e 2 =e 2 = ex
d d 2y
2 dy  2 x
put x = e z and D = we get 24. Given + 1 +
−  y = xe
dz dx 2
x dx  x2
2 2
[D(D − 1) + D − 9] y = 0 Here P = − , Q = 1 + , R = xe x
x x2
(D 2 − 9) y = 0 A.E. is m = ± 3
1 dP P 2
To convert it in normal form X = Q − −
3z −3z 2 dx 4
so solution is y = c1e + c 2e
2 1 2  1
c2 X = 1+ −  − =1
or y = c1x 3 + x2 2 x2 x2
3
x
d 2y dy
d y 2
dy 2 26. Given cos x + sin x − 2y cos 3 x = 0
16. Given that − 4x + (4x 2 − 3) y = e x dx 2 dx
dx 2 dx
2 Here P = tan x, Q = −2 cos 2 x, R = 0
Here, P = −4x, Q = 4x 2 − 3, R = e x
Changing the independent variable from x to z, we
u = e −1/ 2 ∫ Pdx get
= e −1/ 2 ∫ − 4x dx d 2y dy
+ P1 + Q1y = R1
2 dz
2 dz
= ex Q −2 cos 2 x
1 dP P 2 1 1 Choose Q1 = = = Constant = −2
X = Q− − = 4x 2 − 1 − (−4) − .16x 2 = 1  dz 
2
 dz 
2
2 dx 4 2 4    
1 1
 dx   dx 
∫ Pdx 2 ∫ −4 x dx 2 2 dz
and Y = R e 2 = ex e 2 = ex e− x = 1 so = cos x or z = sin x
dx
d 2v d 2z dz
So, normal form is + Xv = Y +P
2
dx 2 so P1 = dx dx =0
2
d 2v  dz 
⇒ +v=1  
dx 2  dx 

d 2y d 2y dy
21. Given differential equation is sin 2 x = 2y and 28. Given + (tan x − 1)2 − n (n − 1) sec4 xy = 0
2
dx dx 2 dx
y = cot x is a solution so put y = v cot x Here Q = − n (n − 1) sec4 x, P = (tan x − 1)2, R = 0
dy dv
= cot x − v cosec 2 x d 2y dy
dx dx To convert it into + P1 + Q1y = R1 choose Q
Again differentiating dz 2 dz
54

Q − n (n − 1) sec4 x 31. Given differential equation is


such that Q1 = =
2 2
 dz   dz  d 2y 2dy
    − + y = e x log x
 dx   dx  dx 2 dx
= constant = − n (n − 1) A.E. is m 2 − 2m + 1 = 0 ⇒ m = 1, 1
dz
so = sec2 x ⇒ z = tan x
dx So. C.F. is y = c1e x + c 2xe x
d 2z dz Here, u = e x , v = xe x so Wronskian W is
+P
2 dx
So P1 = dx u v ex xe x
2 W= = x
 dz  u1 v1 e e x + xe x
 
 dx 
= e 2x (1 + x − x) = e 2x
2 tan x sec2 x + (tan x − 1)2 sec2 x
=
sec4 x d 2y
dy
33. Given x 2 − y = x 2e x
+x
2 tan x + 1 + tan 2 x − 2 tan x dx 2
dx
or P1 = =1
d
sec2 x put x = e 2 and D ≡ we have
dz
d 2y 3 dy a 2 1 z
29. Given + + y= [D(D − 1) + D − 1] y = e 2z . e e
2 x dx x 6
dx x8
z
3 a2 1 ⇒ (D 2 − 1) y = e 2z . e e
Here, P = ,Q= ,R =
x x6 x8 A.E. is m 2 − 1 = 0, m = 1, − 1
To change the variable from x to z, we choose c2
So C.F. is y = c1e z + c 2e − z = c1x +
2 x
a 1
Q 6 So, u = x, v = and Wronskian W is
Q1 = = x = constant = a 2 x
2 2
 dz  dz
  1
    u v x
 dx   dx  W= = x = − 1 − 1 = −2
u1 v1 1 x x x
dz 1 1 1 −
so = i.e. z = − x2
dx x 3 2x 3
Now transformed equation is 35. Given y = vx 3 be the solution is
2
d 2y d y dy
+ P1
dy
+ Q1y = R1 x2 +x − 9y = 0. Differentiating y = vx 3 we
dz 2 dz dx 2 dx
get
d 2z
dz −3 3 1
+P + . dy dv
dx 2 dx x 4 x x3 = 0 = x3 + 3x 2v
where P1 = = dx dx
2 2
 dz   dz 
    d 2y dv d 2v
 dx   dx  Again differentiating + 6x 2= x3+ 6xv
2
1 dx dx 2 dx
Q R 8 1 put these values in given equations we have
Q1 = 2
= a , R1 = = x = = −2z
2 2 1 x2
 dz   dz  d 2v dv dv
 
 dx 
 
 dx  x6 x5 + 6x 4 + 6x 3v + x 4 +3x 3v − 9x 3v = 0
dx 2 dx dx
d 2y
so, we get + a 2y = −2z d 2v dv d 2v 7 dv
dz 2 or x5 + 7x 4 =0 ⇒ + =0
dx 2 dx dx 2 x dx
⇒ (D 2 + a 2) y = −2z
i. e.  D2 + 7 D v = 0
30. Since y = A cos 2x + B sin 2x be the complete  
 x 
primitive of (D 2 + 4) y = 4 tan 2x
37. Given y = v sec x
Here u = cos 2x, v = sin 2x so Wronskian W is dy dv
so = sec x + sec x tan xv
u v cos 2x sin 2x dx dx
W= = =2
u1 v1 −2 sin 2x 2 cos 2x
55
dA dB
d 2y d 2v dv − A cos x − B sin x − sin x + cos x
again = sec x + 2 sec x tan x dx dx
dx 2 dx 2 dx
+ A cos x + B sin x = x
+ (sec3 x + sec x tan 2 x) v dA dB
i. e. − sin x + cos x =x ...(3)
put these values in dx dx
d 2y dy
− 2 tan x + 5y = e x sec x Multiplying (2) by cos x and (3) by sin x and
2 dx
dx Subtracting we get
we have (D 2 + 6) v = e x dA
= − x sin x
dx
d 2y dy
40. Given + 2 sin 2 xy
+ (3 sin x − cot x) Integrating A = x cos x − sin x + c
dx 2 dx
= e − cos x sin 2 x 50. The complete primitive is y = Ax + Bxe 2x
Here, P = 3 sin x − cot x, Q = 2 sin 2 x, Differentiate w.r.t.x,
dy dA
R = e − cos x sin 2 x = A + B (e 2x + 2xe 2x ) + x + xe 2x
dx dx
Choose Q1 such that
Choose A and B such that
Q 2 sin 2 x dA dB
Q1 =
2
=
2
= Constant = 2 x + xe 2x =0 ...(1)
 dz   dz  dx dx
    dy
 dx   dx  Then = A + B (1 + 2x) e 2x
dz dx
∴ = sin x
dx d 2y dA dB 2x
and = + e (1 + 2x)
i. e., z = − cos x dx 2 dx dx
R e − cos x sin 2 x +2Be 2x + 2B (1 + 2x) e 2x
So, R1 = =
2
 dz  sin 2 x Put these values in given equation
 
 dx  d 2y 2 (1 + x) dy 2 (x + 1)
− + y=x
= e − cos x = e z dx 2 x dx x2
dA dB
42. Given that [3x 2D 2 + (2 + 6x − 6x 2) D − 4] y = 0 We get, + e 2x (1 + 2x) =x ...(2)
dx dx
or [(3x 2D 2 + 6xD + 2D) − (6x 2D + 4)] y = 0 dA 1
Solving (1) and (2), we get =−
dx 2
or [D (3x 2D + 2) − 2 (3x 2D + 2)] y = 0 1
⇒ A= − x + c
or (D − 2) (3x 2D + 2) y = 0 2
d 2y
2 dy  2
44. Given that [xD 2 + (x − 1) D − 1] y = x 2 52. Given equation is − + 1 +  y=0
dx 2
x dx  x2
or [xD(D + 1) − (D + 1)] y = x 2 2 2
Here P = − , Q = 1+ ,R= 0
or (xD − 1) (D + 1) y = x 2 x x2
1 1 2
− ∫ Pdx − ∫ − dx
47. Differentiate y = A cos x + B sin x ...(1) Choose u = e 2 =e 2 x =x
dy dA dB
= − A sin x + B cos x + cos x + sin x Put y = uv in given equation it reduces to normal for
dx dx dx
d 2v
Choose A and B such that + Xv = Y
dx 2
dA dB
cos x + sin x =0 ...(2) 1 dP P 2 2
dx dx Where X = Q − − = 1+
2 dx 4 x2
dy
Then = − A sin x + B cos x 1 2  1 4
dx −  − . =1
2 x2 4 x2
d 2y dA dB 1
so, = − A cos x − B sin x − sin x + cos x
dx 2 dx dx ∫ Pdx
and R = R e2 = 0, so we get
Put these value is given differential equation d 2v
+ v = 0 ⇒ (D 2 + 1) v = 0
(D 2 + 1) y = x we have dx 2
56

54. Given y = A cos x + B sin x be the solution of Choose A and B such that
d 2y x
dA 1 dB
+ =0 ...(2)
+ y = x , then
dx 2 dx x dx
dy dA dB dy B
= − A sin x + B cos x + cos x + sin x then = A−
dx dx dx dx x2
Choose A and B such that d 2y dA 1 dB 2
and = − + B
2 dx x dx x 3
2
dA dB dx
cos x + sin x =0 ...(1)
dx dx Putting these values in equation (1) we get
dy
= − A sin x + B cos x
dA 1 dB
dx − = ex ...(3)
d 2y dA dB dx x 2 dx
⇒ = − A cos x − B sin x − sin x + cos x
dx 2 dx dx
Solving (2) and (3) we have
dA 1 x
= e
Put these values in given differential equation, we get dx 2

dA dB 1 x
− sin x + cos x =x ...(2) Integrating A = e +c
dx dx 2
dB d 2y dy
Solving (1) and (2) we have = x cos x 63. Given + 2 tan x + (1 + 2 tan 2 x) y
dx dx 2 dx
So, B = x sin x + cos x + c = sec x tan x

d 2y 2 dy Here, P = 2 tan x, Q = 1 + 2 tan 2 x, R = sec x tan x


56. Given + + n2y = 0
dx 2 x dx 1 1
∫ Pdx ∫ 2 tan xdx
2 Y = R e2 = sec x tan xe 2
Here, P = , Q = n2, R = 0
x
Y = sec x tan x. elog sec x = sec2 x tan x
1 1
− ∫ Pdx −∫ dx 1 1
Choose u = e 2 =e x = 65. Given  D 2 − D + 4x 2  y = x 4
x  x 
Put y = uv in given equation, it reduces to its normal 1
Here, P = − , Q = 4x 2, R = x 4
form x
d 2v Q 4x 2
+ Xv = Y Choose Q1 = = = constant = 1
dx 2  dz 
2
 dz 
2
   
1 dP P 2  dx   dx 
Where X = Q − − = n2
2 dx 4 dz
= 2x ⇒ z = x2
1 1  1 4 dx
− −  − . = n2
2 x2 4 x2 Put z = x 2, now equation is
d 2y dy
58. If y = ex is a part of C.F. of + P1 + Q1y = R1
dz 2 dz
(D 2 − cot xD − Q) y = e x sin x then 1 + P + Q = 0
d 2z
dz  − 1  2x
+P 2+  
⇒ 1 − cot x − Q = 0 ⇒ Q = 1 − cot x 2 dx  x
Where P1 = dx = =0
2
B  dz  4x 2
60. Given y = Ax + be the general solution of  
x  dx 

(x 2D 2 + xD − 1) y = x 2e x ...(1) d 2y dy
66. Given − cot x − y sin 2 x = 0
dx 2 dx
Differentiating we get
Q − sin 2 x
dy B dA 1 dB Choose Q1 = = = constant = −1
= A− +x + 2 2
dx x 2 dx x dx  dz   dz 
   
 dx   dx 
57

dz −1
So, = sin x, z = − cos x given equation reduces to 1 1  3D D 2 
 5
dx P.I. .5 = 1+ + .5 =
(D 2 + 3D + 2) 2  2 2  2

d 2y dy
+ P1 + Q1y = R, where
dz 2 dz 5
∴ Solution is x = c1e − t + c 2e −2t +
2
d 2z dz 5
+P so, lim x =
2 dx cos x − cot x.sin x
P1 = dx = =0 t→ ∞ 2
2
 dz sin 2 x
  d 2y 2 dy  2
 dx  71. Given equation is − + 1 +  y=0
dx 2 x dx  x2
70. (D 2 + 3D + 2) x = 5
2 2
Here, P = − , Q = 1 + ,
A.E. is m 2 + 3m + 2 = 0 x x2
−3 ± 9− 8 −3 ± 1 1 dP P 2
⇒ m= = So, X = Q − −
2 2 2 dx 4
∴ m = −1, − 2
2 1 2  4
so C.F. is c1e − t + c 2e −2t = 1+ −  − =1
x2 2  x 2  4x 2

mmm
58
Unit-II
C HAPTER
8 Partial Differential Equations of the First Order
PARTIAL DIFFERENTIAL EQUATIONS f (x , y, z , p, q) = 0
1. Equations which contains one or more partial which is the required partial differential
derivatives are called partial differential equations of first order.
equations. Thus the partial differential equation 2. Elimination of Arbitrary Functions
contains at least two independent variables. Consider f (u, v) = 0 ...(1)
∂z ∂z ∂ 2z where u and v are functions of x , y and z.
We shall denote = p, = q, =r
∂x ∂y ∂x 2 Differentiating (1) partially w.r.t. independent
∂ 2z ∂ 2z variables x and y regarding z as dependent
= s and =t variable, we get
∂ x∂ y ∂y 2
∂f  ∂u ∂u  ∂f  ∂v ∂v 
 +p  +  + p  = 0 ...(2)
2. The order of a partial differential equation is the ∂u  ∂x ∂z  ∂v  ∂x ∂z 
order of the highest order derivative occurs in ∂f  ∂u ∂u  ∂f  ∂v ∂v 
the differential equation.  +q  +  +q  =0 ...(3)
∂u  ∂y ∂z  ∂v  ∂y ∂z 
3. The degree of a partial differential equation is ∂f ∂f
Eliminating and between(2) and (3) we get
the degree of the highest order derivative which ∂u ∂v
occur in it after the differential equation has  ∂u ∂v ∂v ∂u   ∂v ∂u ∂u ∂v 
p −  + q − 
been rationalised i. e. made free form redicals  ∂y ∂z ∂y ∂z   ∂x ∂z ∂x ∂z 
and fractions so far as derivatives are ∂u ∂v ∂u ∂v
= . − .
concerned. ∂x ∂y ∂y ∂x
i. e. Pp + Qq = R ...(4)
DERIVATION OF A PARTIAL DIFFERENTIAL ∂u ∂v ∂v ∂u
EQUATION where, P= −
∂y ∂z ∂y ∂z
The partial differential equation can be formed by two
∂v ∂u ∂u ∂v
methods Q= −
∂x ∂z ∂x ∂z
1. Elimination of arbitrary constants ∂u ∂v ∂u ∂v
Consider the equation and R= −
∂x ∂y ∂y ∂x
f (x , y, z , a, b) = 0 ...(1)
Equation (4) is the required partial differential
Where a, b are arbitrary constants and let z be equation.
the function of x and y
Differentiating equation (1) partially w.r.t. x CLASSIFICATION OF PARTIAL DIFFERENTIAL
and y respectively we get
EQUATIONS OF FIRST ORDER
∂f ∂f 1. A first order partial differential equation
+p =0 ...(2)
∂x ∂z f (x , y, z , p, q) = 0 in which dependent variable
∂f ∂f is z which is a function of two independent
and +q =0 ...(3)
∂y ∂z variables x and y, is called a linear if the
Eliminating two constants a and b from dependent variable z and its partial differential
∂z ∂z
equations, (1), (2) and (3) we obtained coefficients and all occur in first degree.
∂x ∂y
59

2. A first order partial differential equation in 3. The general solution of the linear partial
which dependent variable is z which is a differential equation
function of two independent variables x and y, Pp + Qq = R ...(1)
is called a non-linear equation if p and q do not If f (u, v) = 0 where f is an arbitrary function
occur in it first degree. and u(x , y, z ) = c1 and v(x , y, z ) = c2 form a
3. A first order partial differential equation in solution of the equations
which dependent variable is z which is a dx dy dz
= =
P Q R
function of two independent variables x and y,
is called a quasi-linear equation if f is a linear 4. If the linear equation with n independent
expression in p and q but not necessarily linear variables x 1, x 2 , ..., x n be
in z. P1p1 + P2p 2 + .... Pnp n = R ...(1)

4. A first order partial differential equation in Where P1, P2 , ...., Pn and R are functions of
which dependent variable is z which is a x 1, x 2 , ...., x n and z. Then the general solution
function of two independent variables x and y of (1) is given by f (u 1, u 2 , ..., u n) = 0
is called a semi-linear equation if two Where ui (x 1, x 2 , ..., x n, z ) = ci , i = 1, 2, ..., n are
coefficient of p and q are function of x and y any independent integrals of the auxiliary
only and they do not depend on z. The terms dx 1 dx 2 dx dz
equations = = ... 4 =
that do not involve p and q contains the terms P1 P2 Pn R
that are not of first degree in z.
5. The general solution of Pp + Qq = R is the
LAGRANGE’S LINEAR PARTIAL DIFFERENTIAL family of surfaces such that the normal to a
EQUATION
surface at any point must be perpendicular to
1. The partial differential equation Pp + Qq = R,
the tangent to a curve of the family of the
where P, Q and R are functions of x , y and z is dx dy dz
called Lagrange’s linear partial differential equations = =
P Q R
equation of first order.
6. The equation Pp + Qq = R represents a family
2. Consider Lagrange’s partial differential
equation of surfaces orthogonal to the family of surfaces
Pp + Qq = R ...(1) represented by Pdx + Qdy + Rdz = 0.
Let u = a be an integral of (1) 7. For the solution of Lagrange’s equation
Differentiating partially it w.r.t. x and y we get Pp + Qq = R form the auxiliary equations
∂u ∂u ∂u ∂u dx dy dz
+ p = 0 and +q =0 = =
∂x ∂z ∂y ∂z P Q R
∂u ∂u Find two independent integrals say u = a and
∂y v = b then the general solution of the partial
i. e. p = − ∂x , q = −
∂u ∂u differential equation Pp + Qq = R is given by
∂z ∂z f (u, v) = 0 , when f is an arbitrary function.
Put these values in (1) we get
∂u ∂u ∂u
T H E I N TE G R A L S O F TH E N O N - L I N EA R
P +Q +R =0 ...(2) EQUATIONS
∂x ∂y ∂z
1. Any relation which contains as many arbitrary
Thus if u = a satisfies (1) it also satisfies (2). constants as there are independent variables
Conversely if u = a is in integral of (2) then it is and is a solution of a partial differential
also an integral of (1). Hence (1) and (2) are equation of the first order is called a complete
equivalent. solution or a complete integral of that equation.
60

If f (x , y, z , a, b) = 0 ...(1) Put p = a and q = b in (3), we get equation (1)


be the solution of f (x , y, z , p, q) = 0 ...(2) By equation (3), let b = φ (a)
Then (1) is complete solution of complete So complete integral of (1) is
integral of (2). z = ax + φ(a). y + c ...(4)
General Integral :
2. A particular integral of (2) can be obtained by
Put c = ψ (a) in (4) we get
given particular values to a and b in (1).
z = ax + φ(a) y + ψ (a) ...(5)
3. The equation of the envelope of the surfaces Differentiating it partially w.r.t. a we get
represented by (1) can be obtained by 0 = x + φ ′ (a)y + ψ ′ (a) ...(6)
eliminating a and b between
The general integral is obtained by eliminating
∂f ∂f
f = 0, = 0 and =0 a between (5) and (6).
∂a ∂b
Singular Integral : The singular solution (integral) is
This equation of the envelope is called the obtained by eliminating a and c between the complete
singular integral. integral (4) and the equations formed by differentiating
(4) partially w.r.t. a and c i. e.
CHARPIT’S METHOD z = ax + φ(a) y + c
1. Charpit’s method is the general method for 0 = x + φ ′ (a) y
solving partial differential equations of order 0=1
one but of any degree. which is not exist so there is no singular solution.
Consider the differential equation 2. Equation involving only p, q and z i. e.
f (x , y, z , p, q) = 0 ...(1) f ( p, q, z ) = 0 ...(1)
with dz = pdx + qdy ...(2)
Assume that z = f (x + ay) be the trial solution
The Charpit’s auxiliary equations are
dp dq dz of (1) then
= =
∂f ∂f ∂f ∂f ∂f ∂f z = f ( X ) where X = x + ay
+p +q −p −q
∂x ∂z ∂y ∂z ∂p ∂q ∂z dz ∂X dz ∂X
∴ p= = = ∵ =1
dx dy ∂f ∂x dX ∂x dX ∂x
= = = ...(3)
∂f ∂f 0 ∂z dz ∂X dz
− − and q= = . =a
∂p ∂q ∂y dX ∂y dX
Any two of equation (3) give a relation of p and  dz dz 
So, equation (1) reduces to f  z , ,a  =0
q which is solved for p, q with equation (1).  dX dX 
Substitute these values in equation (2) and then Which is an ordinary differerntial equation of
integrating it, we get the complete solution of order one. Integrating it we get the complete
the given partial differential equation. integral.
3. Equation of the form f (x, p) = φ (y, q).
SPECIAL METHODS OF SOLUTION APPLICABLE
TO CERTAIN STANDARD FORMS Consider f (x , p) = φ (y, q) = a ...(1)

1. Equations involving only p and q and not x , y, z . Then we get p = f 1(x , a), q = f 2 (y, a)
Let the equation is f ( p, q) = 0 ...(1) We know dz = pdx + qdy
The complete integral is z = ax + by + c ...(2) so, dz = f 1(x , a) dx + f 2 (y, a) dy
Where a and b are connected by Integrating we get
f (a, b) = 0 ...(3) z=
By (2),
∫ f1(x , a) dx + ∫ f2(y, a) dy + b
∂z ∂z Which is the complete integral. There is no any
p= = a and q = =b
∂x ∂y singular integral.
61

4. Equation of the form f (x , y, z , p, q) = 0 ...(1)


z = px + qy + f ( p, q) ...(1) is also a solution of the first order partial
The complete integral of Clairaut’s equation (1) differential equation
is φ (x , y, z , p, q) = 0 ...(2)
z = ax + by + f (a, b) Then the differential equations (1) and (2) are
∂F ∂f said to be compatible.
∴ =0 ⇒ x+ =0 ...(2)
∂a ∂a
∂F ∂f 2. The condition that the equation (1) and (2) are
and =0 ⇒ y+ =0 ...(3)
∂b ∂b compatible is
Singular integral is obtained by eliminating a ∂( f , g ) ∂( f , g ) ∂( f , g ) ∂ (f , g)
+p + +q =0
and b from (1), (2) and (3). ∂(x , p) ∂(z , p) ∂(y, q) ∂ (z , q)

3. The first order partial differential equations


COMPATIBLE SYSTEMS OF FIRST ORDER
PARTIAL DIFFERENTIAL EQUATIONS p = P(x , y), q = Q(x , y) are compatible if and
1. If every solution of the first order partial ∂P ∂Q
only if =
differential equation ∂y ∂x

EXERCISE
MULTIPLE CHOICE QUESTIONS 8.
y
The differerntial equation of z = f   is :
x
1. The partial differential equation of z = (x + a) (y + b)
(a) px + qy = 0 (b) pq + xy = 0
is :
(c) py + qx = 0 (d) pq = x + y
(a) z = p + q (b) z = px + qy
(c) z = xyp (d) z = pq 9. The differential equation of z = (x 2 + a) (y 2 + b) is :

2. The partial differential equation of z = a (x + y) + b (a) pq = xyz (b) q = px + y


(c) p = qy + z (d) pq = 4xyz
is :
(a) pq = 0 (b) p = q 10. The differential equation of z = f (x 2 + y 2) is :
(c) p + q = 0 (d) p − q = 0 (a) xp = yq (b) xp 2 = yq2
3. The order of r − 2s + p 3 = 0 is : (c) yp 2 = xq2 (d) yp = xq
(a) 1 (b) 2 (c) 3 (d) 0 11. The auxiliary equation of Pp + Qq = R is :
4. 3 2
The degree of r + 2s + t = 0 is : (a) Pdx + Qdy + Rdz = 0
(a) 1 (b) 2 (b) Pdp + Qdq + Rdz = 0
dp dq dz dx dy dz
(c) 3 (d) None of these (c) = = (d) = =
P Q R P Q R
5. The differential equation zyp − xyq = x (z − 2y) is : 12. The order of the differerntial equation
(a) Linear (b) Quasi-linear ∂ 2z ∂ 2x
+ u2 = f (x, y) is :
2
(c) Semi-linear (d) Non-linear ∂x ∂y 2
6. The differential equation pq = z is : (a) 1 (b) 2
(a) Linear (b) Non-linear (c) 3 (d) None of these
(c) Semi-linear (d) Quasi linear 13. The solution of z(xp − yq) = y 2 − x 2 is :
The differential equation x 2p + (y 3 + x 3) q = x 2 x 
7. (a) f  , x 2 + y 2 + z 2  = 0
y 
+ y 2 + z 2 is :
(b) f (xyz, x 2 + y 2 + z 2) = 0
(a) Linear (b) Semi-linear
(c) f (yz, x 2 + y 2 + z 2) = 0
(c) Non-linear (d) Quasi-linear
(d) f (xy, x 2 + y 2 + z 2) = 0
62

14.
y− z
The solution of   z − x  q = x − y is : 23. The solution of px 2 + qy 2 = z 2 is :
 p+  
 yz   zx  xy 1 1  1 1
(a) − = f  −  (b) y − z = f (x − y)
(a) f (x + y + z, xyz) = 0 y z  x y
(b) f (x − y − z, xy) = 0 1 1  1 1
(c) + = f +  (d) y + z = f (x + y)
1 1 1  y z  x y
(c) f  + + , xyz  = 0
x y z 
24. The solution of z(px + qy) = y 2 − x 2 is :
 1 
(d) f  , xyz  = 0
x + y+ z  (a) f (xyz, x 2 + y 2 + z 2) = 0

z (b) f (xyz, x + y + z) = 0
15. The solution of p + q = is : (c) f (xy, x + y + z) = 0
a
a x 
y (d) f  , − x 2 − y 2 + z 2  = 0
(a) z = e y f (x + y) (b) z = e a f (x − y) y 
(c) z = e − y f (x − y) (d) z = e y f (x − y) 25. The solution of − p1 + p 2 + p 3 = 1 is :
16. The solution of p tan x + q tan y = tan z is : (a) f (x1 + x 2, x1 + x 3, x1 + y) = 0
sin y sin x  sin z  sin x  (b) f (x1 + x 2, x1 + x 3, x1 + z) = 0
(a) = f   (b) = f   (c) f (x1 + x 2, x1 + x 3, x1 + x) = 0
sin z  sin z  sin x  sin y 
(d) f (x1 + x 2, x 2 + x 3, x 3 + x1) = 0
sin z  sin x  sin z  sin z 
(c) = f   (d) = f  
sin y  sin y  sin y  sin y  26. The family of surfaces orthogonal to the family of
surfaces of (y + z) p + (z + x) q = x + y is :
17. The solution zy 2p − zx 2q = x 2y is : (a) xyz + c = 0 (b) xy + yz + zx = c
(a) f (x 3 + y 3, y 2 + z 2) = 0 (c) x + y + z = 0 (d) x 2 + y 2 + z 2 = 0
(b) f (x 3 + y 3 + z 3, xyz) = 0 27. The partial differential equation from z = (a + x)2 + y
2 2 2 is :
(c) f (x + y + z , xyz) = 0
p p2
(d) f (x + y + z, xyz) = 0 (a) z = +y (b) z = +y
2 4
18. The complete integral of 3p 2 = q is : q2 q
(c) z = +y (d) z = + y
(a) z = ax + ay + b (b) z = ax + 3a 2y + b 4 2
a p
(c) z = ax + + b (d) z = ax + a 2y + b −
y 28. The complete integral of q = e a is :
b b

19. The complete integral of xp + yq = z is : (a) z = ye a +c (b) z = bx + ye a + c
b b
(a) f (xy, yz) = 0 (b) f (x, y) = 0 − − x
(c) z = bx + ye a +c (d) z = ye a +c
 x y
(c) f (xy, z) = 0 (d) f  ,  = 0
 y z 29. The complete integral of p + q = 1 is :
2
(a) z = ax + (1 − a) y + b
20. The singular integral of p 2 + q2 = 1 is :
(b) z = ax + ay + b
(a) z = 0 (b) x = 0 (c) z = ax + (1 + a) 2 y
(c) y = 0 (d) Not exist (d) z = ax + (1 − a) y + b
21. The solution of p 2 = zq is : 30. The complete integral of p (1 + q) = qz is :
2 2
(a) z = be x + ay
(b) z = be ax + xy (a) az = 1 + be ax + y (b) az = be ax + y
x + ay
(d) az = be x + ay
2
(c) z = be ax + a y
(d) z = be axy + b (c) az = 1 + be

22. The complete integral of z = px + qy + p 2 + q2 is : 31. The differential equations p = 4x + 3y + 1 and


q = 3x + 2y + 1 are :
(a) z = ax + by + a 2 + b2 (b) z = px + qy + a 2 + b2
(a) Exact (b) Compatible
(c) z = ax + by + ab (d) z = ax + by + a + b (c) Not compatible (d) Non-linear
63

32. The equations f (x, y, p, q) = 0 and g(x, y, p, q) = 0 are 42. The differential equations p = P(x, y) and q = Q(x, y)
compatible if : are compatible iff :
∂f ∂f ∂f ∂g ∂f ∂g ∂P ∂Q ∂P ∂Q
(a) = (b) + = + (a) = (b) =
∂x ∂y ∂x ∂x ∂y ∂y ∂x ∂y ∂y ∂x

(c)
∂ ( f , g) ∂ ( f , g)
+ = 0 (d)
∂ ( f , g) ∂ ( f , g)
= ∂ 2P ∂ 2P ∂ 2P ∂ 2P
(c) = (d) =
∂ (x, p) ∂ (y, q) ∂ (x, p) ∂ (y, q) ∂x 2
∂y 2
∂y 2
∂x 2
33. The differential equations p = 7 x + 8y − 1 and 43. The partial differential equation of
q = 3x + 5y − 6 are : z = f (x + ay) + φ (x − ay) is :
(a) Exact (b) Compatible ∂ 2z ∂ 2z ∂ 2z ∂ 2z
(a) = a2 (b) + = a2
2 2 2
(c) Non-linear (d) Not compatible ∂x ∂y ∂x ∂y 2
34. The equation Pp = Qq = R is called : ∂ 2z ∂ 2z ∂ 2z ∂ 2z
(c) − = a2 (d) a 2 =
2 2 2
(a) Charpit’s equation (b) Bernoulli’s equation ∂x ∂y ∂x ∂y 2
(c) Lagrange’s equation (d) Monge’s equation 44. The Charpit’s auxiliaries equations of
p(1 + q2) = q (z − b) is :
35. The differential equations p = 2x − y, q = 5 − x − 2y
dp dq dp dq
are : (a) = (b) =
p 2 q2 q p
(a) Compatible (b) Non-compatible
dp dq dp dq
(c) Exact (d) Non-linear (c) = (d) =
zp bz z pq
36. The singular integral of z = px + qy + log pq is : 45. The Charpit’s auxiliary equation of px + qy = pq is :
(a) z = 2 + log xy (b) z = qx + log y dx dy dx dy
(a) = (b) =
x + p y+ q x −p y−q
(c) z = qy + log x (d) z = −2 − log xy
dx dy dx dy
(c) = (d) =
37. The singular integral of z = px + qy + p 2 + q2 is : q+ x p + y q− x p − y
(a) x 2 + y 2 (b) −(x 2 + y 2) 46. The Charpit’s auxiliary equation of − px 2 + 2zx
 x 2 + y2   x 2 + y2 
(c) −   (d) −   −2qxy + pq = 0 is :
 2   4 
  dp dx dq dx
(a) = (b) =
38. The singular integral of z = px + qy − p 2q is : 0 x2 − q 0 x2 − q
x y dq dx dq dx
(a) z = (b) z = x y (c) = (d) =
2 0 x2 − q 0 x
xy
(c) z = (d) z = xy 47. The Charpit’s auxiliary equation of p 2 + q2 − 2px
2 −2qy + 1 = 0 is :
39. The differential equation of z = ax + a 2y 2 + b is : dp dq dp dq
(a) = (b) =
x y p q
(a) q = 2yp 2 (b) p = 2xq2 dp dq dp dq
(c) = (d) =
(c) q = 2 xyp (d) p = 2xyq q p y x
40. The differential equation of z = Ae pt sin px is : 48. The Charpit’s auxiliary equation of p = (qy + z)2 is :
dp dq dp dx
∂ 2z ∂ 2z ∂ 2z ∂ 2z (a) = (b) =
(a) + =0 (b) + =0 p q p x
2 2 2
∂y ∂t ∂x ∂t 2
dp dy dp dy
(c) = (d) =
∂ 2z ∂ 2z ∂ z ∂ 2z p y p −y
(c) + =0 (d) + =0
2 2 ∂x ∂y 2
∂x ∂y 49. The complete integral of p 2 − 3q2 = 5 is :
41. The differential equation (x + 2y) p + xp − pq = x + y (a) z = 5 − 3a 2 x + ay + b
is : (b) z = 5 x + ay + b
(a) Linear (b) Semi-linear
(c) z = 3a 2 x + ay + b
(c) Quasi-linear (d) Non-linear
(d) z = 5 + 3a 2 x + ay + b
64

50. The complete integral of p 2q2(px + qy − z) = 2 is : 57. The Charpit’s auxiliary equations of p 2 + q2 − 2px
2 −2qy + 2xy = 0 are :
(a) z = ax + y − 2a 2b2 (b) z = ax + by −
2 2 dp dq dx
a b (a) = =
2 y − p q− x p − x
(c) z = ax + by + 2 ab (d) z = x + y −
a 2b2 dp dq dx
(b) = =
51. The Charpit’s equation of (p + q ) x = pz is : 2 2 x −p y−q p+ x

dp dq dp dq dp dq dx
(a) = (b) = (c) = =
−q 2 pq p 2 pq y + p q+ x p + x
dp dq dp dq dp dq dx
(c) = (d) = (d) = =
p q2 q 2
p 2 x + p y+ q p−x

52. The Charpit’s equation of px + qy + z − xq2 = 0 is : 58. The complete integral of pe y = qe x is :

(a)
dx dq
= (b)
dx − dq
= (a) z = ae x + be y
x q 2x q
(b) az = e x + be y
dx dq dx dq
(c) = (d) =−
x 2 q2 x 2q (c) bz = ae x + e y
(d) z = a (e x + e y ) + b
53. The Charpit’s auxiliary equation of p 2x + q2y = z
is : 59. Lagrange’s auxiliary equations of z (xp − yq)
dp dq dx = y 2 − x 2 are :
(a) = =
p2 − p q2 − q −2px dx dy dz
(a) = =
x − y y2 − x 2
dp dq dx
(b) = = dx dy dz
2 2 −1px (b) = =
p +p q −q
x y y − x2
2
dp dq dx dx dy dz
(c) = = (c) = =
2 2 −2px
p −p q +q xz − yz y 2 − x 2
dp dq dx dx dy dz
(d) = = (d) = =
2 2 2px xz yz y 2 − x 2
p −p q −q
54. The complete integral of py = 2xy + log q is : 60. Lagrange’s auxiliary equations of Pp + Qq = R are :
2
(a) z = x + ax + log y + b dx dy dx dy dz
(a) = = dz (b) = =
P Q Q P R
(b) z = 2xy + log b
dx dy dz dx dy dz
e ay (c) = = (d) = =
(c) z = x 2 + ax + +b P Q R R Q P
a
(d) z = x 2 + x + e ay + b 61. The degree of the equation p 4 + q2x 6 + z 6 = 0 is :
q (a) 6 (b) 4
55. The complete integral of z = px + qy + − p is :
p (c) 2 (d) 1
(a) z = ax + by + a − b
(b) az = a (ax + by) + b − a 2 62. The order of the equation r 2 − 2s + t 3 = 0 is :
(a) 3 (b) 2
b2 − a 2
(c) z = ax + by + (c) 1 (d) None of these
a
(d) z = ax + by + a + b 63. If in the Lagrange’s auxiliary equations of
56. The complete integral of pq = xy is : (x + 2z) p + (4 zx − y) q = 2x 2 + y,
(a) az = x 2 + y 2 + 2ab
whose y, x, − 2z as multipliers then :
(b) 2az = ax 2 + by 2 (a) x 2 − y − z = c (b) x 2 − yz = c
2 2 2
(c) 2az = a x + y + 2ab
(c) xz − y 2 = c (d) xy − z 2 = c
2 2
(d) az = ax + by + ab
65

64. If in the Lagrange’s auxiliary equations of (a) 4 (b) 3


2 2 2 2 (c) 2 (d) 1
x (y + z) p − y (x + z) q = z (x − y ),
1 1 1 72. The solution of zp + x = 0 is :
choose , , as multiplier then :
x y z (a) x 2 + y 2 = f (z)
(a) x + y + z = c (b) xyz = c (b) y 2 + z 2 = f (x)

(c)
xy
=c (d)
xz
=c (c) x 2 + y 2 + z 2 = f (xyz)
z y
(d) x 2 + z 2 = f (y)
65. If in the Lagrange’s auxiliary equations of 2y(z − 3) p
1 73. The Charpit’s auxiliary equation of
+ (2x − z) q = y (2x − 3), choose , y, − 1 as
z z = px + qy + p 2 + q2 is :
multipliers then :
dp dq dp dq
(a) x + y 2 − 2z = c (b) y 2 + x − 2z = c (a) = (b) =
2 2 p q
p +p q +q
(c) z + x 2 − 2y = c (d) y + z 2 − 2x = c dp dq
(c) pdp = qdq (d) =
66. The Charpit’s auxiliary equations of px + qy 0 0

= z (1 + pq)1/ 2 are : 74. The singular integral of p + q =1 is :


dp dq dp dq (a) x = 0 (b) y = 0
(a) = (b) = (c) x + y = 0 (d) Not exist
p−q p+ q p2 q2
dp dq dp dq 75. The partial differential equation of az = xy is :
(c) = (d) =
p q 2 2 2 2 (a) px = z (b) qx = z
p −q p +q
(c) py = z (d) px = qy
67. The Charpit’s auxiliary equations of z 2 = pqxy are :
dx dy dx dy 76. The differential equation of z = f (xy) is :
(a) = (b) = (a) p = q (b) p + q = x + y
q p px qy
(c) py = qx (d) px = qy
dx dy dx dy
(c) = (d) = 77. The differential equation of z = f (x − y) is :
x y p q
(a) p + q = 0 (b) p = q
68. The Charpit’s auxiliary equations of p 2 + q2 − 2px
(c) p = x (d) q = y
−2qy + 2xy = 0 are :
1
dx dy dx dy 78. The differential equation of z = y 2 + 2 f  + log y
(a) = (b) = x 
x + q y+ q x −q y−q
is :
dx dy dx dy
(c) = (d) =
x2 y2 q p (a) px + qy = 2y 2 (b) px − qy = 2y 2

69. In a linear partial differential equation all the partial (c) p 2x + qy = y 2 (d) px 2 + qy = 2y 2
derivatives occuring in it are of degree : 79. The differential equation
(a) 1 (b) 2
(x 2 − yz) p + (y 2 − 2x) q = z 2 − xy is :
(c) 3 (d) Any finite value
(a) Quasi-linear (b) Non-linear
70. The order of the partial differential equation of
(c) Semi-linear (d) Linear
z = φ (x + iy) + ψ (x − iy) is :
1 2 2y
80. The differential equation z = axe y + a e + b is :
(a) 1 (b) 2 2
(a) px = z 2 (b) q = px + pq
(c) 3 (d) 4
2
(c) q = px + p (d) p = 2x + q2
71. The degree of the partial differential equation of
z = ax + a 2y 2 + b is :
66

ANSWERS
MULTIPLE CHOICE QUESTIONS
1. (d) 2. (b) 3. (b) 4. (c) 5. (b) 6. (b) 7. (b) 8. (a) 9. (d) 10. (d)
11. (d) 12. (b) 13. (d) 14. (a) 15. (b) 16. (c) 17. (a) 18. (b) 19. (d) 20. (d)
21. (c) 22. (a) 23. (a) 24. (d) 25. (b) 26. (b) 27. (b) 28. (c) 29. (a) 30. (c)
31. (b) 32. (c) 33. (d) 34. (c) 35. (a) 36. (d) 37. (d) 38. (b) 39. (a) 40. (b)
41. (d) 42. (b) 43. (d) 44. (b) 45. (d) 46. (b) 47. (b) 48. (d) 49. (d) 50. (b)
51. (a) 52. (d) 53. (a) 54. (c) 55. (b) 56. (c) 57. (a) 58. (d) 59. (c) 60. (c)
61. (b) 62. (b) 63. (d) 64. (b) 65. (a) 66. (c) 67. (a) 68. (b) 69. (a) 70. (b)
71. (c) 72. (d) 73. (d) 74. (d) 75. (a) 76. (d) 77. (a) 78. (d) 79. (a) 80. (c)

HINTS AND SOLUTIONS


1. Given that z = (x + a) (y + b) dx dy dz
= =
∂z zx − zy y 2 − x 2
Differentiate partially w.r.t. x, p = = y+ p
∂x dx dy
Taking first two we get + =0
∂z x y
again differentiate partially w.r.t. y, q = =x+a
∂y Integrating log x + log y = log c1
⇒ xy = c1
Eliminate a and b from the above equations we get
Chosing x, y, z as multipliers we get
z = pq xdx + ydy + zdz
i. e. xdx + ydy + zdz = 0
3. The partial differential equation is 0
3 Integrating we get
∂ 2z ∂ 2z  ∂ z 
−2 +   = 0 so its order is 2.
2 ∂ x∂ y  ∂ x  x 2 + y2 + z 2 = c 2
∂x
y So solution is f (xy, x 2 + y 2 + z 2) = 0
8. Given that z = f  
x z
15. Given p + q =
∂z y  y  a
p= = f ′   .  − 
∂x  x   x2 Lagranges auxiliary equations are
∂z y 1 dx dy dz
and q= = f ′   = =
z
∂y x x 1 1
a
Dividing these we get
By first and second dx = dy ⇒ x − y = c1
p y adz
=− ⇒ px + qy = 0 By second and third dy = = y = a log z + log c 2
q x z
10. Given that z = f (x 2 + y 2) ey
log c 2 = y − a log z = log e y − log z a = log
∂z za
p= = 2xf ′(x 2 + y 2) y/ a
∂x ⇒ z = c 3e
∂z So solution is z = e y / a f (x − y)
and q= = 2yf ′(x 2 + y 2)
∂y
17. Given zy 2p − zx 2q = x 2y
Dividing we get yp = xq
So Lagrange’s auxiliary equation is
13. Given differential equation is zxp − zyq = y 2 − x 2 dx dy dz
= =
So, Lagrange’s auxiliary equation is zy 2 − zx 2 x 2y
67
p
By first and second x 2 dx + y 2 dy = 0 −
a
28. Given q = e so solution is z = ax + by + c
Integrating we get x 3 + y 3 = c1 −
b
where d = e a
dx dy dz b
19. Lagrange’s auxiliary equation is = = −
x y z So solution is z = ax + ye a +c
By first and second log x = log y + log c1 30. Given p(1 + q) = qz
x dz adz
⇒ = c1 put p= ,q=
y dX dX
where X = x + ay
and by second and third log y = log z + log c 2
dz  dz   dz  z
y 1 + a  =a 
⇒ = c2 dX  dX   dX 
z dz
⇒ 1+ a = az
 x y dX
So solution is f  ,  = 0
 y z adz
= dX ⇒ log (az − 1) = X + log b
az − 1
20. Given p 2 + q2 = 1 ⇒ az − 1 = be X ⇒ az = 1 + be x + ay
2 2
i. e. f (p, q) = 0 so a + b = 1
36. Given equation is z = px + qy + log pq
So its solution is z = ax + by + c
This is Clairant’s equation so its solution is
i. e. z = ax + 1 − a 2 y + c z = ax + by + log ab ...(1)
differentiate w.r.t. a and c we get Differentiating w.r.t. a and b respectively we get
0 = x and 0 = 1 which is not possible so no singular b −1 −1
0= x+ ⇒ a= and b =
solution exist. ab x y
dz dz put in (1) we get singular solution is
21. Given p 2 = zq, put p = and q = a
dX dX 1
z = −1 − 1 + log = −2 − log xy
where X = x + ay xy
2
 dz  = az dz 38. Given that z = px + qy − p 2q
 
 dX  dX Its solution is z = ax + by − a 2b ...(1)
dz dz
⇒ = az ⇒ = aX Diff. partially w.r.t. a and b respectively we get
dX z
0 = x − 2ab and 0 = y − a 2
Integrating log z = aX + log b or z = be aX
x
2 i. e., a = y and b =
i. e., z = be a( x + ay) = be ax + a y
2 y
25. Given − p1 + p 2 + p 3 = 1 put these values in equation (1) we get the singular
solution is
Lagrange auxiliary equations are 1 x
z = x y+ x y−y =x y
dx1 dx 2 dx 3 dz 2 2 y
= = =
−1 1 1 z 40. Given that z = Ae pt sin px
By (i) and (ii) x1 + x 2 = c1 ∂z ∂z
then = Ape pt cos px and = Ape pt sin px
By (i) and (iii) x1 + x 3 = c 2 ∂x ∂t

By (i) and(iv) x1 + z = c 3 ∂ 2z ∂ 2z
= − Ap 2e pt sin px and = Ap 2e pt sin px
2
So solution is f (x1 + x 2, x1 + x 3, x1 + z) = 0 ∂x ∂t 2

26. The family of surfaces orthogonal to the family of ∂ 2z ∂ 2z


so, + =0
2
surface Pp + Qq = R is Pdx = Qdy + Rdz = 0 ∂x ∂t 2
so we have (y + z) dx + (z + x) dy + (x + y) dz = 0 43. Given z = f (x + ay) + φ (x − ay)
Integrating we get ∂z
then = f ′(x + ay) + φ′(x − ay)
xy + yz + zx = c ∂x
68

∂ 2z i. e. dz = (2x + a) dx + e ay dy
= f ′′(x + ay) + φ′′(x − ay)
∂x 2 Integrating we get
∂z e ay
Again = af ′(x + ay) − aφ′(x − ay) z = x 2 + ax + +b
∂y a
∂ 2z p y
= a 2 f ′′(x + ay) + a 2φ′′(x − ay) 56. Given pq = xy ⇒ = =a
∂y 2 x q

∂ 2z ∂ 2z y
So we get a 2 + =0 so p = ax and q =
a
∂x 2 ∂y 2
y
put in dz = pdx + qdy = axdx + dy
45. Given that f = px + qy − pq so Charpit’s equations a
dx dy Integrating we get 2az = a 2x 2 + y 2 + 2ab
= = .....
∂f ∂f
− −
∂p ∂q 58. Given pe y = qe x ⇒ pe − x = qe − y = a
dx dy so p = ae x and q = ae y
or = = ....
−(x − q) −(y − p)
put these values in dz = pdx + qdy = ae x dx + ae y dy
dx dy
or = Integrating we get z = ae x + ae y + b
q− x p − y
63. Given that (x + 2z) p + (4zx − y) q = 2x 2 + y
47. Let f = p 2 + q2 − 2px − 2qy − 1 = 0
Charpit’s auxiliary equations are If we choose y, x, − 2z as multipliers then we get
dp dq ydx + xdy − 2zdz
= = .... 0
∂f ∂f ∂f ∂f
+p +q
∂x ∂z ∂y ∂z Since Lagrange’s auxiliary equations
dx dy dz
dp dq = =
or = = .... x + 2z 4zx − y 2x 2 + y
−2p + p.0 −2q + q.0
dp dq i. e. xdy + ydx − 2zdz = 0
or =
p q Integrating xy − z 2 = c

50. Given that p 2q2(px + qy − z) = 2 65. The Lagrange’s auxiliary equations are
2 dx dy dz
or z = px + qy − which is a Clairauts equation = =
p 2q2 2y (z − 3) 2x − 2 y (2x − 3)

so its solution is 1
If we choose , y, − 1 are multipliers we get
2 2
z = ax + by −
a 2b2 1
dx + ydy − dz
2
52. Given that f = px + qy + z − xq2 yz − 3y + 2xy − yz − 2xy + 3y
Charpit’s auxiliary equation is 1
So we get dx + ydy − dz = 0
dp dq dx dy 2
= = = = ....
∂f ∂f ∂f ∂f ∂f ∂f
+p +q − − Integrating we get x + y 2 − 2z = c
∂x ∂z ∂y ∂z ∂p ∂q
dq dx 67. Given that f = z 2 − pqxy
or =
q + q.1 − x Charpit’s auxiliary equation are
dq dx dx dq ∂x ∂y
or =− ⇒ =− = = ....
2q x x 2q ∂f ∂f
− −
54. Given that yp = 2xy + log q ∂p ∂q
1 dx dy
or p − 2x = log q = a so we get = = ....
y qxy pxy
⇒ p = 2x + a and q = e ay dx dy
or =
so put these value in dz = pdx + qdy q p
69

71. Given differential equation is z = ax + a 2y 2 + b 74. Given p + q = 1


∂z ∂z
= a = p and = 2a 2y = q so its solution is z = ax + by + c
∂x ∂y
where a + b = 1
Eliminate a between p = a and q = 2a 2y we get
i. e. b = 1 − a
q = 2p 2y so its degree is 2.
So required solution is z = ax + (1 − a) y + c
72. Given zp + x = 0 its Lagrange’s equation is
Integrating w.r.t. c we have 0 = 1
dx dy dz
= =
z 0 −x so not S.S. exist.
By second term dy = 0 ⇒ y = c1 77. z = f (x − y)
and by first and third term xdx + zdz = 0 Differentiating w.r.t. x and y respectively
2 2
Integrating x + z = c 2 p = f ′(x − y) and q = − f ′(x − y)
2 2
so solution is x + z = f (y) so, p+ q= 0
mmm
70
Unit-II
C HAPTER
9
Linear Partial Differential Equation of Second
and Higher Order with Constant Coefficients
LINEAR PARTIAL DIFFERENTIAL EQUATION functions is called the complementary function
1. A partial differential equation in which the of the partial differential equation.
dependent variable and its derivatives appear 2. consider f (D, D ′ )z = 0
only in the first degree and are not multiplied i. e., (D n + A1D n− 1D ′+ A2 D n− 2 D ′ 2
together, their coefficients all being constants or
functions of x and y is called linear partial + ... + AnD ′ n) z = 0 ...(1)
differential equation. Then auxiliary equation (A.E.) is
For example, m n + A1m n− 1 + ... + An = 0 ...(2)
n n n
∂ z ∂ z ∂ z It contains n roots say m1, m2 , ... mn
+ A1 + ... An + ....
∂x n ∂x n− 1∂y ∂y n
Case I : If all roots are distinct then general solution
∂z ∂z
+M +N + Tz = f (x , y) ...(1) of (1) is
∂x ∂y
z = φ 1(y + m1x ) + φ 2 (y + m2 x )
Where the coefficients A1, A2 , ..., An, M , N, T
are constants or functions of x and y. +... + φ n(y + mnx )
2. If the coefficients of various terms in equation where φ 1, φ 2 , ..., φ n are arbitrary functions.
(1) are constants then it is called a linear partial Case II : If a root m repeated r times then C.F. is
differential equation with constant coefficients.
φ 1(y + mx ) + x φ 2 (y + mx ) + ... + x r− 1φ r(y + mx )
3. A linear differential (partial) equation of order n
with constant coefficients is
DETERMINATION OF PARTICULAR INTEGRAL
∂ nz ∂ nz ∂ nz (P.I.)
+ A1 + ... + An = f (x , y)
∂x n ∂x n− 1∂y ∂y n 1. Consider f (D, D ′ ) z = f (x , y) so its P.I. is
where A1, A2 .... An are all constants. 1
f (x , y)
or (D n + A1D n− 1D ′ + .... + AnD ′ n)z = f (x , y) f (D, D ′ )

or f (D, D ′ )z = f (x , y) 1
V is defined as the function which
Here f (D, D ′ ) is a homogeneous function in f (D, D ′ )
D, D ′ of degree n. given V when it is operated upon by f (D, D ′ ).
4. If u is the complementary function and v a 2. The shortcut method for P.I. of
particular integral of a linear partial differential
f (D, D ′ )z = φ(ax + by) is given by
equation f (D, D ′ )z = f (x , y) then u + v is a
general solution of the equation. Case I : When f (a, b) ≠ 0 and f (D, D ′ ) is a
homogeneous function of degree n,then
DETERMINATION OF THE COMPLEMENTARY 1
FUNCTION (C.F.) P.I. = ...... ∫ f (u) dudu... du
f (a, b) ∫∫∫
1. The general solution of f (D, D ′ )z = 0 which
contains the correct number of arbitrary Where u = ax + by and R.H.S. contains a
multiple integral of n th order.
71

1 1
Case II : When f (a, b) = 0 we have Case I : e ax + by = e ax + by if f (a, b) ≠ 0
f (D, D ′ ) f (a, b)
1 xn
φ(ax + by) = φ(ax + by) 1
(bD − aD ′ )n n ! bn Case II : sin(ax + by), we put
f (D, D ′)
3. The general method of finding the P.I. of
D 2 = − a 2 , DD ′ = − ab, D ′ 2 = − b2
(D − mD ′ ) z = φ(x , y)
provided eliminator is not zero. Similarly for
or p − mq = φ(x , y) cos (ax + by)
Lagrange’s auxiliary equations are 1
Case III : x my n = [ f (D, D ′ )]−1 x my n
dx dy dz f (D, D ′ )
= =
1 −m φ(x , y) Which can be evaluated after expanding
By first two we get y + mx = a (constant) [ f (D, D′ )]−1 in ascending power of D or D′.
1
and by first and last term we get Case IV : (e ax + by . V)
f (D, D ′ )
dz = φ(x , y) dx = φ(x , a − mx ) dx 1
= e ax + by V
z= ∫ φ(x , a − mx ) dx f (D + a, D ′+ b)

1 Case V : If f (a, b) = 0 then


Thus z = φ(x , y) = ∫ φ(x , a − mx ) dx 1 x
(D − mD ′ ) P.I. f (ax + by) = f (ax + by)
f (D, D ′ ) ( f ′ )( a, b)
NON-HOMOGENEOUS LINEAR EQUATION WITH Where f′=
∂f
CONSTANT COEFFICIENT ∂D
1. A linear differential equation which is not If ( f ′ )( a, b) = 0 then
homogeneous is called a non-homogeneous x2
linear equation. P.I. is f (ax + by) and so on.
( f ′′)( a, b)
Consider f (D, D ′ )z = f (x , y) ...(1)
Where f (D, D ′ ) is non-homogeneous. EQUATIONS REDUCIBLE TO LINEAR FORM
Let WITH CONSTANT COEFFICIENTS
f (D, D ′ ) = (D − m1 D ′ − k 1) (D − m2 D ′−k 2 ) ∂ nz ∂ nz
1. Consider A0 x n + A1x n− 1y
... (D − mnD ′ − k n) ∂x n ∂x n− 1∂y
Then C.F. of equation (1) is ∂ nz
+ ... Any n = f (x , y)
z = e k 1 x φ 1(y + m1x ) + e k 2 x φ 2 (y + m2 x ) ∂y n

+ ... + e k n x φ n(y + mnx ) Put x = eu and y = ev


i. e. u = log x and v = log y we get
2. If (D − mD ′−k ) occurs r times in f (D, D ′ ) then ∂ ∂
the C.F. corresponding to this factor is x≡ ≡D
∂x ∂u
e kx [φ 1(y + mx ) + xφ 2 (y + mx ) ∂2
x2 ≡ D(D − 1)
+ ... + x r− 1φ r(y + mx )] ∂x 2
If f (D, D ′ ) is irreducible then a trial method is ∂3
x3 ≡ D (D − 1) (D − 2)
used to find solutions. ∂x 3
3. To find the particular integral P.I. ∂ ∂
Similarly y = ≡ D ′z , D ′ ≡
so f (D, D ′)z = f (x , y) we have ∂y ∂v
1 ∂2 ∂3
P.I. is f (x , y) y2 ≡ D ′ (D ′−1), y 3 ≡ D ′ (D ′−1) (D ′−2)
f (D, D ′ ) ∂y 2 ∂y 3
72

∂2 then D 2 z = Ah 2 e hx + ky and D ′ z = kAe hx + ky


xy ≡ DD ′
∂ x∂ y put these values in (1) we get
Substituting in given equation it reduces to (h 2 − k ) Ae hx + ky = 0
f (D, D ′ ) z = V
⇒ h2 = k ...(3)
Which is an equation having constant So, C.F. of (1) is
coefficients. 2
z = Ae hx + h y
C.F. OF NON-HOMOGENEOUS LINEAR PDE Since all values of k satisfy equation (1) so
WHEN LINEAR FACTORS ARE NOT POSSIBLE general solution of (1) is
2
1. Consider (D 2 − D ′ )z = 0 ...(1) z = ΣAe hx + h y
Let its trial solution is z = Ae hx + ky ...(2)

EXERCISE
MULTIPLE CHOICE QUESTIONS 7. The P.I. of (D 2 − DD ′−6D ′2 ) z = xy is :
1. The general solution of the differential equation xy x 2 x 2y + x 3
3 2 2 (a) + (b)
(D − 4D D ′ + 3DD ′ ) z = 0 is : 2 4 6

(a) c1 + c 2e x + c 3e 3x x 3y x 4 x 2y x 4
(c) + (d) +
6 24 6 12
(b) φ1(y) + φ2(y + x) + φ3(y + 3x) 1
8. x 3 is equal to :
(c) φ1(x) + φ2(y + x) + φ3(y + 3x) D13 + D 23 + D 33 − 3D1D 2D 3
(d) (c1 + c 2x) e x + c 3e 3x x6 x6 x6 x6
(a) (b) (c) (d)
2. The complementary function of log s = x is : 12 120 60 80

(a) φ1(y) + yφ2(y) (b) φ1(x) + xφ2(x) 9. The solution of (D 2 − 4DD ′+4D ′2 ) z = 0 is :
(c) φ1(x) + φ2(y) (d) φ1(x) + yφ2(x) (a) (c1 + c 2x) e 2x (b) c1e x + c 2e 2x
3. The auxiliary equation of 2r + 5s + 2t = 0 is : (c) φ1(y + 2x) + φ2(y + x)
2 2 (d) φ1(y + 2x) + xφ2(y + 2x)
(a) 2 m + 5 m + 2 = 0 (b) 2 m + 5 m − 2 = 0
10. The solution of (D 3 − 3D 2D ′+2DD ′2 ) z = 0 is :
(c) 2 m 2 − 5 m + 2 = 0 (d) 2 m 2 − 5 m − 2 = 0
(a) φ1(y) + φ2(x) + φ3(y + x)
4. The roots of A.E. of (D 4 − D ′4 ) z = 0 are :
(b) φ1(y) + φ2(y + x) + φ3(y + 2x)
(a) 1, 1, 1 + i, 1 − i (b) 1, − 1, 1 + i, 1 − i (c) φ1(y) + φ2(x) + φ3(y + 2x)
(c) 1, 1, ± i (d) 1, − 1, ± i (d) φ1(x) + φ2(y + x) + φ3(y + 2x)
2 2
5. The solution of the differential equation r − s = 0 11. The roots of A.E. of
is : (D 4 − 2D 3D ′+2DD ′3 − D ′4 ) z = 0 are :
x −x
(a) φ1(y + ix) + φ2(y − ix) (b) c1e + c 2e
(a) 1, 1, 1, − 1 (b) 1, 1, 2, − 1
(c) (c1 + c 2x) e − x (d) φ1(y + x) + φ2(y − x) (c) 1, 2, − 1, − 2 (d) 1, 1, 1, 1

6. The particular integral of (D 2 − a 2D ′2 ) z = x 2 is : 12. The P.I. of the differential equation r − t = x − y is :

x2 x3 x 3 x 2y x2 y
(a) (b) (a) − (b) −
12 6 6 2 6 2
x3 x4 y 3 xy 2 x 3 xy
(c) (d) (c) − (d) +
12 12 6 2 6 2
73

13. The P.I. of 4r − 4s + t = 16 log (x + 2y) is : 24. The solution of s + p − q = 0 is :


2
(a) x log (x + 2y) (b) 2x log (x + 2y) (a) e − x φ1(y) + e y φ2(x) (b) e x φ1(x) + e − y φ2(y)
x −y
(c) x log (x + 2y) (d) 2x 2 log (x + 2y) (c) e φ1(y) + e φ2(x) (d) e − x φ1(x) + e y φ2(y)
1 The P.I. of (D 2 − D ′) z = 2y − x 2 is :
14. The value of e x − y is : 25.
2 2
2D − DD ′−3D ′
(a) xy (b) x 2y (c) xy 2 (d) x + y
ex − y xe x − y x 2e x − y xe y − x
(a) (b) (c) (d) 1
5 5 5 5 26. sin x is equal to :
D + D′
15. The particular integral of r − 2as + a 2t = f (y + ax) is : (a) − sin x (b) cos x
f (y + ax) (c) − cos x (d) sin x
(a) f (y + ax) (b)
2
27. The P.I. of (D13 + D 23 + D 33 − 3D1D 2D 3) z = −3xyz
xf (y + ax) x 2 f (y + ax)
(c) (d) is :
2 2
xyz xy 2z x 2yz x 2y 2z 2
16. The P.I. of r + s − 2t = (2x + y)1/ 2 is : (a) (b) (c) (d)
8 8 8 8
(2x + y)1/ 2 (2x + y)3/ 2 1
(a) (b) 28. The value of (x + y) is :
15 15
D 2 + 3DD ′+2D ′2
5/ 2 7/ 2
(2x + y) (2x + y)
(c) (d) (x + y)2 (x + y)3
15 15 (a) (b)
18 36
17. The roots of A.E. of (D 3 − 7 DD ′2 −6D ′3 ) z = 0 are : (x + y)2
(c) (d) None of these
(a) 1, 2, 3 (b) −1, − 2, − 3 36
(c) −1, 2, − 3 (d) −1, − 2, 3 1
29. The value of 12xy is :
18. P.I. of r + s − 6t = y cos x is : D − 2DD ′−15D ′2
2

(a) cos y − x sin y (b) cos y + y sin x (a) x 4 + 2x 3y (b) x 3 + 2x 4 y


(c) sin x − y cos x (d) sin x + x cos x
(c) x 4 + 2xy 3 (d) None of these
1 x
19. (y − 1) e is equal to : 1
D 2 − DD ′−2D ′2 30. The value of e ax + by , provided f (a, b) ≠ 0 is :
f (D, D ′)
(a) e x (b) xe x
1 1
(c) ye x
(d) (x + y) e x (a) e ax + by (b) ea + b
f (− a, − b) f (a,b)
20. The solution of p + q = sin x is : 1 2 2 1
(c) ea + b (d) e ax + by
(a) φ (y − x) − cos x (b) φ (y − x) + sin x f (a, b) f (a, b)
(c) φ (y − x) + cos x (d) φ (y + x) − sin x
31. The P.I. of (D 2 − D ′2 −3D + 3D ′) z = e x + 2y is :
21. The solution of (D − mD ′−k) z = 0 is :
(a) e x + 2y (b) xe x + 2y
(a) e − kx φ (y + mx) (b) e − kx φ (y − mx)
(c) − xe x + 2y (d) None of these
(c) e kx φ (y − mx) (d) e kx φ (y + mx)
32. The auxiliary equation of x 2r + 2xys + y 2t = 0 is :
22. The solution of (D − D ′2 ) z = 0 is :
2 (a) (D − D ′) (D − D ′−1) z = 0
(a) φ1(y + x) + φ2(x) (b) Σ Ae k x + ky
(b) (D − D ′) (D − D ′+1) z = 0
(c) φ(y + x) + φ2(y) (d) Σ Ae kx + ky
(c) (D + D ′) (D − D ′+1) z = 0
1 (d) (D + D ′) (D + D ′−1) z = 0
23. The value of x is :
(D − D ′−1) (D − D ′−2)
33. The P.I. of x 2r − y 2t = xy is :
x+3 2x + 3
(a) (b) x
2 4 (a) xy (b) log x
2x − 3 x−3 y
(c) (d)
4 2 (c) xye x (d) xy log x
74

34. The P.I. of x 2r − y 2t + px − qy = log x is : 1


43. The value of sin (4x + y) is :
3x 3 (D 2 − 5DD ′+4D ′2 )
e (log x)
(a) (b) y1
6 6 y
(a) cos (4x + y) (b) − cos(4x + 5)
(log x) 2
e 2x 2 2
(c) (d) x y
6 6 (c) − cos (4x + y) (d) − cos (4x + y)
3 3
2 2
35. The solution of x r − y t = 0 is :
1
y 44. φ (x, y) is equal to :
(a) φ1(xy) + xφ2(xy) (b) φ1(xy) + φ2   D − mD ′
x
(a) ∫ φ(x, a − mx) dx (b) ∫ φ(x, a + mx) dx
y y
(c) φ (xy) + xφ2   (d) φ1(xy) + yφ2  
x x (c) ∫ φ(x, a − my) dx (d) ∫ φ (x, a + my) dx
hx + ky 2
36. If z = ΣAe be the solution of (D − D ′) z = 0
45. The solution of (4D 2 + 12DD ′+9D ′2 ) z = 0 is :
then :
(a) φ1(2y + 3x) + φ2(2 y − 3x)
(a) h2 = k (b) h = k 2
(c) h3 = k (d) h = k (b) φ1(2y − 3x) + xφ2(2y − 3x)
(c) e 3x φ1(2y) + e −3x φ2(2y)
∂ 2z ∂ 2z ∂ 2z
37. The equation x 2 + 4 xy + y2 = 0 can
∂x 2 ∂y 2 ∂y 2 (d) [φ1(2y) + xφ2(2y)] e −3x
be reduced into linear equation with constant
46. The P.I. of r + 3s + 2t = 6 (x + y) is :
coefficients if we substitute :
(a) 3xy − 2x 2 (b) xy − 3x 2
(a) x = X + h, y = Y + k (b) x = e X , y = e Y
(x − y)3
(c) x = log X, y = log Y (d) x = e 2X , y = e 2Y (c) (d) 3x 2y − 2x 3
6
38. The solution of r − a 2t = 0 is : 47. The value of
1
φ(ax + by) is :
(a) c1e ax + c 2e − ax (bD − aD ′)2
(b) φ1(y + ax) + xφ2(y + ax) x2 x
(a) φ (ax + by) (b) φ (ax + by)
(c) φ1(y + ax) + φ2(y − ax) 2! b2 b
(d) (c1 + c 2x) e − ax
x2 x2
(c) φ′(ax + by) (d) φ′′(ax + by)
39. The roots a A.E. of r + (a + b) s + abt = xy is : 2! b2 2! b2
(a) a, b (b) a, − b
1
(c) −a, b (d) − a, − b 48. φ (x + y) is equal to :
(D − D ′ ) 2
1 2 2
40. The value of {−4 π (x + y )} is :
D 2 + D ′2 x x2
(a) φ (x + y) (b) φ (x + y)
(a) −2x 2y 2 (b) −2 π 2x 2y 2 2 2
(c) −2 πx 2y 2 (d) −2πxy x3 x
(c) φ (x + y) (d) φ (x + y)
1 3 3
41. x r φ (ax + by) is equal to :
(bD − aD ′) 49. The P.I. of r − 2s + t = sin (2x + 3y) is :
xr x r+1 1
(a) φ(ax + by) (b) φ(ax + by) (a) 2 sin (2x + 3y) (b) sin (2x + 3y)
r+1 r+1 2
x r+1 xr
(c) φ (ax + by) (d) φ (ax + by) 1
b (r + 1) b (r + 1) (c) sin (2x + 3y) (d) − sin (2x + 3y)
3
42. The solution of r + t = 0 is : 50. The P.I. of r + s − 6t = y sin x is :
(a) φ1(y + ix) + φ2(y − ix)
(a) − y sin x + cos x (b) − x sin y − cos y
(b) φ1(y + x) + φ2(y − x)
(c) φ1(y − x) + φ2(y − x) (c) − y sin x − cos x (d) − x sin y + cos y
(d) φ1(y + ix) + xφ2(y − ix)
75

1 If Σ Ae hx + ky be the solution of (D 2 − D ′−1) z = 0


51. (e ax + by . V) is equal to : 60.
f (D, D ′)
then :
1 1
(a) e ax + by V (b) e ax + by V (a) k = h + 1 (b) k = h2 − 1
f (D, D) f (a, b)
(c) k = h − 1 (d) k = h2 + 1
1
(c) Ve ax + by 1 1
f (D, D ′) 61. e y is equal to :
2
1 (D − DD ′+ D ′−1)
(d)e ax + by V
f (D + a, D ′ + b) (a) −xe y (b) xe y
(c) −ye x (d) None of these
52. If z = Ae hx + ky be the solution of
1
(2D 4 − 3D 2D ′ + D ′2 ) z = 0 then : 62. e y − x is equal to :
2
(D − DD ′+ D ′−1)
(a) h = k 2 (b) k = h2
ey − x − ye x − y
(c) h = −k (d) h = k (a) (b)
2 2
1 −x 2 y− x y2 − x + y
53. cos (x + 2y) is equal to : (c) e (d) − e
(D 2 − DD ′+ D ′ – 1) 2 2
1 1 1
(a) cos (x + 2y) (b) − sin (x + 2y)
2 2 63. e 2x + 3y is equal to :
1 1 (D 2 + DD ′−2D ′2 +2D + 2D ′)
(c) sin (x + 2y) (d) − cos (x + 2y)
2 2 1 1
(a)− e 2x + 3y (b) − e 2x + 3y
54. The complementry function of 8 6
(D 2 − DD ′−2D ′2 +2D + 2D ′) z = xy is : 1 2x + 3y 1
(c) e (d) − e 2x + 3y
10 10
(a) φ (y + x) + e 2x φ2(y + 2x)
64. The P.I. of r − s + q − z = sin (x + 2y) is :
(b) φ1(y − x) + e 2x φ2(y + 2x)
1 1
(a) − cos (x + 2y) (b) − sin (x + 2y)
(c) φ1(y − x) + e −2x φ2(y + 2x) 2 2
(d) φ1(y + x) + e −2x φ2(y + 2x) 1
(c) cos (x + 2y) (d) None of these
1 2
55. The value of e x − y is:
D 2 − D ′2 + D + 3D ′−2 1
65. x 2y is equal to :
2
1 1 (D − D ′−1)
(a) e x − y (b) − e x − y
4 4 (a) x 2 − y 2 − 2y + 4 (b) x 2 + y 2 − 2x + y + 4
1 1 x −y
(c) − e x − y (d) e
6 6 (c) x 2 + x 2y + 2y + 4 (d) x 2 − x 2y − 2y + 4

56. If Σ Ae hx + ky be the solution of (D 2 − DD ′−2D) z = 0, 66. If Σ Ae hx + ky be the solution of r − s + 2q − z then :


then k : 1 − h2 1+ h
(a) k = (b) x =
(a) h (b) h − 1 (c) h − 2 (d) h − 3 2− h 2− h
57. The roots of A.E. of r + s − 6t = 0 are : 1− h 1 + h2
(c) k = (d) k =
2− h 2− h
(a) −2, − 3 (b) 2, 3 (c) −2, 3 (d) 2, − 3
67. The general solution of (D − 2D ′2 − 1) z = 0 is :
58. The solution of r − s + q − z = 0 is :
2
(a) φ1(y) + e − x φ2(y + x) (b) e x φ1(x) + e − x φ2(y − x) (a) Σ Ae( 2k +1) x + ky (b) Σ Ae( 2k −1) x + ky
2
(c) e x φ1(y) + e x φ2(y + x) (d) e x φ1(y) + e − x φ2(y + x) (c) Σ Ae( 2k −1) x + ky (d) Σ Ae( 2k +1) x + ky

59. The P.I. of (DD ′+ aD + bD ′+ ab)z = e mx + ny is : 1


68. (2y − x) is equal to :
2
mx + ny mx + ny (D − D ′ )
e e
(a) (b)
mn + ab (m + n)(a + b) (a) xy − y (b) xy 2 − y
e mx + ny e mx + ny y2
(c) (d) (c) yx − y 2 (d) −x
(n + a) (m + b) (m − n) (a − b) x
76

If Σ Ae hx + ky be the solution of (D 2 − 4DD ′ 1


69. (c) − (x + y)3/ 2 (d) − y 2(x + y)3/ 2
3
+ D − 1) z = 0, then k = :
h2 + 2h − 1 h2 + h − 1 76. The P.I. of (D 2 + D ′2 ) z = 30 (2x + y) is :
(a) (b)
4 4h (2x + y)3
(a) (2x + y)3 (b)
h2 − 4h + 1 3
(c) (d) None of these (2x + y)3
2h (c) (d) None of these
6
70. The P.I. of x 2r − y 2t + px − qy = (log x)2 is : 1
77. cos mx sin y is :
log x (log x)3 D 2 + D ′2
(a) (b)
6 6 cos (mx + ny) + cos (mx − ny)
(a)
(log x)4 (log x)4 (m + n)
(c) (d)
6 12 cos (mx + ny) + cos (mx − ny)
(b)
1
71. f (x, y) = (m 2 + n2)
(D − mD ′) cos (mx + ny) + cos (mx − ny)
(c)
(a) ∫ f (x, c − mx) (b) ∫ f (c, c − mx) 2 (m 2 + n2)
(c) ∫ f (x, c + mx) (d) ∫ f (x, mx) (d) None of these
78. The P.I. of (D 2 − 2DD ′+ D ′2 ) z = x 3 is :
72. The P.I. of log s = x + y is :
(a) log (x + y) (b) e x + y x5 x5 x5 x5
(a) (b) (c) (d)
(x + y)2 10 20 5 15
(c) x + y (d)
2 1
1 79. (x + 3y)1/ 2 is :
73. 3 3
x y is equal to : (D − 4DD ′+3D ′2 )
2
D 3 − D ′3
(x + 3y)1/ 2 (x + 3y)3/ 2
x 3y x9 x 6y 3 x6 (a) (b)
(a) + (b) + 60 20
9 10080 120 10080
(x + 3y)5/ 2 (x + 3y)7 / 2
(c) (d)
x 3y + x 9 x 6y 3 x9 60 120
(c) (d) +
10080 120 10080 1
80. e x is :
74. The P.I. of (x 2D 2 − y 2D ′2 ) z = x 2y is : (D + 6DD ′+ D ′2 )
2
1 1
(a) xy 2 (b) xy 2 (c) yx 2 (d) x 2y xe x
2 2 (a) e x (b)
y
1 1/ 2
75. (x + y) is equal to :
D 3 − 4D 2D ′+5DD ′2 −2D ′3 (c) (x + y) e x (d) ye x
x x2
(a) − (x + y)3/ 2 (b) − (x + y)3/ 2
3 3

ANSWERS
MULTIPLE CHOICE QUESTIONS
1. (b) 2. (c) 3. (a) 4. (d) 5. (d) 6. (d) 7. (c) 8. (b) 9. (d) 10. (b)
11. (a) 12. (a) 13. (d) 14. (b) 15. (d) 16. (c) 17. (d) 18. (c) 19. (c) 20. (a)
21. (d) 22. (b) 23. (b) 24. (c) 25. (b) 26. (c) 27. (d) 28. (b) 29. (a) 30. (d)
31. (c) 32. (d) 33. (d) 34. (b) 35. (c) 36. (a) 37. (b) 38. (c) 39. (d) 40. (c)
41. (c) 42. (a) 43. (c) 44. (a) 45. (b) 46. (d) 47. (a) 48. (b) 49. (d) 50. (c)
51. (d) 52. (b) 53. (c) 54. (c) 55. (b) 56. (c) 57. (b) 58. (d) 59. (c) 60. (b)
61. (a) 62. (a) 63. (d) 64. (c) 65. (d) 66. (a) 67. (d) 68. (c) 69. (b) 70. (d)
71. (a) 72. (b) 73. (d) 74. (d) 75. (b) 76. (a) 77. (c) 78. (b) 79. (c) 80. (a)
77

HINTS AND SOLUTIONS


−1
1. Given differential equation is 1  D ′2 
= 1 −  (x − y)
2
(D 3 − 4D 2D ′ + 3DD ′2 ) z = 0 D  D2 
or (D 3 − 3D 2D ′ − D 2D ′ + 3DD ′2 ) z = 0 1  D ′2 
=  1 + + ... (x − y)
2 2
or [D 2(D − 3D ′) − DD ′(D − 3D ′)] z = 0 D  D 

or D (D − D ′) (D − 3D ′) z = 0 1 x 3 x 2y
= (x − y) = −
2 6 2
so its solution is φ1(y) + φ2(y + x) + φ3(y + 3x) D
2 1
2. Given log s = x ⇒ s=
∂ z
=x 14. ex − y
∂ x∂ y 2D 2 − DD ′−3D ′2
1
i. e. DD ′ = x = ex − y .1
So C.F. is φ1(x) + φ2(y) 2 (D + 1)2 − (D + 1) (D ′−1) − 3 (D ′−1)2
1
4. Given (D 4 − D ′4 ) z = 0 = ex − y .1
2D 2 − 3D ′2 − DD ′+5D + 5D ′
4
A.E. is m − 1 = 0 −1
1  3D ′2 D ′ 2D D ′ 
⇒ (m 2 + 1) (m 2 − 1) = 0 = ex − y  1 − − + +  .1
5D  5D 5 5 D 
i. e. m = 1, − 1, i, − i
1 x
−1 = ex − y .1 = e x − y
1 1  a 2D ′ 2  5D 5
6. x2 =  1 −  x2
( D 2 − a 2D ′ 2 ) D2  D2  1
16. (2x + y)1/ 2
1  a 2D ′ 2  D 2 + DD ′−2D ′2
=+ 1 + + ... x 2
2 2 1
D  D  = u1/ 2du du where u = 2x + y
4 + 2 − 2 ∫∫
1 x4
= x2 =
D 2 12 1 5/ 2 2.2 u 5/ 2 (2x + y)5/ 2
= u = =
1 4 3.5 15 15
8. P.I. is x3
D13 + D 23 + D 33 − 3D1D 2D 3 18. P.I. is
1
y cos x
−1 D + DD ′−6D ′2
2
1  D 23 D 33
3D 2D 3 
= 1+ + −  x3 1
  y cos x
D13  3
D1 D13
D12  =
(D − DD ′) (D + 3D ′)
1  D 23 D 33 3D 2D 3  1 3
= 1− − + + .... x 3 = x 1
  = (a + 3x) cos xdx where, y − 3x = a
D13  D13 D13 D12  D13 D − 2D ′ ∫
6
x 1
= − [a sin x + 3x sin x + 3 cos x]
120 D − 2D ′
10. Given differential equation is 1
= [(y − 3x) sin x + 3x sin x + 3 cos x]
(D 3 − 3D 2D ′+2DD ′2 )z = 0 D − 2D ′

D (D 2 − 3DD ′+2D ′2 ) z = 0 = ∫ [(b − 2x) sin x + 3 cos x] dx where, y + 2x = b

or D (D − D ′) (D − 2D ′) z = 0 = − b cos x − 2 (− x cos x + sin x) + 3 sin x


so its solution is = − y cos x + sin x
z = φ1(y) + φ2(y + x) + φ3(y + 2x)
20. (D + D ′)z = sin x
∂ 2z ∂ 2z 1
12. Given that r − t = − = x−y C.F. is φ (y − x) and P.I. sin x = − cos x
∂x 2 ∂y 2 D + D′
1
so its P.I. is (x − y) So solution is z = φ (y − x) − cos x
D 2 − D ′2
78

Differential equation is (D − D ′2 ) z = 0 1
22. ...(1) = e x + 2y
(D + D ′−3) (D − D ′)
It has no factor so solution is z = Σ Ae hx + ky
1 1
=− e x + 2y = − e x + 2y .1
put in (1) we get (h − Ak 2) Σ Ae hx + ky = 0 (D + D ′−3) D + 1 + D ′+2 − 3
i. e. h = k2 1 1 D′
−1
k 2 x + ky = − e x + 2y .1 = e x + 2y  1 −  .1
So required solution is Σ Ae D + D′ D D
x + 2y 1 x + 2y
1 = −e .1 = − xe
23. x D
(D − D ′−1) (D − D ′−2)
 2 ∂ 2z ∂ 2z 
1
−1 33.  x − y2  = xy
= (1 − D + D ′)−1 1 − D + D ′  .x  ∂x 2
∂y 2 
 
2  2 2
∂ ∂
1 D D′ put x = e X , y = e Y and = D, = D′
= (1 + D − D ′+ ...)  1 + − + .... x ∂X ∂Y
2  2 2 
we get [D(D − 1) − D ′(D ′−1)] z = e X + Y
1 3  1 3  2x + 3
=  1 + D + .... x =  x +  = (D − D ′) (D + D ′−1) z = e X + Y
2 2  2 2 4
1 1
P.I. is eX + Y = eX + Y
−1 (D − D ′) (D + D ′−1) D − D′
1 1  D2 
25. (2y − x 2) = −  1 −  (2y − x 2) 1 1
D2 − D′ D′  D ′  = eX + Y .1 = e X + Y .1
D + 1 + D ′−1 D + D′
−1
1  D2  1 = eX + Y
1 D′
.1 = Xe X + Y = xy log x
=−  1 + + .... (2y − x 2) = − (2y − x 2 − 2y) 1 + 
D′  D′  D ′ D D
1 ∂ 2z ∂ 2z
∂z ∂z
=− (− x 2) = x 2y 34. Given x 2 − y2 −y +x
= log x
2 ∂ x 2 ∂y
D′ ∂x ∂y
∂ ∂
1 put x = e X , y = e Y , D = , D′ =
27. (−3xyz) ∂X ∂Y
D13 + D 23 + D 33 − 3D1D 2D 3
we get [D(D − 1) − D ′(D ′−1) + D − D ′]z = X
−1
1  D12 D 22 D 32  (D 2 − D − D ′2 + D ′+ D − D ′) z = X
=− 1 − − −  (−3xyz)
3D1D 2D 3  3D 2D 3 3D1D 3 3D1D 2  or (D − D ′ ) (D + D ′ ) z = X
−1
1 1  D ′2 
1  D12 D 22 D 32  So P.I. is .X = 1 −
2
 .X
= 1 +

+ + + .... (xyz)
 (D 2 − D ′ 2 ) D  D2 
D1D 2D 3  3D 2D 3 3D1D 3 3D1D 2 
1 X 3 (log x)3
1 x 2y 2z 2 = .X = =
= (xyz) = D 2 6 6
D1D 2D 3 8
hx + ky
1 36. If z = ΣAe be the solution of (D 2 − D ′) z = 0
29. 12xy
D 2 − 2DD ′−15D ′2 then it satisfies so
−1
1  2D ′ 15D ′ 2 Σ Ae 2e hx + ky − ΣkAe hx + ky = 0
= 1 − −  12xy
2 h2 = k
D  D D2  i. e.

1  2D ′ 15D ′2  ∂ 2z ∂ 2z
= 1 + + + ... 12xy 38. Given − a2 =0
2 D 2 ∂x 2
∂y 2
D  D 
1  1  12x 3y 24 x 4 i. e. D 2 − a 2D ′ 2 = 0
=  12xy + 24 x  =
2
+
D D  6 4.6
i. e. (D + aD ′) (D − aD ′) = 0
= 2 x 3y + x 4
So. C.F. is φ1(y + ax) + φ2(y − ax)
1 x + 2y 39. Given r + (a + b) s + abt = xy
31. e
D 2 − D ′2 −3D + 3D ′
1 A.E. is m 2 + (a + b) m + ab = 0
= e x + 2y
(D + D ′ ) (D − D ′ ) − 3 (D − D ′ ) i. e. (m + a) (m + b) = 0 or m = − a, − b
79
1
40. [−4 π (x 2 + y 2)] 64.
1
sin (x + 2y) =
1
sin (x + 2y)
2 2
D + D′ D 2 − DD ′+ D ′−1 D′
1
4π  D ′2  2 D′
=− 1 +  (x + y 2) =− sin (x + 2y)
2 D′2
D  D2 
D′ 1
4π  D ′2  =− sin (x + 2y) = cos (x + 2y)
=− 1 − + ... (x 2 + y 2) 4 2
2
D  D2 
66. If Σ Ae hx + ky be the solution of (D 2 − DD ′+2D ′−1)
4π  2 2 2  4π 2 2 2
=−  x + y − 2 .1 = − 2 (x + y − x ) z = 0 then it satisfied i. e.
D2  D  D
(h2 − hx + 2k − 1) Σ Ae hx + ky = 0
4π y 2x 2
=− y 2 = −4 π = − 2 π x 2y 2 so h2 − hx + 2k − 1 = 0
2 2
D
1 k (2 − h) = 1 − h2
43. sin (4x + y)
D − 5DD ′+4D ′2
2 1 − h2
or k=
1 2− h
sin (4x + y)
(D − 4D ′) (D − D ′) −1
−1 1 1  D2 
= cos (4x + y) 68. (2y − x) = −  1 −  (2y − x)
3 (D − 4D ′) D2 − D′ D′  D ′ 
x 1  D2  1
= − cos (4x + y) =−  1 + + ... (2y − x) = − (2y − x)
3 D′  D′ D′

1 6 (x + y)3 = − (y 2 − xy) = xy − y 2
46. 6 (x + y) =
2 2 (1 + 3 + 2) 3
D + 3DD ′+2D ′
(x + y)3 70. Given x 2r 2 − y 2 + px − qy = (log x)2
= ∂ ∂
3 put x = e X , y = e Y , D = , D′ = we get
1 ∂X ∂Y
49. sin (2x + 3y)
(D 2 − 2DD ′+ D ′2 ) [D(D − 1) − D ′(D ′−1) + D − D ′] z = X 2
1 or (D − D ′) (D + D ′) z = X 2
= sin (2x + 3y)
(D − D ′ ) 2 P.I. is
1
− X2
=
1
[− sin (2x + 3y)] = − sin (2x + 3y) (D − D ′ 2 )
2

(2 − 3)2 −1
1  D ′2  1
1 1 =  1 −  . X2 = X2
53. cos (x + 2y) = cos (x + 2y) D2  D2  D 2
2 D ′
(D − DD ′+ D ′−1)
D′ 1 X 4 (log x)4
= cos (x + 2y) = − D ′ cos (x + 2y) = =
2 4 12 12
D′
1
= + sin (x + 2y) 72. Given s = e x + y
2 ∂ 2z
1 1 ⇒ = ex + y
55. x − y = − ex − y ∂ x∂ y
D 2 − D ′2 + D + 3D ′−2 4
i. e. DD ′ = e x + y
58. Given (D 2 − DD ′+ D ′−1) z = 0 1 x +y
So, P.I. is e = ex + y
or (D − 1) (D − D ′+1) z = 0 DD ′
−1
So its solution is z = e x φ1(y) + e − x φ2(y + x) 1 1 
3 3 D ′3 
1 1 73. x y =  1 −  x 3y 3
61. ey = ey D 3 − D ′3 D3  D3 
D 2 − DD ′+ D ′−1 (D + 1) (D − 1) − D ′(D − 1)
1  D ′3 
1 −1 = 1 + + .... x 3y 3
= ey = ey 3 3
(D − D ′+1) (D − 1) D − D ′+1 D  D 
1 1 1 D ′3
= − ey .1 = − e y .1 = x 3y 3 +x 3y 3
3
D − (D ′+1) + 1 D − D′ D D6
1 D′
−1
1 x 6y 3 1 x 6y 3 x9
= − ey 1 −  .1 = − e y . .1 = − xe y = + 6x 3 = +
120 D 6 120 10080
D D D
80
1 1
75. (y + x)1/ 2 =− [cos (mx + ny) + cos (mx − ny)]
3 2 2 3
(D − 4D D ′+5DD ′ −2D ′ ) 2 (m + n2)
2

1
= (y + x)1/ 2 78.
1
x3 =
1
x3
2
(D − D ′) (D − 2D ′) (D 2 − 2DD ′+ D ′2 ) (D − D ′ ) 2
2 1
=− ( y + x ) 3/ 2 −2
3 (D − D ′ ) 2 1  D′ 1  2D ′
= 1 −  x3 = 1 + + ... x 3
2 2 D2  D D2  D 
2x x
=− ( y + x ) 3/ 2 = − ( y + x ) 3/ 2
3 2! 3 1 x5
= x3 =
1 D2 20
76. P.I. is .30 (2x + y)
2 2
(D + D ′ ) 1
79. (x + 3y)1/ 2
1 (2x + y)3 (D − 4DD ′+3D ′2 )
2
= 30 = (2x + y)3
(4 + 1) 6 1 2.2
(x + 3y)5/ 2
1 1 − 41
. .3 + 3.9 3.5
77. cos mx sin ny
D 2 + D ′2 4 (x + 3y)5/ 2
1 = (x + 3y)5/ 2 =
= cos (mx + ny) 16 × 15 60
2 (D 2 + D ′ 2 )
1 1 1
+ cos (mx − ny) 80. ex = . e x + 0y
2 2
2
2 (D + D ′ ) 2 D + 6DD ′+ D ′ D + 6DD ′+ D ′2
2

−1 1
= cos (mx + ny) = ex = ex
2 (m 2 + n2) 1+ 0+ 0
1
− cos (mx − ny) mmm
2 (m + n2)
2
Unit-81III
C HAPTER
10
Partial Differential Equations of Second Order
with Variable Coefficients : Monge’s Method
PARTIAL DIFFERENTIAL EQUATION OF SECOND ∂ 2u ∂ 2u ∂ 2u
A +B +C
ORDER ∂x 2 ∂ x∂ y ∂y 2
1. If a partial differential equation contains at least  ∂u ∂u 
+ f  x , y, u, ,  =0 ...(1)
one of the second order partial differential  ∂x ∂y 
coefficients r , s, t but not of higher order is Where A is positive.
called second order PDE. Its general form is Here φ = AS12 + BS1S2 + CS22
f (x , y, z , p, q, r , s, t) = 0 So the equation (1) is
After finding the general solution by usual (i) Elliptic if B 2 − 4 AC < 0
method, the given geometrical conditions may (ii) Hyperbolic if B 2 − 4 AC > 0
be used to find arbitrary functions.
(iii) Parabolic if B 2 − 4 AC = 0
CLASSIFICATIONS OF SECOND ORDER PDE
MONGE’S METHOD
1. Consider a linear PDE of second order in n- 1. Consider Rr + Ss + Tt = V ...(1)
independent variables
Where r , s, t have usual meaning and R, S, T , V
n n ∂ 2u n ∂u are functions of x , y, z , p and q
Σ Σ aij + Σ bi + cu = 0 ...(1)
i=1 j =1 ∂x i ∂x j i = 1 ∂x i ∂p ∂p
Then we have dp = dx + dy = rdx + sdy
∂x ∂y
Where aij , bi , c are constants or function of ∂q ∂q
x 1, x 2 , ..., x n and dq = dx + dy = sdx + tdy
∂x ∂y
∂ ∂2 dp − sdy dq − sdx
Let δi = and δiδ j = ∴ r= and t = put these
∂x i ∂x i ∂x j ds dy
n n values in equation (1) we get
Consider, φ = Σ Σ aij δi δ j ...(2)
i=1 j =1 (Rdpdy + Tdqdx − Vdxdy)
− S (Rdy 2 − Sdxdy + Tdx 2 ) = 0
Then partial differential equation (1) is
So the Monge’s subsidiary equations are
(i) Elliptic if φ is positive for all real values of δi and
Rdy 2 − Sdxdy + Tdx 2 = 0
it reduced to zero only when all δi are zero.
Rdpdy + Tdqdx − Vdxdy = 0 ...(2)
(ii) Hyperbolic if φ can be both positive or negative.
and dz = pdx + qdy
(iii) Parabolic if the determinant It may be possible to obtain either one or two
a 11 a 12 .... a 1n
relations between x , y, z , p, q called intermediate
a 21 a 22 .... a 2 n integrals.
∆= =0
.... .... .... .... So we get general solution of (1).
a n1 a n2 .... a nn
2. Consider Rr + Ss + Tt + U (rt − s 2 ) = V
2. Consider a PDE of second order in two where, R, S, T , U , V are functions of x , y, z p and q
independent variables Then λ-quadratic equation is
λ 2 (UV + RT ) + λSU + U 2 = 0
82

EXERCISE
MULTIPLE CHOICE QUESTIONS 10. The equation t
∂ 2u
+2
∂ 2u
+x
∂ 2u ∂u
+ is
2
1. The solution of S = 0 is : ∂t ∂x ∂t ∂x 2 ∂x
(a) xf (x) + φ(y) (b) f (x) + yφ (y) hyperbolic if :
(c) xf (x) + y φ (y) (d) f (x) + φ(y) (a) tx = 1 (b) tx = 0 (c) tx > 1 (d) tx < 1
2 2
2. z = log x log y + f (x) + φ(y) be the solution of : ∂u ∂u
11. The equation x 2 − + u is parabolic if :
(a) xyt = 1 (b) xyr = 1 ∂t 2 ∂x 2
(c) xys = 1 (d) xyps = 1 (a) x > 0 (b) x < 0
3. The solution of t = sin xy is : (c) x = 0 (d) For any value of x
sin xy 12. The solution of xr = (n − 1) p is :
(a) z = + f (x) + φ(y)
x2 (a) f (y) + x nφ (y) (b) x n + f (y) + φ(y)
− sin xy
(b) z = + yf (x) + φ (x)
(c) x n f (y) + φ(y) (d) x n f (x) + φ(y)
x2
sin xy 13. The Lagrange’s equation for t + s + q = 0 is :
(c) z = − + yf (x) + φ(x)
x dx dy dz dx dy dz
sin xy (a) = = (b) = =
(d) z = − + yf (x) + φ (x) 1 −1 f (x ) + z 1 1 f (x ) − z
y
dx dy dz dx dy dz
4. The equation of surface satisfying t = 6x 3y (c) = = (d) = =
1 1 f (x ) + z 1 1 z − f (x )
containing the two lines y = z = 0, y = z = 1 is :
(a) x 3y 3 (b) xy + y (1 − x 3) 14. The solution of r = 2y 2 is :

(c) xy + 1 − x 3 (d) x 3y 3 + y (1 − x 3) (a) z = x 2y 2 + xf (y) + φ(y)


(b) z = x 2y 2 + f (y) + yφ(y)
5. The solution of xr = p is :
(a) f (y) + φ (y) (b) xf (y) + φ (y) (c) z = x 2y 2 + f (y) + φ(y)
2 x2 (d) x 2y 2 + xf (y) + yφ(y)
(c) x f (y) + φ (y) (d) f (y) + φ (y)
2
15. The solution of t + s + q = 0 is :
∂ 2u ∂ 2u ∂ 2u
6. The equation + + = 0 is : (a) e x = f (x) + yφ (y) (b) ze x = f (x) + yφ(y)
∂x 2 ∂y 2 ∂z 2
(a) Parabolic (b) Hyperobolic (c) ze x = xf (x) + yφ(y) (d) ze x = f (x) + φ (x − y)
(c) Elliptic (d) Circular ∂ 2u ∂ 2u ∂ 2u 1 ∂ 2u
2 2 16. The equation + + − = 0 is :
2 2 2 2
7. The equation
∂u
= C2
∂u
is : ∂x ∂y ∂z c 2 ∂t
∂t 2 ∂x 2
(a) Circular (b) Hyperbolic
(a) Elliptic (b) Hyperbolic (c) Parabolic (d) Elliptic
(c) Parabolic (d) None of these 17. The solution of r = 6x is :
8. If z = φ1(y + x) + xφ2(y + x) be the general solution (a) x 3 + xf (y) + φ (y) (b) x 3 + f (y) + yφ (y)
2
of r + t − 2s = 0 with bz = y when x = 0, then
(c) x 2 + xf (y) + φ (y) (d) z = x 2 + xf (y) + yφ (y)
φ1(x + y) is :
18. The equation r = q is :
(a) (x + y) (b) b (x + y)2
(a) Circular (b) Elliptic
1 (x + y)2
(c) (x + y) (d) (c) Parabolic (d) Hyperbolic
2 b
∂ 2u ∂ 2u ∂ 2u ∂u
9. The solution of xr + Zp = 0 is : 19. The equation t +2 +x + is parabolic
2
1 ∂t ∂ x∂ t ∂x 2 ∂x
(a) − xf (y) + φ (y) (b) − f (y) + φ (y)
x if :
(c) z = − f (y) + φ (y) (d) f (y) − yφ(y) (a) tx > 1 (b) tx < 1 (c) tx = 0 (d) tx = 1
83

20. If z = φ1(2x + y) + xφ2 (2x + y) be the general (a) Circular (b) Parabolic
solution of r − 4s + 4t = 0 with z = x = 0 and (c) Hyperbolic (d) Elliptic
z − 1 = x − y = 0 then φ2(y + 2x) is equal to : 27. The differential equation xu tt + tu xt + u tt = 0 is
(a) 2x + y (b) 3 (2x + y) hyperbolic if :
3
(c) −3 (2x + y) (d) (a) t 2 > 4x (b) t 2 < 4x
2x + y 2
(c) t = 4x (d) None of thse
21. The Lagrange’s auxiliary equation for p + r + s = 1
2
is : 28. The solution of 2qy + y t − 1 = 0 is :

(a)
dx dy
= =
dz (a) yz = xe x − f (x) + yφ (x)
x y x − z + f (y)
(b) yz = e x − f (x) + φ(x)
dx dy dz
(b) = = (c) z = y log y − f (x) + φ(y)
1 1 x + y + f ( z)
dx dy dz (d) yz = y log y − f (x) + yφ(x)
(c) = =
x y x + y + f ( z) 29. The solution of q + xs − 4x − 2y = 2 is :
(d)
dx dy
= =
dz (a) 2x 2y + xy 2 + 2xy
1 1 x − z + f (y) (b) 2x 2y + xy 2 + 2xy + f (x) + φ(y)
x
22. The Lagrange’s auxiliary equation for s − t = is : (c) 2x 2y + 2xy + f (x) + yφ(y)
y2
(d) 2x 2y + xy 2 + f (x) + φ(y)

(a)
dx dy
= =
dz
(b)
dx dy dz
= = 30. In reducing the equation r = x 2t to canonical form
1 −1 f (x ) − x −1 1 x choose u and v such that :
y y x x
(a) u = y + x, v = y − x (b) u = y + , v = y −
dx dy dz dx dy dz 2 2
(c) = = (d) = =
1 −1 z + f (x ) −1 1 x
− f ( z) x2 x2
y (c) u = + y, v = y −
2 2
∂ 2u ∂ 2u ∂ 2u 1 ∂u y2 y2
23. The equation + + − = 0 is : (d) u = x + ,v = x −
∂x 2 ∂y 2 ∂z 2 c 2 ∂t 2 2
(a) Elliptic (b) Parabolic 31. In reducing the equation r + 2s + t = 0 to canonical
(c) Hyperbolic (d) None of these form choose u and v such that :
y y
∂ 2u ∂ 2u (a) u = x − y, v = x + y (b) u = x − , v = x +
24. The equation x 2 − + u is hyperbolic if : 2 2
∂t 2 ∂x 2 x x x−y x+y
(c) u = − y, v = + y (d) u = ,v =
(a) x > 0 (b) x < 0 2 2 2 2
(c) x = 0 (d) For all value of x
32. The equation x 2u tt + 3u xt + nu xx +17u t − 100u = 0
25. The solution of ar − xy = 0 is :
in third quadrant is :
x 2y
(a) az = + f (y) + yφ(y) (a) Parabolic (b) Elliptic
2
(c) Circular (d) Hyperbolic
x 3y
(b) az = + xf (y) + φ(y)
6 ∂ 2u ∂ 2u ∂ 2u
33. The equation x +t + = 0 is parabolic
(c) az = xy + xf (y) + φ(y) ∂t 2 ∂ x∂ t ∂ t 2
3
x y if :
(d) az = + f (y) + φ(y)
6 (a) t 2 > 4x (b) t 2 < 4x (c) t 2 > 2x (d) t 2 = 4x
2
∂u
26. The differential equation x 2 + y2 34. The solution of S = e x + y is :
∂x 2
(a) e x + y + f (y) + φ(x) (b) e x + y + xf (y) + φ(x)
∂ 2u ∂ 2u
+2 (x − y) + x 2 + y2 = 0 in the second
∂ x∂ y ∂y 2 (c) e x + y + f (y) + xφ (x) (d) e x + y + xf (y) + yφ (x)
quadrant is :
84

35. In reducing the equation r + x 2t = 0 to canonical 44. The equation r + t = 0 is :


form choose : (a) Parabolic (b) Hyperbolic
(c) Circular (d) Elliptic
x2 x2
(a) u = y + ,v = y−
2 2 x 3y x 3
45. az = + + xf (y) + φ(y) be the solution of :
(b) u = y + ix v = y − ix 6 3
(a) r = 2axy (b) ar = xy + 2x
(c) u = y + ix 2, v = y − ix 2
(c) ar = xy + y (d) ar = 2x + y
ix 2 ix 2 46. The solution of sy + p = cos (x + y) − y sin (x + y) is :
(d) u = y + ,v = y−
2 2 (a) yz = y sin (x + y) + f (x) + g (y)
36. In reducing the equation x 2r − 2xys + y 2t − xp (b) yz = sin (x + y) + f (x) + g (y)
8y (c) yz = cos (x + y) + f (x) + g(y)
−3yq − = 0 to canonical form choose u and v
x (d) None of these
such that : 47. To find Monge’s subsidiary equation is equal to :
y dq − sdx dq + sdy
(a) u = xy, v = x − y (b) u = xy, v = (a) (b)
x dy dx
(c) u = x + y, v = x − y (d) u = x + iy, v = x − iy dq − sdy dq + sdx
(c) (d)
37. To find Monge’s subsidiary equation, r is equal to : dx dy
dq − sdx dp − sdy
(a) (b) 48. One of the Monge’s subsidiary equation of
dy dx
dp + sdy dq + sdx r − t cos 2 x + p tan x = 0 is :
(c) (d)
dx dy (a) dx − cos xdy = 0 (b) dy − cos xdx = 0
2 (c) dy + cos xdx = 0 (d) dx + cos xdy = 0
38. One of the Mong’s subsidiary equation of r = a t is :
(a) dpdy + a 2dqdx = 0 (b) dy 2 + a 2dx = 0 49. Monge’s subsidiary equation of pt − qs − q3 = 0 is :
(c) dy 2 − a 2dx = 0 (d) dp dy + a 2dq = 0 (a) pdq − q3dy = 0 (b) pdq − q2dy = 0

39. In reducing the equation (y − 1) r − (y 2 − 1) s (c) pdq − qdy = 0 (d) pdx − qdy = 0
2x 3
+ y (y − 1) t + p − q = 2ye (1 − y) to canonical form 50. Monge’s subsidiary equation of (r − s) y + (s − t) x
+ q − p = 0 is :
choose u and v such that :
(a) dy − dx = 0 (b) xdy + ydx = 0
(a) u = x + y, v = ye x (b) u = x + y, v = x − y
(c) ydy − xdx = 0 (d) xdx + ydy = 0
y x
(c) u = xe ,v = ye (d) u = xe x , v = ye y
51. Monge’s subsidiary equation of (q + 1)s − (p + 1) t = 0
40. One of the Monge’s subsidiary equation of is :
t − r sec4 y = 2q tan y is : (a) pdq = 0 (b) (p − 1) dq = 0
(a) dx − dy tan y = 0 (b) dx − dy sec2 y = 0 (c) (q − 1) dy + (p − 1) dx = 0
(c) dx + dy tan y = 0 (d) dx + dy cos y = 0 (d) (p + 1) dq = 0

41. The Monge’s subsidiary equation of pt − qs = q3 is : 52. The λ− quadratic equaton of rt − S2 + 1 = 0 is :


(a) dx + pqdy = 0 (b) dy + pdx = 0 (a) λ2 + 1 = 0 (b) λ2 − 1 = 0
2
(c) pdx + qdy = 0 (d) pdx − qdy = 0 (c) λ + 2 = 0 (d) λ2 − 2 = 0
42. Monge’s subsidiary equation of (r − s) y + (s − t) x 53. The λ− quadratic equation for 2st (rt − s 2) = 1 is :
+ q − p = 0 is : (a) λ2 − 2λ + 1 = 0 (b) λ2 − 1 = 0
(a) xdy − ydx = 0 (b) xdx + ydy = 0 2
(c) λ + 2λ + 1 = 0 (d) λ2 − 2λ − 1 = 0
(c) dx + dy = 0 (d) None of these
54. The λ− quadratic equation for 5r + 6s + 3t
43. The Monge’s subsidiary equation of a 2t(rt − s 2) = 0
+2 (rt − s 2) +3 = 0 is :
is :
(a) dq + adx = 0 (b) dp + ady = 0 (a) 4λ2 + 4λ + 1 = 0 (b) 4λ2 − 4λ + 1 = 0
2
(c) dp − ady = 0 (d) None of these (c) 9λ − 12 λ + 4 = 0 (d) 9λ2 + 12λ + 4 = 0
85

55. The Monge’s subsidiary equation of (x + y) (r − t) 60. The Lagrange’s auxiliary equations for r + s = 1 is :
+ 4p = 0 is : dz dz
(a) dx = dy = (b) dx = − ydy =
(a) dy − dx = 0 (b) dy − xdy = 0 x + f (y) f (y)
(c) ydy − dx = 0 (d) ydy − xdx = 0 dz z
(c) − dx = dy = (d) dx = dy =
x − f (y) x − (y)
56. The Monge’s subsidiary equation of q2r − 2pqs
61. The solution of r = sin xy is :
+ p 2t = 0 is : xy
(a) − cos + yf (y) + φ (x)
(a) ydx − xdy = 0 (b) qdpdx + pdqdy = 0 y2
(c) xqdx − pydy = 0 (d) q2dpdy + p 2dqdx = 0 sin xy
(b) − + xf (y) + φ (y)
57. The Monge’s subsidiary equation of 2s + (rt − s 2) = 1 y2
sin xy
is : (c) − + yf (y) + φ( x )
y2
(a) dy + dp = 0 (b) dx − dq = 0
xy
(c) xdx − dq = 0 (d) dx − qdq = 0 (d) − cos + xf (y) + φ (y)
y2
58. The Monge’s subsidiary equation of r − 2s + t
62. The solution of t = xy is :
= sin (2x + 3y)ss is :
xy 3 xy 3
(a) dy + dx = 0 (b) dy + xdx = 0 (a) + f (x) + φ(y) (b) + xf (x) + φ(y)
6 6
(c) dx + ydy = 0 (d) xdx + ydy = 0
xy 3 xy 3
59. The λ quadratic equation of r − x 2t = 0 is : (c) + xf (x) + φ (x) (d) + yf (x) + φ(x)
6 6
(a) λ2 + x 2 = 0 (b) λ2 − x 2 = 0
2 2
(c) λ + x + 2 = 0 (d) None of these

ANSWERS
MULTIPLE CHOICE QUESTIONS
1. (d) 2. (c) 3. (b) 4. (d) 5. (d) 6. (c) 7. (b) 8. (d) 9. (b) 10. (d)
11. (c) 12. (c) 13. (b) 14. (a) 15. (d) 16. (b) 17. (a) 18. (c) 19. (d) 20. (d)
21. (d) 22. (a) 23. (b) 24. (a) 25. (b) 26. (c) 27. (a) 28. (d) 29. (b) 30. (c)
31. (a) 32. (d) 33. (d) 34. (a) 35. (d) 36. (b) 37. (b) 38. (c) 39. (a) 40. b)
41. (c) 42. (c) 43. (b) 44. (d) 45. (b) 46. (a) 47. (a) 48. (b) 49. (a) 50. (c)
51. (d) 52. (b) 53. (c) 54. (d) 55. (a) 56. (d) 57. (b) 58. (a) 59. (b) 60. (a)
61. (b) 62. (d)

HINTS AND SOLUTIONS


∂ 2z ∂ 2Z
1. S= =0 3. t= = sin xy
∂ x∂ y ∂y 2
∂z
Integrating w.r.t. y we get = f (x ) Integrating w.r.t.y, taking x as constant
∂x
∂z cos xy
Again integrating w.r.t. x we get =− + f (x )
∂y x
z= ∫ f (x)dx + φ(y) Again integrating w.r.t. y we get
or z = f (x) + φ(y) sin xy
z=− , + yf (x) + ψ (x)
x2
86

4. Given that t = 6x 3y with y = z = 0, y = z = 1 ∂ 2z


2 n−1
∂q 12. Given differential equation is ∂x =
i. e. = 6x 3y, integrating w.r.t. y we get ∂z x
∂y ∂x
∂z ∂z
q= = 3x 3y 2 + f (x) Integrating we get log = (n − 1) log x + log f (y)
∂y ∂x
∂z
Again integrating w.r.t. y, or = x n−1 f (y)
∂x
z = x 3y 3 + yf (x) + φ(x) ...(1)
Again integrating w.r.t. x we get
Putting y = 0, z = 0, in (1) we get φ (x) = 0 xn
z= f (y) + ψ(y) = x nφ(y) + ψ(y)
and putting y = z = 1 in (1) we get n
1 ≠ x 3 + f ( x ) + φ( x ) 13. t+ s+ q= 0
⇒ f (x ) = 1 − x 3 ∂ 2z ∂ 2z ∂z
i. e. + + =0
2 ∂ x ∂ y ∂y
so by (1) we get the required surface i. e., ∂x
z = x 3y 3 + y (1 − x 3) ∂q ∂p ∂z
or + + =0
∂y ∂y ∂y
∂ 2u ∂ 2u ∂ 2u
6. Given + + =0 Integrating w.r.t. y we get
2 2
∂x ∂y ∂z 2
q + p + z = f (x )
So φ = S12 + S22 + S32. Here φ is +ve for all real i. e., p + q = f (x ) − z
so Lagrange’s equations are
values of S1, S2, S3 and it reduces to zero only when
dx dy dz
S1 = S2 = S3 = 0. Hence it is elliptic. = =
1 1 f( x ) − z
∂ 2u ∂ 2u
7. Given equation is = c2 15. Given that t + s + q = 0
∂t 2 ∂x 2 ∂q ∂p ∂z
or + + =0
Here, A = C, B = 0, C = −1 ∂y ∂y ∂y
So, B 2 − 4 AC = 0 + 4C2 = 4C2 > 0 so it is hyperbolic. Integrating w.r.t. y q + p + z = f (x)
or p + q = f (x ) − z
8. Given r − 2s + t = 0
So Lagrange's equations are
⇒ (D 2 − 2DD ′+ D ′2 ) z = 0 dx dy dz
= =
(D − D ′)2 z = 0 1 1 f( x ) − z

So its solution is By first and second member we get x − y = 9


By first and last members we get
z = φ1(y + x) + xφ2(y + x)
dz
+ z = f( x )
y2 dx
Putting x = 0, z = we get
b dx
So its I.F. is e ∫ = ex
2 2
y (y + x )
= φ1(y) so φ(y + x) = Solution is ze x = ∫ e x f (x) dx + b = φ(x) + b
b b
10. Given differential equation is i. e., ze x = φ(x) + ψ (x − y)
2 2 2
∂u ∂u ∂ u ∂u
t
2
+2 +x + ∂ 2u ∂ 2u ∂ 2u 1 ∂ 2u
∂t ∂ x∂ t ∂x 2 ∂x 16. Given that
2
+
2
+
2

2
=0
∂x ∂y ∂z C2 ∂t
Here, A = t, B = 2, C = x
1
So B 2 − 4 AC = 4 − 4 tx So φ = S12 + S22 + S32 − S42
C2
It is hyperbolic if B 2 − 4 AC > 0
So φ may be both positive or negative. Hence the
⇒ 4 − 4t x > 1
equation is hyperbolic.
i. e. tx < 1
87

18. Given r = q 28. Given differential equation is 2yq + y 2t = 1


2
∂ z ∂z ∂z ∂ 2z
i. e.,
2
− =0 i. e. 2y + y2 =1
∂x ∂y ∂y ∂y 2
Here, A = 1, B = 0, C = 0 ∂z
Integrating w.r.t. y we get y 2 = y + f (x )
2 ∂y
So B − 4 AC = 0 − 0 = 0
∂z 1 f ( x )
so equation is parabolic. or = +
∂y y y2
20. Since z = φ1(y + 2x) + x φ2(y + 2x) be the solution of 1
given differerntial equation Again integrating w.r.t. y, z = log y − f ( x ) + φ( x )
y
put z = x = 0 we get φ1(y + 2x) = 0 or yz = y log y − f (x) + yφ(x)
and put z − 1 = x − y = 0 30. The given equation is r − x 2t = 0
we get φ1(y + 2x) + xφ2(y + 2x) = 1 Comparing it with Rr + Ss + Tt + f (x, y, z, p, q) = 0

Solving these φ2(y + 2x) =


1
=
3
=
3 We have R = 1, S = 0, T = − x 2
x 3x 2x + y
x ∂p ∂q x So quadratic equation Rλ2 + Sλ + T = 0 becomes
22. Given s − t = ⇒ − =
λ2 − x 2 = 0
y2 ∂y ∂y y 2
x ⇒ λ = x, − x
Integrating w.r.t. y we get p − q = − + f (x ) dy dy
y So we get + λ1 = 0 and + λ2 = 0
So its Lagrange’s equation is dx dx
dx dy dz dy dy
= = i. e. + x = 0 and −x = 0
1 −1 − x + f (x ) dx dx
y x2 x2
2 2 2 Integrating y + = constant and y − = constant
∂u ∂u ∂u 1 ∂u 2 2
23. Given that + + − =0
∂x 2 ∂y 2 ∂z 2 c 2 ∂t x2 x2
So, x = y + and v = y −
∂ 2u ∂u 2 2
Comparing ΣΣaij + Σbi + cu = 0
∂x i∂x j ∂x i 32. Given that x 2u tt + 3u x t + x 4xx + 17ut − 100 u = 0
we get a11 = a22 = a33 = 1, a44 = 0 Here, A = x 2, B = 3, C = x
and all a45 = 0 Vi ≠ j So, B 2 − 4 AC = 9 − 4x 3
a11 a12 a13 a14 1 0 0 0 In the third quadrant x is −ve so B 2 − 4 AC > 0
a21 a22 a23 a24 0 1 0 0 i. e. it is hyperbolic.
So = =0
a31 a32 a33 a34 0 0 1 0
∂ 2z
a41 a42 a43 a44 0 0 0 0 34. S= = ex + y
∂ x∂ y
Hence equation is parabolic. ∂z
Integrating w.r.t. y we get = e x + y + f (y)
∂u 2 2
∂u ∂x
26. Given x 2 + y 2 + 2 (x − y) Again integrating w.r.t x we get
∂x 2 ∂ x∂ y
z = e x + y + xf (x) + f (y)
2 2 ∂ 2u
+ x +y =0 or z = e x + y + φ ( x ) + f (y)
∂y 2
35. Given that r + x 2t = 0
Here A = x 2 + y 2 , B = 2(x − y), C = x 2 + y2
Comparing with Rr + Ss + Tt + f (x, y, z, p, q) = 0
So, B 2 − 4 AC = (4x 2 + 4y 2 − 8xy) − (4x 2 + 4y 2) and Letting Rλ2 + Sλ + T = 0 we get
= −8xy λ2 + x 2 = 0
In second quadrant x is –ve & y is +ve ⇒ λ = ± ix
2 dy dy
So B − 4 AC > 0 so + ix = 0 and − ix = 0
dx dx
i. e. hyperbolic.
88

ix 2 49. Given equation is pt − qs = q3


or y+ = constant
2 dq − sdx
put t= we get
ix 2 dy
and y− = constant
2 (pdq − q3dy) − s (pdx + qdy) = 0
2 2
ix ix
So, u = y+ ,v = y− So Monge’s subsidiary equation are pdx + qdy = 0
2 2
and pdq − q3dy = 0
38. Given that r = a 2t
∂p ∂p 52. Given equation is rt − s 2 + 1 = 0
We know dp = dx + dy = rdx + sdy
∂x ∂y Comparing Rr + Ss + Tt + U (rt − s 2) = V we have
∂q ∂q R = 0 = S = T, U = 1, V = −1
dq = dx + dy = sdx + tdy
∂x ∂y −
So λ quadratic equation is
dp − sdy dq − sdx
So, r = and t = λ2(UV + RT) + λSU + U 2 = 0
dx dy
or λ2(−1) + λ (0) + 1 = 0
Put these values in given equation we get
dp − sdy  dq − sdx  2 or λ2 = 1 ⇒ λ = 1, − 1
=  a
dx  dy 
54. Given 5r + 6s + 3t + 2 (rt − s 2) + 3 = 0
2 2 2 2
or (dpdy − a dqdx) − s (dy − a dx ) = 0
Comparing Rr + Ss + Tt + U (rt − s 2) = V
So Monge’s subsidiary equation is
R = 5, S = 6, T = 3, U = 2,. V = −3
dy 2 − a 2dx 2 = 0
So λ− quadratic equation is
and dpdy − a 2dqdx = 0 λ2(UV + RT) + λSU + U 2 = 0
dp − sdy dq − sdx or aλ2 + 12λ + u = 0
40. Put r = and t = in given equation
dx dy
57. Given equation is 2s + (rt − s 2) = 1
t − r sec4 y = 2q tan y we have
Comparing with Rr + Ss + Tt + U (rt − s 2) = V
[dqdx − dpdy sec4 y − 2q tan ydxdy]
R = 0, S = 2, T = 0, U = 1, V = 1
− s [dx 2 − dy 2 sec4 y]
The λ quadratic equation is
So Monge’s subsidiary equation is dx − dy sec2 y = 0
λ2(UV + RT) + λSU + U 2 = 0
dp − sdy dq − sdx 2
42. Put r = and t = in given equation or λ + 2λ + 1 = 0 ⇒ λ = −1, − 1
dx dy
(r − s) y + (s − t)x + q − p = 0 So intermediate integral is
λ 2Rdy + Udx + λ 2Vdq = 0
 dp − sdy − s  y +  s − dq − sdx  x + q − p = 0
    i. e. dx − dq = 0
 dx   dy 
59. Given r − x 2t = 0
or {ydpdy − xdxdq + (q − p) dxdy} − s {ydy 2
Comparing Rr + Ss + Tt + f = 0,
−(x − y) dxdy − xdx 2} = 0 R = 1, S = 0, T = − x 2
So Monge’s subsidiary equation is dx + dy = 0 So, λ-quadratic equation is
45. Consider ar = xy + x Rλ2 + Sλ + T = 0 ⇒ λ2 − x 2 = 0
2
⇒ r=a
∂ z
= xy + 2x ∂ 2z
2 62. Given t = = xy
∂x ∂y 2
Integrating w.r.t. x we get ∂z xy 2
2 2
Integrating w.r.t. y, = + f (x )
∂z yx 2x ∂y 2
a = + + f (y)
∂x 2 2 Again integrating w.r.t. y we get
Again integrating we get xy 3
z= + yf (x) + φ(x)
3 3 6
x y x
az = + + xf (y) + φ(y)
6 3 mmm
Unit-89III
C HAPTER
11 Legendre’s Functions
LEGENDRE’S EQUATION 3. The Legendre’s function of first kind is defined as
1. The differential equation of the form 1.3.5...(2n − 1)  n n (n − 1) n− 2
Pn(x ) = x − 2 (2n − 1) x
d 2y dy n! 
(1 − x 2 ) − 2x + n (n + 1) y = 0 is called
dx 2 dx n (n − 1) (n − 2) (n − 3) n− 4 
+ x + ....
Lagendre’s differential equation. 2.4. (2n − 1) (2n − 3) 
It can be writted It is also called Legendre’s polynomial and may
d  2 dy  be denoted by
(1 − x )  + n (n + 1) y = 0 n/ 2
dx  dx  2n − 2r!
Pn(x ) = Σ (−1)r x n− 2 r
r=0 n
where n is constant. 2 r !n − 2 r !n − r !
2. For solving n
if n is even
n  2
d 2y dy where   = 
(1 − x 2 ) − 2x + n (n + 1) y = 0 ...(1)  2  n − 1
if n is odd
dx 2 dx  2

4. The Legendre’s function of second kind i. e.
We assume that y = Σ a rx k − r, a 0 ≠ 0
r=0 Legendre’s polynomial is
dy ∞
Differentiate it = Σ a r(k − r )x k − r− 1 n!  − n− 1 (n + 1) (n + 2)
dx r= 0 Qn(x ) = x +
1.3.5....(2 n + 1)  2 (2n + 3)
d 2y ∞
and = Σ a r(k − r ) (k − r − 1) x k − r− 2 (n + 1) (n + 2) (n + 3) (n + 4) − n− 5 
2
dx r=0 x − n− 3 + x 
2.4. (2n + 3) (2n + 5) 
Put these values in equation (1) we get
following cases : 5. The most general solution of the Legendre’s
Case I : When k = n we have equation is y = APn(x ) + BQn(x ).
 n(n − 1) n− 2 6. It may be shown that Pn(x ) is the coefficient of
y = a 0 x 4 − x
 2 (2n − 1) h n in the expansion of (1 − 2xh + h 2 )−1/ 2 in
n (n − 1) (n − 2) (n − 3) n− 4  ascending powers of h i. e.
+ x + .... ...(2) ∞
2.4 (2n − 1) (2n − 3)  Σ h nPn(x ) = (1 − 2xh + h 2 )−1/ 2
which is the first solution of Legendre’s n=0
equation (1). where P0 (x ) = 1
Case II : When k = −(n + 1) we have
Here, (1 − 2xh + h 2 )−1/ 2 is called the
 (n + 1) (n + 2) − n− 3
y = a 0 x − n− 1 + x generating function of the Legendre
 2 (2n + 3)
polynomials.
(n + 1) (n + 2) (n + 3) (n + 4) x n− 5 
+  ...(3) 7. The Laplace first integral for Pn(x ) is
2.4 (2n + 3) (2n + 5)  1 π
Pn(x ) = ∫ { x ± (x 2 − 1) cos φ} n dφ
which is the second solution of Legendre’s π 0
equation (1). where n is a positive integer.
90

8. The Laplace second integral for Pn(x ) is the last term being 3P1 or P0 according as n is
1 π dφ even or odd.
Pn(x ) = ∫
π 0 [x ± (x 2 − 1) cos φ]n+ 1 4. The Christoffel’s symmation formula is
n
Σ (2r + 1) Pr(x )Pr(y) = (n + 1)
where n is a positive integer. r=0
9. The orthogonal properties of Legendre’s Pn + 1(x )Pn(y) − Pn+ 1(y) Pn(x )
polynomials are (x − y)
+1 5. The Rodrigue’s formula is
(i) ∫ P (x )Pn(x ) dx = 0 if m ≠ n
−1 n 1 dn 2
Pn(x ) = (x − 1)n
+1 2 2 n ! 2n dx n
(ii) ∫−1[Pn(x )] dx =
2n + 1 6. The Murphy’s formula is
n (n + 1)  1 − x 
RECURRENCE FORMULAE Pn(x ) = 1 −  
11
.  2 
1. The Recurrence formulae are 2
n (n − 1) (n + 2) (n + 1)  1 − x 
 
(i) (2n + 1) xPn = (n + 1) Pn+1 + nPn−1 +
1.21. .2  2  ....
 
or nPn = (2n − 1) xPn−1 − (n − 1) Pn− 2
(ii) nPn = xPn′ − Pn′−1 where dashes denote LEGENDRE’S POLYNOMIALS
differentiation w.r.t.x. Put n = 0, 1, 2, 3, 4, ... respectively Rodrigue’s
(iii) (2n + 1) Pn = Pn′+ 1 − Pn′− 1 formula
(iv) (n + 1) Pn = Pn′+ 1 − xPn′ 1 dn
Pn(x ) = (x 2 − 1)n we get
(v) (1 − x 2 )Pn′ = n (Pn− 1 − xPn) 2nn ! dx n
1 d 2
(vi) (1 − x 2 ) Pn′ = (n + 1) (xPn − Pn− 1) P0 (x ) = 1, P1(x ) = (x − 1) = x ,
2 dx
2. Beltrami's result is
3x 2 − 1
2 P2 (x ) =
(2n + 1) (x − 1)Pn′ = n (n + 1) (Pn+ 1 − Pn− 1) 2
3. The Christoffel’s Expension is 5x 3 − 3x
P3 (x ) = , Py (x )
Pn′ = (2n − 1) Pn− 1 + (2n − 5) Pn− 3 2
+(2n − 9) Pn− 5 + .... 35x 4 − 30x 2 + 3
= + ...
8

EXERCISE
MULTIPLE CHOICE QUESTIONS 4. P0(x) is equal to :
(a) 1 (b) −1 (c) x (d) −x
1. In the expansion of (1 − 2xh + h2)−1/ 2 in ascending
5. The Legendre’s differential equation is :
power of h, Pn(x) is the coefficient of : (a) [(1 + x 2) D 2 + 2xD + n] y = 0
n −n n −n
(a) x (b) x (c) h (d) h
(b) [(1 − x 2) D 2 − 2xD + n] y = 0
2. The value of Pn(1) is :
(c) [(1 − x 2) D 2 + 2xD + n (n + 1)] y = 0
n 2n
(a) −1 (b) 1 (c) (−1) (d) (−1)
(d) [(1 − x 2) D 2 − 2xD + n (n + 1)] y = 0
3. If n is odd then Pn(0) is equal to :
+1
(a) 0 (b) (−1)n 6. If m ≠ n then ∫ Pm (x) Pn(x) dx is equal to :
−1
(−1)n/ 2 n! 2
(c) (d) None of these (a) 0 (b) 1 (c) −1 (d)
2n 2n + 1
91
1 2 +1
7. ∫−1 P1(x) dx is equal to : 19. If m = n then ∫ Pn(x)2 dx is equal to :
−1
3 1 2 1 1
(a) (b) (c) 2 (d) (a) (b)
2 2 3 2n + 1 2n − 1
8. The generating function of the Legendre's 2 2
(c) (d)
polynomial is : 2n + 1 2n − 1
(a) (1 − 2xh + h2)1/ 2 (b) (1 + 2xh + h2)−1/ 2 20. The value of (n + 1) (nxPn − Pn+1) is :
(c) (1 + 2xh + h ) 2 1/ 2
(d) (1 − 2xh + h ) 2 − 1/ 2 (a) (1 + x 2) Pn (b) (1 − x) Pn

9. The value of Pn(−1) is : (c) (1 + x) Pn (d) (1 − x 2) Pn′


1
(a) 1 (b) −1 (c) (−1)n (d) (−1)2n 21. ∫−1 Pn(x) dx, where n ≠ 0 is equal to :
10. The value os P2(x) is : 3
(a) 0 (b) 1 (c) 2 (d)
2 1 2
2x − 1 3x − 1 22. If m < n then ∫ x m Pn(x) dx is equal to :
(a) 1 (b) −1 (c) (d)
3 2 −1
1
11. The polynomial x + 3 in terms of Legendre’s (a) x (b) x 2 − 1 (c) (d) 0
m+ n
polynomial is : 23. All the roots of Pn(x) = 0 are :
(a) P2(x) + 3 P1(x) (b) 3P1(x) + 2 (a) Same (b) Distinct
(c) 2P2(x) − 2P1(x) (d) P1(x) + 3 P0(x) (c) Two roots are same (d) Zero
1
1 dx 24. ∫−1 xPn(x) Pn−1(x) dx is equal to :
12. Pn(x) = (x 2 − 1)n is called :
2n n! dx n 2 2
(a) (b)
(a) Recursion formula (b) Christophell’s formula 2n2 + 1 4 n2 + 1
2 2
(c) Legendre’s formula (d) Rodrigue’s formula (c) (d)
4n2 − 1 2n2 − 1
13. The value of (n + 1) Pn+1 + nPn−1 is : 1 d n
(a) (n − 1) xPn (b) (2n − 1) xPn 25. The value of [(x 2 − 1)n] is equal to :
2n n! dx n
(c) (2n + 1) xPn (d) None of these (a) Pn(x) (b) Pn(− x) (c) P− n(x) (d) 0
14. The value of xPn′ − Pn′−1 is : 26. The value of Pn′(−1) is :
(a) xPn (b) nPn (c) xPn+1 (d) nPn+1 (−1)n(n + 1)
(a) (−1)n (b)
1 2
15. ∫−1 P0(x) dx is equal to : (−1)n−1 n (n + 1) (−1)n n (n + 1)
(c) (d)
(a) 1 (b) 0 (c) 2 (d) −2 2 2
27. The value of Pn′+1 − Pn′−1 is :
16. If a be the root of Legendre’s polynomial Pn(x) of
(a) nPn (b) (2n − 1) Pn
degree n then Pn+1(a) and Pn−1(a) are of :
(c) (2n − 1) Pn−1 (d) (2n + 1) Pn
(a) Same signs (b) Opposite signs
28. All the roots of Pn(x) = 0 are :
(c) Both zero (d) None of these (a) Real only
17. The value of Pn′(1) is : (b) Complex only
n(n + 1) (c) Real and complex both
(a) n (b)
2 (d) None of these

(c)
n (n − 1)
(d) n2 − 1 29. The polynomial x 2 + x + 1 in terms of Legendre's
2 polynomial :
2 1
18. If n is even then Pn(0) is equal to : (a) P2(x) + P1(x) + P0(x)
n (n + 1) 3 3
(a) (−1)n (b) (−1)n 1
2 (b) P2(x) + P0(x)
3
(−1)n/ 2 n! (−1)n n! 2 4
(c) P2(x) + P1(x) + P0(x)
(c) (d)
2  n  !2 3 3
2n( n / 2!)  
 2 2 1
(d) P2x + P0(x)
3 3
92

1 42. The value of n(Pn−1 − xPn) is :


30. The value of ∫ xP1(x)P0(x) dx is :
−1
(a) (1 − x 2) Pn′ (b) (1 + x 2) Pn
2 3
(a) 0 (b) (c) (d) 1
3 2 (c) (1 − x 2) Pn (d) None of these
1 3
31. 2
) Pm′ Pn′dx , where m, n are distinct the 43. The polynomial x in terms of Legendre
∫−1(1 − x polynomial is :
integers is equal to : 2 3
m+ n m−n (a) P3(x) + P1(x)
(a) 0 (b) (c) m 2 + n (d) 5 5
mn m 2n2 1 2
(b) P3(x) + P2(x) + P(x)
1 5 5
32. The value of ∫ xP2(x) dx is equal to :
−1 1 3 2 2
(c) P3(x) + P2(x) (d) P3(x) + P2(x)
1 1 3 5 5 5
(a) 1 (b) (c) (d) 0
2 3 1
1 44. The value of ∫ x 2P3(x) dx is equal to :
2 −1
33. The value of ∫ [P0(x)] dx is equal to :
−1 3 2
(a) 1 (b) (c) (d) 0
1 3 2 3
(a) 0 (b) (c) 2 (d)
3 2 1
1 45. The value of ∫ P2(x)P3(x) dx is equal to :
34. The value of ∫ P1(x) P2(x) dx is equal to : −1
−1 5 2 3
3 2 (a) (b) (c) (d) 0
(a) (b) 0 (c) 2 (d) 6 5 5
2 3
46. 2P2 + P0 is equal to :
35. The value of P1(x) is :
(a) xP1 (b) 3xP1 (c) 3P1 (d) 3x
3x 2 − 1
(a) (b) 2x − 1 47. The value of xP1′ − P0′ is :
2
(c) x (d) 1 (a) P0 (b) P1 (c) P2 (d) xP1

36. The value of Pn′+1 − xPn′ is : 48. The value of P2′ − xP1′ is
(a) nPn (b) (n + 1) Pn (a) P1 (b) P2 (c) 2P1 (d) 2P2
(c) (n − 1) Pn (d) nPn−1 1 2
49. The value of ∫ (P1′) dx is :
−1
37. The roots of Pn(x) = 0 lies between :
(a) 0 (b) 1 (c) 2 (d) 3
(a) 0 to 1 (b) −1 to +1
1− x
(c) – ∞ to +∞ (d) All zeros 50. The expansion of Pn(x) in powers of is called :
2
38. The polynomial x 2 + 1 in terms of Legendre (a) Legendre formula
polynomial is : (b) Murphy’s formula
3 2
(a) P2( x ) + P0(x) (b) P2(x) + 3P(x) (c) Recurrence formula
2 3
2 4 (d) Christoffel’s summication formula
(c) P2(x) + P1(x) + P0(x)
3 3 51. P8′ + 9P9 is equal to :
2 4 (a) xP9′ (b) (x + 1) P9
(d) P2(x) + P0(x)
3 3 (c) P9′ − x (d) P9′ + x
1 1
39. The value of ∫ x P1(x)P2(x) is :
−1
52. The value of ∫ (1 − x 2) (P0′)2 dx is :
−1
3 4 4 (a) 0 (b) 1 (c) 2 (d) 3
(a) 0 (b) (c) (d)
10 13 15 1 2
1 53. ∫−1 x P2(x)dx is equal to :
40. The value of ∫ (1 − x 2)P1′P2′ dx is :
−1
4 4 4 4
2 3 (a) (b) (c) (d)
(a) 0 (b) 1 (c) (d) 3 7 15 13
3 2
54. P2′ (1) is equal to :
41. The value of P3(1) is equal to :
(a) 0 (b) 1 (c) 2 (d) 3
(a) 1 (b) −1 (c) 5 (d) 3
93

55. If −1 ≤ x ≤ 1 then : 59. P3′ − xP2′ is equal to :


(a) 0 ≤ Pn(x) ≤ 1 (b) −∞ ≤ Pn(x) < ∞ (a) 2P3 + P2 (b) 3P2
(c) |Pn(x)|≤ 1 (d) 1 ≤ |Pn(x)| (c) 3P1 (d) 3P0
56. If Pn(x) and Qn(x) are Legendre’s function of first 60. P3′ − P1′ is equal to :
and second kind the general solution of Legendre’s (a) 5P2 (b) 3P2
equation is :
(c) 5P0 (d) 3P0
(a) APn(x) (b) BQn(x)
(c) APn(x) + BQn(x) (d) APn(x)Qn(x) 61. The value of Pn′(0) is :
1 π (a) nPn(0) (b) nPn−1(0)
57. Pn(x) = ∫ [x ± (x 2 − 1) cos φ]n dφ, where n is
π 0 (c) (n + 1) Pn(0) (d) None of these
positive integer is called : 62. The value of Pn′(0) is :
(a) Recurrence formula (b) Laplace first integral (a) nPn+1 (b) (n + 1) Pn+1(0)
(c) Beltrami’s result (c) −(n + 1) Pn+1 (d) nPn(0)
(d) Laplace second integral
63. P5(0) is equal to :
1 π dφ 2 5 1
58. Pn(x) = , where n is
π ∫0 [x ± (x 2 − 1) cos φ]n+1 (a) 0 (b)
3
(c)
3
(d)
2
positive integer is called : 64. P4 (0) is equal to :
(a) Laplace first integral (b) Legendre first integral 2 5 3
(a) 0 (b) (c) (d)
(c) Laplace second integral 3 8 8
(d) Christoffel’s formula

ANSWERS
MULTIPLE CHOICE QUESTIONS
1. (c) 2. (b) 3. (a) 4. (a) 5. (d) 6. (a) 7. (d) 8. (d) 9. (c) 10. (d)
11. (d) 12. (d) 13. (c) 14. (b) 15. (c) 16. (b) 17. (b) 18. (c) 19. (c) 20. (d)
21. (a) 22. (d) 23. (b) 24. (c) 25. (a) 26. (c) 27. (d) 28. (a) 29. (c) 30. (b)
31. (a) 32. (d) 33. (c) 34. (b) 35. (c) 36. (b) 37. (b) 38. (d) 39. (d) 40. (a)
41. (a) 42. (a) 43. (a) 44. (d) 45. (d) 46. (b) 47. (b) 48. (c) 49. (c) 50. (b)
51. (a) 52. (a) 53. (c) 54. (d) 55. (c) 56. (c) 57. (b) 58. (c) 59. (b) 60. (a)
61. (b) 62. (c) 63. (a) 64. (d)

HINTS AND SOLUTIONS


∞ ∞
2. Since Σ hnPn(x) = (1 − 2xh + h2)−1/ 2 3. Since Σ hnPn(x) = (1 − 2xh + h2)−1/ 2
n= 0 n= 0
∞ Put x = 0 ΣhnPn(0) = (1 + h2)−1/ 2
Put x = 1, Σ hnPn(1) = (1 − 2h + h2)−1/ 2 = (1 − h)−1
n= 0 = [1 − (− h2)]−1/ 2
∞ 1 1.3
= 1 + h + h2 + ... + = Σ hn = 1+ (− h2) + (− h2)2
n= 0 2 2. x
1.3.5... (2r − 1)
Equating the coefficient of hn on both sides we get + ... + (− h2)r + ...
2.4...2r
Pn(1) = 1
Equating the coefficient of h4 on both sides we have
Pn(0) = 0 if n is odd.
94

1 2 1.3.5.... (2m − 1) (−1)m 2m !


7. ∵ ∫ Pm (x) Pn(x) dx = P2m (0) = = (−1)m
−1 2x + 1 2.4.6. .... 2m 2 (m !)2
2m

when m = n, put m = n = 1 Put n = 2m we get


1 2 2
So ∫−1 [P1(x)]2 dx = = (−1)n/ 2 n!
2+ 1 3 Pn(r) = .
2
n
9. ∵ ΣhnPn(x) = (1 − 2xh + h2)−1/ 2, put h → − h 2n  !
2 
Σ(−1)n hnPn(x) = (1 + 2xh + h2)−1/ 2 ...(1)
20. This is a recurrence formula.
Again put x → − x in first equation
1 dn

1 21. By Rodrigue's formula Pn(x) = (x 2 − 1)n
n
Σh Pn(− x) = (1 + 2xh + h ) 2 2 ...(2) 2 n! dx n
n

Integrating from −1 to +1
But equation (1) and (2)
n
Σh Pn(− x) = Σ(−1) h Pn(x)n n 1 1 1 dn 2
∫−1 Pn(x) dx = 2n n! ∫−1 dx n (x − 1)n dx
n
Equating coefficient of h on both sides
1
Pn(− x) = (−1)n Pn(x) 1  d n− 1 2 
= (x − 1)n  = 0
n 
⇒ Pn(−1) = (−1)n 2 n!  dx n − 1
 −1
10. Rodrigue’s formula is 24. From Recurrence formula I
1 dx
Pn(x) = (x 2 − 1)n (n + 1) Pn+1(x) + nPn−1(x)
2n n! dx n xPn(x) =
2n + 1
1 d2
So, P2(x) = (x 2 − 1)2 1 1
2 2! dx 2
2 ∴ ∫1 xPn(x) Pn−1(x) dx = 2n + 1
1 d
= [4x(x 2 − 1)] 1
8 dx ∫−1[(n + 1) Pn+1(x) + nPn−1(x)] Pn−1(x) dx
1 1 1 1
= (12x 2 − 4) = (3x 2 − 1) = [(n + 1)∫ Pn+1(x)Pn−1(x) dx
8 2 2n + 1 −1
1
11. ∵ P0(x) = 1, P1(x) = x + n∫ [Pn−1(x)2 dx]
−1
So, f (x) = x + 3 = P1(x) + 3P0(x)
1  2  2n
1 1 = 0 + n =
15. 2n + 1  2 (n − 1) + 1  4n2 − 1
∫−1 P0(x)dx = ∫−11 dx = 2
26. Since Pn(x) is the solution of Legendre’s differential
16. By recurrance formula I
equation
(2x + 1) xPn(x) = (n + 1) Pn+1(x) + nPn−1(x)
Given a is root of Pn(x) so Pn(a) = 0. so, (1 − x 2)Pn′′(x) − 2xPn′(x) + n (n + 1) Pn(x) = 0
Put in first equation (x = a) Put x = −1 we get,
(2n + 1) a . 0 = (n + 1) Pn+1(a) + nPn−1(a) 2Pn′(−1) + n (n + 1) Pn(−1) = 0
Pn+1(a) −n 1 1
or = Pn′(−1) = − n(n + 1) Pn(−1) = − n (n + 1) (−1)n
Pn−1(a) n + 1 2 2
n− 1 n
= (−1) (n + 1)
Since n is +ve integer so R.H.S. is negative i. e., 2
Pn+1(a) and Pn−1(a) must be of opposite signs.
29. Given polynomial f (x) = x 2 + x + 1
1
∞ −
18. Since Σ hnPn(x) = 1 − 2 xh + h2
n= 0
( ) 2 and we know that P0(x) = 1, P1(x) = x,
3x 2 − 1
1 1 P2(x) =
− − 2
Put x = 0, Σhn Pn(0) = (1 + h2) 2 = [1 − (− h2)] 2
2 1
(− h)2 1.3 1.3.5 ... (2r − 1) (h2)r So, x 2 = P2(x) +
= 1+ + (− h2)2 + ... 3 3
2 2.4 2.4...2r
2 1
∴ f (x ) = x 2 + x + 1 = P2(x) + + P1(x) + 1
Equating the coefficient of h2m on both sides we get 3 3
95

2 4 5x 3 − 3x
or f (x) = P2(x) + P1(x) + P0(x) 44. Put P3(x) = and then integrating
3 3 2
1 2
31. ∫−1(1 − x ) Pm′ Pn′dx = [(1 − x 2)Pm′ Pn]1−1 we get the required result.
1 d 45. By orthogonal property
− ∫ Pn [(1 − x 2)Pm′ ] dx
−1
dx 1
1 d ∫−1 Pm(x)Pn(x) dx = 0 if m ≠ n
= − ∫  Pn {(1 − x 2)Pm′ } ...(1)
−1  dx 
Put m = 2, n = 3 we get
1
By Legendre’s equation
d
∫−1 P2(x) P3(x)dx = 0
{(1 − x 2)Pm′ } = − m (m + 1) Pm so by equ. (1)
dx 48. By Recurrence formula Pn′+1 − xPn′ = (n + 1) Pn
1 2 1 Put n = 1 we get P2′ − xP1′ = 2P1
∫−1(1 − x ) Pm′ Pn′dx = − ∫ [− Pnm(m + 1) Pm ] dx
−1
51. By recurrence formula nPn = xPn′ − Pn′−1
1
= m (m + 1) ∫ PnPm dx = 0 (∵m ≠ n) Put n = 9, 9P9 = xP9′ − P8′
−1

33. Put P0(x) = 1 we get required result. i. e. P8′ + 9P9 = xP9′

36. By Recurrence formula Pn′+1 − xPn′ = (n + 1) Pn 3x 2 − 1


54. ∵ P2(x) =
2
38. Given polynomial is f (x) = x 2 + 1
⇒ P2′(x) = 3x
1
∵ P0(x) = 1, P2(x) = (3x 2 − 1)
2 ∴ P2′(1) = 3
2 59. By Recurrence formula
So, 3x − 1 = 2P2(x)
2P2(x) + 1 Pn′+1 − xPn′ = (n + 1) Pn
⇒ x2 =
3 Put n = 2 we get
2P2(x) + 1 2P (x) 4 P3′ − xP2′ = 3P2
So, f (x) = +1= 2 +
3 3 3
60. By Recurrence formula Pn′+1 − Pn′−1 = (2n + 1) Pn
2 4
= P2(x) + P0(x)
3 3 Put n = 2 we get P3′ − P1′ = 5P2

39.
1
∵ ∫ xPn(x)Pn−1(x)dx =
2n 61. By Recurrence formula (1 − x 2)Pn′(x) = n [Pn−1(x)
−1 2
4n − 1 −xPn(x)]
So put n = 2 we get Put x = 0 we get Pn′(0) = nPn−1(0)
1 4 4 62. By Recurrence formula
∫−1 xP2Pn(x) = 16 − 1 = 15
(1 − x 2)Pn′ = (n + 1) (xPn − Pn+1)
5x 3 − 3x
41. ∵ P3(x) = by Rodrigue’s formula
2 Put x = 0 we get Pn′(0) = −(n + 1) Pn+1(0)
5− 3 63. Since Pn(0) = 0 for all odd value of n so P5(0) = 0
So, P3(1) = =1
2 (−1)n/ 2 n!
3
64. Since Pn(0) = for n is even.
5x − 3x 2
n
43. ∵ P3(x) = , 2n  !
2 2 
3x 2 − 1 n! 4.3.2 3
P2(x) = , P1(x) = x, P0(x) = 1 So P4 (0) = = =
2
16(2)2 16 × 4 8
2P3 + 3x 2 3
So, x3 = = P3(x) + P1(x) mmm
5 5 5
96
Unit-III
C HAPTER
12 Bessel’s Functions
BESSEL’S EQUATION y = AJ − n(x ) + BJ n(x )
1. The differential equation of the form where A and B are arbitrary constants.
d 2 y 1 dy  n2 
+ + 1 −  y=0 RECURRENCE FORMULAE OF Jn (x)
dx 2 x dx  x2
1. The recurrence formulae of J n(x ) are
is called Bessel’s differential equation.
(i) xJ n′ (x ) = nJ n(x ) − xJ n+ 1(x )
2. Consider the Bessel’s differential equation
(ii) xJ n′ (x ) = −nJ n(x ) + xJ n− 1(x )
d 2 y 1 dy  n2 
+ + 1 −  y=0 ...(1) d n
dx 2 x dx  x2 or (x J n) = x n J n−1
dx

Assume that its series solution is y = Σ a rx k + r (iii) 2 J n′ (x ) = J n− 1(x ) − J n+ 1(x )
r=0
∞ (iv) 2nJ n(x ) = x [ J n− 1(x ) + J n+ 1(x )]
dy
then = Σ a r(k + r ) x k + r− 1 d −n
dx r= 0 (v) [x J n(x )] = − x − n J n+ 1(x )
dx
d 2y
and = Σa r(k + r ) (k + r − 1)x k + r− 2
2
dx GENERATING FUNCTION FOR Jn (x)
Putting these values in equation (1) we get
1. When n is a positive integer, J n(x ) is the
Σa r[{(k + r )2 − n 2 } x k + r− 2 + x k + r] = 0  1
e x z − 
Case I : If k = n we get 2
coefficient of Z n in the expansion of 
xn  x2 2
y= 1 + (−1) 2
2n !(n + 1)  2 1 ! (n + 1) in ascending and descending powers of z.

x4  2. J n(x ) is the coefficient of z − n multiplied by (−1)n


2
+(−1) + ...  1
4
2 2 ! (n + 1) (n + 2)  x  z−  / 2
in the expansion of e  z .
This is known as Bessel’s function of the first  1
xz − 
kind of order n.  2

n+ 2 r

r x 1 Thus, e 2 = Σ z n J n(x )
So, J n(x ) = Σ (−1)   −∞
r=0  2 r ! + (n + r + 1)
Case II : If k = −n we get ORTHOGONAL PROPERTIES
∞ −n+ 2r
x 1  0 α ≠β
J − n(x ) = Σ (−1)r   1
r=0  2 r ! + (−n + r + 1) ∫0 xJ n(αx ) J n(βx ) dx =  1 [ J n+1(α)]2 α =β
 2
The solution of Bessel’s equation are called
Bessel’s functions. where α, β are the roots of J n(x ) = 0.

3. When n is not an integer the most general


solution of Bessel’s equation is
97

EXERCISE
MULTIPLE CHOICE QUESTIONS  1
x z−
 z 
1. The Bessel’s equation is :
11. The coefficient of z n in the expansion of e 2 is :
d 2y xdy
(a) + + x 2y = 0 (a) Pn(x) (b) Pn′(x) (c) J n(x) (d) J n′ (x)
2 dx
dx
12. If α, β are the roots of J n(x) = 0 and α ≠ β then
d 2y 1 dy  n2  1
(b) + + 1 −  y = 0
dx 2 x dx 

x2 ∫0 xJ n(αx)J n(βx) dx is :
d 2y xdy n2 (a) 0 (b) 1
(c) + − y = 0 (d) None of these
2 dx x 2 (c) [J n+1(α)]2 (d) J n(α) + J n(β)
dx
a 2
2. If n is a positive integer then J n(x) is equal to : 13. The value of where α is roots of
∫0 xJ n(αx) dx
(a) J − n(x) (b) J n(− x)
J n(x) = 0 :
(c) (−1)n J n(x) (d) None of these
a2 2
(a) 0 (b) J n+1(aα)
3. If n is even then J n(x) is : 2
(a) Odd (b) Even 2
J n+1(aα)
(c) aJ n+1(aα) (d)
(c) May be odd or even (d) None of these 2
4. J n(− x) is equal to : 14. The most general solution of Bessel’s equation is :
(a) J − n(x) (b) −J n(x) (a) AJ n(x) (b) AJ − n(x)
(c) (−1)n J n(x) (d) (−1)n+1 J n(x) (c) AJ n(x) + BJ − n(x) (d) AJ n(− x) + BJ n(x)
5. nJ n(x) − xJ n+1(x) is equal to: 15. The solution of the differential equation
(a) J n′ (x) (b) − xJ n′ (x) d 2y 1 dy
(c) − J n′ (x) (d) x J n′ (x) + + y = 0 is :
dx 2 x dx
6. J n−1 − J n+1 is equal to : (a) J n(x) (b) J1(x) (c) J 2(x) (d) J 0(x)
(a) 2J n (b) 2J n′ (c) 2 J n+1 (d) J n′ −1 ∞ r 2r
(−1) x
16. Σ is equal to :
7. J 1 (x) is equal to : r= 0 (2r r !)2
2
(a) J 0(x) (b) J1(x) (c) J n(x) (d) J r (x)
πx  2 
(a)   sin x (b)   cos x
17. If n is odd then J n(x) is:
 2   πx  (a) Even (b) Odd
πx 2  (c) May be even or odd (d) 0
(c) cos x (d)   sin x
2  πx  d −n
18. (x J n) is equal to :
dx
8. − nJ n + xJ n−1 is equal to :
(a) − x − nJ n+1 (b) x − nJ n+1
(a) xJ n(x) (b) −xJ n(x)
(c) −x nJ n (d) x − nJ n
(c) xJ n′ (x) (d) − xJ n′ (x)
19. The value of J −1/ 2(x) is :
9. The value of [J1/ 2(x)]2 + [J −1/ 2(x)]2 is :
πx  2 
(a)   cos x (b)   cos x
(a) 0 (b) 1  2   πx 
2 πx
(c) (d) 2 πx
πx 2 (c) sin x (d) sin x
πx 2
10. J − n(− x) is equal to :
20. The coefficient of z − n multiplied by (−1)n in the
(a) J n(− x) (b) −J n(x)  1
x  2− 
(c) (−1)n J n(x) (d) (−1)n−1 J n(x)  2
expansion of e 2 is :
98

(a) Pn(x) (b) J n(x) 32. J 2 − J 0 is equal to :


n n
(c) (−1) J n(x) (d) (−1) Pn(x) (a) J 0′ (b) 2J 0′ (c) 2J 0′′ (d) −2J 0′

21. If α,β are the roots of J n(x) = 0 and α = β then 33. For all n ≥ 1 :
1 1
(a) |J n(x)|≤ 1 (b) |J n(x)|≤
∫0 xJ n(αx) J n(βx) dx is : 2
1 (c) 0 ≤ J n(x) ≤ 1 (d) J n(x) = 0
(a) 0 (b) [J n+1(α)]2
2
34. J 0(x) − J 2(x) is equal to :
J (α)
(c) J n(α)J n(β) (d) n (a) 2J1(x) (b) J1′ (x)
J n(β)
(c) 2J1′ (x) (d) − J1′ (x)
22. J 0′ is equal to : d
35. [xJ1(x)] is equal to :
(a) J1 (b) −J1 (c) J 2 − J1 (d) J 2 + 2J1 dx
23. x [J n−1 + J n+1] is equal to : (a) xJ 0(x) (b) J1(x)
(a) nJ n (b) −nJ n (c) 2nJ n (d) −2nJ n (c) xJ1(x) (d) None of these
J′ 36. J n(x) is Bessel’s polynomial then :
24. J 0′′ − 0 is equal to :
x (a) J n(x) = (−1)n J − n(x) (b) J n(x) = (−1)n J n(x)
(a) J 0 (b) J 2 (c) J 3 (d) J 2′ (c) J n(x) = − J − n(x) (d) None of these
25. For n ≥ 1, the result is : 37. The value of J 2(− x) is equal to :
(a) |J 0(x)|≤ 1 (b) 0 ≤ J 0(x) ≤ 1 (a) J 2(x) (b) −J 2(x)
(c) |J 0(x)|≤ ∞ (d) J 0(x) = 0 (c) J −2(x) (d) None of these
d n 38. J −2(x) is equal to :
26. (x J n) is equal to :
dx (a) J 2(− x) (b) −J 2(x)
(a) x nJ n (b) x nJ n+1 (c) −x nJ n (d) x nJ n−1 (c) J 2(x) (d) None of these
x n J1(x)
27. ∫0 x J n−1(x) dx is equal to : 39. lim is equal to :
x→ 0 x
(a) x − nJ n(x) (b) x nJ n(x) (a) 1 (b) 2 (c)
1
(d) 0
2
(c) xJ n+1(x) (d) xJ n(x)
d
π/2 40. [xJ1(x)] is equal to :
28. ∫0 πx J1/ 2(2x) dx is : dx
1 2 (a) J 0(x) (b) J1(x) (c) −J 0(x) (d) xJ 0(x)
(a) 0 (b) (c) (d) 1
π π
41. x[J1 + J 3] is equal to :
29. All the roots of J n(x) = 0 are :
(a) J 2 (b) 2J 2 (c) 3J 2 (d) 4J 2
(a) Same
(b) Distinct If a and b are root of J n(x) = 0 then
(c) More than one are repeated 1 x
42. ∫0 xJ 2(a ) J 2(bx ) dx is equal to :
(d) Repeated only at x = 0
30. The value of J n(x) − xJ 2(x) is : (a) 0 (b) a 2 (c) b2 (d) a 2 + b2
2
(a) xJ1(x) (b) J1′ (x) 43. If J n+1(x) = J n(x) − J 0(x) then n is :
x
(c) xJ1′ (x) (d) − xJ1′ (x)
(a) 0 (b) 1
J n(x) (c) 2 (d) None of these
31. lim x → 0 for n > −1 is equal to :
xn 1
44. ∫ x J 2(x) dx is equal to :
2n 1
(a) (b)
t(n + 1) 2n t(n + 1) J (x ) J (x )
(a) − 1 + c (b) 1 + c
t (n + 1) x x
(c) (d) 2n t (n + 1) J (x )
2n (c) −1 + c (d) J 0(x) + c
x
99

45. The value of ∫ x 2J1(x) dx is : d


46. [J 0(x)] is equal to :
dx
(a) x 2J1(x) + c (b) x 2J 2(x) + c (a) J 0′ (x) (b) J1′ (x)
2
(c) x J −1(x) + c (d) None of these (c) −J1(x) (d) −J 2(x)

ANSWERS
MULTIPLE CHOICE QUESTIONS
1. (b) 2. (c) 3. (b) 4. (c) 5. (d) 6. (b) 7. (d) 8. (c) 9. (c) 10. (c)
11. (c) 12. (a) 13. (b) 14. (c) 15. (d) 16. (a) 17. (b) 18. (a) 19. (b) 20. (b)
21. (b) 22. (b) 23. (c) 24. (b) 25. (a) 26. (d) 27. (b) 28. (d) 29. (d) 30. (c)
31. (b) 32. (c) 33. (b) 34. (c) 35. (a) 36. (a) 37. (a) 38. (c) 39. (c) 40. (d)
41. (d) 42. (a) 43. (d) 44. (a) 45. (b) 46. (c)

HINTS AND SOLUTIONS


n+ 2r
∞ x 1 x4 
2. ∵ J − n(x) = Σ (−1)r   + 
r= 0  2 r ! τ (− n + r + 1) 2. 4(2n + 2) (2n + 4) 

If p is integer then τ(−p) is ∞ for p ≥ 0 so terms in J − n 1


Putting n = − we get
equal to zero till − n + r + 1 < 1 i. e., r < n. Hence 2
∞ (−1)r x
− n+ 2r x  x2 x4 
J − n(x) = Σ   J −1/ 2(x) = 1 − + + ...
r = n r ! τ(− n + r + 1)  2   1  2! 2.41
. .3 
2+   
 2
n+ 2s
∞ (−1)n+ s x
= Σ =   put r = n + s 2  x2 x4  2
s =0 n + s !+ τ (s + 1)  2  J −1/ 2(x) = 1 − + ... = cos x
πx  2! 4!  πx
n+ 2s
(−1)s x
= (−1)n Σ   = (−1)n J n(x) 2
τ (n + s + 1) s !  2  Similarly J1/ 2(x) = sin x
πx
xn  x2 2
7. ∵ J n(x) 1 − So J12/ 2(x) + J −21/ 2(x) =
n 2(2n + 2) πx
2 (n + 1) 

x4  11. Since by generating function J n(x) is the coefficient


+ 
2n (2n + 2) (2n + 4)  e x ( z −1/ z)
of z n in the expension of in ascending or
1 2
Putting n = we get descending power of z for all positive integer n.
2
x 1/ 2  x2 x4  13. By orthogonal property of Bessel’s functions
J1/ 2(x) = 1 − + 
3 2.3 2.4.3.5  0, α ≠β
21/ 2 τ    1
 2 ∫0 xJ n(αx) J n(βx) dx =  1 [J n+1(α)]2, α =β
2
2  x.3 x 5  2
= x − + .... = sin x
where α, β are the roots of J n(x) = 0.
πx  3! 5!  πx
18. By Recurrence formula
8. By Recurrence formula − nJ n(x) + xJ n−1(x) = xJ n′ (x) d
{(x J n(x)} = − x − nJ n+1(x)
−n
xn  x2 dx
9. ∵ J n(x) = 1 −
n 2 (2n + 2)
2 τ (n + 1)  21. By Orthogonal property of Bessel’s function.
100

22. By Recurrence formula xJ n′ = nJ n − xJ n+1 37. ∵ J n(− x) = (−1)n J n(x) for all integers so putting n = 2
Put n = 0 we have xJ 0′ = − xJ1 we get J 2(− x) = J 2(x)
i. e. J 0′ = − J1 38. ∵ J − n(x) = (−1)n J n (x) so putting n = 2 we get
24. By Recurrence formula xJ n′ = nJ n − xJ n+1
J −2(x) = J 2 (x)
putting n = 1, xJ1′ = J1 − xJ 2
xn  x2 
But we know that J 0′ = − J1 39. ∵ J n(x) = 1− + ...
n 2 ( 2n + 2)
2 τ(n + 1)  
Differentiating it we get J 0′′ = − J1′ put it in above
equation x  x2 
J′ Putting n = 1, J1(x) = 1 − ...
we get − xJ 0′′ = − J 0′ − xJ 2 or J 2 = J 0′′ − 0 2τ(2)  8 
x
J (x ) 1 1
25. Since we know that J 02 + 2 (J12 + J 22 + ...+ J n2 ...) = 1 or lim 1 = =
x→ 0 x 2τ (2) 2
and since J12, J 22 ... are all non-negative so
40. By Recurrence formula
J 02(x) ≤ 1 or 1J 0(x)2 ≤ 1 or |J 0(x)|≤ 1 d n
(x J n) = x nJ n−1
dx
d n
27. By Recurrence formula x nJ n−1(x) = [x J n(x)] So putting n = 1,
d
(xJ1) = − xJ 0
dx dx
Integrating from 0 to x we get 41. By Recurrence formula
x n n
∫0 x J n−1(x) dx = [x J n(x)]x0 n
= x J n(x) x[J n−1 + J n+1] = 2nJ n

2 Putting n = 2, x[J1 + J 3] = 4J 2
28. ∵ J1/ 2(x) = sin x
πx 42. By Orthogonal property
1 1
⇒ J1/ 2(2x) = sin 2x ∫0 xJ n(αx) J n(βx) dx = 0.
πx
π/2 π/2 1 If α, β are the roots of J n(x) = 0.
∴ ∫0 πx J1/ 2(2x) dx = ∫0 πx . .sin 2x dx
πx
π/2
44. By Recurrence formula
π/2 cos 2x 
= ∫0 sin 2x dx =  − =1 d −n
(x J n) = − x − nJ n+1(x)
 2  0 dx
 xn x2  Integrating both sides
31. J n(x) = 1 − + ...
n 2. (2n + 2) −n
2 τ(n + 1)  
∫x J n+1(x) dx = −[x − nJ n(x)] + c
J n(x) 1  x2 
lim = lim 1 − + ... Putting n = 1, we get
x → 0 xn x → 0 2n(n + 1)  2 ( 2n + 2) 
 −1 J1(x)
1 ∫x J 2(x) dx = − x −1J1(x) + c = − +c
= x
2n τ(n + 1) 45. By Recurrence formula
33. ∵ J 02 + 2 (J12 + J 22 + ... J n2 + ...) = 1 d n
[x J n(x)] = x nJ n−1
dx
for n ≥ 1, 2J n2(x) ≤ 1
Integrating we get
⇒ 2 |J n(x)|2 ≤ 1 n n
∫ x J n−1(x) dx = x J n(x) + c
1
or |J n(x)|≤ ⇒ |J n(x)|≤ 2−1/ 2 Putting n = 2 we get ∫ x 2J1(x) dx = x 2J 2(x) + c
2
34. By Recurrence formula J n−1(x) − J n+1(x) = 2J n′ (x) 46. By Recurrence formula
d −n
Putting n = 1, J 0 − J 2 = 2J1′ [x J n(x)] = − x − nJ n+1(x)
dx
d n
35. By Recurrence formula (x J n) = x nJ n−1 Putting n = 0 we get
dx
d
d [J 0(x)] = − J1(x)
Put n = 1 we get (xJ ) = xJ 0 dx
dx
mmm
Unit101
-III

C HAPTER
13 Series Solutions of Differential Equations
INTRODUCTION d 2y
and = Σn (n − 1) C nx n− 2
2
1. Consider the linear differential equation of the dx
second order If P(x ) and Q(x ) are not polynomials in x, then
d 2y dy they can be expanded as
+ P(x ) + Q(x ) y = 0 ...(1)
dx 2 dx ∞ ∞
P(x ) Σ Pnx n and Q(x ) = Σ qnx n
Where P and Q are functions of x. Then there n= 0 n= 0
exists the following points. Putting all these values in equation (1) and
(i) The point x = 0 is called ordinary point of equating to zero the coefficient of various
(1) If P(x ) andQ(x ) do not become infinite in powers of x we determine all coefficients of
the neighbourhood of the origin and they can (2). Putting these values in (2) we get the
be expanded in the sum of power series. general solution of (1).
(ii) The point x = 0 is called a singular point of 3. Solution near a regular singular point
the differential equation (1) if it is not an d 2y dy
Consider ≠ P(x ) + Q(x )y = 0 ...(1)
ordinary point. These are of two types. dx 2 dx
(a) Regular singular points (b) Irregular Let its trial solution is
singular points. ∞
y = x m Σ C nx n with C 0 ≠ 0
The origin is called regular singular point if n= 0
both xP(x ) and x 2 Q(x ) can be expanded in dy d 2y
power series of x. Otherwise x = 0 is called an Find and and substitute in (1) then it
dx dx 2
irregular singular points.
reduces to an identity in x. By equating to zero
FROBENIUS METHOD the coefficients of the lowest power of x in this
identity we get quadratic equation in m called
1. The method of finding a solution in a series for
indicial equation. It determines m.
the differential equation (1) near a regular
singular point or near an ordinary point is Now equating to zero the next coefficients of
called frobenius method. various powers of x we can determine all the
coefficients C 1, C 2 , ... in terms of C 0 .
2. Solution near an ordinary point
d 2y dy 4. Consider y ′′ + P(x ) y ′ + Q(x ) y = 0 and P and
Consider + P(x ) + Q(x ) y = 0 ...(1) Q are not analytic (P = ∞ or Q = ∞) at x = 9
2 dx
dx
∞ then x = a is not ordinary point but a singular
Let its trial solution is y = Σ C nx n ...(2) point. If (x − a) P and (x − a)2 Q are infinite at
n= 0
dy x = a then it is called irregular singular point. If
Then = ΣnC nx n− 1
dx (x − a) P and (x − a)2 Q are not infinite at x = a
then it is called regular singular point.
102

EXERCISE
MULTIPLE CHOICE QUESTIONS 10. If y = ΣCr x m + r be the solution of 9x(1 − x) y ′′
2
d y dy −12y′+4y = 0 then roots of indicial equations are :
1. If x = 0 is not an ordinary point of +P
dx 2 dx 3 7
(a) 0, 1 (b) 0, (c) 0, 3 (d) 0,
+ Qy = 0 then it is : 7 3
(a) Regular point (b) Singular point 11. The indicial equation of (D 2 − 1) y = x is :
(c) Essential point (d) None of these
C0 C1
(a) C2 = (b) C2 =
2. If xP(x) and x 2Q(x) can be expanded in a power 2 2
series of x in the nbd of x = 0 then x = 0 is called :
(c) m 2 = 0 (d) m (m − 1) = 0
(a) Regular point (b) Singular point
(c) Ordinary point (d) Regular singular point 12. The singular point of x (x − 1) y ′′ + xy′+ x 2y = 0 is :
3. Singular points of a differential equation are : (a) 0 (b) 1
(a) Unique (b) Two types (c) 2 (d) None of these
(c) Three types (d) May be infinite 13. The singular point of (x − 1)y ′′ + xy′+2y = 0 is :

4. If we put y = Σ Cr x m + r in [xD 2 + (1 + x) D + 2] y (a) 3 (b) 2 (c) 1 (d) 0
i= 0
2
= 0 then indicial equation is : 14. For the equation x (x − 3) y ′′ + 2 (x − 3) y′+ xy = 0
2 the regular singular points are :
(a) C0 (m − 1) (b) C0 m (m − 1)
(c) C0 m 2
(d) C0 m (m + 1) (a) 0 (b) 3 (c) 2 (d) 0, 3
15. For the solution of x 2(x − n) y ′′ + (x − n) y′+ y = 0 at
5. The roots of indicial equation of [x 2D 2 + xD
+ (x 2 − 1)] y = 0 are : x = 0 choose y such that :
∞ ∞
(a) 0, 1 (b) 0, − 1 (c) 1, − 1 (d) 0, 0 (a) y = Σ Cr x m + 2r (b) Σ Cr x m − 2r
r= 0 r= 0
6. The indicial equation of [(1 − x 2) D 2 + 2xD +1] y = 0 ∞ ∞
(c) Σ Cr x m + r (d) Σ Cr x m + r −1
is : r= 0 r= 0
(a) C0 m 2 = 0 (b) C0 m (m − 1) = 0
d 2y
16. For the solution of differential equation (x − x 2)
(c) C0 (m − 1)2 (d) C0 m (m + 1) dx 2
dy
7. The indicial equation of [xD 2 + D + x] y = 0 is : + (1 − 5x) − 4y = 0, y is equal to :
dx
(a) C0 m 2 = 0 (b) C0(m − 1)2
(a) ΣCr x r (b) ΣCr x m + r
2
(c) C0(m + 1) (d) C0(m − 1) (m − 2)
(c) ΣCr x m + 2r (d) None of these
8. For the differential equation (1 − x 2) y ′′ − 2xy′
17. The irregular singular point of (x − 1) (x − 2)3 y ′′
+ p (p + 1) y = 0 the point x = 0 is :
(a) Regular (b) Singular + (x − 1)2 y′+3 (x − 1) y = 0 is :

(c) Ordinary (d) Essential (a) 0 (b) 1 (c) 2 (d) 3


2 2
9. If x = 1 be the ordinary point of y ′′ + (x − 1)2 y′ 18. For the equation (D + x ) y = 0, x = 0 is :
− y (x − 1) y = 0 then the corresponding equation (a) Ordinary point (b) Regular point
whose ordinary point x = 0 is : (c) Singular point (d) None of these
(a) (D 2 + x D
2
− 4x) y = 0 19. The ordinary point of y ′′ −
x
y′+
y
y = 0 is :
2 1− x2 1− x2
(b) (D − 4D) y = 0
(a) 0 (b) 1
(c) (D 2 + xD 2 − 1) y = 0
(c) −1 (d) ± 1 both
(d) None of these
103

1 1 26. The indicial equation of [x 2D 2 + xD + (x 2 − 4)] y = 0


20. At x = 0 the differential equation y ′′ + y′ + y
2 3
x x is :
= 0 has :
(a) C0(m − 2)2 (b) C0(m + 2)2
(a) Regular singular point
(c) C0(m + 2) (m − 2) (d) C0m (m − 2)
(b) Irregular singular point
(c) Ordinary point 27. The roots of the indicial equation of (x 2D 2 + 5xD
(d) Extreme point + x 2) y = 0 are :

21. At x = 1 the equation x (x − 1) y ′′ + xy′+ x 2y = 0, has : (a) 0, 1 (b) 0, 2 (c) 0, 4 (d) 0, −4



(a) Singular point (b) Ordinary point 28. If y = Σ Cr x m + r be the solution of [(x − x 2) D 2
r= 0
(c) Regular point (d) None of these
+ (1 − 5x) D − 4] y = 0 then its indicial equation is :
22. The singular point of x 2(x − 3) y ′′+ 3 (x − 3) y′+5y = 0 (a) C0 m (m − 1) = 0 (b) C0 (m + 1) (m − 1) = 0
is : (c) C0 m (m − 2) = 0 (d) C0 m 2
(a) 1 (b) 0 (c) 3 (d) 5 ∞
23. For (x − 1)y ′′ + xy′+3 = 0, x = 0 is : 29. If y = Σ Cr x m + 2r be the solutions of (2x + x 3) D 2y
r= 0
(a) Singular point (b) Regular singular point − Dy − 6xy = 0 then its indicial equation is :
(c) Irregular singular point (a) C0 m (m − 2) = 0 (b) C0 m (2m − 3) = 0
(d) Ordinary point
(c) C0 m (2m − 1) = 0 (d) None of these
x −1
24. The regular singular point of y ′′ + y′ 30. The indicial equation of 2x 2D 2 − xD + (1−x 2) y = x 2
x
(x + 2) is :
+ y = 0 is :
x2 (a) C0 (2m − 1) (m − 1) = 0
(a) 3 (b) 2 (c) 1 (d) 0
(b) C0 (m − 1) (m − 2) = 0
25. The roots of the indicial equation of
(xD 2 + D − 1) y = 0 are : (c) C0 m 2

(a) 0, 0 (b) 0, 1 (c) 0, − 1 (d) 1, − 1 (d) C0 m (m − 1) = 0

ANSWERS
MULTIPLE CHOICE QUESTIONS
1. (b) 2. (d) 3. (b) 4. (c) 5. (c) 6. (b) 7. (a) 8. (c) 9. (a) 10. (d)
11. (a) 12. (b) 13. (c) 14. (d) 15. (c) 16. (a) 17. (c) 18. (c) 19. (a) 20. (b)
21. (a) 22. (b) 23. (d) 24. (d) 25. (a) 26. (c) 27. (d) 28. (d) 29. (b) 30. (a)

HINTS AND SOLUTIONS


4. Given y = ΣCr x m + r be the solution of Equating to zero the coefficient of the lowest power
of x i. e. x m−1 we get indicial equation (1) C0m 2 = 0
d 2y dy
x + (1 + x) + 2y = 0 …(1)
dx 2 dx
5. Put y = ΣCr x m + 2r be the solution of given equation
dy
so = Σ Cr (m + r)x m + r −1 d 2y dy
dx x2 +x + (x 2 − 1) y = 0
2 dx
dx
d 2y
= ΣCr (m + r) (m + r − 1) x m + r − 2 dy
dx 2 then = ΣCr (m + 2r) x m + 2r −1,
dx
put in (1) we get d 2y
= ΣCr (m + 2r) (m + 2r − 1) x m + 2r − 2
ΣCr [(m + r + 2) x m+ r
+ (m + r) x2 m + r −1
]=0 dx 2
104

put these in given equation we get 14. Given equation is


m + 2r
ΣCr [(m + 2r + 1) (m + 2r − 1) x d 2y 2 dy 1
+ + y=0
+x m + 2r + 2
]=0 dx 2 x (x − 3) dx (x − 3)2
2 1
Equating the coefficient of lowest power is x m we get Here P= and Q =
x (x − 3) (x − 3)2
C0(m + 1) (m − 1) = 0
⇒ m = 1, − 1 It is obvious that xP and x 2Q both are not infinite at
8. Given equation is x = 0 so x = 0 is regular singular point.
2
d y dy Also (x − 3) and (x − 3)2 Q both are not infinite at
(1 − x 2) − 2x + p (p + 1) y = 0
2 dx
dx x = 3 so x = 3 is regular singular point.
2
2x dy p (p + 1)
d y 16. Given equation is
i. e. − + y=0
dx 2
(1 − x 2) dx (1 − x 2) d 2y dy
(x − x 2) + (1 − 5x) − 4y = 0
2x p (p + 1) dx 2 dx
Here, P=− ,Q=
2
1− x 1− x2 Put x m in given equation we get
Since at x = 0
(x − x 2)m (m − 1) x m − 2 + (1 − 5x) mx m −1 − 4x m
neither P nor Q → ∞ so x = 0 is an ordinary point
9. Given that x = 1 be the ordinary point of or m 2x m −1 − (m 2 + 4m + 4) x m

d 2y dy Here the common difference of the power is so its


+ (x − 1)2= −4 (x − 1) y = 0 ∞
dx 2 dx solution is y = Σ Cr x m + r
r= 0
dy dy dt dy
put x = t + 1, = − =
dx dt dx dt d 2y x − 1 dy 3
2 2
17. Given that + + y=0
So,
d y
=
d  dy  d  dy  d y dx 2 (x − 2)3 dx (x − 2)3
  =   =
dx 2 dx  dx  dt  dt  dt 2 x −1 3
Here, P= ,Q=
3
So given equation becomes (x − 2) (x − 2)3
2
d y dy and (x − 2) P and (x − 2)2 Q are infinite at x = 2 so,
+ t2 − 4ty = 0
dt 2 dt
and its ordinary point is t = 0 x = 2 is an irregular singular point.

11. Let y = ΣCr x r be the solution of D 2y − y = x 20. Given differential equation is


dy d y 2 d 2y 1 dy 1
So = ΣCr rx r −1 and = ΣCr r(r − 1) x r − 2 2
+
2 dx
+ y=0
dx dx 2 dx x x3
1 1
Put in given equation we get Here, P= ,Q=
2
x x3
ΣCr [r (r − 1) x r − 2 − x r ] − x = 0
and xP and x 2Q are infinite at x = 0 so x = 0 is a
Equating to zero the coefficient of x 0 we get
regular singular point.
2 A2(2 − 1) − A0 = 0
A0 22. Given that x 2(x − 3) y ′′ + 3 (x − 3) y′+5y = 0
⇒ A2 =
2 d 2y 3 dy 5
or + + y=0
12. Given differential equation dx 2 x 2 dx x 2(x − 3)
d 2y 1 dy x 3 5
+ + y=0 Here, P= and Q =
2 x − 1 dx x − 1
dx x2 x 2(x − 3)
1 x
Here, P= ,Q= Also P and Q are not analytic at x = 0
x −1 x −1
i. e. P = ∞, Q = ∞
So (x − 1) P and (x − 1)2 Q
so x = 0 is a singular point.
i. e. 1 and x (x − 1) both are not infinite at x = 1 so
x = 1 is regular point.
105

24. Given differential equation is ∞


2
30. Put y = Σ Cr x m + 2r in given equation we have
d y x − 1 dy x + 2 r= 0
+ + y=0
2
dx x dx x2 dy
= ΣCr (m + 2r) x m + 2r −1,
x −1 x+2 dx
Here, P= ,Q=
x x2 d 2y ∞
= Σ Cr (m + 2r)(m + 2r − 1) x m + 2r − 2
2 2 r= 0
Also xP and x Q are not ∞ at x = 0 so dx
x = 0 is a regular singular point. We get ΣCr [− x m + 2r + 2 + (2m + 4r − 1)

d 2y dy (m + 2r − 1) x m + 2r ] = 0
26. Put y = ΣCr x m + r in x 2 +x + (x 2 − 4) y = 0
2 dx
dx So equating to zero the coefficient of lowest term i. e.
We get Σc r (m + r + 2) (m + r − 2) x m + r x m is
+ ΣCr x m + r + 2 = 0 C0(2m − 1) (m − 1) = 0
mmm
So equating the lowest degree term i. e. x m we get
C0(m + 2) (m − 2) = 0 which is indicial equation.
B.Sc. Objective Mathematics (Differential Equations and Integral Transform)
106
Unit-III
C HAPTER
14 The Laplace Transform
LAPLACE TRANSFORM (i) L {1} =
1
, p> 0
1. The Laplace transform reduces the problem of p
solving a differential equation to an algebraic τ (n + 1)
(ii) L{ t n} = , p> 0
problem. pn+ 1
If the knernel k ( p, t) is defined by n!
(iii) L { t n} = , p > 0 and n is a positive
0 for t < 0 n
p +1
k ( p, t) =  − pt
e for t ≥ 0
integer.
Thenf ( p) = L { F (t)} = F ( p) 1
∞ (iv) L{ e at } = , p> a
pt p− a
= ∫0 e F (t) dt ...(1)
a
is called the Laplace transform of the function (v) L{sin at} = , p> 0
p + a2
2
F (t). It exists if the integral (1) converger for
some values of p, otherwise it does not exist. p
(vi) L {cos at} = , p> 0
2. The Laplace transform is a linear transformation p + a2
2

i. e. a
(vii) L {sinh at} = , p > | a|
L { a 1F1(t) + a 2 F2 (t)} = a 1L { F1(t)} + a 2 L { F2 (t)} p − a2
2

Where a 1, a 2 are constants. p


(viii) L {cosh at} = , p > | a|
3. A function F (t) is said to be of exponential order p − a2
2

α as t → ∞ if there exists a positive constant µ, a


number α and a finite number to such that PROPERTIES OF LAPLACE TANSFORMS
| F (t)| < µe αt or 1. If L { F (t)} = f ( p) when p > α

| e − αt F (t)| < µ for all t ≥ t0 then L { e at F (t)} = f ( p − a), p > α + a

If a function F (t) is of exponential order α then it This is called first translation or shifting theorem.
is also of β for β > α. 2. If L { F (t)} = f ( p) and
4. If F (t) is a function which is piecewise  F (t − a), t > a
G(t) = 
continuous on every finite internal in the range  0, t< a
t ≥ 0 and satisfies | F (t)| ≤ µe at for all t ≥ 0 and
then L { G(t)} = e − ap f ( p)
for some constant a and µ then the Laplace
transform of F (t) exists for at p > a. This is called second translation or shifting
theorem.
5. A function which is piecewise continuous on
every finite interval in the range t ≥ 0 and is of 3. If L { F (t)} = f ( p), then
exponential order as t → ∞ is known as a 1  p
L{ F (at)} = f 
function of class A. a  a
6. Laplace transforms of some elementary functions This is called change of scale property.
are
107

4. Let F (t) be continuous for all t ≥ 0 and be of t 1


L{ ∫ F (x ) dx } = L{ F (t)}, ( p > 0, p > a)
exponential order a as or t → ∞ and if F ′ (t) is of 0 p
class A, then Laplace transform of the 8. If F (t) is a function of class A and if LF (t) = f ( p),
derivative F ′ (t) exists when p > a and then
L{ F ′ (t)} = pL { F (t)} − F (0)
L{ tF (t)} = − f ′ ( p)
Similarly dn
L{ F ′′(t)} = p2 L{ F (t)} − pF (0) − F ′ (0) Similarly L{ t nF (t)} = (−1)n f ( p)
dpn
Continuing in this manners we have
For all n = 1, 2, 3, ...
L{ F n(t)} = pnL{ F (t)} − pn−1F (0)
F (t) ∞
− pn− 2 F ′ (0) −.... − F n− 1(0) 9. If L{ f (t)} = f ( p), then L = ∫p f (x ) dx
 t 
5. Let F (t) be continuous for all t ≥ 0 and be of F (t)
provided lim   exists.
exponential order as t → ∞ and if F ′ (t) is of t→ 0  t 
class A, then ∞
10. Let L{ F (t)} = f ( p) i. e. ∫ e − pt F (t) dt = f ( p)
lim F (t) = lim pL { F (t)} 0
t→ 0 p →∞
taking limit as p → 0, we have
This is called initial-value theorem.

∫0 F (t) dt = f (0)
6. Let F (t) be continuous for all t ≥ 0 and be of
exponential order as t → ∞ and if F ′ (t) is of 11. Let F (t) be a periodic function with period
class A, then lim F (t) = lim pL{ F (t)} T > 0, F (u + T ) = F (u), F (u + 2T ) = F (u) etc.
t→∞ p→ 0
T − pt
This is called final-value theorem. ∫ e F (t) dt
then L{ F (t)} = 0
7. If F (t) is piecewise continuous and satisfies 1 − e − pt
| F (t)| Σµe at for all t ≥ 0 for some constants a
and µ, then

EXERCISE
MULTIPLE CHOICE QUESTIONS 4. L {(5t − 2)} is :
1. The Laplace transform of t is : 5 − 2p 5p − 2
(a) (b)
p2 p2
1 1 2
(a) (b) (c) (d) 1 5 − 2p 2
p3 p2 p2 (c) (d) None of these
p
2. L {cosh at} is equal to :
5. L {t 5e 3t } is :
1 p
(a) (b) 5 60
2 2
p −a p + a2
2 (a) (b)
(p − 3)6 (p − 3)6
p 1 120 15
(c) (d) (c) (d)
p 2 − a2 p 2 + a2 (p − 3)6 (p − 3)6
3. If L{F (t)} = f (p) then L{F (at)} is : cos at 
6. L   is equal to :
p p  t 
(a) f   (b) af  
 a  a p p
(a) cot −1   (b) tan −1  
1 p  a  a
(c) f   (d) None of these
a  a p
(c) sin −1   (d) Does not exist
 a
108

7.
2
The function e t as t → ∞ is of exponential order : n! n!
(c) (d)
(p + a)n+1 (p − a)n+1
(a) 1 (b) 2 (c) 3 (d) Not exist
8. The value of L {sin t cos t} is : 16. The value of L {e −2t (3 cos 6t − 5 sin 6t)} is :
1 2 1 2 p−6 3p − 24
(a) (b) (c) (d) (a) (b)
p2 + 1 p2 + 2 p2 + 4 p3 + 4 p 2 + 4p + 40 p 2 + 4p + 40
3p − 9
e t r< t≤1 (c) (d) None of these
9. If F (t) ⇒  then L{F (t)} is : p 2 + 4p + 40
0 t>1
17. If L {F (t)} = f (p) then L{F (t) cos at} is :
1 − e− p 1 − e − p +1 f (p − ia) − f (p + ia) f (p − a) + f (p + a)
(a) ,p ≠1 (b) ,p ≠1 (a) (b)
p −1 p −1 2 2
1 + e − p +1 (c)
f (p + ia) + f (p − ia)
(d)
f (p − a) − f (p + a)
(c) ,p ≠1 (d) None of these
p −1 2 2

sin t, 0 < t < π 18. L{sin 2t sin 3t} is : [Kanpur 2012]


10. If F (t) =  then L{F (t)} is : 6p 12p
0 t> π (a) (b)
πp πp
(p + 1) (p 2 + 25)
2
(p 2 + 1) (p 2 + 25)
e e +1
(a) (b) p
p2 + 1 p2 + 1 (c) (d) None of these
(p 2 + 1) (p 2 + 25)
e − πp + 1
(c) (d) None of these 19. If L {F (t)} = f (p) then L {e − t F (3t)} is :
p2 + 1
−3 3

1 
11. The value of L   is : e( p +1) e p
 πt  (a) (b)
(p + 1) (p + 1)
1 1
(a) , p > 0 (b) ,p> 0 3 3
p p −
( p +1)
e e p +1
1 p (c) (d)
(c) ,p> 0 (d) ,p> 0 p (p + 1)
πp π
20. The value of L {t sin at} is :
12. The exponential order of t 2 is :
ap 2ap
(a) 1 (b) 2 (c) 3 (d) Not exist (a) (b)
2 2
(p + a ) (p + a 2)
2
t 2
13. L {e sin t} is equal to : 2ap
(c) (d) None of these
2 2 (p 2 + a 2)2
(a) (b)
(p − 1) (p 2 + 5) (p + 1) (p 2 − 2p + 5)
sin t  −1 1 sin at 
2 2 21. If L   = tan then L  is :
(c) (d)  t  p  t 
(p + 1) (p 2 − 5) (p − 1) (p 2 − 2p + 5)
1  a 1 p
(a) tan −1   (b) tan −1  
cos  t − 2 π  , t > 2 π a  p a  a
 
  3 3
14. If F (t) =   a  a
2π (c) a tan −1   (d) tan −1  
0, t<  p  p
 3
Then L {F (t)} is : 22. L {t ne at } is equal to :
p
−2π
p n! n!
−π (a) (b)
3 3
(a)
e
(b)
e p n+1 (p + a)n+1
p (p 2 + 1) p2 + 1 n! n
(c) (d)
−2π
p p (p − a)n+1 (p + a)n+1
pe 3 pe 2π 3
(c) (d) sin ht 
p2 + 1 p2 + 1 23. The value of L   is :
 t 
15. L {t ne at } is : p+1 p −1
(a) log (b) log
n! n − 1! p −1 p+1
(a) (b)
p n+1 (p + a)n+1
109

1 p −1 3 2 3 2
(c) log (d) None of these (a) − + (b) − +
2 p+1 e e2 e2 e
3 2 3 2
24. If L {F (t)} = f (p) and integral is converges then (c) − (d) +
∞ F (t) e2 e e e2
∫0 t dt is : 33. The value of L{t 2e t } is :
∞ ∞ f (x )
(a) ∫ xf (x) dx (b) ∫ dx 2 1
0 0 x (a) (b)
p3 (p − 1)3

(c) ∫ f (x) dx (d) Not exist 2 p
0 (c) (d)
3

(p − 1) (p − 1)3
25. The value of ∫ te − t sin tdt is :
0 ∞ e− x sin x
34. ∫0 dx is equal to :
1 1 x
(a) 0 (b) 1 (c) (d)
2 4 π π
(a) (b) π (c) 0 (d)
∞ sin t 2 4
26. The value of ∫ dt is :
0 t p
35. If L {J1(t)} = 1 − then L {t J1(t)} is :
π p 2
p +1
(a) π (b) (c) (d) 0
2 π
1 1
∞ (a) (b)
27. The value of ∫ te −3t sin t dt is : p2 + 1 p2 + 1
0
1 2 3 4 1 1
(a) (b) (c) (d) (c) (d)
2 3/ 2
25 25 25 25 (p + 1) (p + 1)5/ 2
2

1 ∞
1 t > a
28. If L{J 0(t)} = then ∫ J 0(t) dt is : 36. If F (t) =  then L{F (t)} is :
2 0
1+ p 0 t < a
(a) π (b) ∞ (c) 0 (d) 1 e ap e − ap e− a / p e− p / a
(a) (b) (c) (d)
 t  1 p p p p
29. If L 2  1  is :
 = 3/ 2 then L 
 π  p  πt  sin  t − π  π
  t>
1 1 1 2 37. If F (t) =   3 3 then,L {F (t)} is :
(a) (b) (c) (d) π
p p p p p 0 t<
 3
sin t, r < t < π
30. If F (t) =  and F (t) has a period of e πp / 3 e πp / 3
0, π < t < 2π (a) (b)
p+1 p2 + 1
2π then L{F (t)} is :
1 1 e − πp / 3
(a) (b) (c) (d) None of these
(p 2 − 1) (1 − e − pπ ) 2
(p + 1) e πp
p2 + 1
1 1
(c) (d) 38. The value of L {t ne nt } is :
(p + 1) (1 + e − pπ )
3 2
(p + 1) e − πp
n! n!
−2t 
(a) (b)
n+1
31.
1 − e
The value of L  p (p − n)n+1
 is :
 t  n! n!
(c) (d)
2
(a) log  1 +  (b)
log 2 (p + n) n+1 (p − n)n
 p p
39. L {t sin at} is equal to :
2  2
(c) log  1 −  (d) log  1 +  (a)
2ap
(b)
2p
 p  p2  2
(p + a ) 2
(p + a 2)2
2

0, 0 < t < 1 2ap


(c) (d) None of these
32. If F (t) = t, 1 < t < 2 then, L{F (t)} is under lim is : (p + a 2)2
2
p→1
0, 2 < t
110

t sin x 1 1
40. L ∫ dx  is equal to : (a) (b)
 0 x  (p + 3) p − 2 (p − 3) p + 2
cot −1 p tan −1 p 1 1
(a) (b) (c) (d)
p p (p − 3) p − 2 (p − 3) p

sin p sin −1 p 44. If L{F (t)} = f (p) then L{F (at)} is :


(c) (d)
p p 1 p
(a) f (ap) (b) af  
a  a
∞ e− t − e −3t
41. The value of ∫ dt is : 1  p 1  a
0 t (c) f  (d) f 
3 3 log 3 a  a a  p
(a) (b) (c) log 3 (d)
p p2 p2  e − at t n−1 
45. L  is equal to :
42.
∞ −3t
cos xt dt is equal to :  n!− 1! 
∫0 te n! 1
7 7 1 1 (a) (b)
(a) (b) (c) (d) (p − a)n (p + a)n
625 225 625 225
1 n!
1 (c) (d)
43. If L {erf t } = then L{e 3t erf t } is equal to : (p − a) n
(p + a)n
p p+1

ANSWERS
MULTIPLE CHOICE QUESTIONS
1. (b) 2. (c) 3. (c) 4. (a) 5. (c) 6. (d) 7. (d) 8. (c) 9. (b) 10. (c)
11. (b) 12. (c) 13. (d) 14. (c) 15. (d) 16. (b) 17. (a) 18. (b) 19. (a) 20. (c)
21. (d) 22. (c) 23. (d) 24. (c) 25. (c) 26. (b) 27. (c) 28. (d) 29. (a) 30. (a)
31. (a) 32. (b) 33. (c) 34. (d) 35. (c) 36. (b) 37. (c) 38. (b) 39. (c) 40. (a)
41. (c) 42. (a) 43. (c) 44. (c) 45. (b)

HINTS AND SOLUTIONS


1
5. L{t 5} =
5!
=
120 1 − pt t ∞ − pt  e −( p −1) t 
6 6 = ∫ e e dt + ∫ e 0 dt =  
p p 0 0
 p −1 0
∴ By first shfting theorem. 1 − e −( p −1)
= ,p ≠1
120
L{e 3t t 5} = p −1
(p − 3)6  t2  2t
12. lim {e − at F (t)} = lim   = lim  at 
cos at cos at  t→ ∞ t→ ∞  e at  t→ ∞  ae 
6. Since lim does not exist so L   not
t→ 0 t  t  2
= lim = 0 if a > 0
exist. t→ ∞ a 2e at
2
7. lim {e − at e t } = lim e t( t − a) = ∞ for all values of a. so F (t) = t 2 is of exponential order.
t→ ∞ t→ ∞
Hence for any value of a, we can not find a number ∵ |t 2| = t 2 < e 3t V t > 0
2 2
µ such that e t < µe at so e t is not of exponential So t 2 is of exponential order 3.
order as t → ∞. φ  t − 2 π  , t > 2 π
  
e t 0 < t ≤ 1 ∞
Let φ(t) = cos t then F (t) =   3 3
9. F (t) =  so L{F (t)} = ∫ e − pt F (t) dt 14.
0 2π
0 t > 1 0, t<
 3
111

p  1
So L{φ(t)} = L {cos t} = = f (p) ∞ 1−
2 1 x − 1 1 x  = 1 log p + 1
p +1 =  log =  log
 2 x + 1  p 2 1 2 p −1
 1+ 
From second shifting theorem we have x p
2π 
−  p −2π
p
 3  3 p 25. Let F (t) = t sin t
L{F (t)} = e . f (p) =e ⋅ d d 1
p2 + 1 L {F (t)} = L{t sin t} = − L{sin t} = −
p dp p 2 + 1
p 6
16. ∵ L {3 cos 6t − 5 sin 6t} = 3 − 5. 2p
p 2 + 62 p 2 + 62 =
(p 2 + 1)2
3p − 30 ∞ 2p
= = f (p) so by first shifting theorem. so, ∫ e − pt t sin t dt =
p 2 + 36 0 (p 2 + 1)2
We have L{e −2t (3 cos 6t − 5 sin 6t)} = f (pt 2) letting p → 1 we get
∞ −t 2 1
3p − 24
=
2
∫0 e t sin t dt = =
4 2
p + 4p + 40
1
26. Let F (t) = sin t ⇒ L{F (t)} = L{sin t} =
17. ∵ L{F (t)} = f (p) then by first shifting theorem p2 + 1
1 sin t  ∞ sin t ∞
L{f (t) cos at} = LF (t) (e iat − e − iat ) L = ∫ e − pt dt = ∫ f (x) dx
 2   t  0 t p
1 ∞ 1
= [L{f (t)e } − L{F (t)e − iat }]
− iat
=∫
π
dx = [tan C]p = − tan −1 p
−1 ∞
2 p x2 + 1 2
1
= [ f (p − ia) − f (p + ia)] put p → 0 we get
2 ∞ sin t π
1 ∫0 t dt = 2
18. F (t) = sin 2t sin 3t = (cos t − cos 5t)
2
1  t  1
So, L{F (t)} = [L(cos t) − L{cos 5t}] 29. Given L2  = 3/ 2
2  π  p
1 p p  t 1
= − Let F (t) = 2 so F ′(t) =
2  p 2 + 1 p 2 + 25  π πt
∴ L{F ′(t)} = pL {F (t)} − F (0)
p  p 2 + 25 − p 2 + 1  12p
= = 1   t  p 1

2 (p 2 + 1) (p 2 + 25)  (p 2 + 1) (p 2 + 25) or L  = pL2 − 0 = =
 πt   π  p 3/ 2 p1/ 2
a
20. L {sin at} = 30. Since F (t) has a period 2π
p 2 + a2 T − pt π − pt 2π
∫ e F (t) dt ∫ e sin t dt + ∫π 0e − pt dt
d  a  2ap L{F (t)} = 0 = 0
∴ L {t sin at} = − =

dp  p 2 + a 2  (p 2 + a 2)2 1 − e − pt 1 − e −2πp
1 π − pt 1 + e − pπ
sin t  1 sin at  = ∫ e sin t dt =
21. Given L = tan −1   = L 1− e −2πp 0 (p 2 + 1) [1 − (e − pπ )2]
 t   p  at 
1
1 a =
= tan −1   (p 2 + 1) (1 − e − pπ )
a  p
∞ − pt
sin at  sin at  a a a 32. L{F (t)} =
So L = aL tan −1   = tan −1   ∫0 e F (t) dt
 t   at  a  p  p 1 − pt 2 − pt ∞ − pt
= ∫ e 0 dt + ∫ e t dt + ∫2 e 0 dt
0 1
23. Here, F (t) = sin ht
2
F (t) sin ht  e − pt e − pt 
so, lim = lim =1 L {F (t)} =  − t −
t→ 0 t t 
 p p 2 1
1
since L{sin ht} = = f (p)
p2 − 1 2 1  −2p  1 1  −p
= − +
2
 e + +
2
 e
sin ht  ∞ ∞ dx p p  p p 
so L = f (x)dx = ∫
 t  ∫p p x2 −1
112
3 2 t sin x cot −1 p
So, lim L{F (t)} = − + ∴ L∫ dx  =
p→ 1 2 e
e  0 x  p
1
34. L{sin x} = , p > 0 say f (p) 41. Let F (t) = e − t − e −3t
p2 + 1 1 1
sin x L{F (t)} = − = F (p)
and lim exists and is equal to 1. p+1 p+ 3
x→ 0 x
F (t)  ∞ ∞ 1 1 
sin x  ∞ ∞ 1 So, L = F (x) dx = ∫  −  dx
So, L  = ∫p f (x) dx = ∫p 2 dx  t  ∫p p  x + 1 x + 3
 x  x +1 ∞
x + 1  x + 1  p + 1
= [tan −1 x]∞ −1
p = cot p =  log   = lim log   − log  
  x + 3   p x → ∞  x + 3  p + 3
sin x  ∞ − px sin x
Since L = ∫0 e dx 1+ 1
 x  x 
= lim log  x  − log  p + 1 
∞ e − px  

sin x
dx = cot −1 p x→ ∞ 3  p + 3
∫0 x
1+
 x

Putting p = 1, we get  p + 1  p + 3
∞ e − x sin x = − log   = log  
−1 π  p + 3  p + 1
∫0 x dx = cot 1 = 4
∞ − pt  e − t − e −3t   p + 3
So, ∫0 e   dt = log  
35. ∵ L {F ′(t)} = pL{F (t)} − F (0)  t   p + 1
∴ L{J1(t)} = L{− J 0′ (t)} = − L{J 0′ (t)}
∞  e − t − e −3t 
 1  p → 0 we get ∫   dt = log 3
= − {pL{J 0(t)} − J 0(0)} = − p  0
 t 
2
 p + 1 − 1 
p d d  p 
= 1− 42. L {t cos 4t} = − L {cos 4t} = −  
2
p +1 dp dp  p 2 + 16 
∞ − pt p 2 − 16
37. Let φ(t) = sin t or ∫0 e t cos 4t dt = (p 2 + 16)2
φ  t − π  , t > π
   ∞
p = 3 we get ∫ e −3t t cos 4t dt =
7
so F (t)   3 3 put
0
π 625
0, t<
 3 1
43. Given L {erf t } = = F (p)
1 p p+1
So, L{φ(t)} = L{sin t} = = f (p)
p2 + 1 So, L{e 3t erf t } = F (p − 3)
By second shifting theorem we have
π 1 1
−   p. f ( p) = =
 3 e − πp / 3 (p − 3) p − 3 + 1 (p − 3) p − 2
L{F (t)} = e = ,p> 0
p2 + 1 (n − 1)!
45. Since L {t n−1} = = F (p)
39. L{sin at} =
a pn
p 2 + a2  e − at t n−1  1
So, L = L{e − at t n−1}
d  a  ( n − 1)! ( n − 1)!
∴ L{t sin at} = −    
dp  p 2 + a 2  1 1 (n − 1)!
= f (p + a) = .
2ap (n − 1)! (n − 1)! (p + a)n
=
(p 2 + a 2)2 1
=
sin t  (p + a)n
40. Since we know that L −1
 = cos p
 t  mmm
F (p) t
Since if L{F (t)} = f (p) then L{∫ F (x) dx} =
0 p
Unit113
-IV

C HAPTER
15 The Inverse Laplace Transform
INVERSE LAPLACE TRANSFORM PROPERTIES OF INVERSE LAPLACE TRANSFORMS
1. If L{ F (t)} = f ( p) then F (t) is called an inverse 1. If L−1{ f ( p)} = F (t) then
Laplace transform of f ( p) and is written as L−1{ f ( p − a)} = e at F (t) = e at L−1{ f ( p)}
−1
F (t) = L { f ( p)}
This is called first translation or shifting
Here L−1 is called the inverse Laplace theorem.
transformation operator. 2. If L−1{ f ( p)} = F (t)
2. If f 1( p) and f 2 ( p) be the Laplace transforms of  F (t − a), t > a
F1(t) and F2 (t) respectively and C 1, C 2 are two where G (t) = 
 0, t< a
constants, then
This is called second translation or shifting
L−1{ C 1 f 1( p) + C 2 f 2 ( p)} = C 1L−1{ f 1( p)} theorem.
+ C 2 L−1{ f 2 ( p)}
3. If L−1{ f ( p)} = F (t) then
This is called Linearity property. 1  t
L−1{ f (ap)} = F 
3. The inverse Laplace transforms of some a  a
functions are This is called change of scale property.
 1 If L−1{ f ( p)} = F (t) then
(i) L−1   = 1
 p
 dn 
 1 L−1{ f n( p)} = L−1  f ( p) = (−1)n t nF (t)
(ii) L−1   = t  dp
n

2
p 
4. If L−1{ f ( p)} = F (t) then
 1  tn tn
(iii) L−1   = or ∞ F (t)
n+ 1
 p  t(n + 1) n ! L−1{ ∫ f (x ) dx } =
p t
 1 
(iv) L−1  =e
at
5. If L−1{ f ( p)} = F (t) and F(0) = 0 then
 p − a
L−1{ pf ( p)} = F ′ (t)
−1   sin at
1
(v) L  =
2 2 a 6. If F (t) is sectionally continuous and of
p + a 
F (t)
 p  exponential order a and such that lim
(vi) L−1  = cos at t→0 t
2 2
p + a  exists then for p > a
 f ( p) t
 1  sin hat L−1   = ∫0 F (x ) dx
(vii) L−1  = p
2 2 a  
p − a 
Continuing it we have
−1   p  f ( p) t t t t
(viii) L  = cos hat L−1  − F (t) dt n
2 2 =
n  ∫0 ∫0 ∫0 ∫0
p − a   p 
114

7. If F (t) and G(t) be two functions of class A, then This is called convolution theorem.
the convolution of two functions F (t) and G(t) 9. If F ( p) and G( p) be two polynomials in p where
denoted by F * G is defined as F ( p) has degree less than that of G( p) and G( p)
t
F*G= ∫0 F (x ) G(t − x ) dx has n distinct factors (zeros)
α r, r = 1, 2, 3, ... n
F × G is commutative, Associative and
i. e. G ( p) = ( p − α 1) ( p − α 2 ) ... ( p − α n)
distributive w.r.t. addition.
n F (α )
 F ( p) α rt
8. If F (t) and G(t) be two functions of class A and then L−1  r
 r= 1 G′ (α ) e
= Σ
 G ( p)
let L−1{ f ( p)} = F (t) and L−1{ g ( p)} = G(t) then r
t This is called Heaviside’s Expansion theorem.
L−1{ f ( p) g ( p)} = ∫0 F (x ) G(t − x ) dx = F * G

EXERCISE
MULTIPLE CHOICE QUESTIONS 7. The value of L−1 {(2p + 3)−1/ 2} is :
 1  1 e 3/ 2t
1. L−1   is equal to : (a) (b)
3 πt
 p t
e2 . 2 πt
1 1 1
(a) (b) (c) πt (d) 3
t
t t πt e2 1
(c) (d)
2πt 3
−1  1  t
2. L   is : e2 πt
2
 p + 4 
cos 2t cos h2t sin 2t sin h2t  p  t sin t  32p 
(a) (b) (c) (d) 8. If L−1  = then L−1   is :
2 2 2 2 2 2 2 2 2
(p + 1)  (16p + 1) 
−1  4  t t t t
3. The value of L   is : (a) sin (b) sin
(p − 2)  2 2 3 3
e 2t t
(a) e 2t (b) 4e 2t (c) 2e 2t (d) t t
sin
4 (c) sin (d) 2
4 4 t
 −1 
 e p  cos (2 t )  e− a / p  2
4. If L−1  = then L−1   for a > 0
 p  πt  p  e −3p
9. The inverse Laplace transform of is :
  p3
is :
(a) (t − 3) H(t − 3) (b) tH(t − 3)
t k
cos 2 cos 2
k t (t − 3)2
(a) (b) (c) (t − 3)2 H(t − 3) (d) H (t − 3)
πt πt 2
cos 2 kt  pe − ap 
(c) (d) None of these 10. The value of L−1  is :
πt 2 2
p − w 
 e− p 
5. L−1  is equal to : (a) (t − a) H(t − a) (b) cos w (t − a) H(t − a)
2 
p  (c) cos hw(t − a) H(t − a) (d) None of these
(a) (t − 1) e − t (b) (t + 1) e − t
 1 
(c) (t − 1) H(t − 1) (d) (t + 1) + (1) 11. L−1   is equal to :
 p(p + 3) 
 1 
6. L−1   is equal to : (a) 1 + e −3t (b) 1 − e −3t
2
 p − 6p + 10 
(a) e t cos t (b) e 3t sin t e −3t − 1 1 − e −3t
(c) (d)
(c) e 3t cos t (d) e t sin t 3 3
115

  1   1 
12. L−1 log  1 −   is : 20. L−1  is :
n
  p2   (p + a) 

(a)
2 (1 − cos ht)
(b)
1 − sin ht e − at t n−1 e − at t n
(a) (b)
t t (n − 1)! n!
1 + cos ht
(c) (d) None of these e − at t n+1
t (c) (d) None of these
(n − 1)!
 1 
13. L−1   is : e − pπ
2 21. The inverse Laplace transform of is :
 p (p + 1) 
p2 + 1
−t −t
(a) t + e (b) t − 1 + e
−t
(a) (t − π) H (t − π) (b) t H (t − π)
(c) t + 1 + e (d) t − e − t
(c) − sin t H (t − π) (d) cos t H (t − π)
 p + 2
14. L−1 log  is :   1 
 p + 1 22. L−1 log  1 +  is :
2 
  p 
e t + e 2t e t + e −2t
(a) (b) 2 (1 − cos t) 1 − sin t
t t (a) (b)
t t
e − t − e −2t e − t + e −2t log (1 + t)
(c) (d) (c) (d) None of these
t t t
 1  If L−1 {f (p)} = F (t) then L−1 {f (p − a)} is :
15. L−1   is equal to : 23.
(p − 1) (p + 3)  (a) e − at F (t) (b) e at F (t)
t −2t t −2t
e −e e +e 1 t
(a) (b) (c) F   (d) (−t)n F (t)
3 3 a  a
e − t − e −2t
(c) (d) None of these  f (p) 
2 24. If L−1 {f (p)} = F (t) then L−1   is :
 p 
 19p + 37  x F (x ) x
16. The value of L−1   is : (a) ∫ dx (b) ∫ F (x) dx
0 x 0
(p + 1) (p − 2) 
(c) F ′(t) (d) (−1)n t nF (t)
(a) 3e 2t − 5e − t (b) 3e 2t + 5e − t
t
(c) 5e 2t + 3e − t (d) 5e 2t − 3e − t 25. The value of ∫ sin x cos (t − x) dx is equal to :
0
∞ −x 2 (a) sin t + cos t (b) t sin t − 1
17. ∫0 e dx is equal to :
t sin t
(c) t cos t + 1 (d)
π 2
(a) (b) π
2
26. By convolution, the value of |*||*|x| (n times) is :
π t tn t n−1
(c) π (d) (a) (b) t n (c) (d)
2 (n − 1)! n! (n − 1)!
 2p + 1 
18. The value of L−1   is :  p 
 p (p + 1)  27. The value of L−1   is :
2
 p + 36 
sin 2 t
(a) sin t + 1 (b) cos t + 1 (c) e − t + 1 (d) (a) sin 6t (b) sin h 6 t
2 (c) cos 6t (d) cos h 6 t
 3  1 1
19. L−1 −  is equal to : 28. L−1  cos  is :
2
 9p − 16   p p

(a)
1
cos h
4t 1
(b) − sin h
4t t2 t2
(a) 1 − (b) 1 − + ...
4 3 4 3 (L )2 2+ ...
L2
1 4t 1 4t
(c) sin h (d) − cos h 1 + t2 1 + t2
4 3 4 3 (c) + ... (d) + ....
(L2)2 L2
116

 1   e − ap 
29. L−1  is equal to : 38. L−1  , a > 0 is :
3 2 
 (p − a)   p 
e at te at t 2e at t 3e at (a) at when t > a and 0 otherwise
(a) (b) (c) (d)
2 2 2 2 (b) (t − a) when t > a and 0 otherwise
−1  p+ 3 (c) (t + a) when t > a and 0 otherwise
30. The value of L log  is :
 p+ 2 (d) None of these
e −2t + e −3t e −2t − e −3t  3p + 7 
(a) (b) 39. The value of L−1   is :
2 t 2
 p + 2p − 3 
e −2t + e −3t e −2t − e −3t
(c) (d) (a) 4e 3t − e − t (b) e 3t + e −2t
t 2
(c) 4e −3t − e t (d) 2e 3t − e − t
−1  p  t sin t −1  1 
31. If L  = then L  is :
2 2 2 2 2  1 
 ( p + 1)   ( p + 1)  40. The value of L−1   is :
sin t + t cos t  p(p + 1) 
(a) sin t + t cos t (b)
2 (a) e − t (b) e t (c) 1 + e − t (d) 1 − e − t
t sin t − cos t sin t − t cos t
(c) (d) 1 p 
2 2 41. If L−1 {f (p)} = F (t) then L−1  F    is :
2  2 
−1  1 
32. The value of L   is : t 1 t
(p + 1) (p − 2)  (a) F   (b) F  
 2 2  2
e 2t − e − t et − e− t 1
(a) (b) (c) F (2t) (d) F (t)
3 3 2
e 2t + e − t et + e− t
(c) (d) 42. L−1 {pf (p) − F (0)} is equal to :
3 3
 p + 5  (a) F (t) (b) F ′(t) (c) F ′′(t) (d) tF ′(t)
33. L−1   is equal to :
2  
 p + 1 43. L−1 
1
2  is equal to :
(a) sin t − 2 cos t (b) 3 sin t − 2 cos t  9p + 6p + 1 
(c) 3sin t + cos t (d) 2 sin t − 3 cos t te − t / 3 te t/ 3
(a) (b)
−1 −1 9 3
34. If L {f (p)} = F (t) then L {f (ap)} is :
1 t t te − t / 2
(a) F   (b) a F   (c) (d) None of these
a  a  a 9
1
(c) atF (t) (d) F (at)  1 
a 44. The value of L−1   is :
2
 e − πp   2(p − 1) + 32 
35. L−1   is equal to :
2
 p + 1 e 4 t sin t e t sin 8t
(a) (b)
(a) sin t H (t − π) (b) cos t H(t − π) 2 4
(c) t sin(t − π) (d) sin(t − π) H (t − π) e t cos 4t e t sin 4t
(c) (d)
8 8
36. If L−1 {f (p)} = F (t) and F(0) = 0 then L−1 {pf (p)} is :
F (t)  2p − 6 
(a) (b) tF (t) 45. L−1   is equal to :
t 2
x  p − 3p + 2 
(c) ∫ F (x, dx) (d) F ′(t)
0 e 2t
(a) 4e t − 2e 2 t (b) 4e t −
 −1  2
−1  e 2  cos 2 t  e − ap 
37. If L  = then L−1   is : 4e t + e 2t 3e 2t
(c) (d)
 p πt  p  2 2
 
 1
cos (2 at ) cos (2 πt ) 46. L−1 tan −1  is :
(a) (b) p
πt at 
1 cos 2 t tan t cos t sect sin t
(c) (d) None of these (a) (b) (c) (d)
a πt t t t t
117

 1   1 
47. The value of L−1   is : 48. L−1  is :
2 2 2
 p (p + 1)   p(p + 1) 
(a) sin t (b) t + sin t (c) t − sin t (d) t − sin t (a) e − t (t + 1) (b) 1 − e − t (t + 1)
(c) 1 + e − t (d) e t (t − 1)

ANSWERS
MULTIPLE CHOICE QUESTIONS
1. (d) 2. (c) 3. (b) 4. (a) 5. (c) 6. (b) 7. (a) 8. (c) 9. (d) 10. (c)
11. (d) 12. (a) 13. (b) 14. (c) 15. (a) 16. (d) 17. (d) 18. (c) 19. (b) 20. (a)
21. (c) 22. (a) 23. (b) 24. (b) 25. (d) 26. (d) 27. (c) 28. (a) 29. (c) 30. (b)
31. (d) 32. (a) 33. (b) 34. (a) 35. (d) 36. (d) 37. (a) 38. (b) 39. (a) 40. (d)
41. (c) 42. (b) 43. (a) 44. (d) 45. (a) 46. (d) 47. (d) 48. (b)

HINTS AND SOLUTIONS


1
−1
1  2a 2p  t t

or L−1  = sin
−1 
1  −1  1  t2 t 2 1 2 2 2 a a
1. L   = L  1/ 2  = 1 = =  ( a p + 1) 
 p   π tπ
p   
t  2 put a = 4, we get
 32 p  t t
 e −1/ p  cos 2 t L−1  = sin
4. Given that L−1  = 2 2 4 4
1/ 2   (16p + 1) 
p  πt
 −1   p 
t 10. Since L−1  = cos hω t
 pk  cos 2 2 2
∴ −1  e
L  =1 k p − ω 
1/ 2 
(pk)  k πt  pe − ap 
L−1  = cos hω(t − a) H (t − a)
  k 2 2
p − ω 
 −1  t
 kp  cos 2 k
−1  e
 1
or L  = 12. Let f (p) = log  1 − 
 p  πt  p2 
 
1  p 2 − 1
put k = we get = log   = −2 log p + log (p 2 − 1)
a  2 
 p 
 −a 
 e p  cos 2 at 1 p 
L−1  = ∴ f (p) = −2  − 
 p  πf  p p 2 − 1
 
 1  −1  1  ∴ L−1 {f (p)} = −2 (1 − cos ht)
6. L−1  =L  
2 2
 p − 6p + 10  (p − 3) + 1  or − tL−1 {f (p)} = −2 (1 − cos ht)
 1 
e 3t L−1  3t
 = e sin t   1  2
2
 p + 1 or L−1 log  1 −   = (1 − cos ht)
  p2   t
 p  t sin t
8. Given that L−1  =
2 2 2 14.
 p + 2
Let f (p) = log 
(p + 1)   = log (p + 2) − log (p + 1)
 p + 1
 ap  11 t t
We have L−1  = sin 1 1
2 2 2 f ′(p) = −
(a p + 1)  2 a a a
p+ 2 p+1
118

t
∴L−1 {f ′(p)} = e −2t − e − t 26. Since F * G = ∫0 F(x)G (t − x) dx
or = tL−1 {f (p)} = e −2t − e − t t
|*|= ∫0 11. dx = t
 p + 2 1 − t
∴ L−1 {f (p)} = L−1 log −2t
 = (e − e ) t t2
p + 1 t |*|*|= t * 1 =
 ∫0 x.1 dx = 2
 19p + 37 
16. Given L−1    t2  1x2 t3
|*|*|*|=   * 1 = ∫0 1dx =
(p + 1) (p − 2)   2 2 3!
 
Here F (p) = 19p + 37
Proceeding similarly, we have
G (p) = (p + 1) (p − 2) = p 2 − p − 2
t n− 1
Here G (p) has 3 distinct zeroes, |*|*|....*| (n times) = , n = 1, 2, 3,...
(n − 1)!
α1 = −1, α 2 = 2
 p  t sin t
G′(p) = 2p − 1 31. Given L−1  = = F (t)
2 2 2
By Heaviside’s expansion formula, we have  ( p + 1) 
 19p + 37  F (−1) − t  1  1 p 
L−1  = e ∴ L−1  = L−1  .
(p + 1) (p − 2)  G′(−1) 2 2 p 2 2
 ( p + 1)   ( p + 1) 
F (2) 2t
+ e = −3e − t + 5e 2t t 1 t sin t − t cos t
G′(2) = ∫0 F(x) dx = 2 ∫0 x sin x dx = 2
∞ − tx 2 dx
17. Let F (t) = ∫0 e  1 
35. ∵ L−1   = sin t
∞ 2
L{ F (t)} =
2
L{ e − tx } dx  p + 1
∫0
∞  e − pπ 
∞ dx  1 x  L−1   = sin (t − π)H (t − π)
= ∫0 = tan −1  2
p + x2  p p 0 (p + 1) 
π = − sin t H (t − π)
=
2 p
 −1 
−1 
π  π 1 1 π  p  cos 2 t
−1  e
∴ F (t) = L  = = 37. Since L  =
 2 p  2 πt 2 t 1/ 2  πt
p 
∞ − tx 2 1 π  
or ∫0 e . dx =
2 t  −1  t
put t = 1 we get  e pk  1 cos 2 k
−1 
∴ L  =
∞ −x 2 π 1/ 2 
(pk)  k πt
∫0 e dx =
2   k
 1 
21. Since L−1   = sin t  −1  t
2
 p + 1  pk  cos 2 k
−1  e
or L  =
1/ 2 
e − pπ 
−1  p  πt
L   = sin (t − π) H (t − π)  
2
 p + 1
= − sin t H(t − π)  −a 
1 e p  cos 2 at
25. Let. F (t) =
t
Put k = we get L−1  =
∫0 sin x cos (t − x) dx a  p  πt
 
By convolution theorem we have
L {F (t)} = L{sin t} L{cos t} 1
38. L−1   = t
1 p p 2
= . = p 
p 2 + 1 p 2 + 1 (p 2 + 1)2
 e − ap  t − a t > a
So, L−1  = (t − a)H (t − a) =
−1   t sin t
p 2   
∴ F (t) = L 
2 2
=  p  0 t≤ a
 ( p + 1)  2
119

 1  f ′(p) = − sin t
43. L−1  
2
 ( 9p + 6pt)  But L {f ′(p)} = − tL−1 {f (p)}
−1

 
  ∴ − tL−1 {f (p)} = − sin t
 1  1 1
= L−1   = L−1  
2 sin t
2
(3p + 1)  9  p + 1  L−1 {f (p)} =
   t
3  
  1   sin t
1 − t / 3 −1  1  t or L−1 tan −1    =
= e L =   = − e− t / 3   p t
9 2 9
p
 
 1 
 1   1  47. L−1   = sin t
44. L−1  = e t L−1  2
(p + 1) 
2 2 2 2
 2( p − 1) + 3   2p + 3 
 1  t
e t −1  1  e t sin 4t e t sin 4t L−1  = ∫0 sin x dx = 1 − cos t
= L  2
2 = =  p(p + 1) 
2  p + 16  2 4 8
 1  t
45. Here F (p) = 2p − 6 L−1  = ∫0(1 − cos x) dx = t − sin t
2 2
 p (p + 1) 
and G(p) = p 2 − 3p + 2
 1   1 
G(p) = (p − 1) (p − 2) 48. L−1  = L−1 
2 2
 p(p + 1)  (p + 1 − 1) (p + 1) 
∴ G′(p) = 2p − 3
 1 
By Heaviside expansion formula we have = e − t L−1 
2
 2p − 6  F (1) t F (2) 2t (p − 1) p 
L−1  = e + e
2  1 
 p − 3p + 2  G′(1) G′(2) Now, L−1  t
=e
−4 t −2 2t  p − 1
= e + e
−1 1  1  t x
∴ L−1  = ∫0 e dx = e t − 1
t
= 4e − 2e 2t  p(p − 1) 

 1  1  t
46. Let f (p) = tan −1   = cot −1 p L−1  = ∫0(e
x
− 1) dx = e t − t − 1
 p 2
 p ( p − 1) 
1  1 
f ′(p) = − ∴ L−1  = e − t (e t − t − 1) = 1 − e − t (t + 1)
p2 + 1 2
 p( p + 1) 
 1 
So L−1 {f ′(p)} = tL−1 −  mmm
2
 p + 1
120
Unit-IV
C HAPTER
16 Applications of Laplace Transform
If y(x , t) is a function of x and t then
SOLUTION OF ORDINARY DIFFERENTIAL EQUATION
1. The Laplace transform is very useful in solving  ∂y 
(i) L  = py(x , p) − y(x , 0)
ordinary linear differential equations with  ∂t 
constant coefficient.  ∂ 2y 
(ii) L = p2 y(x , p) − py (x , 0) − yt(x , 0)
Consider a linear differential equation with 2
 ∂t 
constant coefficients.
 ∂y  dy
d ny d n− 1y (iii) L  =
+ A1 + ...  ∂x  dx
dt n dt n− 1
 ∂ 2y  d 2y
dy (iv) L =
+ An− 1 + Any = F (t) ...(1) 2 2
dt  ∂x  dx
Where F (t) is a function of independent where, L { y(x , t)} = y(x , p)
variable t.
Let y(0) = C 0 , y(0) = C 1, ..., y n− 1(0) = C n− 1 SOLUTION OF INTEGRAL EQUATIONS
...(2) 1. Laplace transform is also useful in solving
be the given initial or boundary conditions. various integral equations such as
Where C 0 , C 1, C 2 , ..., C n− 1 are all constants. b
F (t) = y( f ) + ∫a k (u, t)F (u) dx
To solve equation (1) we take the Laplace
transform of both sides of equation (1) and where y(t) and k (u, t) are known a and b are
using contition (2) we obtain an algebraic either constants or functions of t. Here the
equation called subsidiary equation form function F (t) which appears under the integral
which
sign is to be determined.
y( p) = L{ y(t)} is determined. The required
solution can be obtained by finding the inverse t F (u) du
2. An equation of the form G(t) = ∫0 (t − u)n
Laplace transform of y( p).
2. The Laplace transform is very useful in solving is called Abel’s integral equation with r < n < 1.
the differential equations having the terms
t my n(t) is variable coefficients, where Laplace 3. An integral equation of the form
t
transform is F (t) = y(t) + ∫0 k (t − u) F (x ) dx or
dm
L{ t my n(t)} = (−1)m [L{ y n(t)}]
dp m F (t) = y(t) + k (t) * F (t) is called an integral
equation of convolution type.
3. The Laplace transform can also be used in
solving two or more simultaneous ordinary 4. An integral equation in which various
differential equations.
derivatives of the unknown function F (t) can
SOLUTION OF PARTIAL DIFFERENTIAL EQUATIONS also be present is called integer-differential
1. Laplace transform is also useful in solving equation such as
t
partial differential equations when the F ′ (t) = F (t) + y(t) + ∫0 k (t − u) F (u) du
boundary conditions are given
121

EXERCISE
MULTIPLE CHOICE QUESTIONS 10. The Laplace transform of 3
∂ 2y
=
∂y
with
∂y 2 ∂t
1. If y(x, t) is a functio.n of x and t then L   is : dx
 ∂t  y(x, 0) = 30 cos 5 x is :
(a) y(x, p) − y(x, 0) (b)
dy (a) (D 2 + 3) y = cos 5 x
dt p
(b)  D 2 −  y = −10 cos 5 x
(c) py(x, p) − y(x, 0) (d) p 2y(x, p) − py(x, 0)  3
2. The solution of (D + 1) y = 1, y = 2 when t = 0 is :  2 p
(c)  D +  y = 10 cos 5 x
 3
(a) 1 + e t (b) 1 + e − t (c) 1 − e − t (d) 1 − e t
p
3. The solution of (D 2 + 1) y = 0, y(0) = 0, y′(0) = 1 is : (d)  D 2 −  y = cos 5 x
 3
(a) cos t (b) e t + 1 (c) sin t (d) e − t t
11. The solution of F (t) = a sin t − 2∫ F (u) cos (t − 9)du is :
4. If we take Laplace transform of (D + 2)2 y = 4e −2t 0

with y(0) = −1, y′(0) = 4, L{y} is equal to : (a) e − t (b) te t (c) ate − t (d) at 2e t
2 2 t
4 − 2p − p 2p + 1 12. The solution of F (t) = 1 + 2∫ F (t − u) e −2u du is :
(a) (b) 0
(p + 2)3 (p + 2)3
(a) 2t (b) 1 + 2t (c) 1 − 2t (d) 2t + t
2p 2 − 2p + 1
(c) (d) None of these ∂y
(p + 2)3 13. L   is equal to :
 ∂x 
5. The solution of D 3y = 0 with (a)
dy
(b) py (x, p)
y(0) = y′(0) = 0, y ′′(0) = 1 is : dx
t2 t2 + 1 (c) py(x, p) − y (x) dy (d) y (x, p)
(a) t (b) t 2 (c) (d)
2 2 14. The solution of D 2y = t when y(0) = y′(0) = 0 is :
6. The solution of (D 2 + D) x = 2, x(0) = 3, x ′(0) = 1 is : t2 t3 t
(a) (b) (c) (d) Not exist
(a) 1 + 2t + e t
(b) 2 + t − e −t 3 6 6
(c) 1 + 2t + e − t (d) 2 + 2t + e − t 15. L−1 {py(x, p) − y (x, 0)} is :
∂y ∂y dy dy ∂y ∂y
7. The Laplace transform of = 2 + y, (a) (b) (c) (d)
∂x ∂t dt dx ∂x ∂t
y(x, 0) = 6e −3x is :  d 2y 
dy 16. L−1  is equal to :
(a) − (2p + 1) y = −12e −3x 2
dx  dx 
dy
(b) − py = 12e −3x d 2y d 2y ∂ 2y ∂ 2y
dx (a) (b) (c) (d)
dx 2 dx 2 ∂x 2 ∂x 2
dy
(c) + 2py = e −3x (d) None of these
dx 17. The solution of y ′′ + y = t with y′(0) = 1, y ( π) = 0 is :
2
8. The solution of (D + 1) y = 0 under the condition (a) t cos t + π (b) π sin t + t
dy (c) t sin t + π (d) π cos t + t
y = 1, = 0 when t = 0, is:
dt ∂y
(a) e t + 1 (b) cos t + 1 (c) sin t − 1 (d) cos t 18. L   is equal to :
 ∂x 
9. For (D 2 − D − 6) y = 2, t > 0 with y(0) = 1, y′(0) = 0 . (a) y(x, p) − y(x, 0) (b)
dy
L{y} is equal to : dx
(c) py(x, p) − y (x, 0) (d) y(x, p)
p2 − p + 2 p2 + 1
(a) (b)
2
p(p − p − 6) 2
(p − p − 6) ∂ 2y ∂y
19. The Laplace transform of = with
2 ∂t
1 p+1 ∂x
(c) (d) y(x, 0) = 3 sin 2 πx is :
p (p 2 + 1) p2 − p − 6
122

t
(a) (D 2 + p) y = 3 sin 2 πx 28. The solution of F ′(t) = sin t + ∫0 F(t − u) cos u du with
(b) (D + p) y = −3 sin 2πx F (0) = 0 is :
(c) (D − p) y = 3 sin 2πx
t t2
(d) (D 2 − p) y = −3 sin 2 πx (a) 0 (b) t (c) (d)
2 2
t t
20. The solution of F (t) = 4t − 3∫ F (u) sin(t − u) du is : 29. The solution of ∫ F (u)F (t − u)du = t is :
0 0
3 3
(a) t + sin 2t (b) t − cos 2t t2
2 4 (a) 0 (b) ± t (c) ± (d) ± 1
1 2
(c) t + sin 3t (d) t − sin 3t
2 t f (u) du
t 30. The solution of ∫ = t is :
21. The solution of y(t) = 1 + ∫0 y(u) (t − u) du : 0 (t − u)1/ 3
(a) sin t (b) cos t (c) sin ht (d) cos ht 3 t1/ 3 3 3 t1/ 2
(a) (b)
22. L−1 {p 2y(x, p) − py (x, 0) − y + (x, 0)} is : 2π π

d 2y ∂ 2y d 2y ∂ 2y 3 3 t1/ 3 3 t1/ 2
(a) (b) (c) (d) (c) (d)
dx 2 ∂x 2 dt 2 ∂t 2 2π 2π
31. The transformed differential equation after Laplace
∂ 2y ∂y
23. The solution of 2 = with y(x, 0) = 0, transform of y ′′t + y′− y = 0 if y(0) = 0, y′(0) = 1 is :
∂x 2 ∂t
dy  2 1 dy 1
y = L{y} is : (a) −  p −  y = − (b) − py =
dp  p p dp p
(a) y = C1e px / 2 + C2e − px / 2
dy 1 dy 1
p p (c) − (p − 2) y = − (d) + py = −
x −x dp p dp p
(b) y = C1e 2 + C2e 2
32. After solving (D − 2) x − (D − 2) y = sin t and
(c) y = C1e px + C2e − px
(D 2 + 1) x + 2Dy = 0, x(0) = x ′(0) = y(0) = 0, L{x} is :
(d) None of these 2p 2p
(a) (b)
∂ y ∂ 2y (p 2 + 1) (p − 1) (p 2 + 1) (p 2 − 3p + 2)
24. Laplace transform of = , x > 0, t > 0,
∂t ∂x 2 2p
(c) (d) None of these
y − L{y}, y(x, 0) = 0 is : (p + 1) (p 3 − 3p − 2)
2

(a) (D 2 + 1) y = 0 (b) (D 2 − 1)y = 0 ∂u ∂u


33. Taking Laplace trasnform of − = 1 − e− t,
(c) (D 2 − p)y = 0 (d) (D 2 + p) y = 0 ∂x ∂t
t F (u) du r < x < 1, t > 0, u(x, 0) = x, new equation is :
25. The solution of ∫ = 1 is : 1
0 t −u (a) (D − p)u =
p(p + 1)
π t 1
(a) (b) (c) (d) πt 1
t π π t (b) (D + p) u = +x
p(p − 1)
∂ 2y ∂ 2y 1
26. The Laplace transform of − = 0 with (c) (D + p) u = +x
2
∂x ∂t 2 p+1
∂y
y= 0= at t = 0 is : 1
∂t (d) (D − p)u = −x
p(p + 1)
(a) L{y} = C1e px + C2e − px
34. The solution of (D 2 − 2D + 2) y = 0, y = Dy = 1 at
(b) L{y} = C1e pt + C2e − pt
t = 0 is :
p p
(c) L{y} = C1 e2
x
+ C2
− x
e 2 (a) e t + e − t (b) e t cos t
t
(c) e sin t (d) e − t sin t
(d) None of these
t 35. If ty ′′ + 2y′+ ty = 0, y(0) = 1, y( π) = 0 then L{y} is
27. The solution of F (t) = 1 + ∫0 f (u) sin(t − u) du is : given by :
t t2
t2 t2 (a) tan p + A (b) tan −1 p + A
(a) 1 + (b) 2 + (c) 2 − (d) 1 +
2 2 2 2 (c) − tan p + A (d) − tan −1 p + A
123

36. The differential of y = L{y} for [tD 2 + (t − 1) D − 1] y = 0 an (p 2 + n2) n (p 2 + n2)


(a) (b)
2 2
if y(0) = 5, y(∞) = 0 : p +m p 2 − m2
dy 10
(a) + py = a n(p 2 + m 2)
dp p (c) (d) None of these
p 2 + n2
dy 3p 10
(b) + y=
dp p 2 + 1 2
p +p 39. The solution L{y} of (D 2 + 9) y = 18 t if y (0) = y′(0) =
dy 3p + 2 10 0 is :
(c) + y=
dp p 2 + p p2 + p p2 18p 2
(a) (b)
2
(d) None of these p +9 p2 + 9
The value of L{y} for (D 2 + 1) y = 6 cos 2t if 18p
37. (c) (d) p 2 + 9
y (0) = y′(0) = 0 : p2 + 9
p 6p
(a) (b) 40. If (D 2 − 4D + 5) y = t 2 with y(0) = y′(0) = 0 then L{y}
p(p 2 + 1) p(p 2 + 4)
6p is :
(c) (d) None of these 1 2
(p 2 + 1) (p 2 + 4) (a) (b)
p 2 − 4p + 5 p 3(p 2 + 1)
38. If (D 2 + m 2) y = a sin nt, t > 0 with y(0) = y′(0) = 0
2
then L{y} is : (c) (d) None of these
p 3(p 2 − 4p + 5)

ANSWERS
MULTIPLE CHOICE QUESTIONS
1. (c) 2. (b) 3. (c) 4. (a) 5. (c) 6. (d) 7. (a) 8. (d) 9. (a) 10. (b)
11. (c) 12. (b) 13. (a) 14. (b) 15. (d) 16. (c) 17. (d) 18. (b) 19. (d) 20. (a)
21. (d) 22. (d) 23. (b) 24. (c) 25. (c) 26. (a) 27. (d) 28. (d) 29. (d) 30. (c)
31. (a) 32. (c) 33. (d) 34. (b) 35. (d) 36. (c) 37. (c) 38. (a) 39. (b) 40. (c)

HINTS AND SOLUTIONS


2. (D + 1) y = 1, y (0) = 2 3 (D 2 + 1) y = 0, y(0) = 0, y′(0) = 1
Taking Laplace transform Taking laplace transform we get

pL(y) − y(0) + L{y} = L{1} =


1 p 2L{y} − py(0) − y′(0) + L{y} = 0
p
(p 2 + 1) L{y} = 1
1
py − 2 + y = 1
p ⇒ L{y} =
p2 + 1
1 2p + 1
⇒ (p + 1)y = 2 + =  1 
p p y = L−1   = sin t
2
 p + 1
2p + 1 1 1
y= = +
p(p + 1) p + 1 p 5. Laplace transform of D 3y = 0

Taking inverse Laplace p 3L{y} − p 2y(0) − py′(0) − y ′′(0) = 0


 1  −1  1  Given y(0) = y′(0) = 0, y ′′(0) = 1, put these values in
y = L−1  −t
 + L p = e + 1
 p + 1   above equation we get
124

1 F (t) = 1 + 2 t
L{y} =
p3 17. Taking Laplace transform of given equation we have
1  t2 t2
−1  L{y ′′} + L{y} = L{t}
⇒ y=L  = =
3 1
 p  2! 2 p 2L{y} − py(0) − y′(0) + L{y} =
∂y ∂y p2
7. Given equation is = 2 + y, taking the Laplace
∂x ∂t put y (0) = A
transform we have (p 2 + 1)
(p 2 + 1)L{y} = Ap +
∂y ∂y
L  = 2L   + L{y} p
 ∂x   ∂t  p 1
L{y} = A + − y = A cos t + t
dy p2 + 1 p2
or = 2 {py − y(x, 0)} + y
dx
But y( π) = 0
∵ y(x, 0) = 6e −3x
so 0 = Acos π + π
dy
or − (2p + 1) y = −12e −3x ⇒ A= π
dx
∴ y = π cos t + t
8. Taking the Laplace transform of
20. F (t) = 4t − 3F (t) * sin t
(D 2 − D − 6) y = 2 we get
Taking Laplace transform of this equation
L{y ′′} − L{y′} − 6L{y} = L[2]
2 L{F (t)} = 4L{t} − 3L{F (t) * sin t}
p 2L{y} − py(0) − y′(0) − [pL(y) − y (0)] − 6L{y} =
p = 3L{t} − 3L{F (t)}. L{sin t}
2
p 2L{y} − p − pL{y} + 1 − 6L{y} = =
4
− 3L{F (t)}
1
p
p2 p2 + 1
2 p2 − p + 2
(p 2 − p − 6)L{y} = + p −1 = 4(p 2 + 1)
p p ∴ L{F (t)} =
p 2(p 2 + 4)
p2 − p + 2
∴ L{y} = 
p(p 2 − p − 6)  1  −1  1 
F (t) = 4  L−1  +L  2 2 
 2    
 p − 4 p ( p + 4)
∂ 2y ∂y
10. Taking Laplace transform of 3 = we have sin 2t t sin 2t  3
∂x 2 ∂t F (t) = 4  + − = sin 2t + t
 2 4 8  2
 ∂ 2y  ∂y
3L  = L 
∂ x 2  ∂t  ∂y ∂ 2y
  23. Taking Laplace transform of =2
∂t ∂x 2
d 2y
3 = py (x, p) − y(x, 0) ∂y  ∂ 2y  d 2y
dx 2 L  = 2L   or py − y(x, 0) = 2 2
 ∂t  2
 ∂x  dx
d 2y p
− y = −10 cos 5x
2 3 d 2y p 2
dx or − y =0
2 2
dx
12. Given equation is F (t) = 1 + 2F (t) * e −2t
∵ y(x, 0) = 0
Taking Laplace transform of both sides we get
1 so its general solution is
L{F (t)} = L{1} + 2L{F (t) * e −2t } =
p y = C1e x p/ 2
+ C2e − x p/ 2
−2t
+2 [L{F (t)}. Le ]
25. Given F (t) * t −1/ 2 = 1, taking Laplace transform
1 2
= + L{F (t)}
p p+ 2 L{F (t)}. L{t −1/ 2} = L{1}
p+ 2 π 1
or L{F (t)} = ⇒ L{F (t)} =
p2 p1/ 2 p
1  1 1 1
∴ F (t) = L−1   + 2L−1   L{F (t)} =
p  2
p  π p1/ 2
125

1 −1  1  d
∴ F (t) = L  or − {p 2y − py(0) − y′(0)} + 2{py
π 1/ 2  dp
p 
1 dy dy
F (t) = − y(0)} − = 0 −(p 2 + 1) −1 = 0
π t dp dp
dy 1
28. Given equation is F ′(t) = sin t + F (t) * cos t or =−
Taking Laplace transform we have
dp p2 + 1
L{F ′(t)} = L{sin t} + L{F (t) * cos t} so y = A − tan −1 p
1
pL{F (t)} − F (0) = + L{F (t)}. L{cos t} 37. Taking Laplace transform of (D 2 + 1)y = 6 cos 2 t we
p2 + 1
have
1 p
pL{F (t)} = + L{F (t)}. L{y ′′} + L{y} = 6L{cos 2t}
p3 + 1 p2 + 1
2 6p
1 p L{y} − py(0) − y′(0) + L{y} =
L{F (t)} = p2 + 4
p3
 1  t2 ∴ y(0) = y′(0) = 0 so
⇒ F (t) = L−1   = 6p
3 2
p  2 (p + 1) L{y} =
p2 + 4
31. Taking laplace transform of y ′′ + ty′− y = 0
6p
L{y ′′} + L{ty′} − L{y} = 0 ⇒ L{y} =
2 d (p 2 + 1) (p 2 + 4)
p y − py (0) − y′(0) − {L(y′)} − y = 0
dp 39. Taking Laplace transform of (D 2 + 9) y = 18t we
d
(p 2 − 1)y − 1 − {py − y (0)} = 0 have
dp L{y ′′} + 9L{y} = 18L{t}
∵ y′(0) = 1 2 18
p L{y} − py(0) − y′(0) + 9L{y} =
or 2 
(p − 1) y − 1 − y + p
dy  p2
=0
 dp  18
(p 2 + 9)L{y} =
∵ y (0) = 0 p2
dy  p 1 ∴ y(0) = y(0) = 0
or − p −  y = −
dp  2 p 18
So, L{y} =
33. Taking Laplace transform of
∂u ∂u
− = 1 − e− t p 2(p 2 + 9)
∂x ∂t
40. Taking Laplace transform of given equation we have
∂u ∂u
we have L  − L  = L{1 − e − t } L{y ′′} − 4L{y} + 5L{y} = L{t 2}
 ∂x   ∂t 
du 1
− {pu − u (x, 0)} = −
1 p 2L{y} − py(0) − y′(0) − 4{pL(y)
dx p p+1 2
− y (0)} + 5L{y} =
∴ u (x, 0) = x p3
du 1 2
− pu + x = (p 2 − 4p + 5) L{y} =
dx p(p + 1) p3
du 1 ∴ y(0) = y′(0) = 0
− pu = −x
dx p(p + 1) 2
∴ L(y) =
35. Taking Laplace transform of ty ′′ + 2y′+ ty = 0 p 3(p 2 − 4p + 5)
L{ty ′′} + 2L{y′} + L{ty} = 0 mmm
d d
or − L {y ′′} + 2L {y′} − L {y} = 0
dp dp
126
Unit-V
C HAPTER
17 Fourier Transforms
FOURIER TRANSFORM 3. The infinite Fourier sine transform of f (x ),
1. Let f (x ) be a function defined on (– ∞, ∞) and r < x < ∞ is denoted by Fs { f (x )} or f$s ( p) and is
be piecewise continuous in each finite partial defined as
interval and absolutely integrable in (– ∞, ∞), 2 ∞
Fs { f (x )} = f$s ( p) = f (x ) sin px dx
then π ∫0
1 ∞ −ipx
F { f (x )} = ∫−∞ e f (x ) dx The function f (x ) is called the inverse Fourier

sine transform of f$ ( p) i. e.
s
is called the Fourier transforms of f (x ) and
denoted by F { f (x )} or f$( p) f (x ) = Fs−1{ f$s ( p)}

The function f (x ) is called the inverse Fourier Some authors define it as



transform of f$( p) i. e. f$s ( p) = ∫0 f (x ) sin px dx
f (x ) = F { f$( p)}
−1
This is called Fourier sine transform.
2. If f$( p) is the Fourier transform of f (x ) and if f (x ) 4. If f$s ( p) is the Fourier sine transform of the
satisfies the Dirichlet condition in very finite function f (x ) which satisfies the Dirichlet’s
∞ ∞
interval (−l , l ) and is convergent, conditions in (0, e) and is such that ∫ 1 f (x ) dx
∫−∞| f (x ) dx| 0

then at every point of continuity of f (x ), exists, then


1 ∞ 2 ∞$
∫ f$( p)e
−ipx
f (x ) = dp f (x ) = f s ( p) sin px dx at every point of
2π −∞ π ∫0
Some authors also define the Fourier and continuity of f (x )
inverse Fourier transform as follows This is called inverse formula for Fourier sine
∞ transform.
(i) f$( p) = ∫−∞ e
ipx
f (x )dx
5. The infinite Fourier cosine transform of f (x ),
1 ∞ −ipx $
and f (x ) = ∫−∞ e f ( p) dp 0 < x < ∝ is denoted by F { f (x )} or f$ ( p) and is
c c

∞ defined as
(ii) f$( p) = ∫−∞ e
−ipx
f (x ) dx 2 ∞
Fc { f (x )} = f$c ( p) = f (x ) cos px dx
1 ∞
π ∫0
−ipx $
and f (x ) = ∫−∞ e f ( p) dp
2π The function f (x ) is called the inverse Fourier
1 ∞ cosine transform of f$ ( p) i. e.,c
(iii) f$( p) = ∫ e
−ipx
f (x ) dx
2π −∞ f (x ) = Fc−1{ f$c ( p)}
1 ∞
∫−∞ f$( p)e
ipx ∝
and f (x ) = dp Some authors define f$c ( p) = ∫0 f (x ) cos px dx

This is called Fourier cosine transform.
127

6. If f$c ( p) is the Fourier cosine transform of the 4. Modulation Theorem


function f (x ) which satisfies the Dirichlet If f$( p) is the complex Fourier transform of f (x )
conditions in every finite interval (0, l ) and is then the Fourier transform of f (x ) cos ax is
1 $
such that

[ f ( p − a) + f$( p + a)]
∫0 | f (x )| dx exists, then 2
 2 ∞
f (x ) =   ∫ f$c ( p) cos px dp at every point CONVOLUTION
 π 0
1. The function
of continuity of f (x ).
1 ∞
This is called inversion formula for infinite F*G= ∫– ∞ F (u) G (x − u) du

Fourier cosine transform.
is called the convolution or Falting of two
PROPERTIES OF FOURIER TRANSFORM integrable functions F andf G over (−∞, ∞).
1. Linearity Property Some define it as

If f$( p) and g$( p) are Fourier transforms of f (x ) F*G= ∫−∞ F (u), G (x − u) du
and g (x ) respectively, then
2. If F { f (x )} and F { g (x )} are the Fourier
F { af (x ) + by(x )} = af$( p) + bg$( p) transforms of the functions f (x ) and g (x )
2. Change of Scale Property respectively then the Fourier transform of the

(i) If f$( p) is the complex Fourier transform of convolution of f (x ) and g (x ) is the product of
their Fourier transforms i. e.,
f (x ), the complex Fourier transform of f (ax ) is
1  p F { f (x ) * g (x )} = F { f (x )}. F { g (x )}
f$   .
a  a This is called convolution or Falting Theorem.
(ii) If f$s ( p) is the Fourier sine transform of f (x ),
FOURIER TRANSFORM OF THE DERIVATIVES
then the Fourier sine transform of f (ax ) is 1. The Fourier transform of f ′ (x ), the derivative
1 $  p of f (x ) is −ipf$( p), where f$( p) is the Fourier
fs  
a  a transform off (x ).
(iii) If f$c ( p) is the Fourier cosine transform of 2. The Fourier transform of f n(x ), the n th
f (x ), then the Fourier cosine transform of f (ax )
derivative of f (x ) is (−ip)n times the Fourier
1  p
is f$c   . transform of f (x ) provided that the first (n − 1)
a  a
derivatives of f (x ) Vanish as x → ± ∞.
3. Shifting Property
If f$( p) is the complex Fourier transform of f (x ),
then the complex Fourier transform of f (x − a)
is e −ipa f$( p).
128

EXERCISE
MULTIPLE CHOICE QUESTIONS (a)
2 ∞
f (x) sin px dp (b)
2 x
f (x) sin px dx
1. The infinite Fourier cosine transform of f (x) is $fc (p) π ∫0 π ∫0
2 ∞ 2 x
is : (c) f (x) sin px dx (d) f (x) sin px dp
π ∫0 π ∫0
2 x 2 ∞
(a) f (x) cos px dx (b) f (x) cos px dx
π ∫0 π ∫0 10. If Fc {f (x)} = $fc (p) then FC {f (ax)} is :
2 ∞ 1 $  p p
(c) cos px dx (d) None of these (a) fc   (b) af$c  
π ∫0 a  a  a

2. If $f (p) and g$ (p) are Fourier transform of f (x) and 1 $  a 1$


(c) fc   (d) fc (ap)
a  p a
g(x), then F {af (x) + bg(x)} is :
(a) $f (ap) + g$ (bp) (b) af$(p) + g$ (bp) 11. If F {f (x)} = $f (p) thenF {f (x − a)} is :
(c) f$(ap) + bg$ (p) (d) af$(p) + bg$ (p) (a) e − ipa $f (p)
p
(b) e − ip $f  
 a
3. If F {f (x)} = $f (p) then F {f (ax)} is :
p
p 1 p (c) e ipa $f (p) (d) e ipa $
f  
(a) $f   (b) $ f   a
 a a  a
1 $
p 1$ 12. {fs (p + a) − $fs (p − a)} is equal to :
(c) af$  (d) f (ap) 2
 a a
(a) FS {f (x) cos ax} (b) FS {f (x) sin ax}
1, |x |< a (c) FC {f (x) sin ax} (d) FC {f (x) cos ax}
4. If F (x) =  then for p ≠ 0, F {f (x)} is :
0, |x |> a
 2π
(a)
sin pa
(b)
sin pa
(c)
2 sin pa
(d)
2 sin pa 13. If f (x) =  2a , |x |≤ a then F {f (x)} is :
p 2π a 2π p 2π a 2π 0, |x |> a
5. The values of Fs {e − x } is : cos ap sin ap
(a) (b)
ap ap
2 1 2 1
(a) (b) sin p cos p
π p2 + 1 π p2 − 1 (c) (d)
ap p
2 p 2 p
(c) (d) cos x, 0 < x < a
π p2 − 1 π p2 + 1 14. Iff (x) =  then FC {f (x)} is :
0 , x>a
 1  π −p  x  1  sin ap cos ap 
6. If FC  = e then FS   is : (a)  + 
1 + x 2  2 1 + x 2  2π  p p 
2 −p π −p 1  cos (1 + p) a cos (1 − p) a 
(a) 0 (b) e − p (c) e (d) e (b)  + 
π 2 2π  1 + p 1− p 
1  sin (1 + p) a cos (1 − p) a 
7. If $fc (p) = πe − p / 2 then f (x) is : (c)  + 
2π  1 + p 1− p 
21 π 1
(a) (b) 1  sin (1 + p) a sin (1 − p) a 
πx 2 1+ x2 (d)  + 
2π  1 + p 1− p 
2 e− x
(c) (d) None of these 0 0 < x < a
π 2
15. If f (x) = x a ≤ x ≤ b then $fs (p) is :
8 If F {f (x)} = $f (p) then F {f n(x)} is : 0 x > b

(a) − $f n(p) (b) (−p)n $f (p) 2 ∞ 2 ∞
(a) x sin px dx (b) sin px dx
(c) (−ip)n f$(p) (d) None of these π ∫0 π ∫0
2 b 2 b
9. The infinite Fourier sine transform of f (x) i. e., $fs (p) is : (c) x sin px dx (d) sin px dx
π ∫a π ∫a
129

π
16. If $fs (p) = then f (x) is :  a − p 
2 $  
24. If fc (p) =   2  , p < 2a then f (x) is :
(a)
π
x (b)
π  2π
2 2 0 , p ≥ 2a

(c)
π 2
x (d)
π1 sin 2 ax cos 2 ax
(a) (b)
2 2x π πx 2
2
17. If $fc (p) = e − p then f (x) is : cos ax sin 2 ax
(c) (d)
2 2 2 πx πx 2
(a) (x + 1) (b) x
π π e − ap
25. If $fs (p) = then f (x) is :
2 x 2 1 p
(c) (d)
π (x 2 + 1) π (x 2 + 1) π 2 x
(a) tan −1(x) (b) tan −1  
18. If $fc (p) is the Fourier cosine transform of f (x) then 2 π  a
f (x) is : 2 x 2 e − ax
(c) sin −1   (d)
2 ∞$ 2 ∞$ π  a π x
(a) fc (p) cos px dx (b) fc (p) dp
π ∫0 π ∫0
26. If $fs (p) is the Fourier sine transform of f (x) then f (x)
2 ∞$
(c) fc (p) cos px dp (d) None of these
π ∫0 is :
2 ∞$ 2 ∞$
19. If Fs {f (x)} = $fs (p) then Fs {f (ax)} is : (a) fs (p) sin px dp (b) fs (p) sin px dp
π ∫0 π ∫0
p 1$
(a) af$s   (b) fs (ap)
 a a 2 ∞$ 2 ∞$
(c) fs (p) dp (d) fs (p) dx
1 $  p π ∫0 π ∫0
(c) fs   (d) af$s (ap)
a  a 27. If F {f (x)} = $f (p) then F {f (x) cos ax] is :
1, |x |< a (a) $f (p) cos ap (b) $f (p) sin ap
20. If F (x) =  then for p = 0, $f (p) is :
0, |x |> a $f (p − a) + $f (p + a)
cos ap + sin ap 
a 2a 2 2a (c) $f (p)  (d)
(a) 2a (b) (c) (d)  2  2
2π 2π 2π
28. If F and G are two integrable functions over (− ∝, ∝)
21. Fc {e − x } is : then the convolution F * G is :
2 1 2 p 1 ∞
(a) (b) (a)
π 1 + p2 π p2 + 1 2π ∫0 F(u) G (x − u) du
2 1 2 p 1 ∞
(c) (d) (b) ∫−∞ F(u) G (u) du
π p2 − 1 π p2 − 1 2π
x, |x |≤ a 1 ∞
22. If f (x) =  then Fourier transform of f (x) (c) ∫−∞ F(u) G (x − u) du
0, |x |> a 2π
is : 1 ∞
(d) ∫−∞ F(u) G (x + u) du
2 2 2π
(a) (cos ap − sin ap) (b) (p cos ap + sin ap)
π π
x 0< x < 1
i 2 29. If f (x) = 2 − x 1 < x < 2 then Fc {f (x)} is :
(c) − (ap cos ap − sin ap)
p2 π 0 x>2

(d) None of these
2 cos p 2 sin p
π (a) (b)
23 If f$c (p) = then f (x) is : π p2 π p2
2
π π π x 2 sin p 2
(a) 0 (b) x (c) (d) . (c) 2 (1 − cos p) (d) 2 sin p (1 − cos p)
2 2 2 2 π p2 π
130

30. If $fs (p) = e − ap then f (x) is : 38. If F {f (x)}s = $f (p) then F {f (x) cos ax} is :
2 x 2 2 $f (p − a + $f (p + a)
(a) (b) (a + x 2) 1$
π a + x2
2 π (a) (b) f (p − a)
2 2
2 a2 + x 2 $f (p − a) − $f (p + a)
(c) (d) None of these 1$
π x (c) f (p + a) (d)
2 2
1
31. If f$s (p) = then f (x) is :
p 39. FS[e −5x ] is equal to :
2 π
(a) p (b) p p2 + 1 p2 − 1
π 2 (a) (b)
5 25
π 1 2
(c) (d) p(p − 1) p
2 p π (c) (d)
5 2
p + 25
1 , |x |< a
32. If f (x) =  then F {f (x)} is :
−2x
0 , |x |> a 40. Fc {e } is equal to :
2 p
(a) sin ap (b) sin ap
p 2 2 1  2 4 
(a)   (b)  
2 p π  p 2 + 4 π  p 2 + 4
(c) cos ap (d) cos ap
p 2
2 2  4  2
33. If f (x) = e − x then lim fs {f (x)} is : (c)   (d)  
p→ ∞ π  p 2 + 4 p2 − 4  π 
1
(a) 0 (b) 1 (c) (d) 2
2 1
41. If f (x) = then Fs {f (x)} is :
x
34. The convolution of f (x) and g (x) is f (x) * g (x) then
F {f (x) * g(x)} is : π 1 π 1 π
(a) p (b) (c) (d)
(a) F {f (x)} + F {g(x)} (b) F {f (x)} − F {g(x)} 2 p 2 p 2

 f (x )  42. Fs {f (x)} is equal to :


(c) F   (d) F {f (x)} − F {g(x)}
 g (x )  d d
(a) − Fc {f (x)} (b) − Fc {f (x)}
dp dx
35. If F {f (x)} = F (p$ ) then F {f ′(x)} is :
d
(c) − Fs {f (x)} (d) None of these
(a) f$ ′(p) (b) ipf$(p) dp
(c) −pf$(p) (d) −ipf$(p) 43. Fc {xf (x)} is equal to :

x 0< x < 1 d
(a) xf ′(x) (b) Fc {f (x)}
36. If f (x) = 2 − x 1 < x < 2 then find Fs {f (x)} : dp
0 x>2 d
 (c) − xf ′(x) (d) Fs {f (x)}
dp
2 sin p 2 cos p
(a) 2 (1 − cos p) (b) 44. If Fs {f (x)} = (2 π p)1/ then f (x) is :
π p2 π p2
1 1
(a) (b)
2 2 sin p x x
(c) cos p (1 − cos p) (d) 2 (1 − sin p)
π π p2 1
(c) x x (d)
x x
37. If $fc (p) = e − ap then f (x) is :

45. If ∫ f (x) cos px dx = e − p then f (x) is :
0
2 a2 + x 2 2 a
(a) (b) 1 2 1
π a π x 2 + a2 (a) (b)
1+ x2 π 1+ x2
2 π 2 1 21
(c) a (d) a (c) (d)
π 2 π 1+ x2 πx
131

ANSWERS
MULTIPLE CHOICE QUESTIONS
1. (b) 2. (d) 3. (b) 4. (c) 5. (d) 6. (d) 7. (b) 8. (c) 9. (c) 10. (a)
11. (c) 12. (c) 13. (b) 14. (b) 15. (b) 16. (d) 17. (d) 18. (c) 19. (c) 20. (d)
21. (a) 22. (c) 23. (a) 24. (d) 25. (b) 26. (a) 27. (d) 28. (c) 29. (c) 30. (a)
31. (c) 32. (a) 33. (a) 34. (d) 35. (d) 36. (a) 37. (d) 38. (a) 39. (d) 40. (c)
41. (d) 42. (a) 43. (d) 44. (d) 45. (b)

HINTS AND SOLUTIONS


1, |x |< a 1 a ipx 2π
4. Given that f (x) = 
0, |x |> a
=
2π ∫− a e 2a
dx

a
1 ∞ 1  e ipx  e ipa − e − ipa sin pa
Since F {f (x)} = $f (p) = ∫−∞ e ipx f (x) dx =   = =
2π 2a  ip  − a 2ipa pa
a
1 a 1  e ipx 
= ∫− a e − ipx dx =   15. $f (p) = 2 ∞
f (x) sin px dx
2π 2 π  ip  − a π ∫0
s

1  e ipa e − ipa  2  a b ∞ 
=  −  = 0 sin px dx + ∫a x sin px dx + ∫b 0 sin px dx 
2π  ip ip  π  ∫0
2 i sin pa 2 sin pa 2 b
= = = x sin px dx
ip 2 π p 2π π ∫a
2 ∞ cos px π −p
6. Given that f$c (p) = dx = e 16. By Fourier sine inverse formula we have
π ∫0 1 + x 2 2 2 ∞$ 2 ∞π
f (x ) = ∫ fs (p) sin pxdp = sin px dp
Differentiating both sides w.r.t. p we get π 0 π ∫0 2
∞ x sin px π π ∞
−∫ dx = − e − p = sin px dx
0 1+ x2 2 2 ∫0

2 ∞ x π −p ∞ − ipx  e − ipx  1 i
∴ $fs (p) = sin px dx = e Since ∫0 e dx =   = =−
π ∫0 1 + x 2 2  − ix 0 ix x
∞ i
πe − p i. e. ∫0 (cos px − i sin px ) dx =−
7. Here f$c (p) = , so by Fourier cosine inversion x
2 ∞ 1
formula we have
⇒ ∫0 sin px dx = x
π  1
2 ∞$ ∴ f (x ) =  
f (x ) = f c ( p) cos px dp 2x
π ∫0
^ 1 ∞ 1 a ipx
2 ∞ πe − p 20. f (p) = ∫−∞ e
ipx
f (x)dx = ∫− ae dx
= cos px dp
π ∫0 2
2π 2π
∞ 1 a 2a
π  e− p  π 1 ∴ p = 0 so ^
f (D) = ∫− a dx =
=  (− cos px + sin px) = . 2π 2π
2 1 + x 2 2 1+ x2
0
^ 2 ∞ 2 ∞ −x
21. f c (p) = f (x) cos px dx = e cos px dx
 2π π ∫0 π ∫0
13. Here, f (x) =  2a |x |≤ a

0 |x |> a 2  e− x  2 1 
=  (− cos px + p sin px) =  
1 ∞ π 2 π  1 + p2 
F {f (x)} = e ipx f (x) dx 1 + p 0
2π ∫−∞
132

24. By Fourier cosine inversion formula, we have 2 ∞ −x


33. FS {e − x } = ^
f S(p) = e sin px dx
2 ∞$ π ∫0
f (x ) = f c ( p) cos px dp
π ∫0

2  e− x 
=  (− sin px − p cos px)
π 2
2 2a 1  p 1 + p 0
f (x ) =  a −  cos px dp
π ∫0 2π  2 =
p 2
1 + p2 π
2 ∞
+ 0.cos px dp
π ∫2α So lim $fS(p) = 0
p→ ∞
2a 2a
1  p  cos px  2 ∞ 2 1
= a −  sin px  −  36. $fS(p) = f (x) sin px dx = [ x sin px ] dx
 πx  2  
 2 πx 2  0 π ∫0 π ∫0
0
2
1 − cos 2ax 2 sin 2 ax + ∫ (2 − x) sin px dx]
= = = π −1x −2 sin 2 ax 1
2 πx 2 2 πx 2 1
 
2   − x cos px + 1 sin px 
25. By Fourier cosine inversion formula =
π  p p2 0
 e − ap  2 ∞ e − ap sin px 
Fs−1   = f (x ) = dp
 p  π ∫0 p  2 − x 
2
1
+ −   cos px − 2 sin px 
df 2 ∞ − ap   p  p  1
Differentiating w.r.t. x = e cos px dp
dx π ∫0
2 2 1  2 sin p
df 2 a
=  2 sin p − 2 sin 2p  = 2 (1 − cos p)
= π p p  π p2
dx π a2 + x 2
Integrating we get 2 ∞
38. FS {f (x) cos ax} = f (x) cos ax sin px dx
π ∫0
2 x
f = tan −1   + A 21 ∞
π  a = f (x){sin (p + a)x + sin(p − a)x}dx
π 2 ∫0
But when x = 0, f = 0
1 2 ∞
= f (x) sin (p + a)x dx
so A= 0 i. e. f (x) =
2 x
tan −1   2  π ∫0
π  a
2 ∞ 
+ f (x) sin (p − a) x dx 
π ∫0
$f (p) = 2 ∞
29. f (x) cos px dx
π ∫0
c 
1 $
2 1 2 = [ fS(p + a) + $fS(p − a)]
= x cos px dx + (2 − x) cos px dx 2
π ∫0 ∫1
2 ∞ −2x
1 40. FC [e −2x ] = e cos px dx
π ∫0
 
2   x sin px + cos px 
=
π  p p2  0 ∞
 2  e −2x 
=  (−2 cos px + x sin px)
 2 π  4 + x 2
 2− x cos px    0
+ sin px − 
 p p2 1  2 2
 =
π 4 + p2
2 1 2 cos p
= [2 cos p − 1 − cos 2p] = 2 (1 − cos p)
π p2 π p2
1 2 ∞ sin px
41. FS   = $fS(p) = dx
π ∫0 x
a
∞ ipx a ipx
ipx
e   x 
32. F {f (x)} = ∫−∞ e f (x) dx = ∫− a e 1dx =  
 ip  − a put px = t, pdx = dt

2  e ipa − e − ipa  2 sin ap 2 ∞ sin t 2 π π


=  = when p ≠ 0 f$S(p) = dt = =
ip  2  p π ∫0 t tπ 2 2
133


43. The Fourier sine transform of f (x) 45. Given ∫ f (x) cos px dx = e − p
0
2 ∞
Fs {f (x)} = f (x) sin px dx = $fS(p)
π ∫0
2 ∞ 2 −p
or f (x) cos px dx = e = FC {f (x)}
π ∫0 π
d 2
Differentiating w.r.t. p we get FS {f (x)} =  2 −p  2 2 ∞ −p
dp π ∴ f (x) = FC−1  e = . e cos px dp
∞  π  π π ∫0
∫0 xf (x) cos px dx 2 ∞ −p
f (x ) = e cos px dp
or
d
FS {f (x)} = FC {xf (x)} π ∫0
dp ∞
2  e− p 
=  {− cos px + x sin px}
π 1 + x 2 0
2 1
=
π 1+ x2

mmm
134
Unit-V
C HAPTER
18 Finite Fourier Transform
FINITE FOURIER SINE TRANSFORM 1$ 2 ∞ $ πpx
f (x ) = f C (x ) + Σ f C ( p) cos
1. The finite fourier sine transform of f (x ) in the l l p =1 l
l
finite range (0, e) is defined as where f$C (0) = ∫0 f (x ) dx
l πpx
FS { f (x )} = f$S ( p) = ∫ f (x ) sin dx
0 l If the interval is (0, π) then it becomes
where p is an integer. 1 $ 2 α $
f (x ) = f C (0) + Σ f C ( p) cos px
In the finite range 0 < x < π it is defined as π π p =1
π
FS { f (x )} = f$S ( p) =
π
f (x ) sin px dx where f$C (0) = ∫0 f (x ) dx
∫0
2. The finite fourier cosine transform of f (x ) in 3. Let f (x , y) be a function of x , y defined in
(0, l ) is defined as 0 ≤ x ≤ π and 0 ≤ y ≤ π then
l πpx
FC { f (x )} = f$C ( p) =
π π
∫0 f (x ) cos dx f$S ( p, q) = ∫0 ∫0 f (x ,y) sin px sin qydx dy
l
If the range is (0, π) then Similarly,
π
FC { f (x )} = f$C ( p) = ∫0 f (x ) cos px dx f$C ( p, q) =
π π
∫0 ∫0 f (x , y) cos px cos qydx dy
INVERSE FORMULA FOR FINITE FOURIER PROPERTIES OF FINITE FOURIER TRANSFORM
TRANSFORM 1. If f (x ) is continuous and f ′ (x ) is sectionally
1. If f$S ( p) is the finite fourier sine transform of f (x ) continuous then
over (0, l ) then the inverse formula for sine
(i) FS { f ′ (x )} = − pFC { f (x )}, p = 1, 2, 3, ...
transform is
(ii) FC { f ′ (x )} = pFS { f (x )} − f (0)
2 α $ πpx
f (x ) = Σ f S ( p) sin
l p =1 l +(−1)p f (π), p = 0, 1, 2, ...

If interval is (0, π) then it becomes 2. If F (x ) and G(x ) be two functions on the interval
2 α $ (−2π, 2π) then
f (x ) = Σ f S ( p) sin px
π p =1 π
F (x ) * G(x ) = ∫−π F (x − y) G(y) dy is called the
2. If f$C ( p) is the finite fourier cosine transform of
convolution of F (x ) and G(x ).
f (x ) over (0, l ) then the inversion formula for
cosine transform is
135

EXERCISE
MULTIPLE CHOICE QUESTIONS 10. The finite fourier cosine transform of x when p = 0,
is :
1. The finite fourier sine transform of 1 is : π2
1 + (−1)p 1 − (−1)p (a) π (b) − π (c) π 2 (d)
(a) (b) 2
p p x
11. The finite fourier sin transform of is :
1 + p2 1 − p2 4π
(c) (d)
p p
(−1)p (−1)p −1
(a) (b)
2. The finite fourier cosine transform of x is : 4p 4p
1 + (−1)p 1 − (−1)p
(a) (b) (−1)p +1 (−1)p
p p2 (c) (d)
p 4p p
−1 − (−1) −1 + (−1)p
(c) (d)
p2 p2 π
cos  2p 
 3  and 0 < x < 1 then f (x) is :
x 12. If $fC (p) = −
3. The finite fourier sine transform of  1 −  is : (2p + 1)2
 π
1 πp p π
(a) p (b) (c) (d) cos  2p 
p 1− p 1+ p α  3  cos pπx
(a) Σ
4. If $fS(p) is the finite fourier sine transform of f (x) is p =1 (2p + 1)2

(0, π) then f (x) is : π


α
cos  2p 
α
α $f (p) sin px  3  sin pπx
(a) ∫ S (b) Σ $fS(p) sin px (b) Σ
0 1 1 (2p + 1)2
2α 2 α$
(c) Σ $fS(p) sin px dx (d) fS(p) sin px dp
π ∫0
π
π0 cos  2p 
α  3  sin pπx
(c) 1 + Σ
5. The finite fourier cosine transform of f (x) is : 1 (2p + 1)2
πpx l cos πpx
(a) Σf (x) cos (b) ∫ f (x) dx
0 π
l l cos  2p 
α
l px px  3  cos pπx
(c) ∫ f (x) cos dx (d) Σf (x) cos (d) 1 + 2 Σ
0 l l p =1 (2p + 1)2
1, π
r<x<
 2 then $f (0) is : 13. If f (x) = sin mx, m is + ve integer and m = p then
6. If f (x) =  C
π $f (p) is :
−1, <x< π S
 2
2 2 πp  π π
(a) cos πp (b) cos   (a) (b) (c) π (d) 0
p p  2  2 4
1
(c) (d) 0 14. If f (x) = 2x, 0 < x < 4 then $fS(p) is :
p
2 4 4
x
7. The finite fourier cosine transform of  1 −  when (a) cos pπ (b) − cos pπ
 π pπ pπ
p = 0 is : 32 32
π π π (c) sin pπ (d) − cos pπ
(a) (b) (c) π (d) pπ p
4 2 3
8. 2 $
If f (x) = x , 0 < x < 4 then f (0) is : 15. If f (x) = sin nx and(n − p) is given in 0 < x < π then
C
3 64 $f (p) is :
(a) 0 (b) 1 (c) (d) C
64 3 1 1
(a) (b)
9. The finite fourier cosine transform of 1 is : n+ p n− p
1 + (−1)p
(a) p (b) 1 1
p (c) + (d) 0
n+ p n− p
(c) 1 + (−1)p (d) 0
136

1 − cos pπ π π π
16. If $fS(p) = , 0 < x < π then f (x) is : (a) (b) (c) (d) 0
p2 π2 2 4 6
x
The finite fourier sine transform of  1 −  is :
α  1 − cos pπ 
(a) Σ  24.
 sin px  π
p =1  p 
1 1− p
α  1 − cos pπ  (a) 0 (b) p (c) (d)
2 p p
(b) Σ   sin px
π 3 p =1  p2 
x; π
0≤ x <
α cos pπ 25.

If f (x) =  2 then F {f (x)} is :
(c) ∫ sin px dx π S
φ p  π − x; <x< π
 2
2 α 1+ cos pπ 
(d)  sin px pπ 2 pπ
π2 1
∫ 
 p2 
(a) 2 sin (b) sin
2 p 2
2 pπ 2 pπ
17. The finite fourier sine transform of x is : (c) cos (d) sin
p 2 2 2
p
(−1)p +1 (−1)p
(a) (b)
p p x
26. The finite fourier cosine transform of is :
π (−1) p
p (−1) p 4π
(c) (d) −1 − (−1)p −1 + (−1)p
p 4 πp 2 (a) (b)
2
4 πp p2
x
18. The finite fourier cosine transform of  1 −  is : (−1)p
 π (c) (d) None of these
4 πp 2
1 − (−1)p −1 − (−1)p
(a) (b)
πp 2 πp 2 27. If p = n then finite fourier cosine transform of sin nx
1 + (−1) p
−1 − (−1) p is :
(c) (d) 2n n
2
4 πp 4 πp 2 (a) (b)
n2 − p 2 n2 − p 2
1; π

0< x < (c) np 2 (d) 0
19. If f (x) =  2 then $f (p) is :
π C
 − 1; < x < π 28. If p = 1, 2, 3,... then finite fourier cosine transform of
 2
3x 2 is :
2 πp 
(a) sin   (b) p sin ( πp) (a)

(−1)p (b)

(−1)p
p  2  p 2
p
p πp 
(c) sin   (d) None of these 6π p 6π
2  2  (c) (−1) (d) (−1)p
p3 p2
20. If f (x) = sin nx , n is positive integer and p = n then
$f (p) is : 29. If p = 0 then finite fourier cosine transform of
S
π π x2 π
(a) π (b) (c) (d) 0 − is :
2 4 2π 6
2π 2π 3π
21. If f (x) = 2x, 0 < x < 4 then $f (0) is : (a) 0 (b) (c) (d)
C p p2 p3
(a) 0 (b) 1 (c) 4 (d) 16
30. The finite fourier sine transform of x( π − x) is :
22. If n − p is odd then fourier finite cosine transform of 1 − (−1)p 2[1 − (−1)p ]
(a) (b)
sin nx is : p 2
p3
n 2n
(a)
2 2
(b) 2 [1 + (−1)p ] 1 + (−1)p
n +p n − p2
2 (c)
3
(d)
p p3
2n
(c) (d) 0
n2 + p 2 π x2 
31. The finite fourier cosine transform of  − x + 
3 2π 
23. The finite fourier sine transform of sin nx for p ≠ n
is : is :
137

1 1 38. The finite fourier sine transform of Cx in 0 < x < l


(a) p (b) (c) (d) 0 is :
p 2
p l 2C lC
(a) (−1)p +1 (b) −
32. If p = 0 then the finite fourier sine transform of πp πp
f (x) = a is : πC
(c) (−1)p (d) None of these
a pl 2
(a) 0 (b) ap (c) (d) a(−1)p
p
39. If f (x) is continuous and f ′(x) is sectionally
33. If p = 0 then the finite fourier cosine transform of continuous then for p = 1, 2,... FS {f ′(x)} is :
f (x) = Cx in the interval 0 < x < l is :
(a) −FC {f (x)} (b) −pFC {f (x)}
Cl 2
(a) Cpl (b) Cl (c) pl (d) (c) FC {f (x)} (d) pFC {f (x)}
2
34. The finite fourier cosine transform of Cx in (0, l) is : 40. If F (x) and G(x) be two functions on − π < x < π then
Cl Cl 22 the convolution F (x) * G(x) is :
(a) (cos πp − 1) (b) (cos πp − 1) π π
πp π2 (a) ∫ F (x)G(x) dx (b) ∫ F (x) G(y) dy
−π −π
2 π π
cos πp Cl (c) ∫ [F (x) − G(x)] dx (d) ∫ F (x) G(x − y) dy
(c) (d) (cos πp − 1) −π −π
2
Cp π 2p 2
41. If f (x) is continuous and f ′(x) is sectionally
35. If p = 0 then finite fourier cosine transform of 3x 2 is :
continuous then for p = 0, 1, 2,... FC {f ′(x)} is :
(a) 0 (b) π (c) π 2 (d) π 3
(a) pFS {f (x)} − f (0) + (−1)p f ( π)
36. If p = 0 then finite fourier cosine transform of
(b) pFS {f (x)} − f (0)
π x2 
 −x+  is : (c) pFS {f (x)} + (−1)p f ( π)
3 2π 
1 2p (d) None of these
(a) p (b) (c) (d) 0
p2 π 42. The finite fourier cosine transform of f (x) on (0, l) is :
πpx l
37. If p = 0 then finite fourier cosine transform of (a) Σf (x) cos (b) ∫ f (x) cos px dx
l 0
f (x) = a is :
π l l πpx
(a) 0 (b) π (c) aπ (d) (c) ∫ f (x) cos πx dx (d) ∫ f (x) cos dx
0 0 l
a

ANSWERS
MULTIPLE CHOICE QUESTIONS
1. (b) 2. (c) 3. (b) 4. (d) 5. (b) 6. (d) 7. (d) 8. (d) 9. (d) 10. (d)
11. (c) 12. (d) 13. (a) 14. (d) 15. (d) 16. (b) 17. (c) 18. (a) 19. (a) 20. (d)
21. (d) 22. (b) 23. (d) 24. (c) 25. (d) 26. (a) 27. (d) 28. (d) 29. (a) 30. (b)
31. (c) 32. (a) 33. (d) 34. (d) 35. (d) 36. (d) 37. (c) 38. (a) 39. (b) 40. (d)
41. (a) 42. (d)

HINTS AND SOLUTIONS


1. Given f (x) = 1 2. f (x ) = x ,
π π
so $fC (p) =
$f (p) = π
f (x) cos px dx = ∫0 x cos px dx
S ∫0 f (x) sin px dx ∫0
π π π
π  cos px   x sin px  1 π  cos px 
= ∫0 sin px dx = −   =  − ∫0 sin px dx =  
 p 0  p 0 p  p2  0
1 − cos πp 1 − (−1)p (−1) + (−1)p
= = = i. e. p = 1, 2,...
p p p2
138

π x p
3. $f (p) =  cos  2 π 
S ∫0  1 − π  sin px dx = 1+ 2 Σ
 3 cos πpx
π (2p + 1)2
 x cos px  π 1  cos px 
= −  1 −  −  −  −  dx
π  p  0 ∫0  π 
 4
  p   2x cos  pπx  
  
π $f (p) = 4  pπx   4 
1 1  sin px  1 14. S ∫0 2x sin   dx = −  
= −   =  4   π 
p
p π  p2  0 p  4 0
π pπx 
1, 0< x < 4 cos  
 2 4  4 
6. Given f (x) =  + 2∫ dx
π 0 pπ
−1, <x< π
 2 4
π π
Then fC (0) = f (x) cos px dx = 4
∫0 ∫0 f (x) dx  sin  pπx  
 
π/2 π 32 8   4  32
1 dx + =− cos pπ +   =− cos pπ
= ∫0 ∫π / 2(−1) dx = 0 pπ pπ  pπ
 p π
7. When p = 0, cos px = 1  4 0
1 − cos πp
π π x
2
16. Given $fS(p) = , 0< x < π
∴ f$C (p) = ∫0 f (x) cos px dx = 
∫0  1 − π  dx p2 π2
π 2 α $ 2 α  1 − cos pπ 
π 3 ∵ f (x ) = Σ fS(p) sin px = Σ   sin px
x  π π p =1 p 2 π 2 
= −   1 −   = π p =1
 3  π   0 3
2 α  1 − cos pπ 
= Σ   sin px
8. When p = 0, cos px = 1, f (x) = x 2 π 3 p =1 p2 
4 4 2 $f (p) = π  x
πpx  18.
So, $fC (p) = ∫0 f (x) cos   dx = ∫ x dx C ∫0  1 − π  cos px dx
 4  0
π
 x sin px  π 1 sin px
$f (p) = 64 =  1 −  − ∫  −  dx
C  π  p 0 0  π p
3 
π
10. Given f (x) = x and p = 0 1  cos pπ  1
= − = [1 − (−1)p ]
$f (p) = π f (x) cos px dx 2 
π p 0 πp 2
C ∫0
π 20. If p = n and f (x) = sin nx then
π  x2 π2
= x dx =   = $f (p) = π sin nx sin nx dx = π 2
∫0  2 0 2 S ∫ 0 ∫0 sin nx dx
π
$f (p) = π
=
π  1 − cos 2nx  dx = 1  x − sin 2nx  = π
11. S ∫0 f (x) sin px dx ∫0  2

 2  2n  0 2
π x π
= ∫0 sin px dx 22. f$C (p) = ∫0 sin nx cos px dx

1 π
π = [sin (n + p) x + sin (n − p)x] dx
1  cos px  cos px  2 ∫0
π
=  −x  + ∫0 dx 
4π  p 0 p  π
 1  cos (n + p) x cos(n − p) x 
= − − if p ≠ n
π 2  n+ p n− p 
0
1 1  sin px  (−1)p +1
=− cos pπ +  2  = 1  cos(n + p) π cos(n − p) π
4p 4π  p 0 4p ∴ $fC (p) = − −
2  n+ p n− p
1$ 2 ∞ $ πpx 
12. ∵ f (x ) = fC (0) + Σ fC (p) cos   +
1
+
1 
l l p =1  l  n + p n − p 
2 πp If n − p is odd then n + p is also odd so
cos
2 ∞ 3 cos πpx
= 1+ Σ $f (p) = 1  2 + 2  = 2n
1 p =1 (2p + 1)2 C
2  n + p n − p  n2 − p 2
139

π 1 1 1
25. $f (p) = = − (sin px)π0 = ,p> 0
S ∫0 f (x) sin px dx p2 p3 π p2
π/2 π l l
x sin px dx + 33. When p = 0, $fC (p) = ∫0 Cx cos 0xdx = ∫0 Cx dx
= ∫0 ∫π / 2( π − x) sin px dx
π/2 Cl 2
 x (− cos px) sin px  =
= +  2
 p p2 0 π π
35. $f (0) = 2 2
dx = π 3
  cos px  sin px 
π C ∫0 3x cos 0x dx = ∫0 3x
+ ( π − x)  − − 
 p  p2 π / 2 38. Here f (x) = Cx, 0 < x < l

$f (p) = l f (x) sin πpx dx = l πpx
=
2
sin 
πp 
 S ∫0 l ∫0 Cx sin l
dx
2  2 
p l
  πpx  pnπ 
π  x2 π   cos cos  2
29. If p = 0 then $fC (p) = ∫0  −  dx = C  −  xl l  l
l dx  = l C (−1)p +1
 2π 6    πp 
 + ∫0l pπ  πp
π   0 
 x3 π  π2 π2
= − x = − =0 π
 6π 6  0 6 6 39. FS {f ′(x)} = ∫0 f ′(x) sin px dx
$f (p) = π π x2  π
= {f (x) sin px}π0 − p ∫ f (x) cos px dx
31. C ∫0  3 − x + 2 π  cos px dx 0

π
= − pFC {f (x)}, p = 1, 2, 3,...
 π x 2  sin px  41. FC {f ′(x)} =
π
=  − x +   ∫0 f ′(x) cos px dx
  3 2 π  p 
0 π
= [ f (x), cos px]π0 + p ∫ f (x) sin px dx
1 π x
− ∫  −1 +  sin px dx 0
p 0  π = f (x) cos pπ − f (0) + pFS {f (x)}, p = 01
, 2,...
π
1 x cos px  1 π cos px = pFS {f (x)} − f (0) + (−1)p f ( π)
= − −  −1 +  − dx
p   π  p  0 p 2 ∫0 π
mmm
140
Unit-V
C HAPTER
19
Applications of Fourier Transforms in
Initial and Boundary Value Problems
APPLICATION OF INFINITE FOURIER TRANSFORMS APPLICATION OF FINITE FOURIER TRANSFORMS
1. The choice of sine or cosine transform is 1. If the range of one of the variables is finite, finite
decided by the form of boundary conditions at Fourier transforms are applied.
the lower limit of the variable selected for ∂ 2U
exclusion. For the exclusion of from a differential
∂x 2
∂ 2V equation, we require
For the exclusion of from a differential
2
∂x (i) U (0, t) and U (l, t) in finite sine transform.
equation we require (ii) U x (0, t) and U x (l , t) in finite cosine
(i) (V)x=0 in sine transform. transform.

 ∂V 
(ii)   in cosine transform.
 ∂x  x =0

EXERCISE
MULTIPLE CHOICE QUESTIONS ∂V
(a)   in sine transform
 ∂x  x = 0
∂ 2V
1. For the exclusion of from a differential (b) (V)x = 0 in cosine transform
∂x 2
∂V
equation if (V)x = 0 is given then we take (c)   in Laplace transform
 ∂x  x = 0
(a) Fourier sine transform ∂V
(d)   in cosine transform
(b) Fourier cosine transform  ∂x  x = 0
(c) Fourier transform ∂U ∂ 2U
4. The finite fourier sine transform of = ,
(d) Laplace transform ∂t ∂x 2

∂U ∂ 2U U (0, t) = U (u, t) = 0, where 0 < x < 4, t > 0 is given


2. In the solution of =2 , U (0, t) = 0, by
∂t ∂x 2 2 2 2 2
(a) U$ S = Ae − p π t / 16 (b) U$ S = Ae p π t
U (x, 0) = e − x , x > 0, U (x, t) is bonded for x > 0, t > 0
2 2 2 2
(c) U$ S = Ae − p π t (d) U$ S = Ae p π t / 16
the new differential equation after Fourier sine
transform is ∂U ∂ 2U
5. The finite fourier cosine transform of =k
(a) (D − p )U$ S = 0
2
(b) (D + 2p ) U$ S = 0
2 ∂t ∂x 2
∂V
with = 0 when x = 0 and x = π, t > 0 is :
(c) (D − 2p 2) U$ S = 0 (d) (D + p 2) U$ S = 0 ∂x
2
∂ 2V (a) U$ C = Ae − pt (b) U$ C = Ae p kt
3. For the exclusion of from the differential
∂x 2 2
(c) U$ C = Ae − p kt
2
(d) U$ C = Ae − p t
equation we require
141

∂U ∂ 2U ∂U ∂ 2U
6. The fourier transform of = for x ≥ 0, 11. The finite fourier sine transform of = ,
∂x ∂y 2 ∂t ∂x 2
− ∞ < b < ∞ is :
0 < x < π, t > 0, U (0, t) = 1, U ( π, t) = 3 is :
(a) U$ = Ae px (b) U$ = Ae − px
2 2 (a) (D + p 2) U$ = p (1 + cos πp)
(c) U$ = Ae − p x (d) U$ = Ae p x
S
(b) (D + p) U$ S = p (1 + 3 sin pπ)
∂U ∂ 2U
7. The fourier finite cosine transform of = (c) (D + p)U$ = p (1 − 3 sin pπ)
∂t ∂x 2 S

0 < x < 6, t > 0, U x (0, t) = U x (6, t) = 0 is : (d) (D + p ) U$ S = p (1 − 3 cos pπ)


2

2 2 2 2
(a) U$ = Ae − p π t
C (b) U$ = Ae − p π t / 16
C ∂U ∂ 2U
12. The fourier transform of =k , −α < x < α is :
p2π 2t
2 2 − ∂t ∂x 2
(c) U$ C = Ae − p π t / 32 (d) U$ C = Ae 36 (a) U$ = Ae − kpt (b) U$ = A cos hpt
2 2 2

8. The finite fourier sine transform of


∂U
=a
∂U
, (c) U$ = Ae − kp t (d) U$ = A cos kp 2t
2 2
∂t ∂x
∂ 2U ∂ 2U
0 < x < 2, t > 0, U (0, t) = U (2, t) = 0 is : 13. The fourier transform of = C2 for
t ∂t 2 ∂x 2
(a) U$ S = Ae −3pπ 2 − α < x < α, t > 0, is given by :
−3π 2 π 2
t (a) U$ = Ae cpt
(b) U$ S = Ae 2
(b) U$ = A cos cpt + B sin cpt
(c) U$ S = A cos
3 πt
+ B sin
3pπt (c) U$ = A cos hcpt + B sin hcpt
2 2
(d) U$ = Ae − cpt
3pπt 3pπt
(d) U$ S A cos h + B sin h
2 2 14. The fourier sine transform of

∂U ∂ 2U ∂U ∂ 2U
9. The finite fourier sine transform of = = , x > 0, t > 0 with U (0, t) = 0,
∂t ∂x 2 ∂t ∂x 2
0 < x < 6, t > 0 and U (0, t) = U (6, t) = 0 is : 1 0 < x < 1
t
U = when t = 0 and U (x, t) is
−p2 0 x ≥ 1
(a) U$ S = Ae 36 (b) U$ S = Ae − pt
t bounded, is :
−p2π 2
(c) U$ S = Ae −6pt (d) U$ S = Ae 36 (a) U$ S = Ae − pt (b) U$ S = A cos hpt
2
(c) U$ S = A cos hp 2t (d) U$ S = Ae − p t
∂ 2U ∂ 2U
10. The finite fourier sine transform of =4 ,
∂t 2 ∂x 2 ∂ 2U ∂ 2U
15. Solve + =0 for U$ S when
2
0 < x < π, t > 0 and U (0, t) = U ( π, t) = 0 is : ∂x ∂y 2
(a) U$ S = Ae −2pt U (0, y) = 0 = V( π, y) :
t2
4p2 (a) Ae − ipy
(b) U$ S = Ae π
(b) Be − ipy
(c) U$ S = A cos 2pt + B sin 2pt (c) A cos hpy + B sin hpy (d) Ae ipy + Be − ipy
(d) U$ = A cos h2pt + B sin h2pt
S

ANSWERS
MULTIPLE CHOICE QUESTIONS
1. (a) 2. (b) 3. (d) 4. (a) 5. (c) 6. (c) 7. (d) 8. (c) 9. (d) 10. (c)
11. (d) 12. (c) 13. (b) 14. (d) 15. (c)
142

HINTS AND SOLUTIONS


∂U ∂ 2U
2. Taking fourier sine transform of =2 , we dU$ C − p 2 π 2 $
∂t ∂x 2 = U C − {U x (0, t) − U x (6, t) cos pπ}
have dt 62
2 ∞ ∂U 2 ∞ ∂ 2U dU$ C − π 2p 2 $
∫ sin px dx = 2 sin px dx
π ∫0 ∂x 2
0
or = UC
π ∂t dt 36
∞  p 2 π 2t 
d 2 ∞ 2   ∂U
U sin px dx = 2 sin px  So U$ C = Ae − 
dt ∫
π 0
 
π   ∂x 0  36 
 
∞ ∂U
−p∫ cos px dx 
0 ∂x  ∂ 2U ∂ 2U
∂U 8. Taking finite fourier sine transform of =a ,
if → 0 as x → ∞ so ∂t 2 ∂x 2
∂x
dU$ S 2 2 2 α
we have
= −2p (U cos px)α
0 − 2p U sin px dx
dt π π ∫0 2∂ 2U
sin
pπx
dx = a∫
2∂ 2U
sin
pπx
dx
if U → 0 as x = ∞ we get
∫0 ∂t 2 2 0 ∂x 2 2

= t 2p
2
U (0, t) − 2p 2U$ S ∂ 2U$ S 9p 2 π 2 $ 9pπ
or =− US +
π dt 2 4 4
dU$ S 2
or + 2p 2U$ S = 2p U (0, t) = 0 [U (0, t) − U (2, t) cos pπ]
dt π2 d 2U$ S 9p 2 π 2 $
∴ U$ S = Ae −2p t + US = 0
dt 2 4
5. Taking finite fourier cosine transform of
3pπt 3pπt
So, the solution is U$ S = A cos + B sin .
∂U ∂ 2U 2 2
=k
∂t ∂x 2
∂ 2U ∂ 2U
2 10. Taking Laplace transform of =4
2
π ∂U
cos px dx = k ∫
∂U
π
cos px dx ∂t ∂x 2
∫0 ∂t 0 ∂x 2 π ∂U2 π 2
∂U
∫0 sin px dx = 4∫ sin px dx
dU$ C ∂t 2 0 ∂x 2
= k[− p 2U$ C − {U x (0, t) − U x ( π, t) cos πp}]
dt d 2U$ S
dU C = −4p 2U$ S + 4p {U (0, t) − U ( π, t) cos pπ}
or = −kk 2U$ C dt 2
dt
2 ∵ U (0, t) = U ( π, t) = 0, we get
Integrates it U$ = Ae − kp t
C
d 2U$ S
+ 4p 2U$ S = 0
6. Taking the fourier transform of both sides of dt 2
∂U ∂ 2U ⇒ (D 2 + 4p 2) U$ = 0
= we have S
∂x ∂y 2
So the required solution is
∞ ∂U ipy ∞ ∂ 2U
∫– ∞ ∂x e dy = ∫– ∞ ∂y2 e ipy dy U$ S = A cos 2pt + B sin 2pt

11. Taking the finite fourier sine transform of


d ∞
or Ue ipy dy = (−ip)2U$
dx ∫−∞ ∂U ∂ 2U
=
d $ ∂t ∂x 2
2
U = − p 2U$ ⇒ U$ = Ae − p x
dx π ∂U π ∂ 2U
∫0 sin px dx = ∫0 sin px dx
7. Taking finite fourier cosine transform of ∂t ∂x 2
∂U ∂ 2U
= we have dU$ S
∂t ∂x 2 − p 2U$ S + p {U (0, t) − U ( π, t) cos pπ}
dt
6 ∂U πpx 6∂ 2U πpx ∵ U (0, t) = 1, U ( π, t) = 3, we get
∫0 ∂t cos dx = ∫0 ∂x 2 cos dx
6 6
143

dU$ S ∞
= p 2U$ S + p (1 − 3 cos pπ) 2 d ∞
U sin px dx =
2  ∂U 
sin px 
π dt ∫0
dt 
π  ∂x 0
dU$ S
or + p 2U$ S = p (1 − 3 cos pπ) 2 ∞ ∂U
dt −p cos px dx
π ∫0 ∂x
∂U ∂ 2U
12. Taking the fourier transform of =k , we dU$ S 2 ∞ ∂U
∂t ∂x 2 =p cos px dx
dt π ∫0 ∂x
have
∂U
1 ∞ ∂U ipx k ∞ ∂ 2U ipx if → 0 as x → ∞
∫ e dx = ∫ e dx ∂x
2 π −∞ ∂t 2 π −∞ ∂x 2
2 2 2 ∞
d 1 ∞
Ue ipx dx = k (−ip)2U$ = −p [U (x, t) cos px]∞
0 −p U sin px dx
dt 2 π ∫−∞ π π ∫0
dU$ 2
pU (0, t) − p 2U$ S as U → 0 as x → ∞
= −kp 2U$ =
dt π
2
So U$ = Ae − kp t dU$ S 2
+ p 2U$ S = pU (0, t)
∂ 2U ∂ 2U dt π
13. Taking the fourier transform of = c2 , we
∂t 2
∂x 2 dU$ S
+ p 2U$ S = 0 ∵ U (0, t) = 0
have dt
2
1 ∞ ∂ 2U c2 ∞ ∂ 2U So, its solution is U$ = Ae − p t .
∫−∞ ∂t 2 e ipx dx = ∫−∞ ∂x 2 e
ipx
dx S
2π 2π
15. Taking finite fourier sine transform of
d2 1 ∞ ^
∫−∞ Ue ipx dx = c 2(−ip)2 U ∂U 2 2
∂U
dt 2 2π + = 0.
2
d 2U$ ∂x ∂y 2
= − c 2p 2U$
dt 2 π ∂ 2U π ∂ 2U
∫0 sin px dx + ∫0 sin px dx = 0
d 2U$ 2 2$ ∂x 2 ∂y 2
or +c p U =0
dt 2 π
 ∂U sin px  − p π ∂U cos px dx
A.E. is m 2 + c 2p 2 = 0  ∂x  0 ∫0 ∂x
m = ± icp ∂2 π
where solution is U$ = A cos cpt + B sin cpt. + ∫ U sin px dx = 0
∂y 2 0
14. Since (U )x = 0 is given so taking the fourier sine π d 2U$ S
or − p[U cos px]π0 − p 2 ∫ U sin px dx + =0
∂U ∂ 2U
0 dy 2
transform of = , we have
∂t ∂x 2 d 2U S
or − p 2U$ S = 0
2 ∞ ∂U 2 ∞ ∂ 2U dy 2
sin px dx = sin px dx
π ∫0 ∂t π ∫0 ∂x 2 Since U (0, y) = U ( π, y) = 0
So, the solution is U$ S = A cos hpy + B sin hpy.

mmm
144
Unit-V
C HAPTER
20 Fourier Series
FOURIER SERIES EXPANSION IN THE INTERVAL a0 ∞ ∞
( 0, 2π) f (x ) = + Σ a n cos nx + Σ bn sin nx
2 n= 1 n= 1
1. The Fourier series is 1  x0 α +2π
∞ ∞ Where a 0 = ∫ f 1(x ) dx + ∫ f 2 (x )dx 
a0 π α x 0

f (x ) = + Σ a n cos nx + Σ bn sin nx
2 n= 1 n= 1 1 x0
1 2π a n = ∫ f 1(x ) cos nxdx
where a 0 = ∫ f (x ) dx π  α
π 0 α +2π
1 2π +∫ f 2 (x ) cos nx dx 
a n = ∫ f (x ) cos nxdx , x0 
π 0 1  x0
bn = f 1(x ) sin nx dx 
π ∫α
1 2π
bn = ∫ f (x ) sin nxdx 
π 0
α +2π
2. If the Fourier series is +∫ f 2 (x ) sin nx dx 
x0 
∞ ∞
f (x ) = a 0 + Σ a n cos nx + Σ bn sin nx At the point of discontinuity, x = α the Fourier
n= 1 n= 1
series gives f (x ) as
1 2π
Then a 0 = f (x ) dx 1
2π ∫0 f (x ) = [ f (α − 0) + f (α + 0)]
2
1 2π
a n = ∫ f (x ) cos nx dx
π 0 CHANGE OF PERIOD
1 2π
and bn = ∫ f (x ) sin nx dx Let the function f (x ) in (0, 2c)
π 0
∵ 2c is the interval for the variable = x
FOURIER SERIES EXPANSION IN ( −π, π) ∴ 2π is the interval for the variable =
x
. 2π
1. The Fourier series is 2c
∞ ∞ πx
a =
f (x ) = 0 + Σ a n cos nx + Σ bn sin nx c
2 n= 1 n= 1
1 π πx zc
where a 0 = ∫ f (x ) dx So, putting z = or x = , the function f (x )
π −π c π
1 π of period 2c is transformed for the function
a n = ∫ f (x ) cos nx dx
π −π  cz 
f   of period 2π.
1 π  π
and bn = ∫ f (x ) sin nx dx
π −π
FOURIER SERIES EXPANSION OF EVEN OR ODD
FOURIER SERIES FOR DISCONTINUOUS FUNCTIONS FUNCTION IN ( −π, π)
 f (x ), α < x < x 0 1. Let the fourier series expansion of an even
1. Let f (x ) =  1
 f 2 (x ), x 0 < x < α + 2π function in (−π, π) is
a ∞ ∞
in the interval (α, α + 2π) and x 0 is the point of f (x ) = 0 + Σ a n cos nx + Σ bn sin nx
discontinuity. Here the Fourier series is 2 n= 1 n= 1
145

1 π 2 π a0 ∞ nπx
Then, a 0 = f (x ) dx = ∫ f (x ) dx
π ∫− π π 0 f (x ) =
+ Σ a n cos
2 n= 1 T
1 π 2 π
a n = ∫ f (x ) cos nx dx = ∫ f (x ) dx 2 T
π − π π 0 where a 0 = ∫ f (x ) dx ,
T 0
1 π 2 π
2 T nπx
an = ∫−π f (x ) cos nx dx = π ∫0 f (x ) cos nx dx a n = ∫ f (x ) cos dx
π T 0 T
1 π
bn = ∫−π f (x ) sin nx dx = 0 2. The half range Fourier sine series in (0, T ) is
π
∞ nπx
2. Let the Fourier series expansion of odd f (x ) = Σ bn sin
n= 1 T
function [ f (− x ) = − f (x )] is
2 T nπx
a ∞ ∞ where bn = ∫0 f (x ) sin dx
f (x ) = 0 + T T
2 ∫n=1 an sin x + ∫n=1n sin nx
1 π PARSEVAL’S FORMULA
where a 0 = ∫−π f (x ) dx = 0
π 1. If the Fourier series for f (x ) converges
1 π uniformly in (−l , l ) then
an = ∫−π f (x ) cosnx dx = 0,
π l  a 02 ∞ 
2
2 π ∫− l [ f (x )] dx = l  + Σ (a n2 + bn2 )
bn = f (x ) sin nx dx  2 n= 1
π ∫0

2. If the Fourier series for f (x ) converges
HALF RANGE FOURIER COSINE AND SINE uniformly in (0, 2l ) then
SERIES IN ( 0, T)
2l  a2 ∞ 
1. The half range Fourier cosine series in (0, T ) is ∫0 [ f (x )]2 dx = l  0 + Σ (a n2 + bn2 )
 2 n= 1 

EXERCISE
MULTIPLE CHOICE QUESTIONS 4. The coefficient bn in the Fourier series expansion of
1. If the Fourier series of f (x) has only cosine forms x 3 for − π < x < π is :
then f (x) must be :  π 6
(a) 0 (b) (−1)n  − 
(a) Even (b) Odd  n3 n 
(c) May be even or odd (d) Does not exist π  6 π2 
(c) (−1)n (d) 2 (−1)n  − 
−x 3 3 n 
2. The coefficient a0 in the Fourier series for e in n  n
0 < x < 2 π is : 5. The coefficient a0 in the expansion of f (x) = e x in
(1 − e −2π ) n
(a) (b) (1 − e −2π ) (0, 1) is :
n2 + 1 n2 + 1 (a) 2e (b) 2e − 1 (c) 2e − 2 (d) 2e − 3
1 − e −2π (1 − e −2π )
(c) (d) 6. The coefficeint bn in the expansion of f (x) = 1 in
2π 2 πn
r < x < π is :
3. The coefficient an in the Fourier series for x cos x in  4 if n is odd  4 if n is even
− π < x < π is : (a)  nπ (b)  nπ
(a) 0 (b)
2n 0 if n is even 0 if n is odd
n2 − 1 4 nπ
(c) (d)
−n nπ 4
(c) (d) None of thse
n2 − 1
146

7. At the point of discontinuity α, Fourier series π π2 4 4


converges : (a) (b) (c) (d)
4 4 π2 π
(a) f (α + 0) + f (α − 0) (b) f ′(α + 0) + f ′(α − 0)
16. The Fourier series of sin 3x in (− π, π) is :
1 f (α + 0) − f (α − 0)
(c) [ f (α + 0) + f (α − 0)] (d) sin 3x x3 x5
2 2 (a) sin x − (b) x − +
3 3! 5!
8. Fourier coefficient a0 in Fourier series expansion 3 sin 3x sin 3x
(c) sin x − (d) 3 sin x −
represents : 4 4 3
(a) Minimum value of f (x)
17. In the Fourier series expansion of x 2nx (− π, π) the
(b) Maximum value of f (x)
(c) Mean value of f (x) coefficient an is :
4 (−1)n
(d) None of these (a) (−1)n (b)
n2 n2
9. In the Fourier series expansion of f (x) = |x| in
(−1)n 4 (−1)n
(− π, π) the value of a2 is : (c) (d)
n n
4 3 2
(a) 0 (b) − (c) (d) − x + π r ≤ x ≤ π
9π 4π 5π 18. If f (x) =  and f (x) is of period
− x − π – π ≤ x < 0
10. cos πx definied in (0, 1) can be expanded in series
2π then the coefficient a0 is :
of :
(a) 0 (b) 1 (c) π 2 (d) π
(a) Cosine terms only
l
(b) Sine and cosine terms both 19. The coefficient an in the Fourier expansion of − x
2
(c) Sine terms only in (0, l) is :
(d) Constant terms only l nπ n
(a) (b) (c) (d) 0
nπ l lπ
11. In the Fourier series expansion of f (x) = x − x 2 in
 x + 1 −1 < x < 0
(− π, π) the value of an is : 20. If f (x) =  , then coefficient bn in
x − 1 r < x < 1
(−1)n (−1)n+1
(a) (b) the Fourier series is :
n2 n2 1 −2 nπ
n (a) (b) (c) (d) 0
− n (−1) 2 (−1)n+1 nπ nπ 2
(c) (d)
2
n n2 21. The coefficient a0 in the Fourier expansion of
− π − π < x < 0 1 + x − 1 < x ≤ 0
12. If f (x) =  then the coefficient a0 in
x 0< x < π f (x) =  2 2 is :
1 1
 − x 0< x <
the Fourier series expansion is : 2 2
π sin nπ 1
(a) (−1)n (b) (a) 1 (b) (c) 2 (d) 0
2 π 2
− π2 −π 0, 0 < x < C
(c) (d) 22. If f (x) =  then the coefficent an in
2 2 1, C < x < 2C
13. The coefficient a0 in the fourier series for x 2 in the Fourier series is :
nπ 2 1
(a) (b) − (c) − (d) 0
− π < x < π is : 2 nπ nπ
2 3
nπ π The coefficient an in the Fourier expansion of x 2 in
(a) nπ (b) 2nπ 2 (c) (d) 23.
3 3 (− π, π) is :
x 4(−1)n (−1)n
14. In the expansion of 1 − in (0, l) the coefficient a0 is : (a) (b) (c) (−1)n n2 (d) 0
e n2
n2
4 2
(a) (b) (c) 2n2 π 2 (d) 1 24. The Fourier series of cos 3x in (− π, π) is :
2 2 nπ
n π 3 cos 3x cos 3x
(a) cos x + (b) cos x + + ...
15. The coefficient a0 in the Fourier series expansion of 4 4 3!
πx  3 x2 x4
sin   in 0 < x < l is : (c) cos x + cos 3x (d) 1 − + + ...
 l  4 2! 4!
147

25. In the Fourier expansion of x − x 2 in (− π, π) the 2


(c) if n is even otherwise 0
coefficient b2 is : nπ
(a) 1 (b) −2 (c) −1 (d) −4 2
(d) −

26. The coefficient bn in the Fourier series expansion of
f (x) = x 2 in (− π, π) is : 33. The coefficient a0 in the Fourier expansion of |x |in
(−1) n
n(−1) 4 (−2, 2) is :
(a) (b) (a) 1 (b) 2 (c) 3 (d) 0
n n2
2 34. If f (x) = x ( π − x) then the coefficient a0 in the
(c) 2n (d) 0
 −1 − π < x < − π Fourier expansion in the interval (0, π) is :
 2
 π π2 π π2
π π (a) (b) (c) (d)
27. If f (x ) =  0 − < x < , then Fourier 2 3 3 2
 2 2
π
1 <x< π 35. The coefficient a2 in the Fourier expansion of
 2
 πx , 0< x < 1
coefficient a0 is : f (x ) =  is :
 π(2 − x), 1 < x < 2
nπ nπ
(a) cos −π (b) sin −π 1 4 −2 −4
2 2 (a) (b) (c) (d)
nπ π π π π
(c) sin (d) 0
2 36. The Fourier series expansion in complex form is :
x
28. The Fourier coefficient bn in the Fourier expansion ∞ − inπ ∞
(a) Σ Cne t (b) Σ Cne nπx / T
l n= −∞ n= −∞
of − x in (0, l) is : ∞ ∞
2 (c) Σ Cne − nπx / T (d) Σ Cne + tinπx / T
l nπ π n= −∞ n= −∞
(a) (b) (c) (d) 0
nπ l ln
37. The coefficient a0 in the Fourier expansion of
1 + 2x
l<x<0  πx , r< x<1
29. If f (x) =  l , then coefficient a0 is : f (x ) is :
2x  π ( 2 − x ) 1 <x<2
1 − 0< x < l
 l
π2 π3
8 n2 π 2 nπ (a) (b) (c) π (d) 0
(a) 0 (b) (c) (d) 2 3
2 2 8 2
n π
38. If f (x) = x( π − x) then the coefficient bn in (0, π) is :
30. The coefficient an in the Fourier expansion of
πn3
 x + 1 −1 < x < 0 (a)
f (x ) =  is: 8
x − 1 0 < x < 1 8
(b) if n is odd otherwise 0
n2 π −2 n3 π
(a) nπ (b) (c) (d) 0
2 nπ
n3 π 3
(c)
31. The coefficient bn in the Fourier expansion of 8
1 + x − 1 < x ≤ 0 8
(d) if n is even otherwise 0
f (x) =  2 2 is : n3 π
1 1
 − x 0< x <
2 2 39. The coefficient bn in the Fourier expansion of |x |in
2 nπ (−2, 2) is :
(a) (b)
n2 π 2 2 8 (−1)n
(a) (b) (c) n2 π 2 (d) 0
2 2 2 2 2 2
(c) n π (d) 0 n π n π
0 0 < x < C 40. If f (x) = x 4 in (−1, 1) then the Fourier coefficient bn
32. If f (x) =  , then the coefficient bn in
1 C < x < 2C is :
the Fourier expansion is : (a) 1 (b) 2 (c) 3 (d) 0
2 2
(a) (b) − if n is odd otherwise 0
nπ nπ
148

ANSWERS
MULTIPLE CHOICE QUESTIONS
1. (a) 2. (c) 3. (a) 4. (d) 5. (c) 6. (a) 7. (c) 8. (c) 9. (a) 10. (c)
11. (c) 12. (d) 13. (d) 14. (d) 15. (d) 16. (c) 17. (a) 18. (d) 19. (d) 20. (b)
21. (b) 22. (d) 23. (a) 24. (a) 25. (c) 26. (d) 27. (d) 28. (a) 29. (a) 30. (d)
31. (d) 32. (b) 33. (b) 34. (b) 35. (d) 36. (d) 37. (c) 38. (b) 39. (d) 40. (d)

HINTS AND SOLUTIONS


2. f (x) = e − x in (0, 2π) −4 (−1)n
=
1 2π 1 2π − x 1 − e −2π n2
So, a0 = ∫ f (x) dx = ∫ e dx = − π − π < x < 0
2π 0 2π 0 2π 12. f (x ) = 
π r<x< π
3
4. f (x) = x for (− π, π)
1 π 1  0 +1 
a0 = ∫ f (x) dx = (− π) dx + ∫0 x dx 
2 π 2 π
bn = ∫ f (x) sin px dx = ∫ x 3 sin px dx 2π − π 2 π  ∫− π
π 0 π 0
1   2 π2   π
2  − π 3 cos nπ 6 π cos nπ  6 π2   = − π +  = −
 + = 2(−1)n  −  π   2   2
π  n 3  3 n  
n n
x
14. f (x ) = 1 − , 0 ≤ x ≤ l
6. f (x) = 1, r < x < π l
2 π 2 π 2 l 2 l  1 − x  dx
So, bn = ∫ f (x) sin nx dx = ∫ sin nx dx Then a0 = ∫ f (x) dx = ∫  
π 0 π 0 l 0 l 0  l
π l
2  cos nx  2 n 2 x2  2  l2 
−  = − [(−1) − 1] = − + x  = − + l 
π n 0 nπ l  2l  0 l  2l 
 n , if n is odd
=  4 π 2l 1
= 1 −  = 1
0, if n is even l  l 

9. f (x) = |x | in (− π, π) 16. ∵ sin 3x = 3 sin x − 4 sin 3 x


2 π 2 π 3 sin 3x
an = f (x ) cos nx dx = ∫ − x cos nx dx so, sin 3 x =
sin x −
π ∫0 π 0 4 4
+1 1 π 1 0
2  sin nx cos nx  2 18. a0 = ∫ f (x) dx = ∫ (− x − π) dx
=− x +  = [1 − (−1)n] π −π π −π
π  n n2  0 πn2 1 π
4 + ∫ (x + π) dx
= if n is odd and 0 if n is even. π 0
πn2  0 π
1  −x 2   x2  
So, a2 = 0 =  − πx  +  + πx  
π  2   
π  2 0 
 
11. f (x) = x − x 2 in (− π, π)
  2   2  
1 π π
1 π =  − π2 +  + π2  = π
an = (x − x 2) cos nx dx π   2   2 
π ∫− π    

1  2 sin nx  cos nx  20.


1 1 nπx
bn = f (x) sin dx
= (x − x ) n − (1 − 2x )  − 
1 ∫−1
π  n2  t
π 0 1
 − sin nx  
+ (−2) 
= ∫−1(x + 1) sin nxπ dx + ∫0(x − 1) sin nπx dx
 n3   −π
149
0
 cos nπx   − sin nπx   1 2 1 0 1 2
= (x + 1)  − 33. a0 = f (x) dx = ∫ (− x) dx + ∫ x dx
 
 − 
nπ   n2 π 2   −1 2 ∫−2 2 −2 2 0
2 0
1 1 x2 1  −x 2  1 1
  − cos nπx   − sin nπx   2 =   +   = (4 − 0) + (0 + 4) = 2
+ (x − 1)   −  =− 2  2  2  2  4 4
  n2 π 2   n2 π 2   0 nπ 0 −2
π
1 2C nπx 2 π 2  πx 2 x 3 
22. an = f (x) cos dx
C ∫0
34. a0 = ∫ x( π − x) dx =  − 
C π 0 π  2 3 
0
1 C nπx 1 2C nπx
= ∫ 0.cos dx + ∫ 1.cos dx
C 0 C C C C 2  π3 π3  π2
=  − =
2π π  2 3  3
1 C 4 πx 
= sin =0
C  4 π C  0 35. a2 =
2 C
f (x) cos
4 πx
dx
C ∫ 0 C
25. f (x) = x − x 2 in (− π, π) 2 1 nπx 2 nπx 
=  ∫ πx cos dx + ∫1 π (2 − x) cos dx 
1 π 2 0 2 2
∵ bn = ∫ (x − x 2) sin nx dx 
π −π 1
 nπx  nπx  
1 cos nx  
= (x − x 2)  −   x sin  cos 
π  n   = π 2 − − 2  +
 nπ  n2 π 2  
π
 sin nx   cos nx    2  
−(1 − 2x)  −  + (−2)     4  0
 n2   n3   − π 2
 nπx  nπx  
(−2) (−1)n  sin  cos 
= π (2 − x) 2 − (−1) − 2 
n nπ
  n2 π 2  
  
(−2) (−1)2  2  4  1
∴b2 = = −1
2  8 nπ 4 4 
= π cos − − cos nπx 
1 π 1 π/2  n2 π 2 2 n2 π 2 n2 π 2 
27. a0 = f (x) dx =  ∫ (−1)dx
π ∫− π π  −π an =
4 
2 cos

− 1 − cos nπ 
π/2 π  n2 π  2 
+∫ 0 dx + ∫π / 21 dx  4
−π / 2 ∴ a2 = −
π
1 π π
= − π+ π−  = 0 2 C 2 1 2
π  2 2  37. a0 = f (x) dx =  ∫ πxdx + ∫1 π(2 − x) dx 

C ∫0 2 0
1 C 1 l 1 2
29 a0 = F (x) dx = ∫ F (x) dx x2  x2
2C ∫− C 2l − l = π   + π  2x −  = π
 2  0  2 
1
1 0 2x  l 2x  
= 1 +  dx + ∫0  1 −  dx 2 π
2l  ∫− l  l  l   38. bn = x ( π − x) sin nx dx
π ∫0
0 l 2 cos nx   sin nx 
1  x2 1 x2 = ( πx − x 2)  −  − ( π − 2x)  − 2 
= x +  + x −  =0 π  n   n 
2l  l  2l  l 
−l 0 π
1 1/ 2 nπx cos nx 
31. bn = ∫ f (x) sin dx + (−2) 
1 − 1/ 2 1 n3  0
8
2 2 2  (−1)4 2   if n is odd
=  −2 3 + 3  =  n3 π
0 1  π  n n  0 if n is even
= 2∫  + x  sin 2nπx dx
−1/ 2  2 
1 1 4
1/ 2  1  40. bn ∫ f (x) sin nxdx = ∫−1 x sin nx dx = 0
+ 2∫  − x  sin 2nπx dx −1
0 2 
1  1  mmm
= 2 − + 2 =0
 4nπ   4nπ 

You might also like